CIMA’S Official Learning System
CIMA Certificate in Business Accounting,
2006 Syllabus
Fundamentals of
Management
Accounting
Janet Walker
CIMA Publishing is an imprint of Elsevier
Linacre House, Jordan Hill, Oxford OX2 8DP, UK
30 Corporate Drive, Suite 400, Burlington, MA 01803, USA
First edition 2006
Copyright # 2006 Elsevier Ltd. All rights reserved
No part of this publication may be reproduced, stored in a retrieval system
or transmitted in any form or by any means electronic, mechanical, photocopying,
recording or otherwise without the prior written permission of the publisher
Permissions may be sought directly from Elsevier’s Science & Technology Rights
Department in Oxford, UK: phone (+44) (0) 1865 843830; fax (+44) (0) 1865 853333;
e-mail: [email protected]. Alternatively you can submit your request online by
visiting the Elsevier web site at http://elsevier.com/locate/permissions, and selecting
Obtaining permission to use Elsevier material
Notice
No responsibility is assumed by the publisher for any injury and/or damage to persons
or property as a matter of products liability, negligence or otherwise, or from any use
or operation of any methods, products, instructions or ideas contained in the material
herein.
British Library Cataloguing in Publication Data
A catalogue record for this book is available from the British Library
ISBN-13: 978 0 7506 8031 8
ISBN-10: 0 7506 8031 8
Typeset by Integra Software Services Pvt. Ltd, Pondicherry, India
www.integra-india.com
Printed and bound in Great Britain
06 07 08 09 10 10 9 8 7 6 5 4 3 2 1
For information on all CIMA publications
visit our web site at books.elsevier.com
Contents
The CIMA Learning System xi
How to use your CIMA Learning System xi
Guide to the Icons used within this Text xii
Study technique xii
Computer-based assessment xiv
Fundamentals of Management Accounting and computer-based assessment xv
Learning Outcomes and Indicative Syllabus Content xvi
1 Basic Aspects of Cost Accounting 3
Learning Outcomes 3
1.1 Introduction 3
1.2 Why organisations need costing systems 3
1.3 What is meant by ‘cost’? 4
1.4 Cost units 4
1.4.1 Composite cost units 5
1.5 Cost centres 6
1.6 Cost objects 6
1.7 Classification of costs 7
1.7.1 Classification of costs according to their nature 7
1.7.2 Classification of costs according to their purpose:
direct costs and indirect costs 8
1.8 Elements of cost 9
1.9 Cost behaviour 12
1.9.1 Fixed cost 12
1.9.2 Variable cost 14
1.9.3 Semi-variable cost 15
1.9.4 Analysing semi-variable costs 16
1.9.5 Using historical data 18
1.9.6 The importance of time scale in analysing cost behaviour 18
1.10 Summary 19
Revision Questions 21
Solutions to Revision Questions 27
iii
2006.1
2 Accounting for the Value of Inventories 33
Learning Outcomes 33
2.1 Introduction 33
2.2 Valuing inventory at cost 33
2.3 First in, first out (FIFO) 34
2.4 Last in, first out (LIFO) 35
2.5 Cumulative weighted average (AVCO) 35
2.6 Comparison of FIFO, LIFO and AVCO 36
2.6.1 Historical cost compared with economic cost and economic value 37
2.7 Inventory valuation and the effect on gross profit 37
2.8 Periodic weighted average 38
2.9 Materials documentation 39
2.9.1 Perpetual inventory system 39
2.9.2 Recording the receipt of goods 39
2.9.3 Recording the movement of inventory items 39
2.10 Summary 40
Revision Questions 41
Solutions to Revision Questions 45
3 The Analysis of Overhead 53
Learning Outcomes 53
3.1 Introduction 53
3.2 What is an overhead cost? 53
3.2.1 Definition 53
3.2.2 Functional analysis of overhead costs 54
3.3 Overhead allocation and apportionment 54
3.4 Absorption of overheads into saleable cost units 56
3.4.1 General principles 56
3.4.2 Applying the overhead absorption rate 57
3.4.3 Other absorption bases 57
3.4.4 Selecting the most appropriate absorption rate 58
3.5 Predetermined overhead absorption rates 59
3.5.1 Under- or over-absorption of overheads 59
3.5.2 The reasons for under- or over-absorption 61
3.5.3 The problems caused by under- or over-absorption of overheads 61
3.6 Illustrative example 61
3.6.1 Solution 62
3.7 Reciprocal servicing 64
3.7.1 Taking account of reciprocal servicing 64
3.7.2 The usefulness of reapportioned service centre costs 65
3.8 Activity based costing (ABC) 66
3.9 The use of cost information in pricing decisions 66
3.9.1 Marginal cost pricing 66
3.9.2 Full cost-plus pricing 67
FUNDAMENTALS OF MANAGEMENT ACCOUNTING C1
iv
CONTENTS
2006.1
3.9.3 Example: full-cost pricing to achieve a specified return on sales 67
3.9.4 Example: full-cost pricing to achieve a specified return on investment 68
3.9.5 Second example: full-cost pricing to achieve a specified return
on investment 68
3.10 Summary 69
Revision Questions 71
Solutions to Revision Questions 79
4 Cost–Volume–Profit Analysis 87
Learning Outcomes 87
4.1 Introduction 87
4.2 Breakeven or cost–volume–profit analysis 87
4.2.1 The concept of contribution 88
4.2.2 Calculating the breakeven point 88
4.3 The margin of safety 88
4.4 The contribution to sales (C/S) ratio 89
4.5 Drawing a basic breakeven chart 90
4.6 The contribution breakeven chart 92
4.7 The profit–volume chart 92
4.7.1 The advantage of the profit–volume chart 93
4.8 The limitations of breakeven (or CVP) analysis 94
4.9 The economist’s breakeven chart 95
4.10 Using CVP analysis to evaluate proposals 96
4.11 Limiting factor analysis 98
4.11.1 Decisions involving a single limiting factor 98
4.12 Summary 102
Revision Questions 103
Solutions to Revision Questions 111
5 Standard Costing and Variance Analysis 119
Learning Outcomes 119
5.1 Introduction 119
5.2 What is a standard cost? 120
5.3 Performance levels 121
5.3.1 A standard 121
5.3.2 Ideal standard 122
5.3.3 Attainable standard 122
5.3.4 Current standard 122
5.4 Setting standard costs 122
5.4.1 Standard material price 122
5.4.2 Standard material usage 123
FUNDAMENTALS OF MANAGEMENT ACCOUNTING
v
CONTENTS
2006.1
5.4.3 Standard labour rate 123
5.4.4 Standard labour times 123
5.4.5 Variable production overhead costs 123
5.5 Updating standards 123
5.6 Standard costing in the modern business environment 124
5.7 What is variance analysis? 124
5.8 Variable cost variances 124
5.8.1 Direct material cost variances 125
5.8.2 The direct material price variance and inventory (stock) valuation 126
5.8.3 Direct labour cost variances 127
5.8.4 Variable overhead cost variances 128
5.9 Sales variances 129
5.9.1 Sales price variance 129
5.9.2 Sales volume contribution variance 130
5.10 Summary 130
Revision Questions 131
Solutions to Revision Questions 137
6 Further Standard Costing 145
Learning Outcomes 145
6.1 Introduction 145
6.2 Reconciling actual contribution with budgeted contribution 145
6.3 Idle time variances 148
6.4 Interpreting variances 149
6.4.1 The reasons for variances 149
6.4.2 The significance of variances 150
6.5 Standard hour 152
6.6 Labour incentive schemes 153
6.6.1 Bonus schemes 153
6.6.2 Piecework systems 154
6.6.3 Guaranteed minimum wage 155
6.6.4 Differential piece rate 155
6.6.5 Piecework hours 156
6.6.6 Group incentive schemes 156
6.7 Summary 157
Revision Questions 159
Solutions to Revision Questions 163
7 Integrated Accounting Systems 171
Learning Outcomes 171
7.1 Introduction 171
7.2 An integrated accounting system 171
FUNDAMENTALS OF MANAGEMENT ACCOUNTING C1
vi
CONTENTS
2006.1
7.3 Accounting for the cost of labour 172
7.3.1 Deductions from employees’ wages 172
7.3.2 Overtime premium 172
7.3.3 Bonus earnings 173
7.3.4 Idle time 173
7.3.5 Example: analysis of labour costs 173
7.4 Integrated accounts in operation 174
7.4.1 Example: the main accounting entries in an integrated system 174
7.4.2 Accounting for under- or over-absorbed overheads 176
7.4.3 Example: integrated accounts 177
7.5 Standard cost bookkeeping 184
7.6 Recording variances in the ledger accounts 184
7.6.1 General rules for recording variances 184
7.6.2 The income statement (profit and loss account) 185
7.7 Standard cost bookkeeping: an example 185
7.8 Valuing material inventory (stock) at actual cost 192
7.8.1 Which inventory (stock) valuation method is generally preferred? 193
7.9 Summary 193
Revision Questions 195
Solutions to Revision Questions 203
8 Specific Order Costing 209
Learning Outcomes 209
8.1 Introduction 209
8.2 Job costing 209
8.2.1 Job cost sheets and databases 210
8.2.2 Collecting the direct costs of each job 210
8.2.3 Attributing overhead costs to jobs 212
8.2.4 A worked example 213
8.2.5 Preparing ledger accounts for job costing systems 214
8.3 Batch costing 218
8.3.1 Example: batch costing 218
8.4 Contract costing 220
8.4.1 Architect’s certificates and progress payments 220
8.4.2 Retention money 220
8.4.3 Contract accounts 220
8.4.4 Accounting for contract materials 221
8.4.5 Accounting for plant used on the contract 221
8.4.6 Cost classification in contract costing 221
8.4.7 Calculating contract profit and preparing balance sheet entries 222
8.4.8 Contract costing: a worked example 222
8.4.9 Accounting for a loss-making contract 226
FUNDAMENTALS OF MANAGEMENT ACCOUNTING
vii
CONTENTS
2006.1
8.4.10 Contract costing: a second example 227
8.4.11 Contract costing: a final example 230
8.5 Summary 232
Revision Questions 233
Solutions to Revision Questions 239
9 Process Costing 247
Learning Outcomes 247
9.1 Introduction 247
9.2 Process accounts 247
9.3 Losses in process 249
9.4 Abnormal losses and gains 250
9.5 Closing work in progress: the concept of equivalent units 252
9.6 Previous process costs 256
9.7 Opening work in progress 256
9.8 Process costing: a further example 258
9.9 Contrasting process costing and specific order costing 260
9.10 Summary 260
Revision Questions 263
Solutions to Revision Questions 271
10 Presenting Management Information 281
Learning Outcomes 281
10.1 Introduction 281
10.2 Subjective and objective classification 281
10.2.1 Responsibility centres 282
10.2.2 Reporting management accounting information 282
10.3 Coding of costs 282
10.3.1 Composite codes 282
10.3.2 The advantages of a coding system 283
10.3.3 The requirements for an efficient coding system 284
10.4 Preparing financial statements that inform management 284
10.4.1 Value added 284
10.4.2 Contribution 285
10.4.3 Gross margin 286
10.5 Managerial reports in a service organisation 287
10.5.1 Establishing a suitable cost unit 287
10.5.2 Establishing the cost per unit 287
10.5.3 The instantaneous and perishable nature of services 287
10.5.4 Managerial reporting in a charity: example 291
10.6 Summary 292
Revision Questions 293
Solutions to Revision Questions 297
FUNDAMENTALS OF MANAGEMENT ACCOUNTING C1
viii
CONTENTS
2006.1
11 Financial Planning and Control 303
Learning Outcomes 303
11.1 Introduction 303
11.2 The purposes of budgeting 303
11.2.1 Budgetary planning and control 304
11.2.2 What is a budget? 304
11.2.3 The budget period 304
11.2.4 Strategic planning, budgetary planning and operational
planning 305
11.3 The preparation of budgets 305
11.3.1 Coordination: the budget committee 306
11.3.2 Participative budgeting 306
11.3.3 Information: the budget manual 306
11.3.4 Early identification of the principal budget factor 307
11.3.5 The interrelationship of budgets 307
11.3.6 Using computers in budget preparation 308
11.3.7 The master budget 308
11.4 Preparation of functional budgets 309
11.4.1 Budget interrelationships 311
11.5 The cash budget 311
11.5.1 Preparing cash budgets 312
11.5.2 Interpretation of the cash budget 313
11.6 A complete exercise 315
11.7 Rolling budgets 322
11.8 Budgets for non-operating functions 322
11.8.1 Incremental budgeting 323
11.8.2 Zero-based budgeting 323
11.9 Budgetary control information 323
11.9.1 Budget centres 324
11.9.2 Budgetary control reports 324
11.10 Fixed and flexible budgets 325
11.10.1 Flexible budgets: an example 325
11.10.2 Preparing a flexible budget 325
11.10.3 The total budget variance 327
11.10.4 Using flexible budgets for planning 328
11.10.5 Flexible budgets: another example 328
11.10.6 Extrapolating outside the relevant range 331
11.10.7 Example: producing a flexible budget control statement 331
11.11 Using budgets as a basis for rewards 333
11.11.1 Example 333
11.11.2 Factors to consider in the design of budget reward schemes 333
11.12 Summary 334
Revision Questions 335
Solutions to Revision Questions 343
FUNDAMENTALS OF MANAGEMENT ACCOUNTING
ix
CONTENTS
2006.1
Preparing for the Assessment 349
Format of the assessment 349
Revision technique 349
How to tackle the assessment 353
Revision Questions 355
Solutions to Revision Questions 387
Mock Assessment 1 421
Mock Assessment 2 451
Index 479
FUNDAMENTALS OF MANAGEMENT ACCOUNTING C1
x
2006.1
CONTENTS
The CIMA
Learning System
How to use your CIMA Learning System
This Fundamentals of Management Accounting Learning System has been devised as a resource for
students attempting to pass their CIMA computer-based assessment, and provides:
a detailed explanation of all syllabus areas;
extensive ‘practical’ materials;
generous question practice, together with full solutions;
a computer-based assessment preparation section, complete with computer-based
assessment standard questions and solutions.
This Learning System has been designed with the needs of home-study and distance-
learning candidates in mind. Such students require very full coverage of the syllabus topics,
and also the facility to undertake extensive question practice. However, the Learning
System is also ideal for fully taught courses.
The main body of the text is divided into a number of chapters, each of which is
organised on the following pattern:
Detailed learning outcomes expected after your studies of the chapter are complete. You
should assimilate these before beginning detailed work on the chapter, so that you can
appreciate where your studies are leading.
Step-by-step topic coverage. This is the heart of each chapter, containing detailed explanatory
text supported where appropriate by worked examples and exercises. You should work
carefully through this section, ensuring that you understand the material being explained
and can tackle the examples and exercises successfully. Remember that in many cases
knowledge is cumulative: if you fail to digest earlier material thoroughly, you may
struggle to understand later chapters.
Question practice. The test of how well you have learned the material is your ability to
tackle assessment-standard questions. Make a serious attempt at producing your own
answers, but at this stage do not be too concerned about attempting the questions in
computer-based assessment conditions. In particular, it is more important to absorb the
material thoroughly than to observe the time limits that would apply in the actual
computer-based assessment.
Solutions. Avoid the temptation merely to ‘audit’ the solutions provided. It is an illusion
to think that this provides the same benefits as you would gain from a serious attempt of
xi
2006.1
your own. However, if you are struggling to get started on a question you should read
the introductory guidance provided at the beginning of the solution, where provided,
and then make your own attempt before referring back to the full solution.
Having worked through the chapters you are ready to begin your final preparations for
the computer-based assessment. The final section of this CIMA Learning System provides
you with the guidance you need. It includes the following features:
A brief guide to revision technique.
A note on the format of the computer-based assessment. You should know what to
expect when you tackle the real computer-based assessment, and in particular the
number of questions that you will be required to attempt.
Guidance on how to tackle the computer-based assessment itself.
A table mapping revision questions to the syllabus learning outcomes allowing you to
quickly identify questions by subject area.
Revision questions. These are of computer-based assessment standard and should be
tackled in computer-based assessment conditions, especially as regards the time allocation.
Solutions to the revision questions.
Two mock computer-based assessments. You should plan to attempt these just before
the date of the real computer-based assessment. By this stage your revision should be
complete and you should be able to attempt the mock computer-based assessment within
the time constraints of the real computer-based assessment.
If you work conscientiously through this CIMA Learning System according to the
guidelines above you will be giving yourself an excellent chance of success in your
computer-based assessment. Good luck with your studies!
Guide to the Icons used within this text
Key term or definition
Assessment tip or topic likely to appear in the computer-based assessment
Exercise
Question
Solution
Comment or Note
Formula to learn
Study technique
Passing exams is partly a matter of intellectual ability, but however accomplished you are
in that respect you can improve your chances significantly by the use of appropriate study
and revision techniques. In this section we briefly outline some tips for effective study
THE CIMA LEARNING SYSTEM
FUNDAMENTALS OF MANAGEMENT ACCOUNTING C1
xii
2006.1
during the earlier stages of your approach to the computer-based assessment. Later in the
text we mention some techniques that you will find useful at the revision stage.
Planning
To begin with, formal planning is essential to get the best return from the time you spend
studying. Estimate how much time in total you are going to need for each paper you are
studying for the Certificate in Business Accounting. Remember that you need to allow
time for revision as well as for initial study of the material. The amount of notional study
time for any paper is the minimum estimated time that students will need to achieve the
specified learning outcomes in the syllabus. This time includes all appropriate learning
activities, for example, face-to-face tuition, private study, directed home study, learning in
the workplace, revision time, etc. You may find it helpful to read Better Exam Results: a
Guide for Business and Accounting Students by S.A. Malone, Elsevier, ISBN: 075066357X. This
book will provide you with proven study techniques. Chapter by chapter it covers the
building blocks of successful learning and examination techniques.
The notional study time for the Certificate in Business Accounting Paper
Fundamentals of Management Accounting is 130 hours. Note that the standard amount
of notional learning hours attributed to one full-time academic year of approximately
30 weeks is 1,200 hours.
By way of example, the notional study time might be made up as follows:
Hours
Face-to-face study: up to 40
Personal study: up to 65
‘Other’ study e.g. learning in the workplace, revision, etc.: up to
25
130
Note that all study and learning-time recommendations should be used only as a
guideline and are intended as minimum amounts. The amount of time recommended for
face-to-face tuition, personal study and/or additional learning will vary according to the
type of course undertaken, prior learning of the student, and the pace at which different
students learn.
Now split your total time requirement over the weeks between now and the assessment.
This will give you an idea of how much time you need to devote to study each week.
Remember to allow for holidays or other periods during which you will not be able to
study (e.g. because of seasonal workloads).
With your study material before you, decide which chapters you are going to study in
each week, and which weeks you will devote to revision and final question practice.
Prepare a written schedule summarising the above and stick to it!
It is essential to know your syllabus. As your course progresses you will become more
familiar with how long it takes to cover topics in sufficient depth. Your timetable may
need to be adapted to allocate enough time for the whole syllabus.
THE CIMA LEARNING SYSTEM
2006.1
FUNDAMENTALS OF MANAGEMENT ACCOUNTING
xiii
Tips for effective studying
1. Aim to find a quiet and undisturbed location for your study, and plan as far as possible
to use the same period of time each day. Getting into a routine helps to avoid wasting
time. Make sure that you have all the materials you need before you begin so as to
minimise interruptions.
2. Store all your materials in one place, so that you do not waste time searching for items
every time you want to begin studying. If you have to pack everything away after each
study period, keep your study materials in a box, or even a suitcase, which will not be
disturbed until the next time.
3. Limit distractions. To make the most effective use of your study periods you should be
able to apply total concentration, so turn off all entertainment equipment, set your
phones to message mode, and put up your ‘do not disturb’ sign.
4. Your timetable will tell you which topic to study. However, before diving in and
becoming engrossed in the finer points, make sure you have an overall picture of all the
areas that need to be covered by the end of that session. After an hour, allow yourself a
short break and move away from your Learning System. With experience, you will learn
to assess the pace you need to work at.
5. Work carefully through a chapter, making notes as you go. When you have covered a
suitable amount of material, vary the pattern by attempting a practice question. When
you have finished your attempt, make notes of any mistakes you made.
6. Make notes as you study, and discover the techniques that work best for you. Your
notes may be in the form of lists, bullet points, diagrams, summaries, ‘mind maps’, or
the written word, but remember that you will need to refer back to them at a later date,
so they must be intelligible. If you are on a taught course, make sure you highlight any
issues you would like to follow up with your lecturer.
7. Organise your paperwork. Make sure that all your notes, calculations etc. can be
effectively filed and easily retrieved later.
Computer-based assessment
CIMA uses computer-based assessment (CBA) for all subjects for the Certificate in
Business Accounting. The website says:
Objective questions are used. The most common type is ‘multiple choice’, where you
have to choose the correct answer from a list of possible answers, but there are a variety of
other objective question types that can be used within the system. These include true/false
questions, matching pairs of text and graphic, sequencing and ranking , labelling diagrams
and single and multiple numeric entry.
Candidates answer the questions by either pointing and clicking the mouse, moving
objects around the screen, typing numbers, or a combination of these responses. Try the
online demo at www.cimaglobal.com to see how the technology works.
The CBA system can ensure that a wide range of the syllabus is assessed, as a pre-
determined number of questions from each syllabus area (dependent upon the syllabus
weighting for that particular area) are selected in each assessment.
In every chapter of this Learning System we have introduced these types of questions
but obviously we have to label answers A, B, C etc. rather than using click boxes. For
FUNDAMENTALS OF MANAGEMENT ACCOUNTING C1
xiv
THE CIMA LEARNING SYSTEM
2006.1
convenience we have retained quite a lot of questions where an initial scenario leads to a
number of sub-questions. There will be questions of this type in the CBA but they will
rarely have more than three sub-questions. In all such cases examiners will ensure that the
answer to one part does not hinge upon a prior answer.
For further CBA practice, CIMA E-Success CD’s will be available from www.
cimapublishing.com from September 2006.
Fundamentals of Management Accounting and
Computer-based Assessment
The assessment for Fundamentals of Management Accounting is a two-hour computer-
based assessment comprising 50 compulsory questions, with one or more parts. Single
part questions are generally worth 1–2 marks each, but two and three part questions may
be worth 4 or 6 marks. There will be no choice and all questions should be attempted if
time permits. CIMA are continuously developing the question styles within the CBA
system and you are advised to try the on-line website demo at www.cimaglobal.com, to
both gain familiarity with assessment software and examine the latest style of questions
being used.
Syllabus (2006) Paper C01 Fundamentals of
Management Accounting
Syllabus Outline
The syllabus comprises:
Topic and Study Weighting
A Cost Determination 25%
B Cost Behaviour and Break-even Analysis 10%
C Standard Costing 15%
D Cost and Accounting Systems 30%
E Financial Planning and Control 20%
Learning Aims
This syllabus aims to test student’s ability to:
explain and use concepts and processes to determine product and service costs;
explain direct, marginal and absorption costs and their use in pricing;
apply CVP analysis and interpret the results;
apply a range of costing and accounting systems;
explain the role of budgets and standard costing within organisations;
prepare and interpret budgets, standard costs and variance statements.
FUNDAMENTALS OF MANAGEMENT ACCOUNTING
xv
THE CIMA LEARNING SYSTEM
2006.1
Assessment Strategy
There will be a computer-based assessment of 2 hours duration, comprising 50 compulsory
questions, each with one or more parts.
A variety of objective test question types and styles will be used within the assessment.
Learning Outcomes and Indicative Syllabus Content
A Cost Determination 25%
Learning Outcomes
On completion of their studies students should be able to:
(i) explain why organisations need to know how much things cost and why they need
costing systems;
(ii) explain the idea of a ‘cost object’;
(iii) explain the concept of a direct cost and an indirect cost;
(iv) explain why the concept of ‘cost’ is not as simple as it first seems;
(v) distinguish between the historic cost of an asset and the economic value of an asset
to an organisation;
(vi) apply first-in-first-out (FIFO), last-in-first-out (LIFO) and average cost (AVCO)
methods of accounting for stock, calculating stock values and related gross profit;
(vii) explain why first-in-first-out (FIFO) is essentially a historic cost method, while last-
in-first-out (LIFO) approximates economic cost;
(viii) prepare cost statements for allocation and apportionment of overheads, including
reciprocal service departments;
(ix) calculate direct, variable and full costs of products, services and activities using
overhead absorption rates to trace indirect costs to cost units;
(x) explain the use of cost information in pricing decisions, including the calculation of
‘full cost’ based prices to generate a specified return on sales or investment.
Indicative Syllabus Content:
Classification of costs and the treatment of direct costs (specifically attributable to a cost
object) and indirect costs (not specifically attributable) in ascertaining the cost of a ‘cost
object’ (e.g. a product, service, activity, customer).
Cost measurement: historic versus economic costs.
Accounting for the value of materials on FIFO, LIFO and AVCO bases.
Overhead costs: allocation, apportionment, re-apportionment and absorption of overhead
costs. Note: The repeated distribution method only will be used for reciprocal service
department costs.
Full-cost pricing to achieve specified return on sales or return on investment.
THE CIMA LEARNING SYSTEM
FUNDAMENTALS OF MANAGEMENT ACCOUNTING C1
xvi
2006.1
B Cost Behaviour and Break-even Analysis 10%
Learning Outcomes
On completion of their studies students should be able to:
(i) explain how costs behave as product, service or activity levels increase or decrease;
(ii) distinguish between fixed, variable and semi-variable costs;
(iii) explain step costs and the importance of time-scales in their treatment as either
variable or fixed;
(iv) estimate the fixed and variable elements of cost using the high-low method and ‘line
of best fit’ method;
(v) explain the contribution concept and its use in CVP analysis;
(vi) calculate and interpret the break-even point, profit target, margin of safety and
profit/volume ratio for a single product or service;
(vii) prepare break-even charts and profit/volume graphs for a singe product or service;
(viii) calculate the profit maximising sales mix for a multi-product company that has
limited demand for each product and one other constraint or limiting factor.
Indicative Syllabus Content:
Fixed, variable and semi-variable costs.
Step costs and the importance of time-scale in analysing cost behaviour.
High-low and graphical methods to establish fixed and variable components of a semi-
variable cost. Note: regression analysis is not required.
Contribution concept.
Break-even charts, profit volume graphs, break-even point, profit target, margin of
safety, contribution/sales ratio.
Limiting factor analysis.
C Standard Costing 15%
Learning Outcomes
On completion of their studies students should be able to:
(i) explain the difference between ascertaining costs after the event and planning by
establishing ‘standard costs’ in advance;
(ii) explain why planned ‘standard costs’, prices and volumes are useful in setting a
benchmark for comparison and so allowing managers’ attention to be directed to
areas of the business that are performing below or above expectation;
(iii) prepare ‘standard costs’ for the material, labour and variable overhead elements of
cost in a product or service;
(iv) calculate variances for materials, labour, variable overhead, sales prices and sales
volumes;
(v) prepare a statement that reconciles budgeted contribution margin with actual
contribution margin;
THE CIMA LEARNING SYSTEM
FUNDAMENTALS OF MANAGEMENT ACCOUNTING
xvii
2006.1
(vi) interpret statements of variances for variable costs, sales prices and sales volumes
including possible inter-relations between cost variances, sales price and volume
variances, and cost and sales variances;
(vii) describe the possible use of standard labour costs in designing incentive schemes for
factory and office workers.
Indicative Syllabus Content:
Principles of standard costing.
Preparation of standards for the variable elements of cost: material, labour, variable
overhead.
Variances: materials total, price and usage; labour total, rate and efficiency; variable
overhead total, expenditure and efficiency; sales price, contribution margin volume.
Note: Candidates will be expected to calculate the sales contribution margin volume
variance.
Reconciliation of budget and actual contribution margin.
Piecework and the principles of incentive schemes based on standard hours ‘produced’
versus actual hours taken. Note: the details of a specific incentive scheme will be
provided.
D Costing and Accounting Systems 30%
Learning Outcomes
On completion of their studies students should be able to:
(i) explain the principles of production accounts and the integration of the cost
accounts with the financial accounting system;
(ii) prepare a set of integrated accounts, given opening balances and appropriate
transactional information, and show standard cost variances;
(iii) compare and contrast job, batch, contract and process costing;
(iv) prepare ledger accounts for job, batch and process costing systems;
(v) prepare ledger accounts for contract costs;
(vi) explain the difference between subjective and objective classifications of expenditure
and the importance of tracing costs both to products/services and to responsibility
centres;
(vii) design coding systems that facilitate both subjective and objective classification of
costs;
(viii) prepare financial statements that provide relevant management information;
(ix) explain why gross revenue, net revenue, value-added, contribution margin, gross
margin, marketing expense, general and administration expense, etc. might be
highlighted in management reporting;
(x) compare and contrast managerial reports in a range of organisations including
commercial enterprises, charities and government undertakings.
THE CIMA LEARNING SYSTEM
FUNDAMENTALS OF MANAGEMENT ACCOUNTING C1
xviii
2006.1
Indicative Syllabus Content:
Integrated ledgers including over and under absorption of production overhead.
The treatment of variances as period entries in integrated ledger systems.
Job, batch, process and contract costing. Note: Only the average cost method will be
examined for process costing but candidates must be able to deal with differing degrees
of completion of opening and closing stocks, normal and abnormal gains and losses, and
the treatment of scrap value.
Subjective, objective and responsibility classifications and design of coding systems to
facilitate these analyses.
Cost accounting statements for management information in production and service
companies and not-for-profit organisations.
E Financial Planning and Control 20%
Learning Outcomes
On completion of their studies students should be able to:
(i) explain why organisations set out financial plans in the form of budgets, typically for
a financial year;
(ii) prepare functional budgets for material usage and purchase, labour and overheads,
including budgets for capital expenditure and depreciation;
(iii) prepare a ‘master budget’; profit and loss account, balance sheet and cash flow
statement, based on the functional budgets;
(iv) interpret budget statements and advise managers on financing projected cash
shortfalls and/or investing projected cash surpluses;
(v) prepare a flexible budget based on the actual levels of sales and production and
calculate appropriate variances;
(vi) compare and contrast fixed and flexible budgets;
(vii) explain the use of budgets in designing reward strategies for managers.
Indicative Syllabus Content:
Budgeting for planning and control.
Budget preparation, interpretation and use of the ‘master’ budget.
Reporting of actual against budget.
Fixed and flexible budgeting.
Budget variances.
Interpretation and use of budget statements and budget variances.
THE CIMA LEARNING SYSTEM
FUNDAMENTALS OF MANAGEMENT ACCOUNTING
xix
2006.1
1
Basic Aspects of Cost
Accounting
Basic Aspects of
Cost Accounting
1.1 Introduction
In this chapter we will look at some of the fundamental concepts of the framework of cost
accounting. You will learn some basic principles which underpin all of the material in your
Fundamentals of Management Accounting syllabus.
1.2 Why organisations need costing systems
An organisation’s costing system is the foundation of the internal financial information
system for managers. It provides the information that management needs to plan and
control the organisation’s activities and to make decisions about the future. Examples of
1
LEARNING OUTCOMES
After completing this chapter, you should be able to:
"
explain why organisations need to know how much products, processes and services
cost and why they need costing systems;
"
explain the idea of a ‘cost object’;
"
explain the concept of a direct cost and an indirect cost;
"
explain why the concept of cost needs to be qualified as direct, full, marginal, etc. in
order to be meaningful;
"
explain how costs behave as product, service or activity levels increase or decrease;
"
distinguish between fixed, variable and semi-variable costs;
"
explain step costs and the importance of time-scales in their treatment as either
variable or fixed;
"
compute the fixed and variable elements of a semi-variable cost using the high–low
method and ’line of best fit’ method.
3 2006.1
the type of information provided by a costing system and the uses to which it might be put
include the following.
Actual unit costs for the latest period; could be used for cost control by comparing
with a predetermined unit standard cost, which would also be provided by the costing
system. Could also be used as the basis for planning future unit costs and for
decisions about pricing and production levels. For example, a manager cannot make a
decision about the price to be charged to a customer without information which tells
the manager how much it costs to produce and distribute the product to the
customer.
Actual costs of operating a department for the latest period; could be used for cost control
by comparing with a predetermined budget for the department. Could also be used as the
basis for planning future budgeted costs and for decisions such as outsourcing. For
example, a manager might be considering the closure of the packing department and
instead outsourcing the packing operations to another organisation. In order to make this
decision the manager needs to know, among other things, the actual cost of operating the
packing department.
The forecast costs to be incurred at different levels of activity. Could be used for
planning, for decision making and as a part of cost control by comparing the actual costs
with the forecasts. For example, a manager cannot make a well-informed decision about
the appropriate production level for the forthcoming period unless information is
available about the costs that will be incurred at various possible output levels.
This is by no means an exhaustive list of the information that is provided by a costing
system. However it should serve to demonstrate that organisations need costing systems
that will provide the basic information that management requires for planning, control
and decision-making.
1.3 What is meant by ‘cost’?
The word ‘cost’ can be used in two contexts. It can be used as a noun, for example, when
we are referring to the cost of an item. Alternatively, it can be used as a verb, for example,
we can say that we are attempting to cost an activity, when we are undertaking the tasks
necessary to determine the costs of carrying out the activity.
The word ‘cost’ can rarely stand alone and should always be qualified as to its nature and
limitations. You will be seeing throughout this text that there are many different types of
cost and that each has its usefulness and limitations in different circumstances.
1.4 Cost units
The CIMA Terminology defines a cost unit as ‘a unit of product or service in relation
to which costs are ascertained’.
This means that a cost unit can be anything for which it is possible to ascertain the
cost. The cost unit selected in each situation will depend on a number of factors,
including the purpose of the cost ascertainment exercise and the amount of information
available.
STUDY MATERIAL C1
4
BASIC ASPECTS OF COST ACCOUNTING
2006.1
Cost units can be developed for all kinds of organisations, whether manufacturing,
commercial or public service based. Some examples from the CIMA Terminology are as
follows:
Industry sector Cost unit
Brick-making 1000 bricks
Electricity Kilowatt-hour (KwH)
Professional services Chargeable hour
Education Enrolled student
Activity Cost unit
Credit control Account maintained
Selling Customer call
Exercise 1.1
Can you think of at least one other cost unit which could be used for each of these
industries and activities? For example, in controlling the costs of the selling activity we
might monitor the cost per order taken.
The above list is not exhaustive. A cost unit can be anything which is measurable
and useful for cost control purposes. For example, with brick-making, 1,000 bricks is
suggested as a cost unit. It would be possible to determine the cost per brick but perhaps
in this case a larger measure is considered more suitable and useful for control purposes.
Notice that this list of cost units contains both tangible and intangible items. Tangible
items are those which can be seen and touched, for example the 1,000 bricks. Intangible
items cannot be seen and touched and do not have physical substance but they can be
measured, for example a chargeable hour of accounting service.
1.4.1 Composite cost units
The cost units for services are usually intangible and they are often composite cost units,
that is, they are often made up of two parts. For example, if we were attempting to
monitor and control the costs of a delivery service we might measure the cost per tonne
delivered. However, ‘tonne delivered’ would not be a particularly useful cost unit because
it would not be valid to compare the cost per tonne delivered from London to Edinburgh
with the cost per tonne delivered from London to Brighton. The former journey is
much longer and it will almost certainly cost more to deliver a tonne over the longer
distance.
Composite cost units assist in overcoming this problem. We could perhaps use a ‘tonne-
mile’ instead. This means that we would record and monitor the cost of carrying one tonne
for one mile. The cost per tonne-mile would be a comparable measure whatever the length
of journey and this is therefore a valid and useful cost unit for control purposes.
Other examples of composite cost units might be as follows:
Business Cost unit
Hotel Bed night
Bus company Passenger mile
Hospital In-patient day
FUNDAMENTALS OF MANAGEMENT ACCOUNTING
5
BASIC ASPECTS OF COST ACCOUNTING
2006.1
Exercise 1.2
Can you think of some other examples of composite cost units that could be used in these
organisations and in other types of organisation?
1.5 Cost centres
A cost centre is a production or service location, a function, an activity or an item of
equipment for which costs are accumulated.
A cost centre is used as a ‘collecting place’ for costs. The cost of operating the cost
centre is determined for the period, and then this total cost is related to the cost units
which have passed through the cost centre.
For instance, an example of a production cost centre could be the machine shop in
a factory. The production overhead cost for the machine shop might be £100,000 for
the period. If 1,000 cost units have passed through this cost centre we might say
that the production overhead cost relating to the machine shop was £100 for
each unit.
A cost centre could also be a service location, a function, an activity or an item of
equipment. Examples of these might be as follows but you should try to think of some
others:
Type of cost centre Examples
Service location Stores, canteen
Function Sales representative
Activity Quality control
Item of equipment Packing machine
If you are finding it difficult to see how a sales representative could be used as a cost
centre, then work carefully through the following points.
1. What are the costs which might be incurred in ‘operating’ a sales representative for one
period?
Examples might be the representative’s salary cost, the cost of running a company
car, the cost of any samples given away by the representative and so on. Say these
amount to £40,000.
2. Once we have determined this cost, the next thing we need to know is the number of
cost units that can be related to the sales representative.
The cost unit selected might be £100 of sales achieved. If the representative has
achieved £400,000 of sales, then we could say that the representative’s costs
amounted to £10 per £100 of sales. The representative has thus been used as a cost
centre or collecting place for the costs, which have then been related to the cost
units.
1.6 Cost objects
A cost object is anything for which costs can be ascertained. The CIMA Terminology contains
the following description: ‘For example a product, service, centre, activity, customer or
distribution channel in relation to which costs are ascertained.’
STUDY MATERIAL C1
6
BASIC ASPECTS OF COST ACCOUNTING
2006.1
All of the cost units and cost centres we have described earlier in this chapter are
therefore types of cost object. We have seen the quality control activity being treated as a
cost centre, and thus as a cost object.
Exercise 1.3
Notice that CIMA’s examples of cost objects include a customer. Can you think of costs
that might be attributed to a supermarket which is a customer and is treated as a cost
object by a supplier of processed foods?
Solution
Costs that you might have thought of include the following.
The cost of the food products supplied to the customer.
The cost of delivering the food products to the customer.
The cost of funding the credit taken by the customer.
The cost of holding any inventories for the supermarket.
The salary cost of the account manager responsible for the supermarket’s account.
The cost of dealing with the customer’s queries.
1.7 Classification of costs
Costs can be classified in many different ways. It is necessary to be able to classify all
costs, that is, to be able to arrange them into logical groups, in order to devise an
efficient system to collect and analyse the costs. The classifications selected and the level
of detail used in the classification groupings will depend on the purpose of the
classification exercise.
The CIMA Terminology defines classification as the ‘arrangement of items in logical
groups by nature, purpose or responsibility’.
1.7.1 Classification of costs according to their nature
This means grouping costs according to whether they are materials, labour or expense
cost.
Material costs include the cost of obtaining the materials and receiving them within the
organisation. The cost of having the materials brought to the organisation is known as
carriage inwards.
Labour costs are those costs incurred in the form of wages and salaries, together with
related employment costs. In the United Kingdom, there is an additional cost borne by the
employer in respect of employees which is paid to the government: this is called National
Insurance. These costs are documented internally, the amount of the wages and salary
costs being determined by reference to agreed rates of pay and attendance time and output
measures, depending on the method of remuneration being used.
FUNDAMENTALS OF MANAGEMENT ACCOUNTING
7
BASIC ASPECTS OF COST ACCOUNTING
2006.1
Expense costs are external costs such as rent, business rates, electricity, gas, postages,
telephones and similar items which will be documented by invoices from suppliers.
Within each of these classifications there is a number of subdivisions; for example,
within the materials classification the subdivisions might include the following:
(a) Raw materials, that is, the basic raw material used in manufacture.
(b) Components, that is, complete parts that are used in the manufacturing process.
(c) Consumables, that is, cleaning materials, etc.
(d) Maintenance materials, that is, spare parts for machines, lubricating oils, etc.
This list of subdivisions is not exhaustive, and there may even be further subdivisions
of each of these groups. For example, the raw materials may be further divided according
to the type of raw material, for example, steel, plastic, glass, etc.
Exercise 1.4
Can you think of some possible subdivisions for the costs that are classified as labour costs
and as expense costs?
1.7.2 Classification of costs according to their purpose:
direct costs and indirect costs
When costs are classified having regard to their purpose, they are grouped according to the
reason for which they have been incurred. The broadest classification of this type is to
divide costs into direct costs and indirect costs.
A direct cost is one that can be clearly identified with the cost object we are trying to
cost. For example, suppose that a furniture maker is determining the cost of a wooden
table. The manufacture of the table has involved the use of timber, screws and metal
drawer handles. These items are classified as direct materials. The wages paid to the machine
operator, assembler and finisher in actually making the table would be classified as direct
labour costs. The designer of the table may be entitled to a royalty payment for each table
made, and this would be classified as a direct expense.
Other costs incurred would be classified as indirect costs. They cannot be directly
attributed to a particular cost unit, although it is clear that they have been incurred in the
production of the table. Examples of indirect production costs are as follows:
Cost incurred Cost classification
Lubricating oils and cleaning
materials
Indirect material
Salaries of supervisory labour Indirect labour
Factory rent and power Indirect expense
It is important for you to realise that a particular cost may sometimes be a direct cost
and sometimes an indirect cost. It depends on the cost object we are trying to cost.
For example, the salary of the machining department supervisor is a direct cost of that
department because it can be specifically identified with the department. However, it is an
indirect cost of each of the cost units processed in the machining department because it
cannot be specifically identified with any particular cost unit.
STUDY MATERIAL C1
8
BASIC ASPECTS OF COST ACCOUNTING
2006.1
Exercise 1.5
State whether each of the following costs would be a direct cost or an indirect cost of the
quality control activity which is undertaken in a company’s factory.
The salary of the quality control supervisor.
The rent of the factory.
The depreciation of the quality testing machine.
The cost of the samples destroyed during testing.
The insurance of the factory.
Solution
The salary of the quality control supervisor is a direct cost of the quality control activity
because it can be specifically attributed to this cost object.
The rent of the factory is an indirect cost of the quality control activity because it cannot
be specifically attributed to this cost object but must also be attributed to other activities
undertaken in the factory.
The depreciation of the quality testing machine is a direct cost of the quality control
activity because it can be specifically attributed to this cost object.
The cost of the samples destroyed during testing is a direct cost of the quality control
activity because it can be specifically attributed to this cost object.
The insurance of the factory is an indirect cost of the quality control activity because it
cannot be specifically attributed to this cost object but must also be attributed to other
activities undertaken in the factory.
In a later chapter we will return to consider the classification of costs by
responsibility.
1.8 Elements of cost
The elements of cost are the constituent parts of cost which make up the total cost of a
cost object.
In Figure 1.1, the outline cost statement for a single cost unit shows you how the total
or full cost for a unit might be built up. Notice in particular that a number of subtotals can
be highlighted before the total cost figure is determined.
The usefulness of each of these subtotals depends on the management action that is to
be taken based on each of the totals.
Suppose that the cost analysis in Figure 1.1 has been provided by the management
accountant to help us to decide on the selling price to be charged for a luxury wall-
mounted hairdryer: the type that is fixed to the wall for customers’ use in hotel bedrooms.
You have been negotiating with the procurement manager of a chain of hotels in an
attempt to secure a contract to supply a batch of hairdryers. It is very important that you
should win this contract because it is likely that, once this first order has been fulfilled
successfully, the hotel chain will place future orders for hairdryers and for your company’s
other products, when refurbishing its other hotels. Furthermore, other hotel chains may
become interested in your company’s products once they discover that this major chain is
one of your customers.
FUNDAMENTALS OF MANAGEMENT ACCOUNTING
9
BASIC ASPECTS OF COST ACCOUNTING
2006.1
Unfortunately, the hotel’s procurement manager is working within the constraints of a
very strict budget and has made it clear that the highest price that the hotel is prepared to
pay is £25 per hairdryer. The analysis in Figure 1.1 shows that your company’s normal
selling price is considerably higher than this.
The company cannot afford to sell its hairdryers for £25 each if they cost £40 to
produce and sell. Or can it?
Let us look at the sort of costs that might be incurred in manufacturing and selling a
hairdryer, and how each cost would be classified in terms of the above analysis of the
elements of cost.
Direct materials. This is the material that actually becomes part of the finished hairdryer. It
would include the plastic for the case and the packaging materials. If we make another
batch of hairdryers then we will need to purchase another batch of these and other
direct materials.
Direct labour. This is the labour cost incurred directly as a result of making one hairdryer.
If we make another batch of hairdryers then we will need to pay more direct labour cost.
Direct expenses. These are expenses caused directly as a result of making one more batch
of hairdryers. For example, the company might be required to pay the designer of the
hairdryer a royalty of £2 for each hairdryer produced.
The three direct costs are summed to derive the prime cost or total direct cost of
a hairdryer. This is one measure of cost but there are still other costs to be added:
production overheads and other overheads.
Production overheads are basically the same three costs as for direct cost, but they are
identified as indirect costs because they cannot be specifically identified with any particular
hairdryer or batch of hairdryers. Indirect costs must be shared out over all the cost objects
using a fair and equitable basis.
In a later chapter you will see how indirect costs can be shared over all the
production for the period.
Figure 1.1 The build-up of cost
STUDY MATERIAL C1
10
BASIC ASPECTS OF COST ACCOUNTING
2006.1
Indirect materials are those production materials that do not actually become part of the
finished product. This might include the cleaning materials and lubricating oils for the
machinery. The machines must be clean and lubricated in order to carry out production, but
it will probably not be necessary to spend more on these materials in order to manufacture a
further batch. This cost is therefore only indirectly related to the production of this batch.
Indirect labour is the production labour cost which cannot be directly associated with the
production of any particular batch. It would include the salaries of supervisors who are
overseeing the production of hairdryers as well as all the other products manufactured in
the factory.
Indirect expenses are all the other production overheads associated with running the
factory, including factory rent and rates, heating and lighting, etc. These indirect costs
must be shared out over all of the batches produced in a period.
The share of indirect production costs is added to the prime cost to derive the total
production cost of a hairdryer. This is another measure of cost but there are still more
costs to be added: a share of the other overheads.
Selling and distribution overhead includes the sales force salaries and commission, the cost of
operating delivery vehicles and renting a storage warehouse, etc. These are indirect costs
which are not specifically attributable to a particular cost unit.
Administration overhead includes the rent on the administrative office building, the
depreciation of office equipment, postage and stationery costs, etc. These are also indirect
costs which are not specifically attributable to a particular cost unit.
Now that you understand the nature of each of the cost elements which make up the
full cost we can think a bit more about the price to be charged to the hotel chain.
Exercise 1.6
Which of the above costs would be incurred as a result of making another hairdryer?
Solution
The direct cost of £22 would definitely be incurred if another hairdryer was produced.
This is the extra material that would have to be bought, the extra labour costs that would
have to be paid and the extra expenses for royalties that would be incurred.
The £16 production overhead cost would not be incurred additionally if another
hairdryer was produced. This is the share of costs that would be incurred anyway, such as
the cleaning materials, the factory rent and the supervisors’ salaries.
The £2 share of selling, distribution and administration overhead would probably not
be incurred if another hairdryer was produced. This includes the office costs, the
depreciation on the delivery vehicles and the rent of warehousing facilities. This sort of
cost would not increase as a result of producing another hairdryer or batch of hairdryers.
However, there may be some incremental or extra selling and distribution costs, for
example we would probably be entitled to a sales commission for all our hard work in
winning the sale, and there would be some costs involved in delivering the goods to the
hotel chain. For the sake of our analysis let us suppose that this incremental cost amounts
to £1 per hairdryer, rather than the full amount of £2 shown in the cost analysis.
You can see from the discussion in this exercise that in fact the only extra or
incremental cost to be incurred in producing another hairdryer is £23 (£22 direct cost plus
assumed £1 incremental selling and distribution costs).
FUNDAMENTALS OF MANAGEMENT ACCOUNTING
11
BASIC ASPECTS OF COST ACCOUNTING
2006.1
Therefore it may be possible to sell to the hotel chain for £25 per hairdryer, and still be
better off than if the sale was not made at all! At least the extra £2 per hairdryer (£25 – £23
extra cost) would contribute towards the costs which are being incurred anyway the
production overheads, administration overheads, etc.
This discussion has illustrated that the concept of cost needs to be qualified if it is to be
meaningful. We need to know to which cost we are referring when we state something
like, ‘The cost is £40’.
The £40 cost quoted is the full cost, which includes a fair share of all costs incurred on
behalf of the cost object. In our discussion we derived the marginal or incremental cost of
£23 which would be incurred as a direct result of making and selling another hairdryer.
Therefore we have seen that different costs are useful in different circumstances and we
must always qualify what we mean by ‘cost’. Do we mean the direct cost, the marginal
cost, the full cost or some other measure of cost?
When we consider the full cost in this example there is a profit of £10 on this particular
cost unit if it is sold for £50. This is referred to as a profit margin on sales of 20 per cent
(10=50) and a profit mark-up on full cost of 25 per cent (10=40). These are the ‘strictly
correct’ definitions of margin and mark-up. However, in practice, the two terms tend to be
used interchangeably.
The important thing in an assessment question is that you should establish whether
profit is to be calculated as a percentage of cost, or as a percentage of selling price.
1.9 Cost behaviour
Many factors affect the level of costs incurred; for instance inflation will cause costs to
increase over a period of time. In management accounting, when we talk about cost
behaviour we are referring to the way in which costs are affected by fluctuations in the
level of activity.
The level of activity can be measured in many different ways. For example, we can
record the number of units produced, miles travelled, hours worked, meals served,
percentage of capacity utilised and so on.
An understanding of cost behaviour patterns is essential for many management tasks,
particularly in the areas of planning, decision-making and control. It would be impossible
for managers to forecast and control costs without at least a basic knowledge of the way in
which costs behave in relation to the level of activity.
In this section we will look at the most common cost behaviour patterns and we will
consider some examples of each.
1.9.1 Fixed cost
The CIMA Terminology defines a fixed cost as a ‘cost incurred for an accounting
period, that, within certain output or turnover limits, tends to be unaffected by
fluctuations in the levels of activity (output or turnover)’.
Another term that can be used to refer to a fixed cost is a period cost. This highlights
the fact that a fixed cost is incurred according to the time elapsed, rather than according to
the level of activity.
STUDY MATERIAL C1
12
BASIC ASPECTS OF COST ACCOUNTING
2006.1
A fixed cost can be depicted graphically as shown in Figure 1.2.
Examples of fixed costs are rent, rates, insurance and executive salaries.
The graph shows that the cost is constant (in this case at £5,000) for all levels of activity.
However, it is important to note that this is only true for the relevant range of activity.
Consider, for example, the behaviour of the rent cost. Within the relevant range it is
possible to expand activity without needing extra premises and therefore the rent cost
remains constant. However, if activity is expanded to the critical point where further
premises are needed, then the rent cost will increase to a new, higher level.
This cost behaviour pattern can be described as a stepped fixed cost or step cost ( Figure 1.3).
The cost is constant within the relevant range for each activity level but when a critical
level of activity is reached, the total cost incurred increases to the next step.
The possibility of changes occurring in cost behaviour patterns means that it is
unreliable to predict costs for activity levels which are outside the relevant range. For
example our records might show the cost incurred at various activity levels between
100 units and 5,000 units. We should therefore try to avoid using this information as
the basis for forecasting the level of cost which would be incurred at an activity of, say,
6,000 units, which is outside the relevant range.
This warning does not only apply to fixed costs: it is never wise to attempt to
predict costs for activity levels outside the range for which cost behaviour patterns
have been established.
When you are drawing or interpreting graphs of cost behaviour patterns, it is important
that you pay great attention to the label on the vertical axis. In Figures 1.2 and 1.3 the
graphs depicted the total cost incurred. If the vertical axis had been used to represent the
fixed cost per unit, then it would look as shown in Figure 1.4.
Total fixed
cost, £
Activity leve
l
5,000
Figure 1.2 Fixed cost
Total
cost, £
Relevant
range 1
Activity leve
l
Relevant
range 2
Relevant
range 3
Figure 1.3 Stepped fixed cost
FUNDAMENTALS OF MANAGEMENT ACCOUNTING
13
BASIC ASPECTS OF COST ACCOUNTING
2006.1
The fixed cost per unit reduces as the activity level is increased. This is because the same
amount of fixed cost is being spread over an increasing number of units.
1.9.2 Variable cost
The CIMA Terminology defines a variable cost as a ‘cost that varies with a measure of
activity’.
Examples of variable costs are direct material, direct labour and variable overheads.
Figure 1.5 depicts a linear variable cost. It is a straight line through the origin, which
means that the cost is nil at zero activity level. When activity increases, the total variable
cost increases in direct proportion, that is, if activity goes up by 10 per cent, then the total
variable cost also increases by 10 per cent, as long as the activity level is still within the
relevant range.
The gradient of the line will depend on the amount of variable cost per unit.
Exercise 1.7
Figure 1.5 depicts the total variable cost at each activity level. Can you draw a sketch graph
of the variable cost per unit?
Your graph of variable cost per unit should look like Figure 1.6. The straight line parallel
to the horizontal axis depicts a constant variable cost per unit, within the relevant range.
In most assessment situations, and very often in practice, variable costs are assumed
to be linear. Although many variable costs do approximate to a linear function, this
Fixed cost
er unit, £
Activity leve
Figure 1.4 Fixed cost per unit
Total variable
cost, £
Activity leve
l
Figure 1.5 Linear variable cost
STUDY MATERIAL C1
14
BASIC ASPECTS OF COST ACCOUNTING
2006.1
assumption may not always be realistic. A variable cost may be non-linear as depicted in
either of the diagrams in Figure 1.7.
These costs are sometimes called curvilinear variable costs.
The graph of cost A becomes steeper as the activity level increases. This indicates that
each successive unit of activity is adding more to the total variable cost than the previous
unit. An example of a variable cost which follows this pattern could be the cost of direct
labour where employees are paid an accelerating bonus for achieving higher levels of
output. The graph of cost B becomes less steep as the activity level increases. Each
successive unit of activity adds less to total variable cost than the previous unit. An
example of a variable cost which follows this pattern could be the cost of direct material
where quantity discounts are available.
Exercise 1.8
Can you think of other variable costs which might follow the behaviour patterns depicted
in Figure 1.7?
The important point is that managers should be aware of any assumptions that have
been made in estimating cost behaviour patterns. They can then use the information which
is based on these assumptions with a full awareness of its possible limitations.
1.9.3 Semi-variable cost
A semi-variable cost is also referred to as a semi-fixed or mixed cost. The CIMA
Terminology defines it as a cost containing both fixed and variable components and
thus partly affected by a change in the level of activity’.
Variable cost
per unit, £
Activity leve
l
Figure 1.6 Variable cost per unit
Total
varlable
cost, £
Total
varlable
cost, £
Activity level Activity leve
l
Cost A Cost B
Figure 1.7 Non-linear variable costs
FUNDAMENTALS OF MANAGEMENT ACCOUNTING
15
BASIC ASPECTS OF COST ACCOUNTING
2006.1
A graph of a semi-variable cost might look like Figure 1.8.
Examples of semi-variable costs are gas and electricity. Both of these expenditures
consist of a fixed amount payable for the period, with a further variable amount which is
related to the consumption of gas or electricity.
Alternatively a semi-variable cost behaviour pattern might look like Figure 1.9.
This cost remains constant up to a certain level of activity and then increases as the
variable cost element is incurred. An example of such a cost might be the rental cost of a
photocopier where a fixed rental is paid and no extra charge is made for copies up to a
certain number. Once this number of copies is exceeded, a constant charge is levied for
each copy taken.
Exercise 1.9
Can you think of other examples of semi-variable costs with behaviour patterns like those
indicated in Figures 1.8 and 1.9?
1.9.4 Analysing semi-variable costs
The semi-variable cost behaviour pattern depicted in Figure 1.8 is most common in
practice and in assessment situations.
When managers have identified a semi-variable cost they will need to know how much
of it is fixed and how much is variable. Only when they have determined this will they be
able to estimate the cost to be incurred at relevant activity levels. Past records of costs and
their associated activity levels are usually used to carry out the analysis. Your Fundamentals
Total cost, £
Activity level
Fixed cost
Variable cos
t
Figure 1.8 Semi-variable cost
Total cost, £
Variable cos
t
Fixed cost
Activity level
Figure 1.9 Semi-variable cost
STUDY MATERIAL C1
16
BASIC ASPECTS OF COST ACCOUNTING
2006.1
of Management Accounting syllabus requires you to know how to use two common methods
of separating the fixed and variable elements:
(a) The high–low method.
(b) The ‘line of best fit’ method.
The high–low method
This method picks out the highest and lowest activity levels from the available data and
investigates the change in cost which has occurred between them. The highest and lowest
points are selected to try to use the greatest possible range of data. This improves the
accuracy of the result.
Example: the high–low method
A company has recorded the following data for a semi-variable cost:
Activity level Cost incurred
Month (units) (£)
January 1,800 36,600
February 2,450 41,150
March 2,100 38,700
April 2,000 38,000
May 1,750 36,250
June 1,950 37,650
The highest activity level occurred in February and the lowest in May. Since the amount of fixed cost incurred in
each month is constant, the extra cost resulting from the activity increase must be the variable cost.
Activity level
(units) £
February 2,450 41,150
May
1,750 36,250
Increase
700 4,900
The extra variable cost for 700 units is £4,900. We can now calculate the variable cost per unit:
Variable cost ¼
£4;900
700
¼ £7 per unit
Substituting back in the data for February, we can determine the amount of fixed cost:
February £
Total cost 41,150
Variable cost (2,450 units £7)
17,150
Therefore, fixed cost per month
24,000
Now that the fixed and variable cost elements have been identified, it is possible to estimate the total
cost for any activity level within the range 1,750 units to 2,450 units.
The scattergraph method
This method takes account of all available historical data and it is simple to use.
However, it is very prone to inaccuracies that arise due to subjectivity and the likelihood
of human error.
FUNDAMENTALS OF MANAGEMENT ACCOUNTING
17
BASIC ASPECTS OF COST ACCOUNTING
2006.1
1. First a scattergraph is drawn which plots all available pairs of data on a graph.
2. Then a line of best fit is drawn by eye. This is the line which, in the judgement of the
user, appears to be the best representation of the gradient of the sets of points on the
graph. This is demonstrated in Figure 1.10.
The inaccuracies involved in drawing the line of best fit should be obvious to you.
If you had been presented with this set of data, your own line of best fit might have
been slightly different from ours.
3. The point where the extrapolation of this line cuts the vertical axis (the intercept) is
then read off as the total fixed cost element. The variable cost per unit is given by the
gradient of the line.
From Figure 1.10, the fixed cost contained within this set of data is adjudged to
be £200.
The variable cost is calculated as follows:
Cost for zero units ¼ £200
Cost for 150 units ¼ £500
Gradient (i.e. variable cost) ¼
500 200
150 0
¼ £2 per unit
1.9.5 Using historical data
The main problem which arises in the determination of cost behaviour is that the
estimates are usually based on data collected in the past. Events in the past may not be
representative of the future and managers should be aware of this if they are using the
information for planning and decision-making purposes.
1.9.6 The importance of time scale in analysing cost
behaviour
It is important to think about the time period under consideration when we are analysing
cost behaviour patterns. For example, over a long period of time all costs might be
considered to be variable.
Total cost, £
Activity leve
l
16012080400
0
100
200
300
400
500
Figure 1.10 Scattergraph
STUDY MATERIAL C1
18
2006.1
BASIC ASPECTS OF COST ACCOUNTING
Over a number of years, if activity reduces an organisation can move to smaller
premises to reduce rent costs and they can reduce the number of supervisors to reduce
supervisor salary cost. Thus costs which we might normally classify as fixed costs are, in
the longer term, becoming more variable in relation to the level of activity.
However in the shorter term costs such as rent and supervisors’ salaries are fixed. If
demand for a product reduces, the expenditure on rent and on supervisors’ salaries cannot
be reduced immediately in response to the reduction in output. Such decisions require
planning and consideration of factors such as whether the reduction in output is
temporary or actions that might be taken to increase output again.
Similarly, over a number of years if activity increases then rent costs and supervisor
salary costs will increase in response to the change in activity, again demonstrating more
variable behaviour patterns in the longer term.
However the rent and salary cost is not likely to increase in the longer term in a linear
fashion in the way that we have depicted linear variable costs earlier in this chapter. In fact
the behaviour of such costs over a longer period of time is likely to follow the pattern of
the stepped fixed cost depicted in Figure 1.3.
Think also about a cost that we would normally classify as variable, such as direct
labour cost. In the very short term, for example one day, this cost could be regarded
as a fixed cost. If for some reason, perhaps a machine breakdown, we do not produce
any output on a particular day it is unlikely that at short notice we can send home all
the work force and not pay them. Thus the direct labour cost is a fixed cost in the
very short term.
In an assessment you should assume that the time period under consideration is
neither very long nor very short, unless you are given clear instructions to the
contrary.
1.10 Summary
Having read this chapter the main points that you should understand are as follows.
1. Organisations need costing systems that will provide the basic information that
management requires for planning, control and decision-making.
2. A cost unit is the basic unit of measurement selected for cost control purposes.
3. A cost centre is used as a ‘collecting place’ for costs, which may then be further
analysed and related to individual cost units.
4. A cost object is anything for which costs can be ascertained. Examples are a product,
a service, a centre, an activity, a customer and a distribution channel.
5. Costs may be classified in a number of different ways depending on the reason for the
classification exercise. The main classifications are according to their nature (material,
labour, expenses), according to their purpose (direct or indirect) or according to
responsibility.
6. The concept of cost needs to be qualified as direct, full, marginal, etc. in order to be
meaningful.
7. Costs which are not affected by changes in the level of activity are fixed costs or
period costs.
8. A stepped fixed cost is constant within the relevant range for each activity level.
BASIC ASPECTS OF COST ACCOUNTING
FUNDAMENTALS OF MANAGEMENT ACCOUNTING
19
2006.1
9. A variable cost increases or decreases in line with changes in the level of activity.
10. A cost which is partly fixed and partly variable is a semi-variable, semi-fixed or mixed
cost.
11. Observed cost behaviour patterns apply only over the relevant range of activity levels.
12. The fixed and variable elements of a semi-variable cost can be determined using the
high–low method or a scattergraph.
13. It is important to consider the time scale when analysing cost behaviour. In the longer
term, fixed costs tend to become step fixed costs and in the very short term all costs
are fixed.
BASIC ASPECTS OF COST ACCOUNTING
STUDY MATERIAL C1
20
2006.1
Revision Questions
Question 1 Multiple choice
In the multiple choice questions in the actual assessment each option would usually have
an empty box or circle beside it. You would be required to simply place the cursor on the
relevant box and click the mouse to select the correct answer. In this Learning System we
have labelled the four options as A, B, C and D. These letters are for reference purposes
only and to assist us in our discussion of the solutions.
You are advised to try the online demo of cba questions on CIMA’s website at
www.cimaglobal.com=cba so that you will be aware of the way in which the questions will
be presented.
1.1 Cost centres are:
(A) units of output or service for which costs are ascertained.
(B) functions or locations for which costs are ascertained.
(C) a segment of the organisation for which budgets are prepared.
(D) amounts of expenditure attributable to various activities.
1.2 Prime cost is:
(A) all costs incurred in manufacturing a product.
(B) the total of direct costs.
(C) the material cost of a product.
(D) the cost of operating a department.
1.3 Fixed costs are conventionally deemed to be:
(A) constant per unit of output.
(B) constant in total when production volume changes.
(C) outside the control of management.
(D) those unaffected by inflation.
1.4 The following data relate to two activity levels of an out-patient department in a hospital:
Fixed overheads are not affected by the number of consultations per period.
The variable cost per consultation:
(A) is approximately £15.50
(B) is approximately £44.44
1
Number of consultations by patients 4,500 5,750
Overheads £269,750 £289,125
21 2006.1
(C) is approximately £59.94
(D) cannot be calculated without more information.
1.5 P Ltd is preparing the production budget for the next period. Based on previous experience,
it has found that there is a linear relationship between production volume and production
costs. The following cost information has been collected in connection with production:
What would be the production cost for a production volume of 2,700 units?
(A) £5,400
(B) £25,400
(C) £27,000
(D) £39,150
1.6 The following is a graph of cost against volume of output:
To which of the following costs does the graph correspond?
(A) Electricity bills made up of a standing charge and a variable charge.
(B) Bonus payments to employees when production reaches a certain level.
(C) Sales commission payable per unit up to a maximum amount of commission.
(D) Bulk discounts on purchases, the discount being given on all units purchased.
The following information relates to questions 1.7–1.11
Which one of the above graphs illustrates the costs described in questions 1.7–1.11?
Volume Cost
(units) (£)
1,600 23,200
2,500 25,000
Total
cost
Volume of outpu
t
£
£
£
£
£
£
Level of activity
Level of activity
Level of activity
Level of activity Level of activit
y
Level of activit
y
Graph 1
Graph 2
Graph 3
Graph 6
Graph 5Graph 4
REVISION QUESTIONS C1
22
BASIC ASPECTS OF COST ACCOUNTING
2006.1
1.7 A linear variable cost when the vertical axis represents cost incurred.
(A) Graph 1
(B) Graph 2
(C) Graph 4
(D) Graph 5
1.8 A fixed cost when the vertical axis represents cost incurred.
(A) Graph 1
(B) Graph 2
(C) Graph 3
(D) Graph 6
1.9 A linear variable cost when the vertical axis represents cost per unit.
(A) Graph 1
(B) Graph 2
(C) Graph 3
(D) Graph 6
1.10 A semi-variable cost when the vertical axis represents cost incurred.
(A) Graph 1
(B) Graph 2
(C) Graph 4
(D) Graph 5
1.11 A step fixed cost when the vertical axis represents cost incurred.
(A) Graph 3
(B) Graph 4
(C) Graph 5
(D) Graph 6
1.12 Over long time periods of several years, factory rent costs will tend to behave as:
(A) linear variable costs
(B) fixed costs
(C) step fixed costs
(D) curvilinear variable costs
Question 2 Short objective-test questions
2.1 Which of the following are stepped fixed costs?
& Machine rental costs
& Direct material costs
& Royalties payable on units produced
& Depreciation on delivery vehicles
FUNDAMENTALS OF MANAGEMENT ACCOUNTING
23
BASIC ASPECTS OF COST ACCOUNTING
2006.1
2.2 A company increases its activity within the relevant range. Tick the correct boxes
below to indicate the effect on costs.
Total variable costs will: increase &
decrease &
remain the same &
Total fixed cost will: increase &
decrease &
remain the same &
The variable cost per unit will: increase &
decrease &
remain the same &
The fixed cost per unit will: increase &
decrease &
remain the same &
2.3 The variable production cost per unit of product B is £2 and the fixed production
overhead for a period is £4,000. The total production cost of producing 3,000 units
of B in a period is £
.
2.4 In a hotel, which of the following would be suitable cost units and cost centres?
2.5
Suitable as cost centre Suitable as cost unit
Restaurant &&
Guest night &&
Meal served &&
Fitness suite &&
Bar &&
500
400
£
300
200
100
0
0 100 200
Level of activity, unit
s
REVISION QUESTIONS C1
24
BASIC ASPECTS OF COST ACCOUNTING
2006.1
Based on the above scattergraph:
the period fixed cost is £
the variable cost per unit is £
2.6 The following data relates to the overhead costs of a commercial laundry for the
latest two periods.
Overhead costs Number of items laundered
£
5,140 2,950
5,034 2,420
A formula that could be used to estimate the overhead costs for a forthcoming
period is:
Overhead cost ¼£
+(£ number of items laundered)
2.7 Spotless Limited is an office cleaning business which employs a team of part-time
cleaners who are paid an hourly wage. The business provides cleaning services for
a number of clients, ranging from small offices attached to high-street shops to large
open-plan offices in high-rise buildings.
In determining the cost of providing a cleaning service to a particular client,
which of the following costs would be a direct cost of cleaning that client’s office and
which would be an indirect cost?
Direct cost Indirect cost
(a) The wages paid to the cleaner who
is sent to the client’s premises
&&
(b) The cost of carpet shampoo used
by the cleaner
&&
(c) The salaries of Spotless Ltd’s
accounts clerks
&&
(d) Rent of the premises where
Spotless Ltd stores its cleaning
materials and equipment
&&
(e) Travelling expenses paid to the
cleaner to reach the client’s
premises
&&
(f ) Advertising expenses incurred in
attracting more clients to Spotless
Ltd’s business
&&
FUNDAMENTALS OF MANAGEMENT ACCOUNTING
25
BASIC ASPECTS OF COST ACCOUNTING
2006.1
Question 3 Cost classification
A company manufactures and retails clothing.
When determining the cost of units produced, you are required to write the correct
classification for each of the costs below into the box provided, using the following
classifications (each cost is intended to belong to only one classification):
(i) direct materials
(ii) direct labour
(iii) direct expenses
(iv) indirect production overhead
(v) research and development costs
(vi) selling and distribution costs
(vii) administration costs
(viii) finance costs
1. lubricant for sewing machines
2. floppy disks for general office computer
3. maintenance contract for general office photocopying machine
4. telephone rental plus metered calls
5. interest on bank overdraft
6. Performing Rights Society charge for music broadcast throughout
the factory
7. market research undertaken prior to a new product launch
8. wages of security guards for factory
9. cost of denim fabric purchased
10. royalty payable on number of units of product XY produced
11. road fund licences for delivery vehicles
12. postage cost of parcels sent to customers
13. cost of advertising products on television
14. audit fees
15. chief accountant’s salary
16. wages of operatives in the cutting department
17. cost of painting advertising slogans on delivery vans
18. wages of storekeepers in materials store
19. wages of fork lift truck drivers who handle raw materials
20. cost of developing a new product in the laboratory
2006.1
REVISION QUESTIONS C1
26
BASIC ASPECTS OF COST ACCOUNTING
Solutions to
Revision Questions
Solution 1
The best way to approach multiple-choice questions is to work out your own answer
first, before you look at the options. If your answer is not included in the options
then you may be forced to guess. Improve your chances by eliminating the unlikely
answers, or those that you know to be incorrect. Then take a guess from the remaining
choices.
Make sure that you answer every question. You will not be penalised for an incorrect
answer and you might guess correctly!
1.1 Answer: (B)
Cost centres act as ‘collecting places’ for costs before they are analysed further.
1.2 Answer: (B)
Answer (A) describes total production cost. Answer (C) is only a part of prime cost.
Answer (D) is an overhead cost.
1.3 Answer: (B)
The total amount of fixed costs remains unchanged when production volume
changes, therefore the unit rate fluctuates.
1.4 Answer: (A)
With the same amount of fixed overheads at both activity levels, the change in
overheads must be due to extra variable cost.
Variable overhead cost per consultation ¼
£19,375
1,250
¼ £15:50
1
Overheads Consultations
£
High 289,125 5,750
Low
269,750 4,500
Change
19,375 1,250
27 2006.1
1.5 Answer: ( B)
Variable cost per unit ¼
£1,800
900
¼ £2
Substitute in high activity:
Forecast for 2,700 units:
1.6 Answer: (B)
The graph shows a variable cost which starts to be incurred only beyond a certain
volume of output. Only B fits this description of cost behaviour.
1.7 Answer: (B)
Graph 2 depicts a cost which increases in total by equal amounts for each increment
in the level of activity.
1.8 Answer: (A)
Graph 1 depicts a cost which remains the same regardless of the level of activity.
1.9 Answer: (A)
The variable cost per unit remains constant regardless of the level of activity.
1.10 Answer: (C)
Graph 4 depicts a cost which contains a fixed element which is incurred even at zero
activity. Thereafter the cost increases in total by equal amounts for each increment in
the level of activity: this is the extra variable cost incurred.
1.11 Answer: (A)
Graph 3 depicts a cost which remains constant up to a critical level of activity. At
that point the total cost increases by a step to a new, higher level.
Units £
2,500 25,000
1,600 23,200
900 1,800
£
Total cost 25,000
Variable cost ¼2,500 units £2
5,000
Therefore fixed cost
20,000
£
Fixed cost 20,000
Variable cost 2,700 £2
5,400
Total cost
25,400
SOLUTIONS TO REVISION QUESTIONS C1
28
BASIC ASPECTS OF COST ACCOUNTING
2006.1
1.12 Answer: (C)
As activity increases or decreases over a period of several years the rent cost will
remain constant for a range of activity but will then increase or decrease in steps as
critical activity levels are reached when larger or smaller premises are needed.
Solution 2
Always read the question carefully. For example, question 2.1 does not state ‘which one of
the following are stepped fixed costs?’. In fact, there is more than one correct answer.
2.1 Machine rental cost is a stepped fixed cost. For one machine the total rental cost
stays constant until the machine is working at full capacity. Then two machines will
be needed and the rental cost goes up a step to a new higher level. When the two
machines are at full capacity there will be a need to rent three machines, and so on.
Depreciation on delivery vehicles is a stepped fixed cost. Depreciation is
calculated on an annual basis and is unlikely to be affected by the level of activity in
the short term. For one vehicle the annual depreciation is a constant amount. If two
vehicles are required the depreciation cost goes up a step, and so on.
Royalty costs and direct material costs are variable costs.
2.2 As activity increases within the relevant range, the total variable costs will increase and
the total fixed cost will remain the same. The variable cost per unit will remain the same
and the fixed cost per unit will decrease.
2.3 Total production cost ¼(3,000 £2) + £4,000 ¼£10,000.
2.4
2.5 The period fixed cost is £200. The variable cost per unit is:
£500 £200
200 units
¼ £1:50 per unit
2.6
Variable cost per item laundered ¼£106=530 ¼£0.20
Substitute in high activity:
£
Total cost 5,140
Variable cost ¼2,950 items £0.20
590
Therefore fixed cost
4,550
Suitable as cost centre Suitable as cost unit
Restaurant 4
Guest night 4
Meal served 4
Fitness suite 4
Bar 4
Overhead costs Number of
items laundered
£
5,140 2,950
5,034 2,420
106 530
FUNDAMENTALS OF MANAGEMENT ACCOUNTING
29
BASIC ASPECTS OF COST ACCOUNTING
2006.1
A formula that could be used to estimate the overhead costs for a forthcoming
period is:
Overhead cost ¼£4,550 + ( £0.20 number of items laundered)
2.7
The direct costs are (a), (b) and (e) because they can be directly identified with the cost
object under consideration (this particular client). The other costs are indirect because they
would have to be shared among all of the clients serviced by Spotless Limited.
Solution 3
When you are trying to determine whether a cost is direct or indirect in relation to a cost
object, think about whether the cost would need to be shared over several cost objects
or whether it can be attributed directly to a particular cost object. A cost that needs to be
shared must be an indirect cost.
Item Classification Item Classification
1 (iv) 11 (vi)
2 (vii) 12 (vi)
3 (vii) 13 (vi)
4 (vii) 14 (vii)
5 (viii) 15 (vii)
6 (iv) 16 (ii)
7 (vi) 17 (vi)
8 (iv) 18 (iv)
9 (i) 19 (iv)
10 (iii) 20 (v)
Direct cost Indirect cost
(a) The wages paid to the cleaner who
is sent to the client’s premises
4
(b) The cost of carpet shampoo used
by the cleaner
4
(c) The salaries of Spotless Ltd’s
accounts clerks
4
(d) Rent of the premises where
Spotless Ltd stores its cleaning
materials and equipment
4
(e) Travelling expenses paid to the
cleaner to reach the client’s
premises
4
(f ) Advertising expenses incurred in
attracting more clients to Spotless
Ltd’s business
4
BASIC ASPECTS OF COST ACCOUNTING
SOLUTIONS TO REVISION QUESTIONS C1
30
2006.1
2
Accounting for the
Value of Inventories
Accounting for the
Value of Inventories
2.1 Introduction
In this chapter you will learn about the different methods that can be used to value inventory
(stock) and the impact of each of these on the profit reported for the period.
The valuation of inventory, although a cost accounting function, is also required for
financial accounting and you should be aware that the regulations concerning the valuation
of inventory which apply to financial accounting may be a significant influence in
determining the valuation method used.
All of the inventory valuation methods described in this chapter can be applied to finished
goods items ready for sale as well as to raw materials held in stores for use in production.
2.2 Valuing inventory at cost
The general principle is that inventory should be valued at cost. However it can sometimes
be difficult to determine which cost should be used. Have a look at the following example.
Example: Which cost should be used?
The following example illustrates the problem of determining the cost of the inventory
held at a particular time. During September the following items were purchased and issued
from stores.
2
LEARNING OUTCOMES
After completing this chapter, you should be able to:
"
distinguish between the historical cost of an asset and the economic value of an asset
to an organisation;
"
apply first-in-first-out (FIFO), last-in-first-out (LIFO) and average cost (AVCO)
methods of accounting for stock, calculating stock values and related gross profit;
"
explain why first-in-first-out (FIFO) is essentially a historical cost method, while last-in-
first-out (LIFO) approximates economic cost.
33 2006.1
September 1 Opening balance Nil
Bought 100 units @ £5.00 each
2 Issued 50 units
10 Bought 50 units @ £5.50 each
20 Issued 60 units
27 Bought 100 units @ £5.60 each
It is easy to calculate the quantity of items remaining in the stores on 27 September
by comparing the total quantity purchased (250) with the total number issued from
stores (110). The closing inventory quantity is therefore 140 units (250 110). However
what is the cost of the items in inventory?
Unless each item is individually marked with the price at which it was bought and the
balance is identified by individual items at individual prices, it is difficult to know what
value should be placed on the inventory items.
This method of individual pricing does exist (it is known as the specific price method), but
because of the cost of operating such a system it is unsuitable for all but very expensive
items where the inventory quantities and rates of usage are low. Alternative methods are
used instead, each of which will now be explained using the data from the above example.
2.3 First in, first out (FIFO)
This method assumes for valuation purposes that the items received earliest are those
which are issued first. This does not necessarily mean that these are the items which have
physically been issued first.
A stores ledger record is maintained for each inventory item. This record shows the
volume and value of the receipts and issues made during the period, and the remaining
balance in stores after each receipt or issue.
The stores ledger record of the transactions using this method would appear as follows:
Stores ledger record
Receipts Issues Balance
Date Qty Price £ Qty Price £ Qty Price £
September
1 Nil Nil
1 100 5.00 500
100 5.00 500
2 50 5.00 250
50 5.00 250
10 50 5.50 275 50 5.00 250
50 5.50 275
100 525
20 50 5.00 250
10 5.50 55
60 305 40 5.50 220
27 100 5.60 560 40 5.50 220
100 5.60 560
140 780
STUDY MATERIAL C1
34
ACCOUNTING FOR THE VALUE OF INVENTORIES
2006.1
Note that the issue made on 20 September is valued as 50 units at £5 each plus ten units
at £5.50 each. This is because the earliest price paid for any of the remaining inventory
held at the time of issue was £5.
You should note the clarity of the entries, particularly those relating to the closing
balances at the end of each day which are stated in chronological order.
You must practice producing neat workings for your stores ledger record. Speed and
accuracy are essential in the assessment and although you will not be awarded marks
for your workings they will help to ensure that you arrive at the correct answer.
2.4 Last in, first out (LIFO)
This method assumes for valuation purposes that the latest price paid for items received is
the one to be used to price issues. Using this method the stores ledger record of the same
transactions would appear as follows:
Stores ledger record
Receipts Issues Balance
Date Qty Price £ Qty Price £ Qty Price £
September
1 Nil Nil
1 100 5.00 500
100 5.00 500
2 50 5.00 250
50 5.00 250
10 50 5.50 275 50 5.00 250
50 5.50 275
100 525
20 50 5.50 275
10 5.00 50
60 325 40 5.00 200
27 100 5.60 560 40 5.00 200
100 5.60 560
140 760
Using this valuation method, the first 50 items issued on 20 September are valued at
£5.50 per unit. This is because the latest price paid at that date was the price paid for the
delivery received on 10 September, which was £5.50 per unit. The remaining ten units are
valued at the price of £5 paid for the delivery received on 1 September.
2.5 Cumulative weighted average (AVCO)
This method calculates a weighted average price each time there is a receipt, using the formula:
Weighted average price ¼
( Value of inventory b/f þvalue of purchases)
(Quantity of inventory b/f þ quantity purchased)
FUNDAMENTALS OF MANAGEMENT ACCOUNTING
35
ACCOUNTING FOR THE VALUE OF INVENTORIES
2006.1
This average value per unit is then used for all issues until another delivery is received
when the average is recalculated. The stores ledger record of the same transactions using
this method is as follows:
Stores ledger record
Receipts Issues Balance
Date Qty Price £ Qty Price £ Qty Price £
September
1 Nil Nil
1 100 5.00 500 100 5.00 500
2 50 5.00 250 50 5.00 250
10 50 5.50 275 100 5.25 525
20 60 5.25 315 40 5.25 210
27 100 5.60 560 140 5.50 770
Note that the price per unit on 10 September is calculated by the formula shown above:
£ð250 þ275Þ
50 þ50
¼ £5:25 per unit
The average value per unit of £5.50 calculated on 27 September (£ 770=140) would be
used to value all issues after this date until another receipt occurs when the average would
be recalculated.
2.6 Comparison of FIFO, LIFO and AVCO
The following table shows the closing inventory valuations and values of issues using each
of the three methods. In each case the value of the issues is obtained by totalling the
individual issue valuations.
Closing inventory valuation Value of issues
££
FIFO 780 555
LIFO 760 575
AVCO 770 565
Points to note about the different inventory valuation methods include the following.
The values for AVCO in the table lie between those for LIFO and FIFO. This should
always occur because AVCO is an averaging method.
Both LIFO and FIFO require records to be kept of each batch of purchases so that the
appropriate price may be attached to each issue.
Price fluctuations are smoothed out with the AVCO method which makes the data easier to
use for decision making, although the rounding of the unit value might cause some difficulties.
Many management accountants would argue that LIFO provides more relevant
information for decision making because it uses the most up-to-date price.
However LIFO may sometimes confuse managers, since the pricing method represents
the opposite to what is happening in reality, that is, the items in store will probably be
physically issued on a FIFO basis.
STUDY MATERIAL C1
36
ACCOUNTING FOR THE VALUE OF INVENTORIES
2006.1
The overriding consideration for the internal cost accounting system is that the
information should be useful for management purposes.
Let us look in more detail at the assertion that LIFO provides more relevant information for
decision making. If we assume that the items in the above example are items for resale, then using
the FIFO method the cost of the items issued on 20 September was £305. If a customer offered
you £315 for them you might well accept the offer on the basis that you had made £10 profit. If
the LIFO method is used the offer would be rejected because the cost of the issue is stated to be
£325 and thus to accept the customer’s offer would be to make a loss. Which is correct?
It is reasonable to believe that in order to make a profit you should be able to replace the
items that you have sold and still have some of the sale proceeds left over. In this example the
latest price paid on 10 September was £5.50 per unit and with the benefit of hindsight we
know that the price on 27 September is £5.60 per unit. It is reasonable therefore to expect that
the cost of replacing the items sold will be at least £5.50 per unit, which totals £330.
Thus, it can be seen that the use of the FIFO method would lead you to a decision
which would cause you to be unable to replace the items sold with the sale proceeds
received. The use of the LIFO method is thus argued to be better for decision-making.
2.6.1 Historical cost compared with economic cost
and economic value
Although we have seen that the LIFO method results in issues from stores being valued at
the most recent prices paid, the costs used are still historical costs. By their nature,
historical costs are out of date and might not reflect the current value of an item.
The economic value of an asset depends on current circumstances and we have just
seen that it can even be misleading to rely on LIFO historical costs when assessing the
worth of an item issued from stores. The issue cost was out of date and the company
might have had difficulty replacing the stock item if did not make a sufficiently high charge
against profit for the cost of replacement.
If the inflation rate is very high or if purchases of stock are made only infrequently then
even the charges to cost of sales resulting from the LIFO method do not provide a good
approximation of economic cost.
However in many circumstances the LIFO method will use up-to-date values, whereas
the FIFO method is essentially an historical cost method.
The economic cost of an asset such as an item of material might be measured in terms of the
benefit forgone by not using the asset in the ‘next best way; this is its opportunity cost.
For example, an item of material might be obsolete. The ‘next best’ alternative for this
item would be to sell it and the value of this option is its net realisable value. The
application of this concept can be seen in the financial accounting rule that inventory
(stock) items should be valued at the lower of cost and net realisable value.
Thus an asset’s economic cost or economic value might be higher or lower than its
historical cost, depending on current circumstances.
2.7 Inventory valuation and the effect on gross profit
An important point to realise is that, since each of the inventory valuation methods
produces a different valuation, the profit reported under each of the methods will be
different. An example will help you to see the difference.
FUNDAMENTALS OF MANAGEMENT ACCOUNTING
37
ACCOUNTING FOR THE VALUE OF INVENTORIES
2006.1
Example
Continuing with the example in Section 2.2, suppose that the units issued from inventory are sold direct
to the customer for £8 per unit. The gross profit recorded under each of the inventory valuation methods
would be as follows.
FIFO LIFO AVCO
£££
Sales revenue: 110 £8
880 880 880
Purchases 1,335 1,335 1,335
Less closing inventory
780 760 770
Cost of goods sold
555 575 565
Gross profit
325 305 315
The prices of receipts are rising during the month. Therefore the FIFO method, which prices
issues at the older, lower prices, results in the highest value of closing inventory and the
highest profit figure. The AVCO method produces results that lie between those for FIFO
and LIFO.
2.8 Periodic weighted average
This method is similar to the cumulative weighted average method described earlier, except
that instead of calculating a new average price every time a receipt occurs an average is
calculated based on the total purchases for the period. This average is then applied to all
issues in the period.
Periodic weighted average price
¼
cost of opening inventory þ total cost of receipts in period
units in opening inventory þ total units received in period
Using the same data, the periodic weighted average cost per unit is:
ð£500 þ£275 þ£560Þ
ð100 þ50 þ 100Þ
¼
£1,335
250
¼ £5:34 per unit
Every issue is priced at £5.34 per unit. The total value of all issues would be 110 units
@ £5.34 ¼£587.40, leaving a closing inventory of 140 units valued at £747.60.
In an assessment you must always use the cumulative weighted average pricing method
unless you are specifically instructed to use the periodic weighted average method.
STUDY MATERIAL C1
38
ACCOUNTING FOR THE VALUE OF INVENTORIES
2006.1
2.9 Materials documentation
2.9.1 Perpetual inventory system
We have seen how a stores ledger record for a particular inventory item shows each receipt
and issue from stores as it occurs. This provides a continuous record of the balance of
each inventory item and is known as a perpetual inventory system.
In practice there are likely to be discrepancies between the actual physical balance in
stores and the balance shown on the stores ledger record. For this reason the inventory in
stores should be counted on a regular basis and checked against the total shown on the
stores ledger record. This ensures that minor discrepancies are corrected as soon as
possible and that the stores ledger record provides a reasonably accurate record of
available inventory.
2.9.2 Recording the receipt of goods
When goods are delivered by a supplier they will normally be accompanied by a delivery
note. It is common for all orders to be delivered to stores (unless there is good reason to
have them delivered elsewhere) and for the storekeeper to be responsible for checking the
delivery and acknowledging its receipt by signing the supplier’s delivery note.
The storekeeper will then raise a goods received note (GRN). This is the document
which is used to record the receipt of goods for the purpose of updating the stores ledger
record. The information recorded on the GRN will include the quantity, code number and
description of the material received, as well as the date of the delivery and the supplier’s
details.
A copy of the GRN will be sent to the purchasing department (so that they know that
the goods ordered have actually been delivered) and to the accounts department (so that
when the time comes to pay the supplier’s invoice the accounts department knows that the
goods have been received).
2.9.3 Recording the movement of inventory items
Whether material is issued to production or issued for indirect purposes, it may only be
issued from stores against a properly authorised material requisition. This document is
used to record the issue of material on the stores ledger record.
The information shown on the material requisition will include the quantity, code
number and description of the items issued, as well as the date of the issue, the cost of the
items issued (based on whatever inventory valuation method is in use: FIFO, LIFO, etc.)
and the cost centre or the job number to be charged with the cost of the items issued.
Sometimes, materials may be issued and subsequently found to be surplus to
requirements. These will be returned to the stores and this movement of materials must be
recorded on a material returned note. This ensures that the recorded inventory quantity
is increased to its correct level and that the recorded cost value of the materials issued to
production is reduced.
FUNDAMENTALS OF MANAGEMENT ACCOUNTING
39
ACCOUNTING FOR THE VALUE OF INVENTORIES
2006.1
This transaction can be thought of as the opposite of an issue to production and the
information contained on a material returned note is similar to that on a material
requisition note. The returned note will indicate which cost centre or job is to be credited
with the cost of the materials returned to stores.
Occasionally, a cost centre might transfer material to another cost centre, without the
material first being sent back to the stores. To ensure that the correct cost centre or job is
charged with the cost of the materials a material transfer note is raised. This note details
the material being transferred, giving the same information as a material requisi-
tion=material returned note, that is description, quantity, cost and so on. The transfer note
also shows which cost centre or job is to be credited with the cost of the material and
which is to be debited. The stores ledger record is not altered because the items transferred
do not physically return to the stores.
2.10 Summary
Having read this chapter the main points that you should understand are as follows.
1. The stores ledger record for each inventory item shows the quantity, cost and total value of
each issue and receipt and of the balance in stores after each movement of inventory.
2. The pricing of issues from inventory has a direct effect on cost of sales, inventory
valuations and reported profits.
3. The FIFO method prices issues at the price of the oldest items remaining in stores. The
LIFO method prices issues at the price of the items in stores that were received most
recently.
4. The weighted average method uses an average price that can be calculated using two
different bases: the cumulative weighted average and the periodic weighted average.
The cumulative weighted average calculates a new average price every time a new batch
is received into stores. The periodic weighted average calculates an average price at the
end of each period. Unless you receive instructions to the contrary you should use the
cumulative weighted average method in an assessment.
5. The inventory valuation methods can be applied to finished goods for sale as well as to
raw materials held in stores for use in production.
6. A number of documents are used to record the receipt and movement of inventory
items. These include a goods received note, material requisition, material returned note
and a material transfer note.
ACCOUNTING FOR THE VALUE OF INVENTORIES
STUDY MATERIAL C1
40
Revision Questions
Question 1 Multiple choice
1.1 P Ltd had an opening inventory value of £2,640 (300 units valued at £8.80 each) on
1 April. The following receipts and issues were recorded during April:
10 April Receipt 1,000 units £8.60 per unit
23 April Receipt 600 units £9.00 per unit
29 April Issues 1,700 units
Using the LIFO method, what was the total value of the issues on 29 April?
(A) £14,840
(B) £14,880
(C) £14,888
(D) £15,300
1.2 A firm has a high level of inventory turnover and uses the FIFO issue pricing system.
In a period of rising purchase prices, the closing inventory valuation is:
(A) close to current purchase prices.
(B) based on the prices of the first items received.
(C) much lower than current purchase prices.
(D) the average of all goods purchased in the period.
1.3 Using the FIFO system for pricing issues from stores means that, when prices are
rising:
(A) product costs are overstated and profits understated.
(B) product costs are kept in line with price changes.
(C) product costs are understated and profits understated.
(D) product costs are understated and profits overstated.
1.4 During a period of rising prices, which one of the following statements is correct?
(A) The LIFO method will produce lower profits than the FIFO method, and lower
closing inventory values.
(B) The LIFO method will produce lower profits than the FIFO method, and
higher closing inventory values.
(C) The FIFO method will produce lower profits than the LIFO method, and lower
closing inventory values.
(D) The FIFO method will produce lower profits than the LIFO method, and
higher closing inventory values.
2
41 2006.1
Question 2 Short objective-test questions
2.1 Opening inventory for a particular component at the beginning of April was zero.
The following receipts and issues were recorded during April.
April 1 Received 100 components at a price of £6.00 each
April 2 Issued 30 components
April 8 Received 30 components at a price of £6.50 each
April 10 Issued 60 components
April 15 Received 100 components at a price of £6.50 each
April 16 Issued 40 components
The weighted average pricing method is used.
Complete the following boxes.
The total value of the components issued on April 10 was £
.
The cost per component issued on April 16 was £
.
2.2 In a period of rising prices, which of the following statements are true? Tick the box
for any statement that is true.
& (a) Reported profits will be higher with FIFO than with LIFO.
& (b) The value of closing inventory will be higher with FIFO than with LIFO.
& (c) LIFO would be the preferable method for financial accounting purposes
because it uses the oldest price.
2.3 BB imports product U and sells the product to retail customers at a price of £14 per
case. BB had no inventory at the beginning of February and during February the
following receipts and sales were recorded.
6 February Received 1,400 cases @ £8.20 per case
15 February Received 900 cases @ £9.10 per case
20 February Sold 780 cases
22 February Received 330 cases @ £9.90 per case
28 February Sold 860 cases
(a) Using the FIFO method of inventory valuation, the gross profit reported for February
would be £
.
(b) Using the LIFO method of inventory valuation, the gross profit reported for February
would be £
.
2.4 Which of the following documents will be used to record the receipt of materials
from stores by a production cost centre, ready for use in production?
& Goods received note
& Material requisition
Question 3 Longer revision question: inventory valuation
Three students, K, L and M, are equal partners in a joint venture which involves
them, on a part-time basis, in buying and selling sacks of product F. The
transactions for the three months ended 30 June were as stated below. You are to
REVISION QUESTIONS C1
42
ACCOUNTING FOR THE VALUE OF INVENTORIES
2006.1
assume that purchases at unit costs given were made at the beginning of each
month and that the sales were made at the end of each month at the fixed price of
£1.50 per sack.
Purchases Sales
Month Sacks Unit cost
£
Sacks
April 1,000 1.00 500
May 500 1.20 750
June 1,000 1.00 200
In July, the student partners held a meeting to review their financial position and share
out the profits but there was disagreement because each partner had priced the issues on a
different basis. K had used FIFO, L had used LIFO and M had used a weighted average,
basing his weighted average on the whole of the three months’ purchases.
Shown below is an extract from K and L’s stores ledger records.
Receipts Sales Balance
Month Qty Price £ Qty Price £ Qty Price £
April 1,000 1.00 1,000 500 500
May 500 1.20 600 750 A 250 B
June 1,000 1.00 1,000 200 C 1,050 D
Requirements
(a) The values shown as A, B, C and D in K’s records, using a FIFO system, would be:
A: £
B: £
C: £
D: £
(b) The values shown as A, B, C and D in L’s records, using a LIFO system, would be:
A: £
B: £
C: £
D: £
(c) The value of the closing inventory in M’s records, using a weighted average based on
the whole of the three months’ purchases, would be £
.
(d) The profit reported by each of the students for the three-month period would be:
K (FIFO): £
L (LIFO): £
M (wt. ave.): £
(e) The pricing method being used by student M is known as a: (tick the correct box)
& periodic weighted average method
& cumulative weighted average method.
(f) Show how the stores ledger record of student M would appear if he recalculated a
revised weighted average price every time sacks are received into stores.
FUNDAMENTALS OF MANAGEMENT ACCOUNTING
43
ACCOUNTING FOR THE VALUE OF INVENTORIES
2006.1
Solutions to
Revision Questions
Solution 1
You will need to think carefully when you are selecting the answer for the narrative
multiple-choice questions. Read each option slowly and ensure that all aspects of the
description are correct before you make your final selection.
1.1 Answer: ( B)
The latest prices are used first:
1.2 Answer: (A)
Using FIFO, the inventory will be valued at the latest prices paid for the items. If
inventory turnover is high, then the items in stores will have been purchased
fairly recently. Therefore, they will be valued at prices which are close to current
purchase prices.
1.3 Answer: (D)
FIFO charges cost of production with the price of the oldest items in stores. When
prices are rising, the charges made to product costs lag behind current prices.
Product costs and charges to cost of sales are therefore understated and profits are
overstated.
1.4 Answer: (A)
LIFO charges the latest prices to cost of sales. Therefore, during a period of
rising prices the LIFO method will produce a higher cost of sales and a lower
profit. Since inventory is valued using the older prices, the LIFO closing
inventory values will be lower.
2
£
600 units from 23 April £9.00 per unit 5,400
1,000 units from 10 April £8.60 per unit 8,600
100 units from opening inventory £8.80 per unit
880
Total
14,880
45 2006.1
Solution 2
2.1 The total value of the components issued on April 10 was £369.00
The cost per component issued on April 16 was £6.40
Workings:
Notes
1.
£420 þ£195
100
¼ £6:15
2.
£246 þ£650
140
¼ £6:40
2.2 (a) True. The FIFO issues from inventory, to be charged as a part of cost of sales, will
be made at the older, lower prices.
(b) True. The FIFO closing inventory will be valued at the most recent prices.
(c) False. The LIFO valuation method is not acceptable under the accounting standard
which regulates the valuation of inventory for financial accounting purposes.
2.3
(a) Using the FIFO method of inventory valuation, the gross profit reported for
February would be £9,296.
(b) Using the LIFO method of inventory valuation, the gross profit reported for
February would be £8,141.
Workings:
(a) FIFO
Receipts Sales Balance
Date Qty Price £ Qty Price £ Qty Price £
6 1,400 8.20 11,480
1,400 8.20 11,480
15 900 9.10 8,190 1,400 8.20 11,480
900 9.10 8,190
2,300 19,670
20 780 8.20 6,396 620 8.20 5,084
900 9.10 8,190
1,520 13,274
22 330 9.90 3,267 620 8.20 5,084
900 9.10 8,190
330 9.90 3,267
1,850 16,541
28 620 8.20 5,084 660 9.10 6,006
240 9.10 2,184 330 9.90 3,267
860 7,268 990 9,273
Gross profit ¼(£14 (780 + 860)) £(6,396 + 7,268) ¼£9,296
Receipts Issues Balance
Date Qty Price £ Qty Price £ Qty Price £
1 Nil Nil
1 100 6.00 600.00 100 6.00 600.00
2 30 6.00 180.00 70 6.00 420.00
8 30 6.50 195.00 100 6.15
1
615.00
10 60 6.15 369.00 40 6.15 246.00
15 100 6.50 650.00 140 6.40
2
896.00
SOLUTIONS TO REVISION QUESTIONS C1
46
ACCOUNTING FOR THE VALUE OF INVENTORIES
2006.1
(b) LIFO
Receipts Sales Balance
Date Qty Price £ Qty Price £ Qty Price £
6 1,400 8.20 11,480
1,400 8.20 11,480
15 900 9.10 8,190 1,400 8.20 11,480
900 9.10 8,190
2,300 19,670
20 780 9.10 7,098 1,400 8.20 11,480
120 9.10 1,092
1,520 12,572
22 330 9.90 3,267 1,400 8.20 11,480
120 9.10 1,092
330 9.90 3,267
1,850 15,839
28 330 9.90 3,267
120 9.10 1,092
410 8.20 3,362
860 7,721 990 8.20 8,118
Gross profit ¼( £14 (780 + 860)) £(7,098 + 7,721) ¼£8,141
2.4 The issue of material from stores to a production cost centre will be recorded on a
material requisition. A goods received note is used to record the original receipt into
stores of goods from the supplier.
Solution 3
Probably the best approach is to draft your own rough stores records as workings.
Remember the need for accuracy; it is your final answer that counts. You will not receive
marks for your workings.
(a) A £800
B £300
C £240
D £1,060
Workings: K’s records using FIFO
Receipts Sales Balance
Month Qty Price £ Qty Price £ Qty Price £
April 1,000 1.00 1,000
500 1.00 500 500 1.00 500
May 500 1.20 600 500 1.00 500
250 1.20 300
750 800 250 1.20 300
June 1,000 1.00 1,000 200 1.20 240 50 1.20 60
1,000 1.00 1,000
1,050 1,060
FUNDAMENTALS OF MANAGEMENT ACCOUNTING
47
ACCOUNTING FOR THE VALUE OF INVENTORIES
2006.1
( b) A £850
B £250
C £200
D £1,050
Workings: L’s records using LIFO
Receipts Sales Balance
Month Qty Price £ Qty Price £ Qty Price £
April 1,000 1.00 1,000
500 1.00 500 500 1.00 500
May 500 1.20 600 500 1.20 600
250 1.00 250
750 850 250 1.00 250
June 1,000 1.00 1,000 200 1.00 200 1,050 1.00 1,050
(c) £1,092
Workings:
Weighted average price for 3 months ¼
£ð1;000þ600þ1;000Þ
1;000þ500þ1;000
¼£1.04 per unit
Value of closing inventory ¼1,050 units £1.04 ¼£1,092
(d) K ( FIFO) £635
L ( LIFO) £625
M (wt. ave.) £667
Workings:
KLM
£££
Sales (1,450 @ £1.50)
2,175 2,175 2,175
Cost of sales:
Purchases 2,600 2,600 2,600
Less closing inventory
1,060 1,050 1,092
1,540 1,550 1,508
Profit
635 625 667
(e) The pricing method being used by student M is known as a periodic weighted average
method.
M’s weighted average is based on the whole of the three month’s purchases, so
this is a periodic average. The method that recalculates the weighted average price
every time a receipt occurs is called the cumulative or perpetual weighted average
method.
SOLUTIONS TO REVISION QUESTIONS C1
48
ACCOUNTING FOR THE VALUE OF INVENTORIES
2006.1
(f) This is the perpetual weighted average method.
Remember that you should use the perpetual weighted average in the assessment
unless you are specifically told otherwise.
Receipts Sales Balance
Month Qty Price £ Qty Price £ Qty Price £
April 1,000 1.00 1,000 1,000 1.00 1,000
April 500 1.00 500 500 1.00 500
May 500 1.20 600 1,000 1.10
1
1,100
May 750 1.10 825 250 1.10 275
June 1,000 1.00 1,000 1,250 1.02
2
1,275
June 200 1.02 204 1,050 1.02 1,071
1. £1,100/1,000 units ¼ £1.10 weighted average price
2. £1,275/1,250 units ¼ £1.02 weighted average price
FUNDAMENTALS OF MANAGEMENT ACCOUNTING
49
ACCOUNTING FOR THE VALUE OF INVENTORIES
2006.1
The Analysis of Overhead
3
The Analysis of
Overhead
3.1 Introduction
In this chapter you will learn about the analysis of indirect costs or overheads. We will be
looking at the three stage process of attributing overheads to individual cost units:
allocation, apportionment and absorption.
You will need a thorough understanding of the contents of this chapter for your studies
of the Fundamentals of Management Accounting syllabus and for many of the syllabuses at later
stages in the CIMA examinations.
3.2 What is an overhead cost?
3.2.1 Definition
An overhead cost is defined in the CIMA Terminology as ‘expenditure on labour, materials
or services that cannot be economically identified with a specific saleable cost unit’.
Overhead costs are also referred to as indirect costs which we discussed in Chapter 1.
Therefore, overhead cost comprises indirect material, indirect labour and indirect
expenses. The indirect nature of overheads means that they need to be ‘shared out’ among
the cost units as fairly and as accurately as possible.
3
LEARNING OUTCOMES
After completing this chapter, you should be able to:
"
prepare cost statements for allocation and apportionment of overheads, including
between reciprocal service departments;
"
calculate direct, variable and full costs of products, services and activities using
overhead absorption rates to trace indirect costs to cost units;
"
explain the use of cost information in pricing decisions, including marginal cost
pricing and the calculation of ’full cost’ based prices to generate a specified return
on sales or investment.
53 2006.1
In this chapter, you will be learning how this ‘sharing out’, or attribution, is accomplished
for production overheads, using a costing method known as absorption costing.
One of the main reasons for absorbing overheads into the cost of units is for inventory
(stock) valuation purposes. Accounting standards recommend that inventory valuations
should include an element of fixed production overheads incurred in the normal course of
business. We therefore have to find a fair way of sharing out the fixed production
overhead costs among the units produced.
3.2.2 Functional analysis of overhead costs
Overhead costs may be classified according to the function of the organisation responsible
for incurring the cost. Examples of overhead cost classifications include production
overhead, selling and distribution overhead, and administration overhead. It is usually
possible to classify the majority of overhead cost in this way, but some overhead costs
relate to the organisation generally and may be referred to as general overhead.
In this chapter we shall focus mainly on production overhead. Production is that
function of the business which converts raw materials into the organisation’s finished
product. The production department is usually divided into a number of departments or
cost centres. Some of these cost centres are directly involved with the production process.
These are called production cost centres and might include, for example, the cutting
department and the finishing department.
Other cost centres in the production department are not directly involved with the
production process but provide support services for the production cost centres. These
are called service cost centres, and examples include the maintenance department and the
stores.
3.3 Overhead allocation and apportionment
The first stage in the analysis of production overheads is the selection of appropriate cost
centres. The selection will depend on a number of factors, including the level of control
required and the availability of information.
Having selected suitable cost centres, the next stage in the analysis is to determine the
overhead cost for each cost centre. This is achieved through the process of allocation and
apportionment.
Cost allocation is possible when we can identify a cost as specifically attributable to a
particular cost centre. For example the salary of the manager of the packing department
can be allocated to the packing department cost centre. It is not necessary to share the
salary cost over several different cost centres.
Cost apportionment is necessary when it is not possible to allocate a cost to a specific
cost centre. In this case the cost is shared out over two or more cost centres according
to the estimated benefit received by each cost centre. As far as possible the basis of
apportionment is selected to reflect this benefit received. For example, the cost of rent
and rates might be apportioned according to the floor space occupied by each cost
centre.
The following example illustrates the allocation and apportionment of production
overhead costs.
STUDY MATERIAL C1
54
THE ANALYSIS OF OVERHEAD
2006.1
Example
The information given below relates to a four-week accounting period of WHW Ltd.
Machining Assembly Finishing Stores
Area occupied (square metres) 24,000 36,000 16,000 4,000
Plant and equipment at cost (£000) 1,400 200 60 10
Number of employees 400 800 200 20
Direct labour hours 16,000 32,000 4,000
Direct wages (£) 32,600 67,200 7,200
Machine hours 32,000 4,000 200
Number of requisitions on stores 310 1,112 100
Allocated costs £ £ £ £
Indirect wages 9,000 15,000 4,000 6,000
Indirect materials 394 1,400 600
Maintenance 1,400 600 100
Power 1,600 400 200
Other costs (in total) £
Rent 2,000
Business rates 600
Insurance on building 200
Lighting and heating 400
Depreciation on plant and equipment 16,700
Wage-related costs 28,200
Factory administration and personnel 7,100
Insurance on plant and equipment 1,670
Cleaning of factory premises 800
57,670
The data above distinguishes between those costs which can and those which cannot be allocated to a
cost centre. The first step is to construct an overhead analysis sheet having separate columns for each
cost centre, together with a column for the total costs, a description of the cost item, and the basis upon
which the cost has been apportioned between the cost centres if applicable.
An explanation of the apportionment method is given beneath the analysis.
Item
Basis of
apportionment
Machining
£
Assembly
£
Finishing
£
Stores
£
Total
£
Indirect wages Allocation 9,000 15,000 4,000 6,000 34,000
Indirect materials Allocation 394 1,400 600 2,394
Maintenance Allocation 1,400 600 100 2,100
Power Allocation 1,600 400 200 2,200
Rent Area occupied 600 900 400 100 2,000
Business rates Area occupied 180 270 120 30 600
Building insurance Area occupied 60 90 40 10 200
Lighting/heating Area occupied 120 180 80 20 400
Depreciation on
plant/equip’t
Plant/equip’t at
cost
14,000 2,000 600 100 16,700
Wage-related costs Total wages 8,320 16,440 2,240 1,200 28,200
Factory admin. and
personnel
No. of employees 2,000 4,000 1,000 100 7,100
Insurance on
plant/equip’t
Plant/equip’t at cost 1,400 200 60 10 1,670
Factory cleaning Area occupied 240 360 160 40 800
39,314 41,840 9,600 7,610 98,364
FUNDAMENTALS OF MANAGEMENT ACCOUNTING
55
THE ANALYSIS OF OVERHEAD
2006.1
You should note that the direct wages costs are not included in the analysis because they are not
overhead costs. Also notice that the apportionment of wage-related costs is based on total wages that
is, the sum of the direct and indirect wages for each cost centre.
The apportioned costs are all calculated using the general formula:
Total overhead cost
Total value of apportionment base
Value of apportionment base of the cost centre being calculated
For example, in the case of depreciation apportioned to the machining cost centre:
£16,700
£1,670,000
£1,400,000 ¼ £14; 000
The result of the initial allocation and apportionment is that the organisation’s production overhead
costs have been identified with cost centres associated with production. However, the service cost centre
(stores) is not directly involved in the manufacture of the saleable cost unit. Nevertheless, it is part of the
production function and the total cost of operating the stores should be attributed to the saleable cost
units. The total cost of the stores must be shared or apportioned between those production cost centres
which derive benefit from the stores service.
If we now return to our example, the next step is to apportion the cost of the stores department to the
production cost centres.
Item
Basis of
apportionment
M/c
£
Ass’y
£
Finish
£
Stores
£
Total
£
B/fwd 39,314 41,840 9,600 7,610 98,364
Stores costs No. of
requisitions
on stores
1,550 5,560 500 (7,610)
40,864 47,400 10,100 98,364
We have now achieved the objective of allocating and apportioning all of the production overhead
costs to the departments directly involved in the manufacture of the saleable cost unit.
3.4 Absorption of overheads into saleable
cost units
3.4.1 General principles
The last stage in the analysis of overheads is their absorption into the cost units produced
in the production cost centres. This is sometimes referred to as overhead recovery.
To begin with, we need to measure the level of production achieved. There are many
measures which may be used, but the most common are:
physical units produced;
labour hours worked;
machine hours operated.
It is quite likely that different production departments will measure their production in
different ways. The objective is to use a measure which reflects the nature of the work
involved. The physical unit measure is in theory the simplest but it is only valid if all of the
items produced require the same amount of resources.
STUDY MATERIAL C1
56
THE ANALYSIS OF OVERHEAD
2006.1
The overhead costs of each production cost centre are then divided by the quantity of
production achieved to calculate the amount of overhead cost to be attributed to each
unit. This is the technique of overhead absorption and we shall illustrate it by extending
our example on allocation and apportionment.
The output of the machining department is to be measured using the number of
machine hours produced, while the output of the assembly and finishing departments is to
be measured using the number of direct labour hours produced. The reasons for this can
be seen from the number of machine and direct labour hours for each department shown
in the original data for the example. The machining department is clearly machine-
intensive, whereas the other departments are labour-intensive.
The absorption rates are calculated by dividing the costs attributed to the department by
its appropriate measure of output.
Machining Assembly Finishing
Production overhead costs
obtained by allocation and
apportionment £40,864 £47,400 £10,100
Number of:
machine hours 32,000
direct labour hours 32,000 4,000
Absorption rates:
per machine hour £1.277
per direct labour hour £1.48125 £2.525
3.4.2 Applying the overhead absorption rate
When using an absorption method based either on direct labour hours or on machine
hours the cost attributed to each unit is obtained by multiplying the time taken per unit by
the absorption rate per hour.
For example, if a particular cost unit took three machine hours in the machining department,
and five direct labour hours in each of the assembly and finishing departments, the overhead
cost absorbed by the cost unit would be as follows:
£
Machining: 3 hours £1.277 3.83
Assembly: 5 hours £1.48125 7.41
Finishing: 5 hours £2.525
12.63
Overhead absorbed by cost unit
23.87
3.4.3 Other absorption bases
In addition to the three bases of absorption mentioned above, a percentage rate based on
any of the following may be used:
direct material cost;
direct labour cost;
prime cost.
FUNDAMENTALS OF MANAGEMENT ACCOUNTING
57
THE ANALYSIS OF OVERHEAD
2006.1
For example, if a direct labour cost percentage is used the absorption rates would be as
follows:
Machining Assembly Finishing
£££
Production overhead costs 40,864 47,400 10,100
Direct wages cost 32,600 67,200 7,200
Direct labour cost percentage 125% 71% 140%
If our cost unit had a labour cost of £12 in the machining department, and £20 in each
of the assembly and finishing departments, the overhead cost absorbed by the cost unit
using this method would be as follows:
£
Machining: 125% £12 ¼ 15.00
Assembly: 71% £20 ¼ 14.20
Finishing: 140% £20 ¼
28.00
Overhead absorbed by cost unit
57.20
The direct material cost and the prime cost methods work in a similar way.
3.4.4 Selecting the most appropriate absorption rate
The data in the last example demonstrate how the calculated total production cost of a
particular cost unit can be dramatically different, depending on the overhead absorption
method selected. It is important that the selected method results in the most realistic
charge for overhead, reflecting the incidence of overheads in the cost centre as closely as
possible within the limits of available data.
You must not make the common mistake of thinking that the best absorption method
in this example would be the one which results in the lowest overhead charge to our cost
unit. Remember that the same total cost centre overhead is being shared out over the cost
units produced, whichever absorption method is selected. If this unit is given a lower
charge for overhead, then other cost units will be charged with a higher amount so that the
total overhead is absorbed overall.
A major factor in selecting the absorption rate to be used is a consideration of the
practical applicability of the rate. This will depend on the ease of collecting the data
required to use the selected rate.
It is generally accepted that a time-based method should be used wherever possible, that is,
the machine hour rate or the labour hour rate. This is because many overhead costs increase
with time, for example indirect wages, rent and rates. Therefore, it makes sense to attempt to
absorb overheads according to how long a cost unit takes to produce. The longer it takes, the
more overhead will have been incurred in the cost centre during that time.
In addition to these general considerations, each absorption method has its own
advantages and disadvantages:
(a) Rate per unit. This is the easiest method to apply but it is only suitable when all cost
units produced in the period are identical. Since this does not often happen in practice
this method is rarely used.
STUDY MATERIAL C1
58
THE ANALYSIS OF OVERHEAD
2006.1
(b) Direct labour hour rate. This is a favoured method because it is time-based. It is most
appropriate in labour-intensive cost centres, which are becoming rarer nowadays and
so the method is less widely used than it has been in the past.
(c) Machine hour rate. This is also a favoured method because it is time-based. It is most
appropriate in cost centres where machine activity predominates and is therefore more
widely used than the direct labour hour rate. As well as absorbing the time-based
overheads mentioned earlier, it is more appropriate for absorbing the overheads
related to machine activity, such as power, maintenance, repairs and depreciation.
(d) Direct wages cost percentage. This method may be acceptable because it is to some extent
time-based. A higher direct wages cost may indicate a longer time taken and therefore
a greater incidence of overheads during this time. However the method will not
produce equitable overhead charges if different wage rates are paid to individual
employees in the cost centre. If this is the case, then there may not be a direct
relationship between the wages paid and the time taken to complete a cost unit.
(e) Direct materials cost percentage. This is not a very logical method because there is no
reason why a higher material cost should lead to a cost unit apparently incurring more
production overhead cost. The method can be used if it would be too costly and
inconvenient to use a more suitable method.
(f ) Prime cost percentage. This method is not recommended because it combines methods
(d) and (e) and therefore suffers from the combined disadvantages of both.
3.5 Predetermined overhead absorption rates
Overhead absorption rates are usually predetermined, that is, they are calculated in
advance of the period over which they will be used.
The main reason for this is that overhead costs are not incurred evenly throughout the
period. In some months the actual expenditure may be very high and in others it may be
relatively low. The actual overhead rate per hour or per unit will therefore be subject to
wide fluctuations. If the actual rate was used in product costing, then product costs would
also fluctuate wildly. Such product costs would be very difficult to use for planning and
control purposes.
Fluctuations in the actual level of production would also cause the same problem of
fluctuating product costs.
To overcome this problem the absorption rate is determined in advance of the period,
using estimated or budget figures for overhead and for the number of units of the
absorption base ( labour hours or machine hours, etc.).
A further advantage of using predetermined rates is that managers have an overhead
rate permanently available which they can use in product costing, price quotations and so
on. The actual overhead costs and activity levels are not known until the end of the period.
It would not be desirable for managers to have to wait until after the end of the period
before they had a rate of overhead that they could use on a day-to-day basis.
3.5.1 Under- or over-absorption of overheads
The problem with using predetermined overhead absorption rates is that the actual
figures for overhead and for the absorption base are likely to be different from the
estimates used in calculating the absorption rate. It is this difference which causes an
FUNDAMENTALS OF MANAGEMENT ACCOUNTING
59
THE ANALYSIS OF OVERHEAD
2006.1
under-/over-absorption of production overhead. We will now return to our example
in Section 3.4 to see how this is calculated, assuming that machine/labour hour rates
have been used to absorb the overheads.
We will assume that all of the values used in the calculations in our example are
estimates based on WHW Limited’s budgets.
The actual costs for the same four-week period have now been allocated and
apportioned using the same techniques and bases as shown in our earlier example, with the
following total actual costs being attributed to each cost centre:
Machining Assembly Finishing
£££
Actual costs 43,528 49,575 9,240
Actual labour and machine hours recorded against each cost centre were:
Machining Assembly Finishing
Number of:
machine hours 32,650
labour hours 31,040 3,925
The amount of overhead cost absorbed into each department’s total number of
saleable cost units will be calculated by multiplying the absorption rate calculated in
Section 3.4 (using the budget data) by the actual number of hours. The amounts
absorbed are thus:
Machining Assembly Finishing
Amount absorbed: £ £ £
32,650 hours £1.277 41,694
31,040 hours £1.48125 45,978
3,925 hours £2.525 9,911
This is compared with the actual cost incurred and the difference is the under-/over-
absorption of production overhead:
Machining Assembly Finishing
£££
Amount absorbed 41,694 45,978 9,911
Actual cost incurred
43,528 49,575 9,240
Over-absorption
671
Under-absorption
1,834 3,597
If the amount absorbed exceeds the amount incurred, then an over-absorption arises;
the opposite is referred to as an under-absorption. The terms under-recovery and over-recovery
are sometimes used.
STUDY MATERIAL C1
60
THE ANALYSIS OF OVERHEAD
2006.1
3.5.2 The reasons for under- or over-absorption
The under- or over-absorption has arisen because the actual overhead incurred per hour
was different from the predetermined rate per hour. There are two possible causes
of this:
(a) The actual number of hours (machine or direct labour) was different from the number
contained in the budget data. If this happens, then we would expect the variable
element of the overhead to vary in direct proportion to the change in hours, so this
part of the absorption rate would still be accurate. However, the fixed overhead would
not alter with the hours worked and this means that the actual overhead cost per hour
would be different from the predetermined rate.
( b) The actual production overhead incurred may be different from the estimate
contained in the predetermined rate. Apart from the expected change in variable
overhead referred to in (a), this would also cause an under- or over-absorption of
overhead.
We will return in a later chapter to learn how any under- or over-absorption is
accounted for in the bookkeeping records.
3.5.3 The problems caused by under- or over-absorption
of overheads
If overheads are under-absorbed then managers have been working with unit rates for
overheads which are too low. Prices may have been set too low and other similar decisions
may have been taken based on inaccurate information. If the amount of under-absorption
is significant, then this can have a dramatic effect on reported profit.
Do not make the common mistake of thinking that over-absorption is not such a bad
thing because it leads to a boost in profits at the period end. If overhead rates
have been unnecessarily high, then managers may have set selling prices unnecessarily
high, leading to lost sales. Other decisions would also have been based on inaccurate
information.
Although it is almost impossible to avoid under- and over-absorption altogether, it is
possible to minimise the amount of adjustment necessary at the year end. This is achieved
by conducting regular reviews of the actual expenditure and activity levels which are
arising. The overhead absorption rate can thus be reviewed to check that it is still
appropriate to absorb the overheads sufficiently accurately by the year end. If necessary
the overhead absorption rate can be adjusted to reflect more recent estimates of activity
and expenditure levels.
3.6 Illustrative example
You can use the following short example to practise the techniques which we have
covered so far in this chapter.
The information given below relates to the forthcoming period for a manufacturer’s
operation. There are four cost centres of which two are involved in production and two
are service cost centres.
FUNDAMENTALS OF MANAGEMENT ACCOUNTING
61
THE ANALYSIS OF OVERHEAD
2006.1
Production depts Service depts
Total A B Canteen Stores
£££££
Allocated costs 70,022 21,328 29,928 8,437 10,329
Other costs:
Rent and rates 4,641
Buildings insurance 3,713
Electricity and gas 6,800
Plant depreciation 28,390
Plant insurance
8,517
122,083
Area occupied (square metres) 7,735 6,188 1,547 3,094
Plant at cost ( £000) 1,845 852 142
Number of employees 600 300 30 70
Machine hours 27,200 800
Direct labour hours 6,800 18,000
Number of stores requisitions 27,400 3,400
Use this information to calculate a production overhead absorption rate for
departments A and B.
3.6.1 Solution
The first step is to prepare an overhead analysis sheet which shows the apportionment of
the overheads, using the most appropriate basis for each.
Overhead item Basis of
apportionment
Total Dept A Dept B Canteen Stores
£££££
Allocated costs 70,022 21,328 29,928 8,437 10,329
Rent and rates
1
Area 4,641 1,934 1,547 387 773
Buildings insurance Area 3,713 1,547 1,238 309 619
Electricity & gas Area 6,800 2,833 2,267 567 1,133
Depreciation Plant cost 28,390 18,450 8,520 1,420
Insurance Plant cost
8,517 5,535 2,556 426
122,083 51,627 46,056 9,700 14,700
Canteen
2
Employees 6,000 3,000 (9,700) 700
Stores
3
Requisitions 13,700 1,700 (15,400)
122,083 71,327 50,756
Notes
1. The rent and rates cost is apportioned as follows. Total area occupied is 18,564 square
metres. Therefore, rent and rates cost £4,641/18,564 ¼£0.25 per square metre.
All of the other apportionments are calculated in the same way.
2. Since the canteen serves all other departments, its costs must be apportioned first, over
the 970 employees in the other departments.
STUDY MATERIAL C1
62
THE ANALYSIS OF OVERHEAD
2006.1
3. Once the stores have received a charge from the canteen, the total stores costs can be
apportioned to the production departments.
Looking at the data for machine hours and direct labour hours in each of the
departments, it appears that the most appropriate absorption base for department A is
machine hours and for department B is direct labour hours. The absorption rates can
now be calculated.
Production department A ¼ £71;327=27;200 ¼ £2:62 per machine hour
Production department B ¼ £50;756=18;000 ¼ £2:82 per direct labour hour
We can now extend the example a little further to practise using the calculated
absorption rates. What is the total production cost of the following job?
Job 847
Direct material cost £487
Direct labour cost £317
Machine hours in department A 195
Direct labour hours in department B 102
The overhead absorption rates can be applied as follows:
Job 847
£
Direct material cost* 487.00
Direct labour cost*
317.00
Prime cost 804.00
Production overhead:
Department A 195 hours £2.62 510.90
Department B 102 hours £2.82
287.64
Total production cost
1602.54
* Remember that direct costs are not affected by the overhead absorption rate selected.
See if you can calculate the under- or over-absorbed overhead in each of the
departments using the following data. The actual overhead incurred would have been
determined by the allocation and apportionment of the actual overhead costs.
Department A Department B
Actual results
Overhead incurred £70,483 £52,874
Direct labour hours 6,740 18,300
Machine hours 27,900 850
The first step is to calculate how much overhead would have been absorbed, based on
the actual hours and the predetermined overhead absorption rate for each department.
This total can then be compared with the actual overhead incurred.
FUNDAMENTALS OF MANAGEMENT ACCOUNTING
63
THE ANALYSIS OF OVERHEAD
2006.1
3.7 Reciprocal servicing
3.7.1 Taking account of reciprocal servicing
In the previous example there were two service cost centres: the canteen and the stores.
The stores personnel made use of the canteen and it was therefore equitable to charge
some of the canteen costs to the stores cost centre. It was not necessary to charge any of
the stores costs to the canteen because there was no indication that the canteen made use
of the stores facilities.
If the canteen had used the stores facilities, then we would say that reciprocal servicing
was taking place, that is, that the service cost centres each used the other’s facilities.
This can lead to a complicated situation because we do not know the total of the stores
costs until a proportion of the canteen costs has been charged to it. Similarly, we do not
know the total of the canteen costs until the total of the stores costs has been apportioned.
There are two methods which can be used to solve this problem. Your Fundamentals of
Management Accounting syllabus requires you to be able to use only the repeated distribution
method. We will use the following example to illustrate this. The other method, using
algebra, is outside the scope of your syllabus.
Example
A company reapportions the costs incurred by two service cost centres materials handling and
inspection to three production cost centres machining, finishing and assembly.
The following are the overhead costs which have been allocated and apportioned to the five cost
centres:
Estimates of the benefits received by each cost centre are as follows:
Department A Department B
££
Overhead absorbed
27,900 £2.62 73,098
18,300 £2.82 51,606
Overhead incurred
70,483 52,874
( Under-)/over-absorption
2,615 (1,268)
£000
Machining 400
Finishing 200
Assembly 100
Materials handling 100
Inspection 50
Machining Finishing Assembly
Materials
handling Inspection
%%%%%
Materials handling 30 25 35 10
Inspection 20 30 45 5
STUDY MATERIAL C1
64
THE ANALYSIS OF OVERHEAD
2006.1
These percentages indicate the use which each of the cost centres makes of the materials handling
and inspection facilities. Calculate the charge for overhead to each of the three production cost centres,
including the amounts reapportioned from the two service centres.
Solution: repeated distribution method
The task of allocating and apportioning the overheads to all cost centres has already been done (the
primary apportionment). The problem now is to reapportion the costs of the service centres (the
secondary apportionment).
Using the repeated distribution method the service cost centre costs are apportioned backwards and
forwards between the cost centres until the figures become very small. At this stage it might be necessary
to round the last apportionments.
In the workings that follow we have chosen to begin the secondary apportionment by apportioning the
inspection costs first. The £50,000 inspection cost is reapportioned according to the percentages provided,
then the total of the materials handling department is reapportioned and so on. The final result would have
been the same if we had chosen instead to begin by apportioning the materials handling costs first.
Machining Finishing Assembly Materials handling Inspection
£££ £ £
Initial allocation 400,000 200,000 100,000 100,000 50,000
Apportion inspection 10,000 15,000 22,500 2,500 (50,000)
Apportion materials
handling 30,750 25,625 35,875 (102,500) 10,250
Apportion inspection 2,050 3,075 4,612 513 (10,250)
Apportion materials
handling 154 128 180 (513) 51
Apportion inspection*
11 16 24
(51)
Total charge for
overhead
442,965 243,844 163,191 0 0
* When the service department cost reduces to a small amount the final apportionment is adjusted for roundings.
The objective has been achieved and all of the overheads have been apportioned to the production
cost centres, using the percentages given. A spreadsheet or similar software package would obviously be
helpful here!
3.7.2 The usefulness of reapportioned service
centre costs
The task of accounting for reciprocal servicing can be fairly laborious, particularly if it must
be performed manually. Managers must therefore ensure that the effort is worthwhile.
Generally, if the service centre costs are significant and they make considerable use of
each other’s services, then accounting for reciprocal servicing is probably worthwhile.
In other cases the reciprocal servicing could be ignored, or alternatively the service centre
which does the most work for the other service centres could be apportioned first. The
other service centres could then be apportioned direct to the production cost centres.
The overriding consideration must be the usefulness to managers of the resulting
information. If the improved accuracy of the overhead absorption rates is deemed to be
worthwhile, then reciprocal servicing should be taken into account in service cost
reapportionment.
In the assessment, you must never ignore the existence of reciprocal servicing
unless you are specifically instructed to do so.
FUNDAMENTALS OF MANAGEMENT ACCOUNTING
65
THE ANALYSIS OF OVERHEAD
2006.1
3.8 Activity based costing (ABC)
Activity based costing (ABC) is a more recent development in cost analysis. It is based on
the idea that to use a single absorption base of either labour or machine hours does not
accurately reflect the cause of the overhead costs being incurred.
Supporters of ABC argue that overhead costs are only loosely related to time and are
not often related to the volume of production. They argue that overheads in a modern
manufacturing environment are related to the complexity of production. The more
complex the production process for a product, the higher are the overheads incurred on its
behalf. For example, a product might require a number of complicated machine set-ups, or
quality control activity might be more intense for some products than for others. An ABC
approach attempts to ensure that overheads are traced to products in a way which more
adequately reflects the overhead cost which has been incurred on their behalf.
Using an ABC approach, overhead costs are accumulated initially in activity cost pools.
These might include, for example, order placing or material handling. Costs would then be
collected and analysed for each activity cost pool and a cost driver would be identified for
each activity. Cost drivers are the factors which cause the cost of an activity to increase.
Using estimates of the costs attributed to each activity cost pool and the number of cost
drivers associated with it, a cost driver rate is calculated. This is similar in principle to the
calculation of absorption rates. For example, if the total cost of the activity of setting up a
machine is £5,000 for a period and the number of machine set-ups for the period is 250,
the cost per set-up is £20 (£5,000/250). Each product that requires the use of this
machine is regarded as having incurred £20 of overhead cost each time the machine is set
up for the product.
This analysis of overhead costs into activities, and their absorption using a variety of
cost drivers, is believed to produce more accurate product costs. The ABC technique can
also be applied to non-production costs as well as to the determination of the costs of
services provided.
3.9 The use of cost information in pricing decisions
3.9.1 Marginal cost pricing
In Chapter 1 we saw how knowledge of the extra or marginal cost of making and selling a
hairdryer provided a manager with important information when deciding what selling price
should be charged for a special order.
If the price charged is higher than the marginal or incremental cost of making and selling a
cost unit then some contribution is earned towards the costs which are being incurred anyway.
These include costs such as certain production and administration overheads.
The problem with marginal cost pricing is that it is difficult to decide on the mark-up
that must be added to the marginal cost in order to ensure that the other costs such as
administration overheads are covered and that the organisation makes a profit.
STUDY MATERIAL C1
66
THE ANALYSIS OF OVERHEAD
2006.1
Marginal cost pricing is useful in a one-off special price decision such as that
discussed concerning the hairdryer in Chapter 1, but it does not help us to decide on the
price to be charged in routine product pricing decisions, in order to cover all costs and
earn a profit.
3.9.2 Full cost-plus pricing
We have seen how the overhead absorption rate can be used to trace indirect costs to cost
units in order to obtain the unit’s full cost.
Full cost-plus pricing involves adding a mark-up to the total cost of a cost unit in order
to arrive at the selling price. Your syllabus requires you to know how to calculate full cost-
based prices to generate a specified return on sales or on investment.
3.9.3 Example: full-cost pricing to achieve a specified
return on sales
This pricing method involves determining the full cost of a cost unit and then adding a
mark-up that represents a specified percentage of the final selling price. The following
example will demonstrate how the method works.
WP Limited manufactures product A.
Data for product A are as follows.
Direct material cost per unit £7
Direct labour cost per unit £18
Direct labour hours per unit 2 hours
Production overhead absorption rate £6 per direct labour hour
Mark-up for non-production overhead costs 5% of total production cost
WP Limited requires a 15 per cent return on sales revenue from all products.
Calculate the selling price for product A, to the nearest penny.
Solution
£ per unit
Direct material cost 7.00
Direct labour cost
18.00
Total direct cost 25.00
Production overhead absorbed ¼2 hours £6
12.00
Total production cost 37.00
Mark-up for non-production costs ¼5% £37.00
1.85
Full cost 38.85
Profit mark-up ¼15/85* £38.85
6.86
Selling price
45.71
* Always read the question data carefully. The 15 per cent required return is expressed as a
percentage of the sales revenue, not as a percentage of the cost.
FUNDAMENTALS OF MANAGEMENT ACCOUNTING
67
THE ANALYSIS OF OVERHEAD
2006.1
3.9.4 Example: full-cost pricing to achieve a specified
return on investment
This method involves determining the amount of capital invested to support a product. For
example, some fixed or non-current assets and certain elements of working capital such as
inventory and trade receivables can be attributed to individual products.
The selling price is then set to achieve a specified return on the capital invested on
behalf of the product. The following example will demonstrate how the method works.
LG Limited manufactures product B.
Data for product B are as follows.
Direct material cost per unit £62
Direct labour cost per unit £14
Direct labour hours per unit 4 hours
Production overhead absorption rate £16 per direct machine hour
Mark-up for non-production overhead costs 8% of total production cost
LG Limited sells 1,000 units of product B each year. Product B requires an investment
of £400,000 and the target rate of return on investment is 12% per annum.
Calculate the selling price for one unit of product B, to the nearest penny.
Solution
£ per unit
Direct material cost 62.00
Direct labour cost
14.00
Total direct cost 76.00
Production overhead absorbed ¼4 hours £16
64.00
Total production cost 140.00
Mark-up for non-production costs ¼8% £140.00
11.20
Full cost 151.20
Profit mark-up (see working)
48.00
Selling price
199.20
Working
Target return on investment in product B ¼£400,000 12% ¼£48,000
Target return per unit of product B ¼£48,000/1,000 units ¼£48
3.9.5 Second example: full-cost pricing to achieve a
specified return on investment
This example demonstrates how the profit mark-up can be determined as a percentage of
the total budgeted cost for a period.
The following data relate to a company which produces a range of products.
Capital invested in company £800,000
Required return on investment each period 15%
Budgeted total cost for next period £1,500,000
One of the company’s products, R, incurs a total cost of £5 per unit.
Calculate the cost-plus selling price of one unit of product R.
STUDY MATERIAL C1
68
THE ANALYSIS OF OVERHEAD
2006.1
Solution
Required profit in period ¼£800,000 15% ¼£120,000
Profit as a percentage of budgeted total cost ¼(£120,000/£1,500,000) 100%
¼8%
This percentage is applied to calculate the mark up for all products produced.
Profit mark-up for one unit of product R ¼£35 8% ¼£2.80
Selling price for one unit of product R ¼£35 + £2.80 ¼£37.80
3.10 Summary
Having read this chapter the main points you should understand are as follows:
1. The three stages in attributing overheads to cost units are allocation, apportionment and
absorption. Allocation involves allotting whole items of cost to a single cost centre.
Apportionment is necessary when it is not possible to allot the whole cost to a single cost
centre. The cost must then be apportioned between cost centres using a suitable basis.
2. The primary apportionment of production overheads involves apportioning the
overhead costs to all cost centres. The secondary apportionment is then necessary to
reapportion the service cost centre costs to the production cost centres.
3. The final totals of the production cost centre overheads are absorbed into product
costs using a predetermined production overhead absorption rate. The absorption basis
should reflect the type of activity undertaken within each production cost centre.
4. The production overhead absorption rate is calculated by dividing the budgeted cost
centre overheads by the budgeted number of units of the absorption base (machine
hours, direct labour hours, etc.).
5. Under- or over-absorption arises at the end of a period when the amount of production
overhead absorbed into cost units is lower than or higher than the actual production
overhead incurred during the period.
6. Reciprocal servicing occurs where service cost centres each do work for the other. In
this situation the service cost centre overheads are reapportioned using the repeated
distribution method.
7. Activity based costing uses a variety of cost drivers to trace overhead costs to products
and services.
8. The full cost of a cost unit can be used as a basis for determining its selling price. The
required return from each cost unit can be calculated to achieve a specified return on
sales or return on investment.
FUNDAMENTALS OF MANAGEMENT ACCOUNTING
69
THE ANALYSIS OF OVERHEAD
2006.1
Revision Questions
Question 1 Multiple choice
1.1 A method of dealing with overheads involves spreading common costs over cost
centres on the basis of benefit received. This is known as:
(A) overhead absorption.
(B) overhead apportionment.
(C) overhead allocation.
(D) overhead analysis.
1.2 An overhead absorption rate is used to:
(A) share out common costs over benefiting cost centres.
(B) find the total overheads for a cost centre.
(C) charge overheads to products.
(D) control overheads.
1.3 Over-absorbed overheads occur when:
(A) absorbed overheads exceed actual overheads.
(B) absorbed overheads exceed budgeted overheads.
(C) actual overheads exceed budgeted overheads.
(D) budgeted overheads exceed absorbed overheads.
Data for questions 1.4 and 1.5
1.4 Based on the data given above, what is the labour hour overhead absorption rate?
(A) £17.50 per hour.
(B) £17.20 per hour.
(C) £18.44 per hour.
(D) £18.76 per hour.
1.5 Based on the data given above, what is the amount of overhead under-/over-
absorbed?
(A) £2,550 under-absorbed.
(B) £2,529 over-absorbed.
(C) £2,550 over-absorbed.
(D) £7,460 under-absorbed.
Budgeted labour hours 8,500
Budgeted overheads £148,750
Actual labour hours 7,928
Actual overheads £146,200
3
71 2006.1
1.6 A management consultancy recovers overheads on chargeable consulting hours.
Budgeted overheads were £615,000 and actual consulting hours were 32,150.
Overheads were under-recovered by £35,000.
If actual overheads were £694,075, what was the budgeted overhead absorption
rate per hour?
(A) £19.13
(B) £20.50
(C) £21.59
(D) £22.68
1.7 P Ltd absorbs overheads on the basis of direct labour hours. The overhead
absorption rate for the period has been based on budgeted overheads of £150,000
and 50,000 direct labour hours.
During the period, overheads of £180,000 were incurred and 60,000 direct labour
hours were used.
Which of the following statements is correct?
(A) Overhead was £30,000 over-absorbed.
(B) Overhead was £30,000 under-absorbed.
(C) No under- or over-absorption occurred.
(D) None of the above.
Question 2 Short objective-test questions
2.1 Match the overhead costs to the most appropriate basis of cost apportionment.
Write the correct letter in the box provided beside each apportionment basis. An
apportionment basis may be selected more than once.
Overhead cost
(a) Canteen costs
(b) Cleaning of factory premises
(c) Power
(d) Rent
(e) Insurance of plant and machinery
Apportionment bases
& Floor area
& Plant and equipment at cost
& Number of employees
& Machine running hours
& Direct labour hours
2.2 Maintenance costs are to be apportioned to production cost centres on the basis of
the following number of maintenance hours worked in each cost centre.
Machining Assembly Finishing
Maintenance hours worked 1,000 700 300
REVISION QUESTIONS C1
72
THE ANALYSIS OF OVERHEAD
2006.1
Complete the following extract from the overhead analysis sheet:
2.3 After the initial overhead allocation and apportionment has been completed, the
overhead analysis sheet for a car repair workshop is as follows:
The costs of the canteen and vending activity are to be reapportioned to the other
activities on the basis of the number of personnel employed on each activity.
The canteen and vending cost to be apportioned to the crash repair activity is
£
2.4 The budgeted fixed overhead absorption rate for last period was £5 per direct labour
hour. Other data for the period are as follows:
The number of direct labour hours budgeted to be worked last period was
2.5 Tick the correct box.
Activity in the packing department of a company manufacturing fine china involves
operatives bubble-wrapping finished items and placing them in boxes which are then
sealed and labelled. Most of the boxes are sealed and labelled by specialised
machines, but about a quarter of them have to be sealed and labelled by hand.
Budgeted activity levels for next period are 3,800 machine hours and 3,600 direct
labour hours. The most appropriate production overhead absorption rate for the
packing department would be a:
Machine hour rate &
Direct labour hour rate &
2.6 Data for the machining cost centre are as follows:
Overhead cost item Total Machining Assembly Finishing
££ £ £
Maintenance cost 38,000
Total overhead Canteen and
cost Vehicle servicing Crash repairs Tyre fitting vending
£££££
233,000 82,000 74,000 61,000 16,000
Vehicle
servicing
Crash
repairs
Tyre
fitting
Canteen and
vending
Number of personnel 20 15 5 2
Actual fixed overhead expenditure £234,500
Actual direct labour hours 51,300
Budgeted fixed overhead expenditure £212,900
Budgeted cost centre overhead £210,000
Actual cost centre overhead £230,000
Budgeted machine hours 42,000
Actual machine hours 43,000
FUNDAMENTALS OF MANAGEMENT ACCOUNTING
73
THE ANALYSIS OF OVERHEAD
2006.1
Complete the following calculation.
2.7 The number of machine and labour hours budgeted for three production cost
centres for the forthcoming period is as follows:
The most appropriate production overhead absorption basis for each cost centre
would be (tick the correct box):
2.8 Production overhead in department A is absorbed using a predetermined rate per
machine hour. Last period, the production overhead in department A was under-
absorbed. Which of the following situations could have contributed to the under
absorption? (tick all that apply)
& the actual production overhead incurred was lower than budgeted.
& the actual production overhead incurred was higher than budgeted.
& the actual machine hours were lower than budgeted.
& the actual machine hours were higher than budgeted.
2.9 The Crayfield Hotel has completed its initial allocation and apportionment of
overhead costs and has established that the total budgeted annual overhead cost of
its linen services activity is £836,000.
The cost unit used to plan and control costs in the hotel is an occupied room
night. The hotel expects the occupancy rate of its 400 rooms, which are available
for 365 nights each year, to be 85 per cent for the forthcoming year.
To the nearest penny, the overhead absorption rate for the linen services activity
is £
per occupied room night.
2.10 GY Limited budgets to produce and sell 3,800 units of product R in the
forthcoming year. The amount of capital investment attributable to product R will
be £600,000 and GY Limited requires a rate of return of 15 per cent on all capital
invested.
Further details concerning product R are as follows.
£
Overhead absorbed
Actual overhead incurred Tick correct box:
Overhead under-/over-absorbed
under-absorbed &
over-absorbed &
Machining Assembly Finishing
Machine hours 50,000 4,000 5,000
Labour hours 10,000 30,000 20,000
Machining Assembly Finishing
Rate per machine hour &&&
Rate per labour hour &&&
Direct material cost per unit £14
Direct labour cost per unit £19
Variable overhead cost per unit £3
Machine hours per unit 8
REVISION QUESTIONS C1
74
THE ANALYSIS OF OVERHEAD
2006.1
Fixed overhead is absorbed at a rate of £11 per machine hour.
Calculate all answers to the nearest penny.
(a) The variable cost of product R is £
per unit.
(b) The total(full) cost of product R is £
per unit.
(c) The selling price of product R which will achieve the specified return on
investment is £
per unit.
Question 3 Overhead analysis and absorption
The Utopian Hotel is developing a cost accounting system. Initially it has been decided to
create four cost centres: Residential and Catering deal directly with customers, while
Housekeeping and Maintenance are internal service cost centres.
The management accountant is in the process of calculating overhead absorption rates
for the next period. An extract from the overhead analysis sheet is as follows:
Other information
The following information is also available:
Requirements
(a) The entries on the overhead analysis sheet shown as A to C are:
(b) The initial overhead allocation and apportionment has now been completed. The cost
centre overhead totals are as follows:
Housekeeping works 70 per cent for Residential and 30 per cent for Catering,
and Maintenance works 20 per cent for Housekeeping, 30 per cent for Catering and
50 per cent for Residential.
Basis of Residential Catering Housekeeping Maintenance Total
apportionment ££ £ £ £
Consumables Allocated 14,000 23,000 27,000 9,000 73,000
Staff costs Allocated 16,500 13,000 11,500 5,500 46,500
Rent and rates A 37,500
Contents ins. Value of equip. B 14,000
Heat and light C 18,500
Residential Catering Housekeeping Maintenance Total
Floor area (sq. metres) 2,750 1,350 600 300 5,000
Value of equipment, etc. £350,000 £250,000 £75,000 £75,000 £750,000
(to the nearest £ )
(to the nearest £ )
(to the nearest £ )
Residential Catering Housekeeping Maintenance Total
££ £ ££
Initial allocation and
apportionment 85,333 68,287 50,370 23,010 227,000
FUNDAMENTALS OF MANAGEMENT ACCOUNTING
75
THE ANALYSIS OF OVERHEAD
2006.1
After the reapportionment of the Housekeeping and Maintenance cost centres, the total
cost centre overheads for Residential and Catering will be, to the nearest £:
Question 4 Overhead absorption rates
QRS Ltd has three main departments Casting, Dressing and Assembly and has
prepared the following production overhead budgets for period 3
During period 3, actual results were as follows:
Requirements
(a) The departmental overhead absorption rates per production hour for period 3 are:
( b) (i) The overheads in the Casting department were (tick the correct box and insert the
value of the over-/under-absorption):
(ii) The overheads in the Dressing department were (tick the correct box and insert
the value of the over-/under-absorption):
(c) The overheads in the Assembly department were over-absorbed. Which of the
following factors contributed to the over absorption?
& the actual overheads incurred were lower than budgeted.
& the actual production hours were higher than budgeted.
Residential £
Catering £
Department Casting Dressing Assembly
Production overheads £225,000 £175,000 £93,000
Expected production hours 7,500 7,000 6,200
Department Casting Dressing Assembly
Production overheads £229,317 £182,875 £92,500
Production hours 7,950 7,280 6,696
Casting £
Dressing £
Assembly £
under-absorbed & over-absorbed &
by £
under-absorbed & over-absorbed &
by £
REVISION QUESTIONS C1
76
THE ANALYSIS OF OVERHEAD
2006.1
Question 5 Overhead analysis
DC Ltd is an engineering company which uses job costing to attribute costs to individual
products and services provided to its customers. It has commenced the preparation of its
fixed production overhead cost budget for year 2 and has identified the following costs:
The stores and maintenance departments are production service departments. An
analysis of the services they provide indicates that their costs should be apportioned as
follows:
Requirements
(a) After the apportionment of the service department costs, the total overheads of the
production departments will be (to the nearest £500):
Machining £
Assembly £
Finishing £
( b) DC Ltd’s overhead absorption rates for year 1 are as follows:
Machining £13.83 per machine hour
Assembly £9.98 per labour hour
Finishing £9.45 per labour hour
Job no. XX34 is to be started and completed in year 1. Data for the job is as follows:
Direct materials cost £2,400
Direct labour cost £1,500
Machine hours and labour hours required for the job are:
DC Ltd adds 10 per cent to total production cost in order to absorb non-production
overhead costs, and profit is calculated as 20 per cent of selling price.
£000
Machining 600
Assembly 250
Finishing 150
Stores 100
Maintenance
80
1,180
Machining Assembly Finishing Stores Maintenance
Stores 40% 30% 20% 10%
Maintenance 55% 20% 20% 5%
Machine hours Labour hours
Machining department 45 10
Assembly department 5 15
Finishing department 4 12
FUNDAMENTALS OF MANAGEMENT ACCOUNTING
77
THE ANALYSIS OF OVERHEAD
2006.1
Requirement
Complete the following statements (to the nearest penny):
(i) The total production overhead cost of job no. XX34 is £
(ii) The total production cost of job no. XX34 is £
(iii) The selling price of job no. XX34 is £
REVISION QUESTIONS C1
78
THE ANALYSIS OF OVERHEAD
2006.1
Solutions to
Revision Questions
Solution 1
Always remember that production overhead absorption rates are predetermined, that is,
they are based on budgeted production overhead and budgeted activity levels.
Over- or under-absorbed overhead ¼overhead absorbed actual overhead incurred.
If actual overhead exceeds the amount absorbed, then there is an under-absorption. If
actual overhead is less than the amount absorbed, there is an over-absorption.
1.1 Answer: ( B)
Answer (A) describes the final stage of charging overheads to cost units.
(C) describes the allotment of whole items of cost to a single cost unit or cost
centre. (D) describes the whole process of overhead allocation, apportionment and
absorption.
1.2 Answer: (C)
An overhead absorption rate is a means of attributing overhead to a product or
service based, for example, on direct labour hours.
1.3 Answer: (A)
Over- or under-absorption of overhead is the difference between absorbed
overheads and actual overheads. Under-absorption occurs when actual overheads
exceed absorbed overheads.
1.4 Answer: (A)
Labour hour overhead absorption rate ¼£148,750/£8,500 ¼£17.50 per hour.
1.5 Answer: ( D)
1.6 Answer: ( B)
Let £x ¼budgeted overhead absorption rate per hour:
£
Overhead incurred 146,200
Overhead absorbed ¼£17.50 7,928 hours
138,740
Under-absorption
7,460
3
79 2006.1
\ x ¼
694,075 35,000
32,150
¼ 20:5
1.7 Answer: (C)
Solution 2
2.1
(a) Number of employees
(b) Floor area
(c) Machine running hours
(d) Floor area
(e) Plant and equipment at cost
2.2 Overhead cost per maintenance hour ¼
£38;000
1;000 þ700 þ300
¼ £19
2.3 Canteen and vending cost per
personnel member in production activities* ¼
£16;000
20 þ15 þ 5
¼ £400
* The canteen and vending personnel are not included because the canteen cannot
give a charge to itself.
The canteen and vending cost apportioned to the crash repair activity is
£400 15 ¼£6,000.
2.4 Direct labour hours budgeted to be worked last period ¼42,580.
Budgeted overhead absorption rate ¼
budgeted fixed overhead expenditure
budgeted direct labour hours
£5 ¼
£212,900
budgeted direct labour hours
Budgeted direct labour hours ¼ £212,900=£5 ¼ 42,580:
£
Actual overheads 694,075
Less: absorbed overheads
32,150x
Difference ¼under-absorption
35,000
£
Absorbed: ð £150; 000=50; 000Þ¼£3=hour 60; 000 180,000
Actual incurred
180,000
Under-/over-absorption
Overhead cost item Total Machining Assembly Finishing
££ £ £
Maintenance cost 38,000 19,000 13,300 5,700
SOLUTIONS TO REVISION QUESTIONS C1
80
THE ANALYSIS OF OVERHEAD
2006.1
2.5 The most appropriate production overhead absorption rate for the packing
department would be a direct labour hour rate.
Although the number of machine hours in the cost centre is significant, we are
told that a quarter of the output is not placed on the machines. No machine hours
would be recorded for this output and the use of a machine hour rate would
mean that this part of the output received no charge for the overheads of the
packing cost centre.
2.6
Overhead absorption rate ¼
£210;000
42;000
¼ £5 per machine hour
2.7 Looking at the number of machine and labour hours budgeted for each cost centre
it is clear that the machining department is machine intensive. Therefore, a rate per
machine hour would be most appropriate for this cost centre.
The assembly and finishing departments are labour intensive. Therefore, a rate per
labour hour would be most appropriate for each of these cost centres.
2.8 Two of the stated factors could have contributed to the under absorption:
the actual production overhead incurred was higher than budgeted; if this did happen then
the predetermined absorption rate would be too low and there would be a
potential under absorption;
the actual machine hours were lower than budgeted; if this occurred then there would be
fewer than expected hours to absorb the production overhead, potentially leading
to under absorption.
2.9 Budgeted number of occupied room nights ¼400 rooms 365 nights 85% ¼
124,100 occupied room nights.
Overhead absorption rate for linen services activity ¼ £836,000/124,100 ¼£6.74
per occupied room night.
2.10
(a) The variable cost per unit of product R is £36.00 per unit.
Direct material £14 + direct labour £19 + variable overhead £3 ¼£36
(b) The total (full) cost of product R is £124.00 per unit.
Variable cost £36 + Fixed overhead (8 hours £11) ¼£124
(c) The selling price of product R which will achieve the specified return on
investment is £147.68 per unit.
Required return from investment in product R ¼£600,000 15% ¼£90,000
Required return per unit sold ¼£90,000/3,800 units ¼£23.68
Required selling price ¼£124.00 full cost + £23.68 ¼£147.68
Solution 3
This is an example of an application of absorption costing in a non-manufacturing
situation. Do not be put off by this. In an assessment you must be prepared to deal with
all sorts of unfamiliar situations. The principles of overhead analysis that you have
£
Overhead absorbed (£5 43,000) 215,000
Actual overhead incurred
230,000
Overhead under-absorbed
15,000
FUNDAMENTALS OF MANAGEMENT ACCOUNTING
81
THE ANALYSIS OF OVERHEAD
2006.1
learned in this chapter can be applied in the same way in this non-manufacturing
environment. Residential and Catering are the equivalent of the production cost centres
that you have learned about, whereas Housekeeping and Maintenance are internal
service departments whose costs will need to be reapportioned.
Maintenance does work for Housekeeping, but notice that Housekeeping does not
provide any service to Maintenance. Therefore, in part ( b), if you apportion the total of
Maintenance first, including the appropriate charge to Housekeeping, you can then
apportion the new total for Housekeeping straight to the departments which deal
directly with customers, that is, Residential and Catering.
(a) A £4,500
B £4,667
C £10,175
Workings:
A: Using floor area as the apportionment basis, the rent and rates cost apportioned to
Housekeeping ¼(600/5,000) £37,500 ¼£4,500.
B: (250,000/750,000) £14,000 ¼£4,667.
C: Using floor area as the apportionment basis, the heat and light cost apportioned to
Residential ¼(2,750/5,000) £ 18,500 ¼£10,175.
( b) Residential £135,318
Catering £91,682
Workings:
Solution 4
A common mistake in part ( b) would be to compare the actual overheads with the
budgeted overheads instead of with the absorbed overheads when calculating the
under- or over-absorption.
Residential Catering Housekeeping Maintenance Total
££ £ £ £
Initial allocation and appt. 85,333 68,287 50,370 23,010
227,000
Maintenance reapportioned
50% to Residential 11,505
30% to Catering 6,903
20% to Housekeeping
4,602 (23,010)
96,838 75,190 54,972
Housekeeping reapportioned
70% to Residential 38,480
30% to Catering
16,492 (54,972)
135,318 91,682
SOLUTIONS TO REVISION QUESTIONS C1
82
THE ANALYSIS OF OVERHEAD
2006.1
(a) Predetermined departmental overhead absorption rates for period 3 ( per production
hour)
( b) (i) The overheads in the Casting department were over-absorbed by £9,183
(ii) The overheads in the Dressing department were under-absorbed by £875.
Workings:
(c) Both factors would have contributed to the over absorption. The amount of overhead
absorbed increased in line with the production hours, which would have led to over
absorption even if the overhead expenditure had remained constant. The fact that the
overhead expenditure was below budget would have increased the amount of over
absorption.
Solution 5
You will need to use the repeated distribution method to deal with the reciprocal
servicing in part (a).
The question mentions job costing, which is the subject of Chapter 8. For now, all you
need to know is that an individual job in this case job XX34 is simply treated as a
cost unit for the purposes of overhead absorption.
(a) Machining: £691,500
Assembly: £299,500
Finishing: £189,000
Workings:
Casting Dressing Assembly
£225;000
7;500
¼ £30
£175;000
7;000
¼ £25
£93;000
6;200
¼ £15
Casting Dressing
££
Overheads absorbed:
£30/hour 7,950 238,500
£25/hour 7,280 182,000
Actual overheads
(229,317) (182,875)
Over-/(under)-absorption
9,183 (875)
Machining Assembly Finishing Stores Maintenance
£000 £000 £000 £000 £000
Allocated costs 600.00 250.00 150.00 100.00 80.00
Stores apportionment 40.00 30.00 20.00 (100.00) 10.00
Maintenance apportionment 49.50 18.00 18.00 4.50 (90.00)
Stores apportionment
2.00 1.50 1.00 (4.50)
Total
691.50 299.50 189.00
FUNDAMENTALS OF MANAGEMENT ACCOUNTING
83
THE ANALYSIS OF OVERHEAD
2006.1
( b) (i) £885.45
(ii) £4,785.45
(iii) £6,580.00
Workings:
* A profit margin of 20 per cent of selling price is the same as a mark-up of 25 per cent of
cost. Check for yourself that the calculated profit margin is in fact 20 per cent of the selling
price.
££
Direct material 2,400.00
Direct labour
1,500.00
Prime cost 3,900.00
Overhead cost:
Machining (45 £13.83) 622.35
Assembly (15 £9.98) 149.70
Finishing (12 £9.45)
113.40
885.45
Total production cost 4,785.45
Non-production overhead (10%)
478.55
Total cost 5,264.00
Profit mark-up (25%)*
1,316.00
Price for Job XX34
6,580.00
THE ANALYSIS OF OVERHEAD
2006.1
SOLUTIONS TO REVISION QUESTIONS C1
84
4
Cost–Volume–Profit Analysis
Cost–Volume–Profit
Analysis
4.1 Introduction
In this chapter you will see how an understanding of cost behaviour patterns and a
focus on identifying the costs that will alter as the result of a course of action
are important in providing effective information as the basis for management
decision-making.
4.2 Breakeven or cost–volume–profit analysis
Cost–volume–profit (CVP) analysis is defined in CIMA’s Terminology as the ‘study of the effects
on future profit of changes in fixed cost, variable cost, sales price, quantity and mix’.
A common term used for this type of analysis is breakeven analysis. However, this is
somewhat misleading, since it implies that the focus of the analysis is the breakeven point
that is, the level of activity which produces neither profit nor loss. You will see in this
chapter that the scope of CVP analysis is much wider than this, as indicated in the
definition. However, you should be aware that the terms ‘breakeven analysis’ and ‘CVP
analysis’ tend to be used interchangeably.
4
LEARNING OUTCOMES
After completing this chapter, you should be able to:
"
explain the concept of contribution and its use in cost–volume–profit (CVP) analysis;
"
calculate and interpret the breakeven point, profit target, margin of safety and
profit/volume ratio for a single product or service;
"
prepare breakeven charts and profit/volume graphs for a single product or
service;
"
calculate the profit maximising sales mix for a multi-product company that
has limited demand for each product and one other constraint or limiting
factor.
87 2006.1
4.2.1 The concept of contribution
In chapter 1 you learned that variable costs are those that vary with the level of activity. If
we can identify the variable costs associated with producing and selling a product or
service we can highlight a very important measure: contribution.
Contribution ¼ sales value variable costs
Variable costs are sometimes referred to as marginal costs and the two terms are
often used interchangeably
Contribution is so called because it literally does contribute towards fixed costs and profit.
Once the contribution from a product or service has been calculated, the fixed costs associated
with the product or service can be deducted to determine the profit for the period.
4.2.2 Calculating the breakeven point
As sales revenues grow from zero, the contribution also grows until it just covers the fixed
costs. This is the breakeven point where neither profits nor losses are made.
It follows that to break even the amount of contribution must exactly match the
amount of fixed costs. If we know how much contribution is earned from each unit sold,
then we can calculate the number of units required to break even as follows:
Breakeven point in units ¼
Fixed costs
Contribution per unit
For example, suppose that an organisation manufactures a single product, incurring
variable costs of £30 per unit and fixed costs of £20,000 per month. If the product sells
for £50 per unit, then the breakeven point can be calculated as follows:
Breakeven point in units ¼
£20,000
£50 £30
¼ 1,000 units per month
4.3 The margin of safety
The margin of safety is the difference between the expected level of sales and the
breakeven point. The larger the margin of safety, the more likely it is that a profit
will be made, that is, if sales start to fall there is more leeway before the organisation
begins to incur losses. (Obviously, this statement is made on the assumption that
projected sales volumes are above the breakeven point.)
In the above example, if forecast sales are 1,700 units per month, the margin of safety
can be easily calculated.
Margin of safety ¼ projected sales breakeven point
¼ 1,700 units 1,000 units
¼ 700 units per month, or 41% of sales ð700=1,700 100%Þ
STUDY MATERIAL C1
88
COST–VOLUME–PROFIT ANALYSIS
2006.1
The margin of safety should be expressed as a percentage of projected sales to put it in
perspective. To quote a margin of safety of 700 units without relating it to the projected
sales figure is not giving the full picture.
The margin of safety can also be used as one route to a profit calculation. We have
seen that the contribution goes towards fixed costs and profit. Once breakeven point
is reached the fixed costs have been covered. After the breakeven point there are
no more fixed costs to be covered and all of the contribution goes towards making
profits grow.
In our example the monthly profit from sales of 1,700 units would be £14,000.
Margin of safety ¼ 700 units per month
Monthly profit ¼ 700 contribution per unit
¼ 700 £20
¼ £14,000:
4.4 The contribution to sales (C/S) ratio
The contribution to sales ratio is usually expressed as a percentage. It can be calculated for
the product in our example as follows.
Contribution to sales ratio (C/S ratio) ¼ £20=£50 100%
¼ 40%
A higher contribution to sales ratio means that contribution grows more quickly as sales
levels increase. Once the breakeven point has been passed, profits will accumulate more
quickly than for a product with a lower contribution to sales ratio.
You might sometimes see this ratio referred to as the profit–volume (P/V) ratio.
If we can assume that a unit’s variable cost and selling price remain constant then the
C/S ratio will also remain constant. It can be used to calculate the breakeven point as
follows (using the data from the earlier example):
Breakeven point in sales value ¼
Fixed costs
C/S ratio
¼
£20,000
0:40
¼ £50,000
This can be converted to 1,000 units as before by dividing by the selling price of
£50 per unit.
Exercise 4.1
A company manufactures and sells a single product which has the following cost and
selling price structure:
£/unit £/unit
Selling price 120
Direct material 22
Direct labour 36
Variable overhead 14
Fixed overhead
12
84
Profit per unit
36
FUNDAMENTALS OF MANAGEMENT ACCOUNTING
89
COST–VOLUME–PROFIT ANALYSIS
2006.1
The fixed overhead absorption rate is based on the normal capacity of 2,000 units per
month. Assume that the same amount is spent each month on fixed overheads.
Budgeted sales for next month are 2,200 units.
You are required to calculate:
(i) the breakeven point, in sales units per month;
(ii) the margin of safety for next month;
(iii) the budgeted profit for next month;
(iv) the sales required to achieve a profit of £96,000 in a month.
Solution
(i) The key to calculating the breakeven point is to determine the contribution per unit.
Contribution per unit ¼ £120 ð£22 þ £36 þ £14Þ¼£48
Breakeven point ¼
Fixed overhead
Contribution per unit
¼
£12 2,000
£48
¼ 500 units
(ii)
Margin of safety ¼ budgeted sales breakeven point
¼ 2,200 500
¼ 1,700 units (or 1,700/2,200 100% ¼ 77% of budgeted sales)
(iii) Once breakeven point has been reached, all of the contribution goes towards profits
because all of the fixed costs have been covered.
Budgeted profit ¼ 1,700 units margin of safety £48 contribution per unit
¼ £81,600
(iv) To achieve the desired level of profit, sufficient units must be sold to earn a
contribution which covers the fixed costs and leaves the desired profit for the month.
Number of sales units required ¼
Fixed overhead þ desired profit
Contribution per unit
¼
ð£12 2,000Þþ£96,000
£48
¼ 2,500 units:
4.5 Drawing a basic breakeven chart
A basic breakeven chart records costs and revenues on the vertical axis and the level of
activityon thehorizontalaxis. Linesaredrawn onthe charttorepresent costs andsales revenue.
The breakeven point can be read off where the sales revenue line cuts the total cost line.
We will use our basic example to demonstrate how to draw a breakeven chart. The data is:
Selling price £50 per unit
Variable cost £30 per unit
Fixed costs £20,000 per month
Forecast sales 1,700 units per month
STUDY MATERIAL C1
90
COST–VOLUME–PROFIT ANALYSIS
2006.1
While you will not be required to draw a graph to scale in the assessment, you may
need to do so in your working life or in future examinations for other subjects.
Learning to draw a chart to scale will provide a firm foundation for your understanding
of breakeven charts. To give yourself some practice, it would be a good idea to follow the
step-by-step guide which follows to produce your own chart on a piece of graph paper.
Step 1. Select appropriate scales for the axes and draw and label them. Your graph should fill as much
of the page as possible. This will make it clearer and easier to read. You can make sure that
you do this by putting the extremes of the axes right at the end of the available space.
The furthest point on the vertical axis will be the monthly sales revenue, that is
1,700 units £50 ¼ £85,000
The furthest point on the horizontal axis will be monthly sales volume of 1,700 units.
Make sure that you do not need to read data for volumes higher than 1,700 units
before you set these extremes for your scales.
Step 2. Draw the fixed cost line and label it. This will be a straight line parallel to the
horizontal axis at the £20,000 level.
The £20,000 fixed costs are incurred in the short term even with zero activity.
Step 3. Draw the total cost line and label it. The best way to do this is to calculate the total
costs for the maximum sales level, which is 1,700 units in our example. Mark this point
on the graph and join it to the cost incurred at zero activity, that is, £20,000
Step 4. Draw the revenue line and label it. Once again, the best way is to plot the extreme points.
The revenue at maximum activity in our example is 1,700 £50 ¼£85,000. This point
can be joined to the origin, since at zero activity there will be no sales revenue.
Step 5. Mark any required information on the chart and read off solutions as required. Check that
your chart is accurate by reading off the measures that we have already calculated in this
chapter: the breakeven point, the margin of safety, the profit for sales of 1,700 units.
The completed graph is shown in Figure 4.1.
£
Variable costs for 1,700 units
(1,700 £30)
51,000
Fixed costs
20,000
Total cost for 1,700 units
71,000
90
£000
80
70
60
50
40
30
20
10
0
0 400 800 1,200
Margin of safety
Fixed cost
PROFIT
LOSS
Total cost
Sales revenue
Variable
cost
Breakeven point
Number of units
1,600
Figure 4.1 Basic breakeven chart
FUNDAMENTALS OF MANAGEMENT ACCOUNTING
91
COST–VOLUME–PROFIT ANALYSIS
2006.1
Your own graph should be considerably larger than this: a full A4 graph-ruled sheet
is recommended to facilitate ease of interpretation.
4.6 The contribution breakeven chart
One of the problems with the conventional or basic breakeven chart is that it is not
possible to read contribution directly from the chart. A contribution breakeven chart is
based on the same principles but it shows the variable cost line instead of the fixed cost
line ( Figure 4.2). The same lines for total cost and sales revenue are shown so the
breakeven point and profit can be read off in the same way as with a conventional chart.
However, it is possible also to read the contribution for any level of activity.
Using the same basic example as for the conventional chart, the total variable cost for
an output of 1,700 units is 1,700 £30 ¼£51,000. This point can be joined to the origin
since the variable cost is nil at zero activity.
The contribution can be read as the difference between the sales revenue line and the
variable cost line.
This form of presentation might be used when it is desirable to highlight the importance
of contribution and to focus attention on the variable costs.
4.7 The profit–volume chart
Another form of breakeven chart is the profit–volume chart. This chart plots a single
line depicting the profit or loss at each level of activity. The breakeven point is where this line
cuts the horizontal axis. A profit–volume graph for our example will look like Figure 4.3.
90
£000
80
70
60
50
40
30
20
10
0
0 400 800 1,200
Variable cost
PROFIT
LOSS
Total cost
Sales revenue
Fixed
cost
Contribution
Breakeven point
Number of units
1,600
PROFIT
LOSS
£000
15
10
5
0
–5
–10
–15
–20
400 800 1,200 1,600
Number of unit
s
Breakeven
point
Loss
Profit
Figure 4.3 Profit–volume chart
STUDY MATERIAL C1
92
COST–VOLUME–PROFIT ANALYSIS
2006.1
The vertical axis shows profits and losses and the horizontal axis is drawn at zero profit
or loss.
At zero activity the loss is equal to £20,000, that is, the amount of fixed costs. The
second point used to draw the line could be the calculated breakeven point or the
calculated profit for sales of 1,700 units.
The profit–volume graph is also called a profit graph or a contribution–volume
graph.
Exercise 4.2
Make sure that you are clear about the extremes of the profit–volume chart axes. Practise
drawing chart to scale on a piece of graph paper.
4.7.1 The advantage of the profit–volume chart
The main advantage of the profit–volume chart is that it is capable of depicting clearly the
effect on profit and breakeven point of any changes in the variables. An example will show
how this can be done.
Example
A company manufactures a single product which incurs fixed costs of £30,000 per annum. Annual sales are
budgeted to be 70,000 units at a sales price of £30 per unit. Variable costs are £28.50 per unit.
(a) Draw a profit–volume graph, and use it to determine the breakeven point.
The company is now considering improving the quality of the product and increasing the selling price
to £35 per unit. Sales volume will be unaffected, but fixed costs will increase to £45,000 per annum and
variable costs to £33 per unit.
(b) Draw, on the same graph as for part (a), a second profit–volume graph and comment on the results.
Solution
The profit–volume chart is shown in Figure 4.4.
The two lines have been drawn as follows:
Situation (a). The profit for sales of 70,000 units is £75,000.
This point is joined to the loss at zero activity, £30,000, that is, the fixed costs.
Situation (b). The profit for sales of 70,000 units is £95,000.
This point is joined to the loss at zero activity, £45,000, that is, the fixed costs.
£000
Contribution 70,000 £(30 28.50) 105
Fixed costs
30
Profit
75
£000
Contribution 70,000 £(35 33) 140
Fixed costs
45
Profit
95
FUNDAMENTALS OF MANAGEMENT ACCOUNTING
93
COST–VOLUME–PROFIT ANALYSIS
2006.1
Comment on the results. The graph depicts clearly the larger profits available from option (b). It also
shows that the breakeven point increases from 20,000 units to 22,500 units but that this is not a large
increase when viewed in the context of the projected sales volume. It is also possible to see that for sales
volumes above 30,000 units the profit achieved will be higher with option (b). For sales volumes below
30,000 units option (a) will yield higher profits (or lower losses).
The profit–volume graph is the clearest way of presenting information like this. If we attempted to draw two
conventional breakeven charts on one set of axes the result would be a jumble, which is very difficult to interpret.
4.8 The limitations of breakeven (or CVP) analysis
The limitations of the practical applicability of breakeven analysis and breakeven charts
stem mostly from the assumptions which underlie the analysis:
(a) Costs are assumed to behave in a linear fashion. Unit variable costs are assumed to
remain constant and fixed costs are assumed to be unaffected by changes in activity
levels. The charts can in fact be adjusted to cope with non-linear variable costs or steps
in fixed costs but too many changes in behaviour patterns can make the charts very
cluttered and difficult to use.
(b) Sales revenues are assumed to be constant for each unit sold. This may be unrealistic
because of the necessity to reduce the selling price to achieve higher sales volumes.
Once again the analysis can be adapted for some changes in selling price but too many
changes can make the charts unwieldy.
(c) It is assumed that activity is the only factor affecting costs, and factors such as inflation
are ignored. This is one of the reasons why the analysis is limited to being essentially a
short-term decision aid.
(d) Apart from the unrealistic situation of a constant product mix, the charts can only be
applied to a single product or service. Not many organisations have a single product or
service and if there is more than one, then the apportionment of fixed costs between
them becomes arbitrary.
£000
20
30
40
50
60
70
80
90
100
10
20
30
40
50
60
70
10
0
–10
–20
–30
–40
–50
Number of units (000)
Breakeven
point (a)
Situation (b)
Situation (a)
Breakeven
point (b)
Loss
Profit
Figure 4.4 Showing changes with a profit–volume chart
STUDY MATERIAL C1
94
COST–VOLUME–PROFIT ANALYSIS
2006.1
(e) The analysis seems to suggest that as long as the activity level is above the breakeven
point, then a profit will be achieved. In reality certain changes in the cost and revenue
patterns may result in a second breakeven point after which losses are made. This
situation will be depicted in the next section of this chapter.
4.9 The economist’s breakeven chart
An economist would probably depict a breakeven chart as shown in Figure 4.5.
The total cost line is not a straight line which climbs steadily as in the accountant’s
chart. Instead it begins to reduce initially as output increases because of the effect of
economies of scale. Later it begins to climb upwards according to the law of diminishing
returns.
The revenue line is not a straight line as in the accountant’s chart. The line becomes less
steep to depict the need to give discounts to achieve higher sales volumes.
However, you will see that within the middle range the economist’s chart does look very
similar to the accountant’s breakeven chart. This area is marked as the relevant range in
Figure 4.5.
For this reason, it is unreliable to assume that the cost–volume–profit relationships
depicted in breakeven analysis are relevant across a wide range of activity. In particular,
Figure 4.5 shows that the constant cost and price assumptions are likely to be unreliable at
very high or very low levels of activity. Managers should therefore ensure that they work
within the relevant range, that is, within the range over which the depicted cost and
revenue relationships are more reliable.
You may recall that we discussed the relevant range in the context of cost
behaviour patterns in Chapter 1.
Relevant
range
Revenue
Total cost
Breakeven
point (2)
Breakeven
point (1)
Activity leve
l
£000
Figure 4.5 The economist’s breakeven chart
FUNDAMENTALS OF MANAGEMENT ACCOUNTING
95
COST–VOLUME–PROFIT ANALYSIS
2006.1
4.10 Using CVP analysis to evaluate proposals
Use your understanding of breakeven analysis and cost behaviour patterns to evaluate the
proposals in the following exercise.
Exercise 4.3
A summary of a manufacturing company’s budgeted profit statement for its next financial
year, when it expects to be operating at 75 per cent of capacity, is given below.
It has been estimated that:
(i) if the selling price per unit were reduced to £28, the increased demand would utilise
90 per cent of the company’s capacity without any additional advertising expenditure;
(ii) to attract sufficient demand to utilise full capacity would require a 15 per cent
reduction in the current selling price and a £5,000 special advertising campaign.
You are required to:
(a) calculate the breakeven point in units, based on the original budget;
(b) calculate the profits and breakeven points which would result from each of the two
alternatives and compare them with the original budget.
££
Sales 9,000 units at £32 288,000
Less:
direct materials 54,000
direct wages 72,000
production overhead fixed 42,000
variable
18,000
186,000
Gross profit 102,000
Less: admin., selling and dist’n costs:
fixed 36,000
varying with sales volume
27,000
63,000
Net profit 39,000
STUDY MATERIAL C1
96
COST–VOLUME–PROFIT ANALYSIS
2006.1
Solution
(a) First calculate the current contribution per unit.
Now you can use the formula to calculate the breakeven point.
Breakeven point ¼
Fixed costs
Contribution per unit
¼
£42,000 þ£36,000
£13
¼ 6,000 units
(b) Alternative (i)
Alternative (ii)
Neither of the two alternative proposals is worthwhile. They both result in lower
forecast profits. In addition, they will both increase the breakeven point and will therefore
increase the risk associated with the company’s operations.
This exercise has shown you how an understanding of cost behaviour patterns and the
manipulation of contribution can enable the rapid evaluation of the financial effects of
£000 £000
Sales revenue 288
Direct materials 54
Direct wages 72
Variable production overhead 18
Variable administration etc.
27
171
Contribution
117
Contribution per unit (9,000 units)
£13
Budgeted contribution per unit £13
Reduction in selling price (£32 £28)
£4
Revised contribution per unit
£9
Revised breakeven point ¼ £78,000=£9 8,667 units
Revised sales volume ¼9,000 (90/75) 10,800 units
Revised contribution ¼10,800 £9 £97,200
Less fixed costs
£78,000
Revised profit
£19,200
Budgeted contribution per unit £13.00
Reduction in selling price (15% £32)
£4.80
Revised contribution per unit
£8.20
Revised breakeven point ¼
£78,000 þ£5,000
£8:20
10,122 units
Revised sales volume ¼9,000 units (100/75) 12,000 units
Revised contribution ¼12,000 £8.20 £98,400
Less fixed costs
£83,000
Revised profit
£15,400
FUNDAMENTALS OF MANAGEMENT ACCOUNTING
97
COST–VOLUME–PROFIT ANALYSIS
2006.1
a proposal. We can now expand it to demonstrate another aspect of the application of
CVP analysis to short-term decision-making.
Exercise 4.4
The manufacturing company decided to proceed with the original budget and has asked
you to determine how many units must be sold to achieve a profit of £45,500.
Solution
Once again, the key is the required contribution. This time the contribution must be sufficient
to cover both the fixed costs and the required profit. If we then divide this amount by the
contribution earned from each unit, we can determine the required sales volume.
Required sales ¼
Fixed costs þrequired profit
Contribution per unit
¼
ð£42,000 þ£36,000 þ £45,500Þ
£13
¼ 9,500 units
4.11 Limiting factor analysis
A limiting factor is any factor which is in scarce supply and which stops the
organisation from expanding its activities further, that is, it limits the organisation’s
activities.
The limiting factor for many trading organisations is sales volume because they
cannot sell as much as they would like. However, other factors may also be limited,
especially in the short term. For example, machine capacity or the supply of skilled
labour may be limited for one or two periods until some action can be taken to alleviate
the shortage.
The concept of contribution can be used to make decisions about the best use of a limited
resource.
4.11.1 Decisions involving a single limiting factor
If an organisation is faced with a single limiting factor, for example machine capacity, then
it must ensure that a production plan is established which maximises the profit from the
use of the available capacity. Assuming that fixed costs remain constant, this is the same as
saying that the contribution must be maximised from the use of the available capacity. The
machine capacity must be allocated to those products which earn the most contribution
per machine hour.
This decision rule can be stated as ‘maximising the contribution per unit of limiting
factor’.
STUDY MATERIAL C1
98
COST–VOLUME–PROFIT ANALYSIS
2006.1
Example
LMN Ltd manufactures three products L, M and N. The company which supplies the two raw materials
which are used in all three products has informed LMN that their employees are refusing to work
overtime. This means that supply of the materials is limited to the following quantities for the next period:
No other source of supply can be found for the next period.
Information relating to the three products manufactured by LMN Ltd is as follows:
Owing to the perishable nature of the products, no finished goods are held.
Requirements
(a) Recommend a production mix which will maximise the profits of LMN Ltd for the forthcoming period.
(b) LMN Ltd has a valued customer to whom they wish to guarantee the supply of 50 units of each product
next period. Would this alter your recommended production plan?
Solution
(a) The first step is to check whether the supply of each material is adequate or whether either or both of
them represent a limiting factor.
There will be sufficient material A to satisfy the maximum demand for the products but material B
will be a limiting factor.
The next step is to rank the products in order of their contribution per unit of limiting factor. The
available material B can then be allocated according to this ranking.
The available material B will be allocated to the products according to this ranking, to give the
optimum production plan for the next period.
Material A 1,030 kg
Material B 1,220 kg
LM N
Quantity of material used per unit manufactured:
Material A (kg) 2 1 4
Material B (kg) 5 3 7
Maximum sales demand (units) 120 160 110
Contribution per unit sold £15 £12 £17.50
L M N Total
Maximum sales demand (units) 120 160 110
Material A required per unit (kg) 2 1 4
Total material A required (kg) 240 160 440 840
Material B required per unit (kg) 5 3 7
Total material B required (kg) 600 480 770 1,850
LM N
Contribution per unit sold £15 £12 £17.50
Material B consumed (kg) 5 3 7
Contribution per kg of material B £3 £4 £2.50
Ranking 2 1 3
FUNDAMENTALS OF MANAGEMENT ACCOUNTING
99
COST–VOLUME–PROFIT ANALYSIS
2006.1
The available material B is allocated to satisfy the maximum market demand for products
M and L. The balance of available material is allocated to the last product in the ranking,
product N.
(b) The recommended production plan in part (a) does not include sufficient product N to satisfy the
requirement of 50 units for the valued customer. Some of the material allocated to product L (second in
the ranking) must be allocated to product N. The recommended production plan will now be as follows:
This recommendation makes the best use of the available material B within the restriction of
the market requirements for each product.
The identification of a limiting factor and the ranking of products to maximise
contribution has been a favourite topic in the multiple-choice questions on past
papers. Make sure that you are well prepared in case this topic comes up in your
assessment.
Exercise 4.5
Gill Ltd manufactures three products E, F and G. The products are all finished on the
same machine. This is the only mechanised part of the process. During the next period the
production manager is planning an essential major maintenance overhaul of the machine.
This will restrict the available machine hours to 1,400 hours for the next period. Data for
the three products are:
No inventories are held.
Fixed production costs are absorbed using a machine hour rate of £2 per machine hour.
Product Recommended
production (units)
Material B
utilised (kg)
M 160 (maximum) 480
L 120 (maximum) 600
N20
140 (balance)
1,220
Product Recommended
production (units)
Material B
utilised (kg)
N 50 350
M 160 480
L78
0,390 (balance)
1,220
Product E Product F Product G
£ per unit £ per unit £ per unit
Selling price 30 17 21.00
Variable cost 13 6 9.00
Fixed production cost 10 8 6.00
Other fixed cost
2 1 3.50
Profit
5 2 2.50
Maximum demand (units/period)
250 140 130
STUDY MATERIAL C1
100
COST–VOLUME–PROFIT ANALYSIS
2006.1
You are required to determine the production plan that will maximise profit for the
forthcoming period.
Solution
The first step is to calculate how many machine hours are required for each product. We
can then determine whether machine hours are really a limiting factor.
Since 2,200 machine hours are required and only 1,400 hours are available, machine
hours are a limiting factor.
The optimum production plan is the plan which maximises the contribution from the
limiting factor.
Do not make the common mistake of allocating the available hours according to the
profit per unit of product or according to the profit per hour.
The next step is to calculate the contribution per hour from each of the products.
The available hours can be allocated according to this ranking.
Product E Product F Product G Total
Fixed production cost per unit
@ £2 per hour
£10 £8 £6
Machine hours per unit 5 4 3
Maximum demand (units) 250 140 130
Maximum hours required 1,250 560 390 2,200
Product E Product F Product G
£££
Selling price per unit 30 17 21
Variable cost per unit
13 6 9
Contribution per unit
17 11 12
Machine hours per unit 5 4 3
Contribution per hour £3.40 £2.75 £4.00
Ranking 2 3 1
Units to be produced Machine hours required
Product G (maximum demand) 130 390
Product E (balance of hours) 202
1,010
1,400
FUNDAMENTALS OF MANAGEMENT ACCOUNTING
101
COST–VOLUME–PROFIT ANALYSIS
2006.1
4.12 Summary
Having read this chapter the main points that you should understand are as follows.
1. Cost–volume–profit (CVP) analysis is the study of the effect on profit of changes in
costs and sales price, quantity and mix. Another common term used in this context is
breakeven analysis.
2. Contribution is calculated as sales value minus variable cost.
3. The ratio of a cost unit’s contribution to its selling price is usually assumed to be
constant. This ratio may be referred to as the contribution to sales (C/S) ratio or the
profit–volume (P/ V) ratio, both of which are usually expressed as a percentage.
4. The breakeven point can be calculated as (fixed costs/contribution per unit) or (fixed
costs/PV ratio).
5. The margin of safety is the difference between the expected level of sales and the
breakeven point. It may be expressed as a percentage of the expected sales.
6. Contribution required to achieve a target profit ¼fixed costs + target profit.
7. A breakeven chart is a pictorial representation of costs and revenues depicting the
profit or loss for the relevant range of activity.
8. A contribution breakeven chart shows the variable cost line instead of the fixed cost
line, so that contribution can be read directly from the chart.
9. A profit–volume (PV) chart depicts a single line indicating the profit or loss for the
relevant range of activity. It is particularly useful for demonstrating the effect on profit
of changes in costs or revenues.
10. Breakeven or CVP analysis has a number of limitations and managers should be aware
of these if they are to apply the technique effectively.
11. A limiting factor is any factor which is in scarce supply and stops the organisation
from expanding its activities further. The decision rule in this situation is to maximise
the contribution per unit of limiting factor.
COST–VOLUME–PROFIT ANALYSIS
2006.1
STUDY MATERIAL C1
102
Revision Questions
Question 1 Multiple choice
1.1 A Ltd has fixed costs of £60,000 per annum. It manufactures a single product which
it sells for £20 per unit. Its contribution to sales ratio is 40 per cent.
A Ltd’s breakeven point in units is:
(A) 1,200
(B) 3,000
(C) 5,000
(D) 7,500.
1.2 B Ltd manufactures a single product which it sells for £9 per unit. Fixed costs are
£54,000 per month and the product has a variable cost of £6 per unit.
In a period when actual sales were £180,000, B Ltd’s margin of safety, in units, was:
(A) 2,000
(B) 14,000
(C) 18,000
(D) 20,000.
1.3 For the forthcoming year, E plc’s variable costs are budgeted to be 60 per cent of
sales value and fixed costs are budgeted to be 10 per cent of sales value.
If E plc increases its selling prices by 10 per cent, but if fixed costs, variable costs
per unit and sales volume remain unchanged, the effect on E plc’s contribution
would be:
(A) a decrease of 2 per cent.
(B) an increase of 5 per cent.
(C) an increase of 10 per cent.
(D) an increase of 25 per cent.
1.4 An organisation currently provides a single service. The cost per unit of that service
is as follows:
£
Selling price
130
Direct materials 22
Direct labour 15
Direct expenses 3
Variable overheads
10
Total variable cost
50
4
103 2006.1
Total fixed costs for the period amount to £1,600,000. How many units of service
(to the nearest whole unit) will the organisation need to provide to customers to
generate a profit of £250,000?
(A) 20,000
(B) 20,555
(C) 23,125
(D) 26,428.
1.5 P Ltd provides plumbing services. Due to a shortage of skilled labour next period the
company is unable to commence all the plumbing jobs for which customers have
accepted estimates.
When deciding which plumbing jobs should be commenced, the jobs should be
ranked according to the:
(A) Contribution to be earned from each job.
(B) Profit to be earned from each job.
(C) Contribution to be earned per hour of skilled labour on each job.
(D) Profit to be earned per hour of skilled labour on each job.
1.6 Z Ltd manufactures three products, the selling price and cost details of which are
given below:
In a period when direct materials are restricted in supply, the most and the least
profitable uses of direct materials are:
Question 2 Short objective-test questions
2.1 OT Ltd plans to produce and sell 4,000 units of product C each month, at a selling
price of £18 per unit. The unit cost of product C is as follows:
Most profitable Least profitable
(A) X Z
(B) Y Z
(C) Z Y
(D) Y X
£ per unit
Variable cost 8
Fixed cost
4
12
Product X Product Y Product Z
£££
Selling price per unit 75 95 95
Costs per unit:
Direct materials (£5/kg) 10 5 15
Direct labour (£4/hour) 16 24 20
Variable overhead 8 12 10
Fixed overhead 24 36 30
REVISION QUESTIONS C1
104
COST–VOLUME–PROFIT ANALYSIS
2006.1
To the nearest whole number, the monthly margin of safety, as a percentage of
planned sales is
%.
2.2 The P/ V ratio is the ratio of profit generated to the volume of sales.
True &
False &
2.3 Product J generates a contribution to sales ratio of 30 per cent. Fixed costs directly
attributable to product J amount to £75,000 per month. The sales revenue required
to achieve a monthly profit of £15,000 is £
.
2.4 Match the following terms with the labels a to d on the graph. Write a, b, c or d in
the relevant boxes.
& Margin of safety
& Fixed cost
& Contribution
& Profit
2.5 Select true or false for each of the following statements about a profit–volume chart.
(a) The profit line passes through the origin.
True &
False &
(b) Other things being equal, the angle of the profit line becomes steeper when the
selling price increases.
True &
False &
(c) Contribution cannot be read directly from the chart.
True &
False &
(d) The point where the profit line crosses the vertical axis is the breakeven point.
True &
False &
(e) Fixed costs are shown as a line parallel to the horizontal axis.
True &
False &
£
0
0
a
c
d
b
Sales
Variable cost
Total cost
Units
FUNDAMENTALS OF MANAGEMENT ACCOUNTING
105
COST–VOLUME–PROFIT ANALYSIS
2006.1
(f) The angle of the profit line is directly affected by the P/V ratio.
True &
False &
2.6 PH Ltd has spare capacity in its factory. A supermarket chain has offered to buy a number
of units of product XZ each month, and this would utilise the spare capacity. The
supermarket is offering a price of £8 per unit and the cost structure of XZ is as follows:
Fixed costs would not be affected.
On a purely financial basis, should the supermarket’s offer be accepted or rejected?
Accept the offer &
Reject the offer &
2.7 The following tasks are undertaken when deciding on the optimum production plan
when a limiting factor exists. Write 1, 2, 3 or 4 in the boxes to indicate the correct
sequence of tasks.
& Rank the products according to the contribution per unit of limiting factor used.
& Calculate each product’s contribution per unit of limiting factor used.
& Identify the limiting factor.
& Allocate the limited resource according to the ranking.
Question 3 Profit statements and breakeven analysis
BSE Veterinary Services is a specialist laboratory carrying out tests on cattle to ascertain
whether the cattle have any infection. At present, the laboratory carries out 12,000 tests
each period but, because of current difficulties with the beef herd, demand is expected to
increase to 18,000 tests a period, which would require an additional shift to be worked.
The current cost of carrying out a full test is:
Working the additional shift would:
(i) require a shift premium of 50 per cent to be paid to the technicians on the additional shift;
(ii) enable a quantity discount of 20 per cent to be obtained for all materials if an
order was placed to cover 18,000 tests;
(iii) increase fixed costs by £700,000 per period.
The current fee per test is £300.
£ per unit
Direct material 3
Direct labour 2
Variable overhead 1
Fixed overhead
3
9
£ per test
Materials 115
Technicians’ wages 30
Variable overhead 12
Fixed overhead 50
COST–VOLUME–PROFIT ANALYSIS
REVISION QUESTIONS C1
106
2006.1
Requirements
(a) The profit for the period at the current capacity of 12,000 tests is £
.
(b) A framework for a profit statement if the additional shift was worked and 18,000 tests
were carried out is as follows (complete the boxes to derive the period profit):
(c) It has been determined that for a capacity of 15,000 tests per period, the test
fee would be £300. Variable costs per test would amount to £140, and period
fixed costs would be £1,200,000. The breakeven number of tests at this capacity
level is
tests.
Question 4 Profit–volume graphs
MC Limited manufactures one product only, and for the last accounting period has
produced the simplified profit and loss statement below:
Requirements
(a) A profit–volume graph is to be drawn for MC Ltd’s product.
(i) The profit line drawn on the graph would cut the vertical axis (y-axis) at the point
where y is equal to £
.
(ii) The profit line drawn on the graph would cut the horizontal axis (x-axis) at the
point where x is equal to £
.
(iii) The margin of safety indicated by the graph would be £
.
(b) The effect of various changes in variables is to be indicated separately on the profit–
volume graph. For each change, indicate whether the angle of the profit line and the
breakeven point will increase, decrease or remain unchanged.
£000
(i) Sales
(ii) Direct materials
(iii) Direct labour
(iv) Variable overhead
(v) Fixed costs
(vi) Profit
££
Sales 300,000
Costs:
Direct materials 60,000
Direct wages
40,000
Prime cost 100,000
Variable production overhead 10,000
Fixed production overhead 40,000
Fixed administration overhead 60,000
Variable selling overhead 40,000
Fixed selling overhead
20,000
270,000
Net profit
30,000
FUNDAMENTALS OF MANAGEMENT ACCOUNTING
107
COST–VOLUME–PROFIT ANALYSIS
2006.1
Question 5 Breakeven charts
The following data is available concerning HF Ltd’s single service Q.
1,000 hours of service Q are provided to customers each month.
Requirements
The management accountant of HF Ltd has prepared the above contribution breakeven
chart for service Q:
Variable changed The angle of the profit line will:
Increase Decrease Remain unchanged
(i) Increase in selling price && &
(ii) Increase in fixed cost && &
(iii) Decrease in variable cost per unit && &
The breakeven point will:
Increase Decrease Remain unchanged
(i) Increase in selling price && &
(ii) Increase in fixed cost && &
(iii) Decrease in variable cost per unit && &
£ per hour of service £ per hour of service
Selling price 50
Variable cost
Direct material 7
Direct labour 8
Variable overhead
5
20
Contribution 30
Fixed overhead
15
Profit
15
£000
A
B
0
0
1,000
E
C
D
Sales revenue
Variable cost
Total cost
Hours of service per month
REVISION QUESTIONS C1
108
COST–VOLUME–PROFIT ANALYSIS
2006.1
The values or quantities indicated by A to E on the chart are:
D hours
E
hours
Question 6 Decision-making, limiting factor
ABC Ltd makes three products, all of which use the same machine, which is available for
50,000 hours per period.
The standard costs of the product, per unit, are:
Fixed costs are £300,000 per period.
Requirements
(a) The deficiency in machine hours for the next period is
hours.
(b) The optimum production plan that will maximise ABC Ltd’s profit for the next
period is:
Product A
units
Product B
units
Product C
units.
Product A Product B Product C
£££
Direct materials 70 40 80
Direct labour:
Machinists (£8/hour) 48 32 56
Assemblers (£6/hour)
36 40 42
Total variable cost
154 112 178
Selling price per unit 200 158 224
Maximum demand (units) 3,000 2,500 5,000
FUNDAMENTALS OF MANAGEMENT ACCOUNTING
109
COST–VOLUME–PROFIT ANALYSIS
2006.1
Solutions to
Revision Questions
Solution 1
Question 1.3 is quite tricky. Try setting up a table of the selling price, variable cost and
contribution before and after the change, perhaps using a selling price of £100.
Remember that fixed costs are not relevant because they do not affect contribution.
1.1 Answer: (D)
Contribution per unit ¼ 40% of selling price ¼ £8
Breakeven point ¼
£60,000
£8
¼ 7,500 units
1.2 Answer: (A)
1.3 Answer: ( D)
Fixed costs are not relevant because they do not affect contribution. Taking a selling
price of, say, £100 per unit, the cost structures will look like this:
Contribution per unit increases by 25 per cent. If sales volume remains unchanged
then total contribution will also increase by 25 per cent.
Before change
£ per unit
After change
£ per unit
Sales price 100 + 10% 110
Variable cost
60 60
Contribution
40 50
4
Contribution per unit ¼£9 £6 ¼£3
Breakeven point ¼
Fixed costs
Contribution per unit
¼
£54,000
£3
¼ 18,000 units
Margin of safety ¼Actual sales breakeven sales ¼
£180,000
£9
18,000 ¼2,000 units
111 2006.1
1.4 Answer: (C)
£1,600,000 þ£250,000
£80
¼ 23,125 units
Working:
1.5 Answer: (C)
The decision rule in a limiting factor situation is to maximise the contribution
per unit of limiting factor.
1.6 Answer: ( B)
Solution 2
2.1
Monthly fixed costs ¼ 4,000 units £4 ¼ £16,000:
Breakeven point ¼
Fixed costs
Contribution per unit
¼
£16,000
£18 £8
¼ 1,600 units
Margin of safety % ¼
Planned sales breakeven sales
Planned sales
100%
¼
4,000 1,600
4,000
100% ¼ 60%:
2.2 False. The P/V ratio is another term for the C/S ratio. It measures the ratio of the
contribution to sales.
2.3
Required sales revenue ¼
Required contribution
C/S ratio
¼
£75,000 þ£15,000
0:30
¼ £300,000:
2.4 c Margin of safety
a Fixed cost
b Contribution
d Profit.
Contribution per unit £
Selling price 130
Variable cost
(50)
Contribution/unit
80
Product X Y Z
Contribution/unit £41 £54 £50
Materials ( kg/unit) 2 1 3
Contribution/kg £20.50 £54 £16.66
Ranking 2 1 3
SOLUTIONS TO REVISION QUESTIONS C1
112
COST–VOLUME–PROFIT ANALYSIS
2006.1
2.5 (a) False. The profit line passes through the breakeven point on the horizontal axis,
and cuts the vertical axis at the point where the loss is equal to the fixed costs.
(b) True. Profits increase at a faster rate if the selling price is higher.
(c) True. A contribution breakeven chart is needed for this.
(d) False. The breakeven point is where the profit line cuts the horizontal axis.
(e) False. No fixed cost line is shown on a profit–volume chart.
(f) True. The higher the P/V ratio or contribution to sales ratio, the higher will be the
contribution earned per £ of sale and the steeper will be the angle of the profit
line.
2.6 Accept the offer. On a purely financial basis, the price of £8 per unit exceeds the
variable cost of £6 per unit. Since the fixed cost would not be affected, the units sold
to the supermarket will each earn a contribution of £2.
2.7 1. Identify the limiting factor.
2. Calculate each product’s contribution per unit of limiting factor used.
3. Rank the products according to the contribution per unit of limiting factor
used.
4. Allocate the limited resource according to the ranking.
Solution 3
In part ( b) do not be tempted to use unit rates to calculate the new level of fixed
costs. The current level of fixed costs is £600,000 per period. This will increase by
£700,000.
Also in part ( b), notice that the shift premium applies only to the technicians working
on the additional shift. It does not apply to all technicians’ wages.
(a) £1,116,000
Workings: profit statement for current 12,000 capacity
£000
Sales 12,000 tests @ £300/test 3,600
Direct materials 12,000 tests @ £115/test (1,380)
Direct labour 12,000 tests @ £30/test (360)
Variable overhead 12,000 tests @ £12/test
(144)
Contribution 1,716
Fixed costs 12,000 tests @ £50/test
(600)
Profit
1,116
FUNDAMENTALS OF MANAGEMENT ACCOUNTING
113
COST–VOLUME–PROFIT ANALYSIS
2006.1
(b) Profit statement for 18,000 capacity, with additional shift
(c)
Breakeven volume ¼
£1,200,000
ð£300 £140Þ
¼ 7,500 tests:
Solution 4
The profit line cuts the vertical axis at the point equal to the fixed costs, that is, the loss
when no sale is made.
The profit line cuts the horizontal axis at the breakeven point. Therefore, for (a)(ii) you
will need to calculate the breakeven point. For (a)(iii), the margin of safety is the
difference between £300,000 sales and the breakeven point.
(a) (i) £120,000
(ii) £240,000
(iii) £60,000.
Workings:
Contribution-to-sales ratio ¼
£ð300,000 100,000 10,000 40,000Þ
£300,000
100 ¼50%
Breakeven point ¼
Fixed costs
C/S ratio
¼
£ð40,000 þ60,000 þ20,000Þ
0:5
¼£240,000
Margin of safety ¼£ð300,000 240,000Þ¼£60,000
£000 £000
Sales 18,000 tests @ £300/test 5,400 (i)
Direct materials 18,000 tests @ £92/test (1,656) (ii)
Direct labour 12,000 tests @ £30/test (360)
6,000 tests @ £45/test
(270)
(630) (iii)
Variable overhead 18,000 tests @ £12/test
(216) (iv)
Contribution 2,898
Fixed costs
(1,300) (v)
Profit
1,598 (vi)
Margin of safety
£60,000
Breakeven point
£240,000
–£120,000
0
50
150
200
250
300
100
£30,000
Profit
£0
Sales revenue, £000
SOLUTIONS TO REVISION QUESTIONS C1
114
COST–VOLUME–PROFIT ANALYSIS
2006.1
(b)
Solution 5
Remember that a contribution breakeven chart shows the variable cost line instead of
the fixed cost line.
This means that contribution can be read directly from the chart, as the difference between
the sales value and the variable cost. This is the main advantage of the contribution
breakeven chart.
A £50,000 (1,000 hours £50 selling price)
B £15,000 (fixed cost at zero activity)
C £15,000 (profit for 1,000 hours see below)
D 500 hours (breakeven point see below)
E 500 hours (margin of safety (1,000 hours 500 hours breakeven))
Workings:
Breakeven point ¼
Fixed costs
Contribution per hour
¼
£15,000
£30
¼ 500 hours
Solution 6
In part ( b) remember to rank the products according to their contribution per machine
hour. Then allocate the available machine hours according to this ranking.
(a) The deficiency in machine hours for the next period is 13,000 hours.
( b) Product A 3,000 units
Product B 2,500 units
Product C 3,142 units
The angle of the
profit line will:
The breakeven
point will:
(i) Increase in selling price Increase Decrease
(ii) Increase in fixed cost Remain unchanged Increase
(iii) Decrease in variable cost per unit Increase Decrease
££
Sales value for 1,000 hours ¼1,000 £50 50,000
Total cost for 1,000 hours:
variable cost 1,000 £20 20,000
fixed cost 1,000 £15
15,000
35,000
Profit for 1,000 hours
15,000
FUNDAMENTALS OF MANAGEMENT ACCOUNTING
115
COST–VOLUME–PROFIT ANALYSIS
2006.1
Workings:
(a) Deficiency in machine hours for next period
(b)
Therefore, make:
Therefore, the company should make 3,142, that is, ð22,000=7Þ units of product C.
Product A Product B Product C Total
Machine hours required per unit 48=8 ¼ 632=8 ¼ 456=8 ¼ 7
Maximum demand (units) 3,000 2,500 5,000
Total machine hours to meet
maximum demand
18,000 10,000 35,000 63,000
Machine hours available
50,000
Deficiency of machine hours
13,000
Product A Product B Product C
£££
Selling price per unit 200 158 224
Variable cost per unit
(154) (112) (178)
Contribution per unit
46 46 46
Machine hours required per unit 6 4 7
Contribution per machine hour £7.67 £11.50 £6.57
Order of production 2 1 3
Machine hours
2,500 units of product B, using machine hours
of (4 2,500) 10,000
3,000 units of product A, using machine hours
of (6 3,000)
18,000
28,000
Machine hours left to make product C
22,000
50,000
SOLUTIONS TO REVISION QUESTIONS C1
116
COST–VOLUME–PROFIT ANALYSIS
2006.1
5
Standard Costing and
Variance Analysis
Standard Costing
and Variance
Analysis
5.1 Introduction
In this chapter we will be looking at standard costs: how they are set and how they are used
as the basis of variance analysis to monitor and control an organisation’s performance.
The CIMA Terminology defines standard costing as a ‘control technique that reports
variances by comparing actual costs to pre-set standards facilitating action through
management by exception’.
The pre-set standards require managers to plan in advance the amount and price of each
resource that will be used in providing a service or manufacturing a product. These pre-set
standards, for selling prices and sales volumes as well as for costs, provide a basis for
planning, a target for achievement and a benchmark against which the actual costs and
revenues can be compared.
The actual costs and revenues recorded after the event are then compared with the pre-
set standards and the differences are recorded as variances. If resource price or usage is
above standard, or if sales volume or selling price is below standard, an adverse variance
will result. If resource price or usage is below standard, or if sales volume or selling price is
above standard, a favourable variance will result.
5
LEARNING OUTCOMES
After completing this chapter, you should be able to:
"
explain the difference between ascertaining costs after the event and planning by
establishing standard costs in advance;
"
explain why planned standard costs, prices and volumes are useful in setting a
benchmark for comparison and so allowing managers’ attention to be directed to
areas of the business that are performing below or above expectation;
"
calculate standard costs for the material, labour and variable overhead elements of
cost of a product or service;
"
calculate variances for materials, labour, variable overhead, sales prices and sales
volumes.
119 2006.1
Careful analysis of the variances and their presentation to management can help to direct
managers’ attention to areas of the business that are performing below or above expectation.
If certain variances are large or significant then managers can concentrate their attention
on these activities where any corrective action is likely to be most worthwhile. If other
variances are small or not significant then managers can ignore these activities, knowing
that they appear to be conforming to expectations. This is the principle of management by
exception that is mentioned in CIMA’s definition of standard costing.
5.2 What is a standard cost?
A standard cost is a carefully predetermined unit cost which is prepared for each cost unit.
It contains details of the standard amount and price of each resource that will be utilised in
providing the service or manufacturing the product.
In order to be able to apply standard costing it must be possible to identify a measurable
cost unit. This can be a unit of product or service but it must be capable of standardising,
for example, standardised tasks must be involved in its creation. The cost units themselves
do not necessarily have to be identical. For example standard costing can be applied in
situations such as costing plumbing jobs for customers where every cost unit is unique.
However, the plumbing jobs must include standardised tasks for which a standard time
and cost can be determined for monitoring purposes.
It can be difficult to apply standard costing in some types of service organisation,
where cost units may not be standardised and they are more difficult to measure.
The standard cost may be stored on a standard cost card like the one shown below but
nowadays it is more likely to be stored on a computer, perhaps in a database. Alternatively
it may be stored as part of a spreadsheet so that it can be used in the calculation of
variances.
A standard cost card showing the variable elements of production cost might look
like this.
Standard cost card: product 176
For every variable cost the standard amount of resource to be used is stated, as well as
the standard price of the resource. This standard data provides the information for a
detailed variance analysis, as long as the actual data is collected in the same level of detail.
Standard costs and standard prices provide the basic unit information which is needed
for valuing budgets and for determining total expenditures and revenues.
£ per unit
Direct materials: 30 kg @ £4.30 129.00
Direct wages: 12 hours @ £11.80
141.60
Prime cost 270.60
Variable production overhead:
12 hours @ £0.75
9.00
Variable production cost
279.60
STUDY MATERIAL C1
120
STANDARD COSTING AND VARIANCE ANALYSIS
2006.1
Exercise 5.1
From the information given below, prepare a standard cost card extract for one unit and
enter on the standard cost card the costs to show subtotals for:
(a) prime cost;
(b) variable production cost.
The following data is given for the standard details for one unit:
Direct materials: 40 square metres @ £6.48/sq m
Direct wages:
Bonding department 48 hours @ £12.50/hour
Finishing department 30 hours @ £11.90/hour
Solution
Standard cost card extract
5.3 Performance levels
5.3.1 A standard
CIMA’s Terminology defines a standard as a ‘benchmark measurement of resource usage or
revenue or profit generation, set in defined conditions’. The definition goes on to describe
a number of bases which can be used to set the standard. These bases include:
a prior period level of performance by the same organisation;
the level of performance achieved by comparable organisations;
the level of performance required to meet organisational objectives.
Use of the first basis indicates that management feels that performance levels in a prior
period have been acceptable. They will then use this performance level as a target and
control level for the forthcoming period.
Budgeted costs and labour
hours per annum:
£ hours
Variable production overhead:
Bonding department 375,000 500,000
Finishing department 150,000 300,000
£ per unit
Direct materials: 40 sq m @ £6.48 259.20
Direct wages:
Bonding 48 hours @ £12.50 600.00
Finishing 30 hours @ £11.90
357.00
Prime cost 1,216.20
Variable production overhead:
Bonding 48 hours @ £0.75 36.00
Finishing 30 hours @ £0.50
15.00
Variable production cost
1,267.20
FUNDAMENTALS OF MANAGEMENT ACCOUNTING
121
STANDARD COSTING AND VARIANCE ANALYSIS
2006.1
When using the second basis management is being more outward looking,
perhaps attempting to monitor their organisation’s performance against ‘the best of the rest’.
The third basis sets a performance level which will be sufficient to achieve the
objectives which the organisation has set for itself.
5.3.2 Ideal standard
Standards may be set at ideal levels, which make no allowance for inefficiencies such as
losses, waste and machine downtime. This type of ideal standard is achievable only under
the most favourable conditions and can be used if managers wish to highlight and monitor
the full cost of factors such as waste, etc. However, this type of standard will almost always
result in adverse variances since a certain amount of waste, etc., is usually unavoidable.
This can be very demotivating for individuals who feel that an adverse variance suggests
that they have performed badly.
5.3.3 Attainable standard
Standards may also be set at attainable levels which assume efficient levels of operation,
but which include allowances for factors such as losses, waste and machine downtime.
This type of standard does not have the negative motivational impact that can arise with
an ideal standard because it makes some allowance for unavoidable inefficiencies. Adverse
variances will reveal whether inefficiencies have exceeded this unavoidable amount.
5.3.4 Current standard
Standards based on current performance levels (current wastage, current inefficiencies) are
known as current standards. Their disadvantage is that they do not encourage any attempt
to improve on current levels of efficiency.
5.4 Setting standard costs
You have already seen that each element of a unit’s standard cost has details of the price
and quantity of the resources to be used. In this section of the chapter we will list some of
the sources of information which may be used in setting the standard costs.
5.4.1 Standard material price
Sources of information include:
(a) quotations and estimates received from potential suppliers;
(b) trend information obtained from past data on material prices;
(c) details of any bulk discounts which may be available;
(d) information on any charges which will be made for packaging and carriage inwards;
(e) the quality of material to be used: this may affect the price to be paid;
(f ) for internally manufactured components, the predetermined standard cost for the
component will be used as the standard price.
STUDY MATERIAL C1
122
STANDARD COSTING AND VARIANCE ANALYSIS
2006.1
5.4.2 Standard material usage
Sources of information include:
(a) the basis to be used for the level of performance (see Section 5.3);
(b) if an attainable standard is to be used, the allowance to be made for losses, wastage,
etc. (work study techniques may be used to determine this);
(c) technical specifications of the material to be used.
5.4.3 Standard labour rate
Sources of information include:
(a) the personnel department, for the wage rates for employees of the required grades
with the required skills;
(b) forecasts of the likely outcome of any trades union negotiations currently in progress;
(c) details of any bonus schemes in operation. For example, employees may be paid a
bonus if higher levels of output are achieved.
5.4.4 Standard labour times
Sources of information include:
(a) the basis to be used for the level of performance (see Section 5.3);
(b) if an attainable standard is to be used, the allowance to be made for downtime, etc.;
(c) technical specifications of the tasks required to manufacture the product or provide
the service;
(d) the results of work study exercises which are set up to determine the standard time to
perform the required tasks and the grades of labour to be employed.
5.4.5 Variable production overhead costs
In Chapter 3 you learned how predetermined hourly rates were derived for production
overhead. These overhead absorption rates represent the standard hourly rates for
overhead in each cost centre. They can be applied to the standard labour hours or machine
hours for each cost unit.
The overheads will be analysed into their fixed and variable components so that a
separate rate is available for fixed production overhead and for variable production
overhead. This is necessary to achieve adequate control over the variable and fixed
elements. Your Fundamentals of Management Accounting syllabus requires you to deal only with
standard variable overhead costs.
5.5 Updating standards
The main purpose of standard costs is to provide a yardstick or benchmark against which
actual performance can be monitored. If the comparison between actual and standard cost
is to be meaningful, then the standard must be valid and relevant.
It follows that the standard cost should be kept as up to date as possible. This may
necessitate frequent updating of standards to ensure that they fairly represent
FUNDAMENTALS OF MANAGEMENT ACCOUNTING
123
STANDARD COSTING AND VARIANCE ANALYSIS
2006.1
the latest methods and operations, and the latest prices which must be paid for the resources
being used.
The standards may not be updated for every small change: however, any significant
changes should be adjusted as soon as possible.
5.6 Standard costing in the modern business
environment
There has recently been some criticism of the appropriateness of standard costing in the
modern business environment. The main criticisms include the following:
(a) Standard costing was developed when the business environment was more stable and
operating conditions were less prone to change. In the present dynamic environment,
such stable conditions cannot be assumed.
If conditions are not stable, then it is difficult to set a standard cost which can be
used to control costs over a period of time.
(b) Performance to standard used to be judged as satisfactory, but in today’s climate
constant improvement must be aimed for in order to remain competitive.
(c) The emphasis on labour variances is no longer appropriate with the increasing use of
automated production methods.
An organisation’s decision to use standard costing depends on its effectiveness in
helping managers to make the correct decisions. It can be used in areas of most
organisations, whether they are involved with manufacturing, or with services such as
hospitals or insurance. For example, a predetermined standard could be set for the labour
time to process an insurance claim. This would help in planning and controlling the cost
of processing insurance claims.
Standard costing may still be useful even where the final product or service is not
standardised. It may be possible to identify a number of standard components and
activities for which standards may be set and used effectively for planning and control
purposes. In addition, the use of demanding performance levels in standard costs may help
to encourage continuous improvement.
5.7 What is variance analysis?
You already know that a variance is the difference between the expected standard cost and
the actual cost incurred. You also know that a unit standard cost contains detail
concerning both the usage of resources and the price to be paid for the resources.
Variance analysis involves breaking down the total variance to explain how much of it is
caused by the usage of resources being different from the standard, and how much of it is
caused by the price of resources being different from the standard. These variances can be
combined to reconcile the total cost difference revealed by the comparison of the actual
and standard cost.
5.8 Variable cost variances
We will use a simple example to demonstrate how the variances are calculated for direct
material, direct labour and variable overhead.
STUDY MATERIAL C1
124
STANDARD COSTING AND VARIANCE ANALYSIS
2006.1
Example
A company manufactures a single product for which the standard variable cost is:
During January, 530 units were produced and the costs incurred were as follows:
You are required to calculate the variable cost variances for January.
5.8.1 Direct material cost variances
(a) Direct material total variance
You should always remember to indicate whether a variance is adverse or
favourable.
This direct material total variance can now be analysed into its ‘price’ and ‘quantity’
elements.
(b) Direct material price variance
The direct material price variance reveals how much of the direct material total variance
was caused by paying a different price for the materials used.
The adverse price variance indicates that expenditure was £8,569 more than standard
because a higher than standard price was paid for each kilogram of material.
£ per unit
Direct material: 81 kg£7 per kg 567
Direct labour: 97 hours£8 per hour 776
Variable overhead: 97 hours£3 per hour
291
1,634
Direct material: 42,845 kg purchased and used; cost £308,484
Direct labour: 51,380 hours worked; cost £400,764
Variable overhead: cost £156,709
£
530 units should cost (£567) 300,510
But did cost
308,484
Total direct material cost variance
7,974 adverse
£
42,845 kg purchased should have cost (£ 7) 299,915
But did cost
308,484
Direct material price variance
8,569 adverse
FUNDAMENTALS OF MANAGEMENT ACCOUNTING
125
STANDARD COSTING AND VARIANCE ANALYSIS
2006.1
(c) Direct material usage variance
The direct material usage variance reveals how much of the direct material total variance
was caused by using a different quantity of material, compared with the standard allowance
for the production achieved.
The favourable usage variance of £595 is the saving in material cost (at standard prices)
resulting from using a lower amount of material than the standard expected for this level
of output.
Check: £8,569 adverse + £595 favourable ¼£7,974 adverse (the correct total variance).
All of the ‘quantity’ variances are always valued at the standard price. Later in this
example you will see that the ‘quantity’ variances for labour and for variable
overhead the efficiency variances are valued at the standard rate per hour.
5.8.2 The direct material price variance and inventory
(stock) valuation
One slight complication sometimes arises with the calculation of the direct material price
variance. In this example the problem did not arise because the amount of material
purchased was equal to the amount used.
However, when the two amounts are not equal then the direct material price variance
could be based either on the material purchased or on the material used. In the example
we used the following method we will call it method A:
Method A Direct material price variance
Alternatively, we could have calculated the variance as follows we will call it method B.
Method B Direct material price variance
kg
530 units produced should have used (81 kg) 42,930
But did use
42,845
Variance in kg
85 favourable
standard price per kg (£7):
Direct material usage variance
£595 favourable
£
Material purchased should have cost X
But did cost
X
Direct material price variance
X
£
Material used should have cost X
But did cost
X
Direct material price variance
X
STUDY MATERIAL C1
126
STANDARD COSTING AND VARIANCE ANALYSIS
2006.1
Obviously, if the purchase quantity is different from the usage quantity, then the two
methods will give different results.
So how do you know which method to use? The answer lies in the inventory valuation
method.
If inventory is valued at standard cost, then method A is used. This will ensure that all
of the variance is eliminated as soon as purchases are made and the inventory will be held
at standard cost.
If inventory is valued at actual cost, then method B is used. This means that the variance
is calculated and eliminated on each bit of inventory as it is used up. The remainder of the
inventory will then be held at actual price, with its price variance still ‘attached’, until it is
used and the price variance is calculated.
If this seems confusing you might find it easier to return and consider the reasoning
after you have studied standard cost bookkeeping in chapter 7, when you will learn which
method is generally preferred.
5.8.3 Direct labour cost variances
(a) Direct labour total variance
This variance can now be analysed into its ‘price’ and ‘quantity’ elements. The ‘price’ part
is called the labour rate variance and the ‘quantity’ part is called the labour efficiency
variance.
(b) Direct labour rate variance
The direct labour rate variance reveals how much of the direct labour total variance was
caused by paying a different rate per hour for the labour hours worked.
The favourable rate variance indicates that expenditure was £10,276 less than standard
because a lower than standard rate was paid for each hour of labour.
Notice the similarity between the method used to calculate the labour rate variance
and the method used to calculate the material price variance.
£
530 units should cost (£776) 411,280
But did cost
400,764
Total direct labour cost variance
10,516 favourable
£
51,380 hours should have cost (£8) 411,040
But did cost
400,764
Direct labour rate variance
10,276 favourable
FUNDAMENTALS OF MANAGEMENT ACCOUNTING
127
STANDARD COSTING AND VARIANCE ANALYSIS
2006.1
(c) Direct labour efficiency variance
The direct labour efficiency variance reveals how much of the direct labour total variance
was caused by using a different number of hours of labour, compared with the standard
allowance for the production achieved.
The favourable efficiency variance of £240 is the saving in labour cost (at standard rates)
resulting from using fewer labour hours than the standard expected for this level of
output.
Check: £10,276 favourable + £240 favourable ¼£10,516 favourable (the correct total
variance).
In the next chapter you will see that a further analysis of the efficiency variance can
be carried out when idle time occurs.
5.8.4 Variable overhead cost variances
(a) Variable overhead total variance
This variance can now be analysed into its ‘price’ and ‘quantity’ elements. The ‘price’ part
is called the variable overhead expenditure variance and the ‘quantity’ part is called the
variable overhead efficiency variance.
(b) Variable overhead expenditure variance
The variable overhead expenditure variance reveals how much of the variable overhead
total variance was caused by paying a different hourly rate of overhead for the hours
worked.
The adverse expenditure variance indicates that expenditure was £2,569 more than
standard because a higher than standard hourly rate was paid for variable overhead.
Hours
530 units produced should take (97 hours) 51,410
But did take
51,380
Variance in hours
30 favourable
standard labour rate per hour (£8)
Direct labour efficiency variance
£240 favourable
£
530 units should cost (£291) 154,230
But did cost
156,709
Total variable overhead cost variance
2,479 adverse
£
51,380 hours of variable overhead should cost (£3) 154,140
But did cost
156,709
Variable overhead expenditure variance
2,569 adverse
STUDY MATERIAL C1
128
STANDARD COSTING AND VARIANCE ANALYSIS
2006.1
(c) Variable overhead efficiency variance
The variable overhead efficiency variance reveals how much of the variable overhead total
variance was caused by using a different number of hours of labour, compared with the
standard allowance for the production achieved. Its calculation is very similar to the
calculation of the labour efficiency variance.
The favourable efficiency variance of £90 is the saving in variable overhead cost (at
standard rates) resulting from using fewer labour hours than the standard expected for this
level of output.
Check: £2,569 adverse + £90 favourable ¼£2,479 adverse (the correct total variance)
Notice that the method used to calculate the variable overhead variances is
identical to the method used to calculate the direct labour variances. In the next
chapter you will see that the calculation of the variable overhead efficiency variance may
be affected by idle time.
5.9 Sales variances
Now that we have seen how to analyse the variable cost variances we will turn our attention
to sales variances. Your syllabus requires you to be able to calculate two variances for sales:
the sales price variance and the sales volume contribution variance. We will demonstrate the
calculation of these variances using the following data.
5.9.1 Sales price variance
The sales price variance reveals the difference in total revenue caused by charging a
different selling price from standard.
The adverse sales price variance indicates that the 82,400 units were sold for a lower
price than standard, which we can see from the basic data.
Variance in hours (from labour efficiency variance) 30 hours favourable
standard variable overhead rate per hour (£3)
Variable overhead efficiency variance
£90
favourable
Budget Sales and production volume 81,600 units
Standard selling price £59 per unit
Standard variable cost £24 per unit
Actual results Sales and production volume 82,400 units
Actual selling price £57 per unit
Actual variable cost £23 per unit
£
82,400 units should sell for (£59) 4,861,600
But did sell for (82,400 units£57)
4,696,800
Sales price variance
164,800 adverse
FUNDAMENTALS OF MANAGEMENT ACCOUNTING
129
STANDARD COSTING AND VARIANCE ANALYSIS
2006.1
5.9.2 Sales volume contribution variance
The sales volume contribution variance reveals the contribution difference which is caused
by selling a different quantity from that budgeted.
Since the analysis of variable cost variances explains all of the variations caused
by differences between actual costs and standard costs, the calculation of the
sales volume variance is based on the standard contribution not on the actual
contribution.
5.10 Summary
Having read this chapter the main points that you should understand are as follows:
1. A standard cost is a carefully predetermined unit cost. It is established in advance to
provide a basis for planning, a target for achievement and a benchmark against which
the actual costs and revenues can be compared.
2. The difference between the standard cost and the actual result is called a variance.
3. The analysis of variances facilitates action through management by exception,
whereby managers concentrate on those areas of the business that are performing
below or above expectations and ignore those that appear to be conforming to
expectations.
4. A number of different performance levels can be used in setting standards. The most
common are ideal, attainable and current.
5. The direct material total variance can be analysed between the direct material price
variance and the direct material usage variance.
6. If inventories (stocks) are valued at standard cost then the material price variance
should be based on the quantity purchased. If inventories are valued at actual cost the
material price variance should be based on the quantity used during the period.
7. The direct labour total variance can be analysed between the direct labour rate
variance and the direct labour efficiency variance.
8. The variable overhead total variance can be analysed between the variable overhead
expenditure variance and the variable overhead efficiency variance.
9. The sales price variance reveals the difference in total revenue caused by charging a
different selling price from standard.
10. The sales volume contribution variance reveals the contribution difference which is
caused by selling a different quantity from that budgeted. The calculation of the
variance is based on the standard contribution not on the actual contribution.
Actual sales volume 82,400 units
Budget sales volume
81,600 units
Sales volume variance in units 800 favourable
standard contribution per unit £(59 24) £35
Sales volume contribution variance
£28,000 favourable
STUDY MATERIAL C1
130
STANDARD COSTING AND VARIANCE ANALYSIS
2006.1
Revision Questions
Question 1 Multiple choice
1.1 A standard cost is:
(A) the planned unit cost of a product, component or service in a period.
(B) the budgeted cost ascribed to the level of activity achieved in a budget centre in
a control period.
(C) the budgeted production cost ascribed to the level of activity in a budget
period.
(D) the budgeted non-production cost for a product, component or service in a
period.
Data for questions 1.2–1.7
Budgeted production of product V is 650 units each period. The standard cost card
for product V contains the following information.
During the latest period 670 units of product V were produced. The actual results
recorded were as follows.
1.2 The ingredients price variance is:
(A) £1,503 favourable
(B) £1,503 adverse
(C) £1,603 favourable
(D) £1,603 adverse
1.3 The ingredients usage variance is:
(A) £100 favourable
(B) £100 adverse
(C) £105 favourable
(D) £860 adverse
£ per unit
Ingredients 12 litres @ £4 per litre 48
Direct labour 3 hours @ £9 per hour 27
Variable production overhead 3 hours @ £2 per hour 6
Ingredients purchased and used 8,015 litres £33,663
Direct labour 2,090 hours £17,765
Variable production overhead £5,434
5
131 2006.1
1.4 The labour rate variance is
(A) £325 favourable
(B) £325 adverse
(C) £1,045 favourable
(D) £1,045 adverse
1.5 The labour efficiency variance is
(A) £680 adverse
(B) £720 adverse
(C) £720 favourable
(D) £1,260 adverse
1.6 The variable overhead expenditure variance is:
(A) £1,254 favourable
(B) £1,254 adverse
(C) £1,534 favourable
(D) £1,534 adverse
1.7 The variable overhead efficiency variance is:
(A) £151 adverse
(B) £160 adverse
(C) £160 favourable
(D) £280 adverse
1.8 ABC Ltd uses standard costing. It purchases a small component for which the
following data are available:
What was the actual purchase price per unit?
(A) 75p
(B) 77p
(C) 93p
(D) 95p.
1.9 During a period 17,500 labour hours were worked at a standard cost of £6.50 per
hour. The labour efficiency variance was £7,800 favourable. The number of standard
labour hours expected for the output achieved was:
(A) 1,200
(B) 16,300
(C) 17,500
(D) 18,700.
1.10 XYZ Ltd uses standard costing. It makes an assembly for which the following
standard data are available:
Actual purchase quantity 6,800 units
Standard allowance for actual production 5,440 units
Standard price 85p per unit
Purchase price variance (adverse) (£544)
Standard labour hours per assembly 24
Standard labour cost per hour £8
REVISION QUESTIONS C1
132
STANDARD COSTING AND VARIANCE ANALYSIS
2006.1
During a period 850 assemblies were made, there was a nil rate variance and an
adverse efficiency variance of £4,400.
How many actual labour hours were worked?
(A) 19,850
(B) 20,400
(C) 20,950
(D) 35,200.
Data for questions 1.11 and 1.12
The standard cost of providing a meal in a fast food restaurant is as follows.
The standard price of the meal is £4.50 and the budgeted sales volume is 4,650 meals each
period.
During period 9 a total of 4,720 meals were sold for £20,768. The actual total variable
cost per meal was £2.30.
1.11 The sales price variance for period 9 was:
(A) £465 favourable
(B) £465 adverse
(C) £472 favourable
(D) £472 adverse
1.12 The sales volume contribution variance for period 9 was:
(A) £147 favourable
(B) £147 adverse
(C) £154 favourable
(D) £154 adverse
Question 2 Short objective-test questions
2.1 Tick the correct box.
A standard which assumes efficient levels of operation, but which includes
allowances for factors such as waste and machine downtime, is known as an:
attainable standard &
ideal standard &
2.2 The standard cost card for product F shows that each unit requires 3 kg of material
at a standard price of £9 per kilogram. Last period, 200 units of F were produced and
£5,518 was paid for 620 kg of material that was bought and used. Calculate the
following variances and tick the correct box to indicate whether each variance is
adverse or favourable.
£
Ingredient cost 1.80
Direct labour cost 0.30
Variable overhead cost 0.20
FUNDAMENTALS OF MANAGEMENT ACCOUNTING
133
STANDARD COSTING AND VARIANCE ANALYSIS
2006.1
2.3 The standard cost card for product K shows that each unit requires four hours of
direct labour at a standard rate of £8 per hour. Last period, 420 units were produced
and the direct labour cost amounted to £15,300. The direct labour efficiency
variance was £160 adverse.
The actual rate paid per direct labour hour is £
.
2.4 Is the following statement true or false?
Standard costing cannot be applied in an organisation that manufactures specialist
furniture to customers’ specifications because every cost unit is unique.
True &
False &
2.5 The budgeted sales of product Y are 230 units per period at a standard sales price of
£43 per unit. Last period the sales volume contribution variance was £1,100
favourable and all units were actually sold for £46 per unit. The sales price variance
was £840 favourable.
The standard variable cost per unit of product Y is £
.
Question 3 Direct cost variances
XYZ Ltd is planning to make 120,000 units per period of a new product. The following
standards have been set:
All direct operatives are paid at the rate of £8 per hour. Attainable work hours are less
than clock hours, so the 500 direct operatives have been budgeted for 400 hours each in
the period.
Actual results for the period were:
Adverse Favourable
(a) the direct material price variance is £
&&
(b) the direct material usage variance is £
&&
Per unit
Direct material A 1.2 kg at £11 per kg
Direct material B 4.7 kg at £6 per kg
Direct labour:
Operation 1 42 minutes
Operation 2 37 minutes
Operation 3 11 minutes
Production 126,000 units
Direct labour cost £1.7 m for 215,000 clock hours
Material A cost £1.65 m for 150,000 kg
Material B cost £3.6 m for 590,000 kg
REVISION QUESTIONS C1
134
STANDARD COSTING AND VARIANCE ANALYSIS
2006.1
Requirements
(a) (i) A realistic labour efficiency variance for the period is £
adverse &
favourable &
(ii) The labour rate variance for the period is £
adverse &
favourable &
(b) (i) The material price variances for the period are
(ii) The material usage variances for the period are:
Material A £
Material B £
adverse & adverse &
favourable & favourable &
Material A £
Material B £
adverse & adverse &
favourable & favourable &
FUNDAMENTALS OF MANAGEMENT ACCOUNTING
135
STANDARD COSTING AND VARIANCE ANALYSIS
2006.1
Solutions to
Revision Questions
Solution 1
Select your answer carefully from the available options. You may in haste select an
option that has the correct absolute value for the variance but is adverse when you
should have selected favourable, or vice versa.
In some of the questions you will need to ‘work backwards’ from variance information
to determine the actual results. This will enable you to test yourself to see if you really
understand how the variances are calculated!
The second question asks for an ingredients price variance. This is calculated in exactly
the same way as a direct material price variance.
1.1 Answer: (A)
A standard cost is a carefully predetermined unit cost which is prepared for each
cost unit.
1.2 Answer: (D)
1.3 Answer: (A)
1.4 Answer: (C)
£
8,015 litres should cost (£4) 32,060
But did cost
33,663
Ingredients price variance
1,603 adverse
Litres
670 units produced should use (12) 8,040
But did use
8,015
Variance in litres
25 favourable
standard price per litre (£4)
Ingredients usage variance
£100 favourable
£
2,090 hours should cost (£9) 18,810
But did cost
17,765
Labour rate variance
1,045 favourable
5
137 2006.1
1.5 Answer: (B)
1.6 Answer: (B)
1.7 Answer: (B)
1.8 Answer: (C)
Purchase price variance per unit purchased ¼ £544=6;800 ¼ 8p adverse per unit.
Actual purchase price ¼85p standard + 8p ¼93p per unit.
1.9 Answer: (D)
Number of hours saved compared with standard ¼ £7,800= £6.50 ¼1,200.
Number of standard labour hours expected ¼17,500 + 1,200 ¼18,700.
1.10 Answer: (C)
Standard labour cost ¼24 hours 850 £8 ¼£163,200
Actual cost ¼£163,200 + £4,400 ¼£167,600
@£8/hour ¼20,950 hours
1.11 Answer: (D)
Hours
670 units produced should take (3) 2,010
But did take
2,090
Variance in hours
80 adverse
standard labour rate per hour (£9)
Labour efficiency variance
£720 adverse
£
2,090 hours should cost (£2) 4,180
But did cost
5,434
Variable overhead expenditure variance
1,254 adverse
Variance in hours (from labour efficiency variance)
80 hours adverse
standard variable overhead rate per hour (£2)
Variable overhead efficiency variance £160 adverse
£
4,720 meals should sell for (£4.50) 21,240
But did sell for
20,768
Sales price variance
472 adverse
SOLUTIONS TO REVISION QUESTIONS C1
138
STANDARD COSTING AND VARIANCE ANALYSIS
2006.1
1.12 Answer: (C)
Solution 2
2.1 A standard which assumes efficient levels of operation, but which includes allowances
for factors such as waste and machine downtime is known as an attainable standard.
2.2
2.3 Efficiency variance in hours ¼ £160=£8 ¼ 20 hours adverse
Actual hours worked ¼20 + 1,680 standard hours (420 4) ¼1,700
Actual rate paid per hour ¼ £15,300=1,700 ¼ £9 per hour
2.4 False. Even though each cost unit is unique, each could involve standardised tasks for
which a standard time and/or cost can be determined for control purposes.
2.5 The standard variable cost per unit of product Y is £21.
Sales price variance per unit sold ¼£46 actual price £43 std. price ¼£3 favourable
Number of units sold ¼£840 sales price variance/£3 ¼280 units
Sales volume variance in units ¼280 actual sales 230 budget sales
¼50 units favourable
Standard contribution per unit ¼£1,100 volume variance/50 ¼£22
Standard variable cost per unit ¼£43 standard price £22 standard
contribution ¼£21
Solution 3
There is an unusual request in part (a): for a realistic labour efficiency variance. This
means that you need to take account of the difference between attainable work hours
and actual clock hours. A realistic efficiency variance should be based on attainable
hours rather than on clock hours.
Actual sales volume 4,720 meals
Budget sales volume
4,650 meals
Sales volume variance in meals 70 favourable
standard contribution per meal
£(4.50 1.80 0.30 0.20)
£2.20
Sales volume contribution variance
£154 favourable
£
620 kg should have cost (£9) 5,580
But did cost
5,518
Direct material price variance
62 favourable
Kg
200 units produced should have used (3 kg) 600
But did use
620
Variance in kg
(20)
adverse
standard price per kg (£9)
Direct material usage variance
(£180) adverse
FUNDAMENTALS OF MANAGEMENT ACCOUNTING
139
STANDARD COSTING AND VARIANCE ANALYSIS
2006.1
The question gives you a hint about the difference between attainable hours and clock
hours: the clock hours budgeted for 120,000 units are more than the standard time
allowance of 1.5 hours per unit. The difference is the lost time or idle time, for which an
allowance should be made when the efficiency variance is calculated.
Do not forget to indicate whether your calculated variances are adverse or favourable.
(a) (i) £36,000 adverse
(ii) £20,000 favourable
Workings:
Standard labour hours per unit ¼ð42 þ37 þ 11Þ=60 ¼ 1:5 hours
Budgeted attainable work hours for the period ¼120,000 units 1.5 hours
¼180,000 hours
Budgeted clock hours for the period ¼500 operatives 400 hours ¼200,000 hours
Attainable hours ¼90 per cent of clock hours
Labour efficiency variance
Labour rate variance
Workings:
Direct material price variance
Hours
126,000 units should have taken ( 1.5 hours) 189,000
But did take (215,000 90%)
193,500
Variance in hours
4,500 adverse
standard labour rate per hour (£8)
Labour efficiency variance
£36,000 adverse
£
215,000 hours paid for should have cost ( £8) 1,720,000
But did cost
1,700,000
Labour rate variance
20,000 favourable
Material A Material B
(b) (i) £0 £60,000 adverse
(ii) £13,200 favourable £13,200 favourable
Material A £
150,000 kg should have cost ( £11) 1,650,000
And did cost
1,650,000
Direct material price variance
SOLUTIONS TO REVISION QUESTIONS C1
140
STANDARD COSTING AND VARIANCE ANALYSIS
2006.1
Direct material usage variance
Material B £
590,000 kg should have cost (£6) 3,540,000
But did cost
3,600,000
Direct material price variance
60,000 adverse
Material A Kg
126,000 units produced should have used (1.2 kg) 151,200
But did use
150,000
Variance in kg
1,200 favourable
standard price per kg (£11)
Direct material usage variance
£13,200 favourable
Material B Kg
126,000 units produced should have used (4.7 kg) 592,200
But did use
590,000
Variance in kg
2,200 favourable
standard price per kg (£6)
Direct material usage variance
£13,200 favourable
FUNDAMENTALS OF MANAGEMENT ACCOUNTING
141
STANDARD COSTING AND VARIANCE ANALYSIS
2006.1
6
Further Standard
Costing
Further Standard
Costing
6.1 Introduction
In this chapter you will be continuing your studies of standard costing and variance analysis.
You will learn how to put all the variances together in a statement which reconciles the
budgeted contribution for a period with the actual contribution achieved.
You will also be learning how to interpret variances and how standard labour costs can
be used in designing incentive schemes.
6.2 Reconciling actual contribution with
budgeted contribution
Now that you have seen how to calculate all the main variable cost and sales variances,
you should be in a position to produce a statement which reconciles the actual and budget
contribution for the period.
First, to get some important practice, you should calculate all of the variances using the
data given in the following example. Then you can learn to put all the variances together in
a reconciliation statement like the one shown at the end of the solution.
LEARNING OUTCOMES
After completing this chapter, you should be able to:
"
prepare a statement that reconciles budgeted contribution with actual contribution;
"
interpret statements of variances for variable costs, sales prices and sales volumes
including possible inter-relations between cost variances, sales price and volume
variances, and cost and sales variances;
"
describe the possible use of standard labour costs in designing incentive schemes
for factory and office workers.
6
145 2006.1
Example
A company produces and sells one product only, the standard variable cost for which is:
The variable production overhead is incurred in direct proportion to the direct labour hours worked. The
budgeted sales volume for May was 2,000 units.
The following were the actual results recorded during May:
Number of units produced and sold: 1,750
You are required to calculate the operating variances and present them in a statement which reconciles
the budget and actual contribution for May.
Solution
Direct material price variance
Direct material usage variance
Direct labour rate variance
£ per unit
Direct material 11 litres at £2 22
Direct wages 5 hours at £6 30
Variable production overhead
10
Total standard variable cost 62
Standard contribution
58
Standard selling price
120
££
Sales revenue 218,750
Direct materials: 19,540 litres purchased and used 41,034
Direct labour: 8,722 hours 47,971
Variable production overhead
26,166
115,171
Contribution
103,579
£
19,540 litres purchased should have cost ( £2) 39,080
But did cost
41,034
Direct material price variance
1,954 adverse
Litres
1,750 units produced should have used ( 11 litres) 19,250
But did use
19,540
Variance in litres
290 adverse
standard price per litre (£2)
Direct material usage variance
£580 adverse
£
8,722 hours should have cost ( £6) 52,332
But did cost
47,971
Direct labour rate variance
4,361 favourable
STUDY MATERIAL C1
146
FURTHER STANDARD COSTING
2006.1
Direct labour efficiency variance
Variable production overhead expenditure variance
Variable production overhead efficiency variance
Sales price variance
Sales volume contribution variance
A reconciliation statement, known as an operating statement, begins with the original budgeted
contribution. It then adds or subtracts the variances (depending on whether they are favourable or
adverse) to arrive at the actual contribution for the month.
Hours
1,750 units produced should take ( 5 hours) 8,750
But did take
8,722
Variance in hours
28
favourable
standard labour rate per hour (£6)
Direct labour efficiency variance
£168
favourable
£
8,722 hours of variable production overhead should cost ( £2) 17,444
But did cost
26,166
Variable production overhead expenditure variance
8,722
adverse
Variance in hours (from labour efficiency variance)
28 favourable
standard variable overhead rate per hour (£2)
Variable production overhead efficiency variance
£56 favourable
£
1,750 units should sell for ( £120) 210,000
But did sell for
218,750
Sales price variance
8,750 favourable
Actual sales volume 1,750 units
Budget sales volume
2,000 units
Sales volume variance in units 250 adverse
standard contribution per unit £58
Sales volume contribution variance
£14,500 adverse
FUNDAMENTALS OF MANAGEMENT ACCOUNTING
147
FURTHER STANDARD COSTING
2006.1
Contribution reconciliation statement for May
Note: Variances in brackets are adverse.
6.3 Idle time variances
You may come across a situation which involves idle time. Idle time occurs when labour is
available for production but is not engaged in active production due to, for example,
shortage of work or material.
During idle time, direct labour wages are being paid but no output is being produced.
The cost of this can be highlighted separately in an idle time variance, so that it is not
‘hidden’ in an adverse labour efficiency variance. In this way, management attention can be
directed towards the cost of idle time.
Variable production overhead variances can also be affected by idle time. It is usually
assumed that variable production overhead expenditure is incurred in active hours only – for
example, only when the machines are actually running, incurring power costs, etc. therefore
variable production overhead expenditure is not being incurred during idle hours. The variable
production overhead efficiency variance is affected in the same way as the labour efficiency
variance.
Example
To demonstrate this, suppose that in the last example you were given the following additional information
about the actual results recorded during May.
Of the 8,722 hours of direct labour paid for, 500 hours were idle because of a shortage of material
supplies.
An idle time variance could be calculated as follows:
Idle time variance
Idle hours standard labour rate per hour
¼ 500 £6
¼ £3; 000 adverse
££
Original budgeted contribution:
2,000 units £58
116,000
Sales volume contribution variance
(14,500)
Standard contribution from actual
sales volume
101,500
Sales price variance
8,750
Cost variances 110,250
Direct material: price (1,954)
usage
(580)
(2,534)
Direct labour: rate 4,361
efficiency
168
4,529
Variable production overhead: expenditure (8,722)
efficiency
56
(8,666)
Actual contribution
103,579
STUDY MATERIAL C1
148
FURTHER STANDARD COSTING
2006.1
This is the standard cost of wages incurred during the idle time.
These idle hours must be eliminated from the calculation of the labour efficiency variance, so that the
efficiency of labour is being measured only during the hours when they were actually working. This gives
a much more meaningful measure of labour efficiency.
Direct labour efficiency variance
The total of these two variances is the same as the original labour efficiency variance (£168 favourable).
The effect on the variable production overhead variances would be as follows:
Variable production overhead expenditure variance
Variable production overhead efficiency variance
The total of £8,666 adverse for the two variable production overhead variances is not affected by
theidletime(youshouldcheckthisforyourself).However,wehavenowmeasuredefciency
during active hours only, and we have allowed variable production overhead expenditure only for
active hours.
6.4 Interpreting variances
6.4.1 The reasons for variances
There are many possible causes of variances, ranging from errors in setting the standard
cost to efficiencies and inefficiencies of operations. Table 6.1 shows the possible causes of
variances. This table is not exhaustive, but it will give you an idea of the range of possible
causes.
In an assessment question, you should review the information given and select
any feasible cause that is consistent with the variance in question: that is, if the
variance is favourable you must select a cause that would result in a favourable
variance.
Hours
1,750 units produced should have taken ( 5 hours) 8,750
But did take (active hours)
8,222
Variance in hours
528 favourable
standard labour rate per hour (£6)
Direct labour efficiency variance
£3,168 favourable
£
8,222 active hours of variable production overhead should cost
( £2)
16,444
But did cost
26,166
Variable production overhead expenditure variance
9,722 adverse
Hours
1,750 units produced should have taken ( 5 hours) 8,750
But did take (active hours)
8,222
Variance in hours
528 favourable
standard variable overhead rate per hour (£2)
Variable production overhead efficiency variance
£1,056 favourable
FUNDAMENTALS OF MANAGEMENT ACCOUNTING
149
FURTHER STANDARD COSTING
2006.1
6.4.2 The significance of variances
Once the variances have been calculated, management has the task of deciding which
variances should be investigated. It would probably not be worthwhile or cost effective to
investigate every single variance. Some criteria must be established to guide the decision as
to whether or not to investigate a particular variance.
Factors which may be taken into account include the following:
(a) The size of the variance. Costs tend to fluctuate around a norm and therefore ‘normal’
variances may be expected on most costs. The problem is to decide how large a
variance must be before it is considered ‘abnormal’ and worthy of investigation.
Table 6.1 Causes of variances
Variance Favourable Adverse
Material price Standard price set too high Standard price set too low
Unexpected discounts available Unexpected general price increase
Lower-quality material used Higher-quality material used
Careful purchasing Careless purchasing
Gaining bulk discounts by
buying larger quantities
Losing bulk discounts by buying
smaller quantities
Material usage Standard usage set too high Standard usage set too low
Higher-quality material used Lower-quality material used
A higher grade of worker used
the material more efficiently
A lower grade of worker used
the material less efficiently
Stricter quality control Theft
Labour rate Standard rate set too high Standard rate set too low
Lower grade of worker used Higher grade of worker used
Higher rate due to wage award
Labour efficiency Standard hours set too high Standard hours set too low
Higher grade of worker Lower grade of worker
Higher grade of material was
quicker to process
Lower grade of material was
slower to process
More efficient working through
improved motivation
Less efficient working due to
poor motivation
Idle time Shortage of work
Machine breakdown
Shortage of material
Variable overhead expenditure Standard hourly rate set too high Standard hourly rate set too low
Overheads consist of a number of items: indirect materials, indirect labour, maintenance
costs, power, etc., which may change because of rate changes or variations in
consumption. Consequently, any meaningful interpretation of the expenditure variance
must focus on individual cost items.
Variable overhead efficiency See labour efficiency variance
Sales price Higher quality product commanded
higher selling price than standard
Increased competition forced a
reduction in selling price below
standard
Sales volume contribution Increased marketing activity led to
higher than budgeted sales volume
Quality control problems resulted
in lower than budgeted sales
volumes
STUDY MATERIAL C1
150
FURTHER STANDARD COSTING
2006.1
A rule of thumb may be established that any variance which exceeds, say, five per
cent of its standard cost may be worthy of investigation. Alternatively, control limits
may be set statistically and if a cost fluctuates outside these limits it should be
investigated.
(b) The likelihood of the variance being controllable. Managers may know from experience that
certain variances may not be controllable even if a lengthy investigation is undertaken
to determine their causes. For example it might be argued that a material price
variance is less easily controlled than a material usage variance because it is heavily
influenced by external factors.
(c) The likely cost of an investigation. This cost would have to be weighed against the
cost which would be incurred if the variance was allowed to continue in future
periods.
(d) The interrelationship of variances. Adverse variances in one area of the organisation may be
interrelated with favourable variances elsewhere. For example, if cheaper material is
purchased this may produce a favourable material price variance. However, if the
cheaper material is of lower quality and difficult to process, this could result in adverse
variances for material usage and labour efficiency.
(e) The type of standard that was set. You have already seen that an ideal standard will
almost always result in some adverse variances, because of unavoidable waste, etc.
Managers must decide on the ‘normal’ level of adverse variance which they would
expect to see.
Another example is where a standard price is set at an average rate for the year.
Assuming that inflation exists, favourable price variances might be expected at the
beginning of the year, to be offset by adverse price variances towards the end of the
year as actual prices begin to rise.
A detailed knowledge of the significance of variances is outside the scope of your
Fundamentals of Management Accounting syllabus. However, you should now be aware that the
use of standard costing systems for control purposes does not end with the calculation of
the variances.
Exercise
In (d) above we mention one possible interrelationship that might exist between cost
variances. Following this example, can you think of a possible interrelationship that might
exist:
(i) between other cost variances;
(ii) between the sales price and sales volume contribution variance;
(iii) between cost and sales variances.
Solution
You might have thought of other, equally valid suggestions in addition to those below.
(i) Possible interrelationship between cost variances
Employing a higher grade of labour than standard might produce an adverse labour
rate variance. However, if these employees are more skilled than standard they may
work more quickly and efficiently, resulting in a favourable labour efficiency variance
and a favourable variable overhead efficiency variance.
FUNDAMENTALS OF MANAGEMENT ACCOUNTING
151
FURTHER STANDARD COSTING
2006.1
(ii) Possible interrelationship between the sales price and sales volume contribution variance
Charging a higher selling price than standard will produce a favourable sales price
variance. However, the higher price might deter customers and thus sales volumes
might fall below budget, resulting in an adverse sales volume contribution variance.
(iii) Possible interrelationship between cost and sales variances
Purchasing a higher quality material than standard might produce an adverse
material price variance. However, the quality of the finished product might be
higher than standard and it might be possible to command higher selling prices,
thus producing a favourable sales price variance. Furthermore, the higher quality
product might attract more customers to buy which could result in a favourable
sales volume contribution variance.
6.5 Standard hour
Sometimes it can be difficult to measure the output of an organisation which
manufactures a variety of dissimilar items. For example, if a company manufactures
metal saucepans, utensils and candlesticks, it would not be meaningful to add together
these dissimilar items to determine the total number of units produced. It is likely that
each of the items takes a different amount of time to produce and utilises a different
amount of resource.
A standard hour is a useful way of measuring output when a number of dissimilar items
are manufactured. A standard hour or minute is the amount of work achievable, at
standard efficiency levels, in an hour or minute.
The best way to see how this works is to look at an example.
Example
A company manufactures tables, chairs and shelf units. The standard labour times allowed to manufacture one
unit of each of these are as follows:
Production output during the first two periods of this year was as follows:
It would be difficult to monitor the trend in total production output based on the number of
units produced. We can see that fifteen units were produced in total in period 1 and eleven units in
period 2. However, it is not particularly meaningful to add together tables, chairs and shelf units
because they are such dissimilar items. You can see that the mix of the three products changed over the
two periods and the effect of this is not revealed by simply monitoring the total number of units
produced.
Standard labour hours per unit
Table 3 hours
Chair 1 hour
Shelf unit 5 hours
Units produced
Period 1 Period 2
Table 7 4
Chair 5 2
Shelf unit 3 5
STUDY MATERIAL C1
152
FURTHER STANDARD COSTING
2006.1
Standard hours present a useful output measure which is not affected by the mix of products. The
standard hours of output for the two periods can be calculated as follows:
Expressing the output in terms of standard labour hours shows that in fact the output level for period 2
was very similar to that for period 1.
It is important for you to realise that the actual labour hours worked during each of
these periods was probably different from the standard labour hours produced. The
standard hours figure is simply an expression of how long the output should have taken to
produce, to provide a common basis for measuring output.
The difference between the actual labour hours worked and the standard labour
hours produced will be evaluated as the labour efficiency variance.
6.6 Labour incentive schemes
Standard labour times can be useful in designing incentive schemes for factory and
office workers. For example, if a standard time has been established for a particular
task an employee might be paid a bonus if the task is completed in less than the
standard time.
Knowledge of the standard labour costs can assist managers in devising a labour
incentive scheme that provides an incentive for the employee while at the same time being
cost-effective for the organisation.
6.6.1 Bonus schemes
A variety of bonus and incentive schemes exist in practice. They are all similar and are
designed to increase productivity.
The schemes rely on the setting of a standard time to achieve a task and the
comparison of the actual time taken with the standard time. The savings which result
from the employee’s greater efficiency are usually shared between the employee
and the employer on a proportionate basis. Usually the employee receives between 30
and 60 per cent of the time saved as a bonus number of hours paid at the normal
hourly rate.
Example
John is a skilled engineer, paid £15 per hour. Each job he does has a standard time allowance and he is
paid 50 per cent of any time he saves each week as a bonus paid at his hourly rate.
Period 1 Period 2
Standard hours
per unit
Units
produced
Standard
hours
Units
produced
Standard
hours
Table 3 7 21 4 12
Chair 1 5 5 2 2
Shelf unit 5 3
15 5 25
Total standard labour
hours produced
41 39
FUNDAMENTALS OF MANAGEMENT ACCOUNTING
153
FURTHER STANDARD COSTING
2006.1
During week 11 John worked for 40 hours and completed jobs having a total standard time allowed
of 47 hours.
John’s earnings were:
* Seven hours were saved against the total standard hours allowed, so 3.5 bonus hours are paid.
A wide variety of incentive and bonus schemes exist. In the assessment you
must read the description of the scheme carefully before you apply it to the data
supplied.
Note that incentive schemes based on a standard time allowance can be applied to
office workers as well as to factory workers. For example, a standard time might be set for
processing an invoice. At the end of a period the number of standard hours of work
represented by the number of invoices processed by a particular employee can be
measured. If the employee has saved time against this standard allowance then a bonus can
be paid to the employee as a reward for performance above standard.
6.6.2 Piecework systems
If remuneration is based on piecework an employee is paid according to the output
achieved, regardless of the time taken.
A payment rate per unit produced is agreed in advance. Knowledge of standard labour
times will help managers to decide on the amount that will be paid for each unit produced.
A variation of the basic piecework principle is for the organisation to set a daily target
level of activity, based on the standard labour time per unit. The employee is then paid a
higher rate per unit for those completed in excess of the target.
Example
Dave is employed on a part-time basis by K Limited. He is paid £0.40 for each unit he produces up to
100 units per shift. Any units produced above this target are paid at £0.50 per unit. Last shift he
produced 108 units. His earnings that shift were:
£
100 @ £0.40 40
8 @ £0.50
4
44
£
40 hours £15 600.00
Bonus 3.5 hours* £15
52.50
Total earnings
652.50
100
0
0
Units produced per shif
t
Total wage cost
per shift, £
STUDY MATERIAL C1
154
FURTHER STANDARD COSTING
2006.1
A sketch graph of this piecework system would look like this (not to scale):
The gradient of the graph becomes steeper when output exceeds 100 units per shift.
6.6.3 Guaranteed minimum wage
A guaranteed minimum wage may be included within a piecework system. It
protects employees by guaranteeing them a minimum weekly wage based on an
hourly rate multiplied by the employee’s number of attendance hours. Note that this
is only applied if the level of piecework earnings is below this guaranteed minimum
level.
Example
If Dave (see Section 6.6.2) had only produced 50 units but was entitled to a guaranteed minimum
wage of £30 per shift, he would receive £30 even though his piecework earnings were only
50 £0.40 ¼£20.
A sketch graph of this piecework system would look like this (not to scale):
The wages cost remains constant at £30 per shift, until output reaches 75 units (75 £0.40 ¼£30).
After this point the wages cost increases according to the rate per unit, as before.
6.6.4 Differential piece rate
Using this system a target number of units is set and different rates per unit are paid
depending upon the total number of units achieved. Usually a daily target is used. For
example:
You should note that it is usual for the higher rates to apply only to the additional units,
not to all of the units achieved.
Total wage cost
per shift, £
Units produced per shif
t
75
0
0
Units produced in a day £
1–100 units 0.40 each
101–129 units 0.42 each
130 units and above 0.44 each
FUNDAMENTALS OF MANAGEMENT ACCOUNTING
155
FURTHER STANDARD COSTING
2006.1
A sketch graph of a differential piece-rate system would look like this (not to scale):
The gradient of the graph becomes progressively steeper with each successive increase
in the rate paid per unit.
6.6.5 Piecework hours
A piecework hour is the same in principle as the standard hour that you learned about
earlier in this chapter. Piecework hours are used to measure the output when employees
are paid according to a piecework scheme and dissimilar items are produced. A standard
piecework time allowance is determined for each unit produced.
Example
Employee number 297 is paid a guaranteed wage of £170 per week plus £3 per piecework hour
produced. Last week the employee produced the following output.
Product Number of units produced Standard piecework hours per unit
R 40 0.7
T 30 0.3
The number of standard piecework hours produced is (40 0.7) + (30 0.3) ¼37
Wages for last week ¼£170 + (37 piecework hours £3) ¼£281
6.6.6 Group incentive schemes
Bonus or incentive schemes based on standard time allowances can be applied to groups
as well as to individuals. Group incentive schemes might be appropriate in circumstances
such as:
when it is not possible to set a standard for and to measure individual performance – for
example, in an office;
when operations are performed by a group or team and not by individuals working
alone for example, road repairs or refuse collections;
where production is integrated and increased output depends on a number of people all
making extra effort for example, in production line manufacture such as that in the
automobile industry.
Units produced per da
y
Daily wages cost, £
0
0
STUDY MATERIAL C1
156
FURTHER STANDARD COSTING
2006.1
Example
A team of three clerks produces a detailed credit control report for a company’s monthly management
meeting. The standard time allowed for production of the report is 18 labour hours. A bonus of £9 per
hour saved against this time allowance is paid to the team, divided equally between the three clerks. The
time taken to produce the report last month was as follows.
Clerk no. Time taken (hours)
12
23
35
Time saved against standard allowance ¼18 hours allowance 10 hours taken
¼8 hours
Bonus payable per clerk ¼(8 £9)/3 ¼£24
6.7 Summary
Having read this chapter the main points that you should understand are as follows.
1. Sales and variable cost variances can be combined in a statement that reconciles the
budgeted contribution with the actual contribution achieved during a period.
Favourable variances are added to the budgeted contribution and adverse variances
are deducted to arrive at the actual contribution.
2. The idle time variance is always adverse. It is calculated as the number of hours idle
multiplied by the standard labour rate per hour. If there is idle time then the variances
for labour efficiency, variable production overhead efficiency and variable production
overhead expenditure should be based on active hours only.
3. It is not always worth investigating every variance. Some criteria must be established to
guide the decision as to whether or not to investigate a particular variance.
4. Variances might be interrelated so that one variance might be a direct result of another
variance. It is important to consider possible interrelationships between variances
before embarking on detailed investigations as to their cause.
5. Knowledge of the standard labour cost can provide the basis for designing incentive
schemes based on standard time allowances or on piecework.
6. A differential piece rate system pays different rates per unit depending on the output
achieved.
FUNDAMENTALS OF MANAGEMENT ACCOUNTING
157
FURTHER STANDARD COSTING
2006.1
Revision Questions
Question 1 Multiple choice
1.1 The following data relates to an employee in production department A:
Normal working day 7 hours
Hourly rate of pay £8
Standard time allowed to produce one unit 6 minutes
Bonus payable at basic hourly rate 50% of time saved
What would be the gross wages payable in a day when the employee produces
82 units?
(A) £33.60
(B) £60.80
(C) £65.60
(D) £84.00
1.2
Outpu
t
£
0
The labour cost graph above depicts:
(A) a piece-rate scheme with a minimum guaranteed wage.
(B) a straight piece-rate scheme.
(C) a time-rate scheme, where the employee is paid for each hour of attendance.
(D) a differential piece-rate scheme.
6
159 2006.1
Data for questions 1.3 to 1.5
The standard direct labour cost of one unit of product Q is £3.00 (0.25 hours £12.00).
The eight employees who make product Q work a 7-hour day. In a recent 3-day period,
results were as follows:
Actual units produced 650 units
Actual wages cost £2,275
During this period, there was a power failure. This meant that all work had to stop for 2 hours.
1.3 If the company reports idle time separately, the labour efficiency variance for the period is:
(A) £126 favourable
(B) £142 favourable
(C) £66 adverse
(D) £126 adverse
1.4 The labour rate variance for the period is:
(A) £259 favourable
(B) £259 adverse
(C) £325 favourable
(D) £325 adverse
1.5 The idle time variance for the period is:
(A) £24 adverse
(B) £24 favourable
(C) £192 adverse
(D) £192 favourable
Question 2 Short objective-test questions
2.1 The direct material usage variance for last period was £3,400 adverse. Which of the
following reasons could have contributed to this variance? (Tick all that apply.)
(a) Output was higher than budgeted. &
(b) The purchasing department bought poor quality material. &
(c) The original standard usage was set too high. &
(d) Market prices for the material were higher than expected. &
(e) An old, inefficient machine was causing excess wastage. &
2.2 If employees are more skilled than had been allowed for in the original standard cost,
which four of the following variances are most likely to result?
(a) favourable material usage; &
(b) adverse material usage; &
(c) favourable labour efficiency; &
(d) adverse labour efficiency; &
(e) favourable labour rate; &
(f ) adverse labour rate; &
(g) favourable variable overhead efficiency; &
(h) adverse variable overhead efficiency; &
REVISION QUESTIONS C1
160
CONTINUOUS OPERATION COSTING
2006.1
2.3 The budgeted contribution for last month was £43,900 but the following variances
arose:
2.4 Extracts from the standard cost card for product N are as follows.
During the latest period, 390 units of product N were produced. Details concerning
direct labour and variable production overhead are as follows:
Direct labour: amount paid for 5,720 hours ¼£68,640
Variable production overhead cost incurred ¼£16,280
Of the 5,720 labour hours paid for, 170 hours were recorded as idle time due to a
machine breakdown.
Calculate the following variances and tick the correct box to indicate whether each
variance is adverse or favourable:
Adverse Favourable
(a) the direct labour rate variance is £
&&
(b) the direct labour efficiency variance is £
&&
(c) the idle time variance is £
&&
(d) the variable production overhead expenditure
variance is £
&&
(e) the variable production overhead efficiency
variance is £
&&
2.5 An office worker who processes insurance claims is paid an hourly wage of £9 per
hour plus a bonus based on the time saved to process claims compared with a
standard time allowance. The bonus paid is 40 per cent of the time saved, at the basic
hourly rate.
£
Sales price variance 3,100 adverse
Sales volume contribution variance 1,100 adverse
Direct material price variance 1,986 favourable
Direct material usage variance 2,200 adverse
Direct labour rate variance 1,090 adverse
Direct labour efficiency variance 512 adverse
Variable overhead expenditure variance 1,216 favourable
Variable overhead efficiency variance 465 adverse
The actual contribution for last month was £
£
Direct labour: 14 hours @ £11 per hour 154
Variable production overhead: 14 hours @ £3 per hour 42
FUNDAMENTALS OF MANAGEMENT ACCOUNTING
161
CONTINUOUS OPERATION COSTING
2006.1
Last week the employee worked 30 hours and processed the following claims.
Number of claims processed Standard hours allowed per claim
Motor insurance 11 2
Household contents 15 1
Travel insurance 4 0.5
(a) The number of standard hours of work produced last week was ______________.
(b) The total wage payable to the employee for the week is (to the nearest
penny) £ __________.
Question 3 Standard costing in a service organisation
Carshine Services employs a number of people providing a car cleaning and valeting
service which operates in the car parks of local supermarkets and railway stations. In an
attempt to control costs and revenues the company has established the following standard
cost and fee per car cleaned and valeted:
Carshine services expects to clean and valet 3,000 cars each month. In March, a total of
2,800 cars were cleaned and the following costs and revenues were recorded:
Requirements
The following cost and sales variances will be recorded for March. Tick the box to indicate
whether each variance is adverse or favourable
£ per car
Materials: shampoo/polish: 0.5 litres @ £2.00 per litre 1.00
Labour: 0.75 hour @ £6 per hour
4.50
Total variable cost 5.50
Standard contribution
4.50
Standard fee per car
10.00
££
Sales revenue 28,050
Shampoo/polish: 1,460 litres 2,800
Labour: 2,020 hours
12,726
15,526
Contribution
12,524
Adverse Favourable
(a) material price: £
&&
(b) material usage: £
&&
(c) labour rate: £
&&
(d) labour efficiency: £
&&
(e) sales price: £
&&
(f ) sales volume contribution: £
&&
REVISION QUESTIONS C1
162
CONTINUOUS OPERATION COSTING
2006.1
Solutions to
Revision Questions
Solution 1
Every bonus scheme is different. In question 1.1 you will need to read the information
carefully to ensure that you understand the principles, then follow these principles to
calculate the correct bonus and do not forget to add the basic pay to the bonus to
arrive at the total amount payable!
1.1 Answer: ( B)
1.2 Answer: (A)
The minimum guaranteed wage is shown as a fixed cost up to a certain output.
Thereafter, the total cost increases at a steady rate, as piecework rates are paid for
increased output.
1.3 Answer: (A)
650 units should take ( 0.25) 162.5 active hours
But did take (7 hours 3days8 employees) (8 2 hours)
152.0 active hours
10.5 (F) h £12.00
Labour efficiency variance
£126 (F)
6
Minutes
Time allowed: 82 units 6 min 492
Time taken: 7 hours
420
Time saved
72
£
Bonus payable:
50% 72 min £8 per hour 4.80
Basic wage: 7 hours £8
56.00
Gross wages payable
60.80
163 2006.1
1.4 Answer: (B)
1.5 Answer: (C)
Idle time variance = 2 hours 8 employees = 16 hours idle £12 per hour = £192
adverse.
Solution 2
2.1 (b) Poor quality material could have led to higher wastage.
(e) Excess wastage causes an adverse material usage variance.
A higher output (a) would not in itself cause an adverse usage variance, because
the expected usage of material would be flexed according to the actual output
achieved.
Setting the original standard usage too high (c) is likely to lead to favourable usage
variances.
Higher market prices (d) would cause adverse material price variances.
2.2 (a) Highly skilled employees may use material more efficiently.
(c) Highly skilled employees may work more quickly.
(f ) Highly skilled employees are likely to be paid a higher hourly rate.
(g) Highly skilled employees may work more quickly.
2.3 The actual contribution for last month was £38,635.
Workings:
When working from the budgeted contribution to the actual contribution, adverse
variances are deducted from the budgeted contribution; favourable variances are
added to the budgeted contribution.
£(43,900 3,100 1,100 + 1,986 2,200 1,090 512 + 1,216 465) ¼£38,635.
2.4
(a) Direct labour rate variance ¼£5,720 adverse
(b) Direct labour efficiency variance ¼£990 adverse
(c) Idle time variance ¼£1,870 adverse
(d) Variable production overhead expenditure variance ¼£370 favourable
(e) Variable production overhead efficiency variance ¼£270 adverse
£
168 hours should cost ( £12.00) 2,016
But did cost
2,275
Labour rate variance
259 (A)
SOLUTIONS TO REVISION QUESTIONS C1
164
FURTHER STANDARD COSTING
2006.1
Workings
(c) Idle time variance ¼170 hours £11 standard rate ¼£1,870 adverse
2.5 (a) The number of standard hours of work produced last week was 39.
(b) The total wage payable to the employee for the week is £302.40.
Basic wage payable ¼30 hours £9 ¼£270
Bonus ¼40% 9 hours saved £9 ¼£32.40
Total wage payable ¼£270 + £32.40 ¼£302.40
(a) £
5,720 hours paid for should cost ( £11) 62,920
But did cost
68,640
Direct labour rate variance
5,720 adverse
(b) Hours
390 units should take ( 14) 5,460
But did take (active hours ¼5,720 170)
5,550
Variance in hours
90 adverse
standard labour rate per hour (£11)
Direct labour efficiency variance
£990 adverse
(d) Variable overhead cost of £
5,550 active hours should be ( £3) 16,650
Actual variable overhead cost
16,280
Variable production overhead expenditure variance
370 favourable
(e) Efficiency variance in hours
(from labour efficiency variance) 90 adverse
standard variable production overhead rate per hour £3
Variable production overhead efficiency variance
£270 adverse
Number
of claims
processed
Standard hours
allowed per claim
Standard hours
produced
Motor insurance 11 2 22
Household contents 15 1 15
Travel insurance 4 0.5
2
Total standard hours produced 39
Time taken
30
Time saved (hours)
9
FUNDAMENTALS OF MANAGEMENT ACCOUNTING
165
FURTHER STANDARD COSTING
2006.1
Solution 3
Do not be put off by the fact that this is a service organisation. An important point to
learn from this question is that the variance calculations in a service organisation are no
different from those in a manufacturing organisation.
Remember to indicate whether your calculated variances are adverse or favourable.
As an additional exercise, have a go at putting together all your calculated variances into
a statement which reconciles the budgeted contribution with the actual contribution for
the month.
(a) £120 favourable
(b) £120 adverse
(c) £606 adverse
(d) £480 favourable
(e) £50 favourable
(f) £900 adverse
Workings:
Material price variance
Material usage variance
Labour rate variance
Labour efficiency variance
£
1,460 litres should have cost ( £2) 2,920
But did cost
2,800
Material price variance
120 favourable
Litres
2,800 cars should have used ( 0.5 litres) 1,400
But did use
1,460
Variance in litres
60 adverse
standard price per litre (£2)
Material usage variance
£120 adverse
£
2,020 hours should have cost ( £6) 12,120
But did cost
12,726
Labour rate variance
606 adverse
SOLUTIONS TO REVISION QUESTIONS C1
166
FURTHER STANDARD COSTING
2006.1
Sales volume contribution variance
Solution to additional exercise
Statement reconciling the budgeted contribution for March with the actual
contribution achieved
£
Budgeted contribution (3,000 cars £4.50) 13,500
Sales volume contribution variance
(900)
Standard contribution from actual
volume achieved
12,600
Sales price variance
50
12,650
Cost variances
Material price 120
Material usage (120)
Labour rate (606)
Labour efficiency 480
(126)
Actual contribution (12,524)
Note: variances in brackets are adverse
Hours
2,800 cars should have taken ( 0.75 hour) 2,100
But did take
2,020
Variance in hours
80 favourable
standard rate per hour (£6)
Labour efficiency variance
£480 favourable
Sales price variance
£
Revenue for 2,800 cars should be (£10) 28,000
But actual revenue was
28,050
Sales price variance
50 favourable
Actual cars cleaned 2,800 cars
Budgeted cars cleaned
3,000 cars
Sales volume variance in cars 200 adverse
standard contribution per car £4.50
Sales volume contribution variance
£900 adverse
FUNDAMENTALS OF MANAGEMENT ACCOUNTING
167
FURTHER STANDARD COSTING
2006.1
7
Integrated
Accounting Systems
Integrated
Accounting Systems
7.1 Introduction
The systems that are used to account for costs will vary between organisations. Each
organisation will design its system to suit its own needs, taking into account factors such
as statutory accounting requirements and management information needs. The
accounting systems that are in use range from very simple manual systems to
sophisticated computerised systems capable of producing detailed reports on a regular or
an ad hoc basis.
In this chapter you will learn about the principal accounting entries within integrated
accounting systems. You will also be applying your knowledge of standard cost variances
when you learn how to record variances in an integrated accounting system.
7.2 An integrated accounting system
The CIMA Terminology defines integrated accounts as a ‘set of accounting records
that integrates both financial and cost accounts using a common input of data for
all accounting purposes’.
Therefore, in an integrated system the cost accounting function and the financial
accounting function are combined in one system, rather than separating the two sets of
accounts in two separate ledgers.
7
LEARNING OUTCOMES
After completing this chapter, you should be able to:
"
explain the principles of manufacturing accounts and the integration of the cost
accounts with the financial accounting system;
"
prepare a set of integrated accounts, given opening balances and appropriate
transactional information, and show standard cost variances.
171 2006.1
The main advantages of integrated systems are as follows:
(a) Duplication of effort is avoided and there is less work involved in maintaining the
system than if two sets of accounts are kept.
(b) There is no need for the periodic reconciliations of the two sets of accounts which are
necessary with non-integrated systems.
(c) Maintaining a single set of accounts avoids the confusion that can arise when two sets
of accounts are in existence which each contain different profit figures.
The main disadvantage of integrated accounts is that a single system is used to provide
information both for external and internal reporting requirements. The need to provide
information for statutory purposes may influence the quality of information which can be
made available for management purposes. For example, it may be more useful for
management purposes to have inventory valued on a LIFO basis. However, this would not
be acceptable for external reporting purposes and the latter requirement may prevail to the
detriment of management information.
7.3 Accounting for the cost of labour
Before we can begin to look at integrated accounts in operation we need to spend some
time discussing the detail of accounting for the cost of labour.
7.3.1 Deductions from employees’ wages
In the United Kingdom, employees pay income tax, usually under the pay-as-you-earn
( PAYE) system. Employers deduct income tax from gross wages before they are paid to the
employee. Employers also deduct a social security tax called National Insurance.Theemployee’s
National Insurance (NI ) contributions are deducted from gross wages to determine the net
wage to be paid to the employee. The employer will pay the deducted tax and NI to the
relevant authorities on behalf of the employee.
In addition, the employer pays employer’s NI contributions based on the level of the employee’s
wages. This, then, is an added cost of employment: it is often referred to as an employment-related cost.
Some organisations treat the cost of employer’s NI as an indirect cost. However, others
regard this related employment cost as part of the wage cost of each direct employee and
would share it among the tasks completed by adding it to the gross wages value, thus
treating it as part of direct wages cost.
7.3.2 Overtime premium
It is common for hours worked in excess of the basic working week to be paid at a higher
rate per hour. The extra amount is usually referred to as overtime premium. This overtime
premium may be caused by the specific request of a customer who requires a job to be
completed early or at a specific time, or may have resulted because of the organisation’s
need to complete work which would not be finished without the working of overtime. In
the situation caused by the customer, the customer should be advised that overtime would
be required and that this cost would be charged to them. Thus, in this situation, the
overtime premium can be clearly identified as being caused by that particular task and is
a direct cost which should be attributed to it. In other more general circumstances the cost
of the overtime premium is regarded as an indirect cost, even the premium that is paid to
direct workers, because it cannot be identified with a specific cost unit.
STUDY MATERIAL C1
172
INTEGRATED ACCOUNTING SYSTEMS
2006.1
7.3.3 Bonus earnings
The earning of bonuses, if paid on an individual task basis, can be clearly attributed to a
particular task and so would be a direct labour cost of this task. However, if the bonus system
accumulates the total standard time and hours worked for a particular pay period and then
calculates the bonus based on these totals, any bonus will usually be treated as an indirect cost.
7.3.4 Idle time
Idle-time payments are made when an employee is available for work and is being paid,
but is not carrying out any productive work. Idle time can arise for various reasons
including machine breakdown, lack of orders or unavailability of materials. Idle time must
be recorded carefully and management must ensure that it is kept to a minimum. Idle time
payments are treated as indirect costs in the analysis of wages.
7.3.5 Example: analysis of labour costs
The wages analysis for cost centre 456 shows the following summary of gross pay:
Direct employees Indirect employees
££
Basic pay ordinary hours 48,500 31,800
Overtime pay basic rate 1,600 2,800
premium 800 1,400
Bonuses paid
5,400 8,700
Total gross pay
56,300 44,700
Which of these are direct labour costs and which are indirect labour costs?
Solution
There is no indication that the overtime and bonuses can be specifically identified with any
particular cost unit. Therefore, the overtime premium and the bonuses are indirect costs, even
the amounts which were paid to direct employees. The wages can be analysed as follows:
It would not be ‘fair’ to charge the overtime premium of direct workers to the cost unit
which happened to be worked on during overtime hours if this unit did not specifically
cause the overtime to be incurred. Therefore, the premium is treated as an indirect cost of
all units produced in the period.
The direct labour cost of £50,100 can be directly identified with cost units and will be
charged to these units based on the analysis of labour time. The indirect costs cannot be
Direct labour cost Indirect labour cost
££
Basic pay 48,500 31,800
Overtime pay basic rate 1,600 2,800
premium 2,200
Bonuses paid
14,100
50,100 50,900
FUNDAMENTALS OF MANAGEMENT ACCOUNTING
173
INTEGRATED ACCOUNTING SYSTEMS
2006.1
identified with any particular cost unit and will be shared out over all units, using the
methods described in Chapter 3.
7.4 Integrated accounts in operation
The following example will demonstrate the double-entry principles involved in an
integrated system. Make sure that you understand which accounts are used to record
each type of transaction, before you move on to the next example, which contains
figures.
7.4.1 Example: the main accounting entries
in an integrated system
Figure 7.1 shows the flow of accounting entries within an integrated system for the following
transactions:
(i) The purchase of raw materials on credit terms.
Debit Raw materials control
Credit Payables (creditors) control
Creditors control
(i)
(iii)
(viii)
(iv)(a)
(iv)(b)
(x)
(x)
(x)
(ix)
(ix)
(ix)
(ii)
(v)
(vi)
(iv)
(iv)
(vii)
Raw materials control
Work in progress control
Finished goods control
Production o/head control
Cost of sales
Profit and loss
Sales
Debtors control
Provision for depreciation
Creditor for PAYE/NI
Cash
Wages control
Admin o/head control
Figure 7.1 Some of the accounting entries in an integrated system
STUDY MATERIAL C1
174
INTEGRATED ACCOUNTING SYSTEMS
2006.1
(ii) The issue to production of part of the consignment received in (i) above.
Direct materials costs are charged to the work in progress account.
(iii) The issue, as indirect materials, of part of the consignment received in (i) above.
Indirect production costs (in this case indirect materials costs) are collected in the
production overhead control account for later absorption into production costs.
(iv) A cash payment of wages, after deduction of PAYE and National Insurance
(a) to direct workers; and ( b) to indirect workers associated with production.
with the net amount of wages actually paid, after deductions.
with the deductions for PAYE and National Insurance.
The wages control account has now been debited with the gross amount of total
wages. This gross amount must then be charged out according to whether it is direct
or indirect wages. The direct wages are charged to work in progress (a). The indirect
wages are collected with other indirect costs in the production overhead control
account ( b) for later absorption into production costs.
Later in the period when the payment is made to the creditor for PAYE/NI, the
relevant entries will be:
Debit PAYE/NI payable
Credit Cash/Bank
(v) Electricity for production purposes, obtained on credit.
Debit Production overhead control
Credit Payables (creditors) control
(vi) Depreciation of machinery used for production.
Debit Production overhead control
Credit Provision for depreciation
These last two items are both production overhead costs which are being
accumulated for later absorption into production costs.
(vii) Cash paid for office expenses.
Debit Administration overhead control
Credit Cash account
Debit Work in progress control
Credit Raw materials control
Debit Production overhead control
Credit Raw materials control
Debit Wages control
Credit Cash/Bank
Debit Wages control
Credit PAYE/NI payable
FUNDAMENTALS OF MANAGEMENT ACCOUNTING
175
INTEGRATED ACCOUNTING SYSTEMS
2006.1
(viii) Absorption of production overhead, using a predetermined rate.
Debit Work in progress control
Credit Production overhead control
Once all of the production overhead has been accumulated in the overhead control
account, a predetermined rate is used to absorb it into the cost of work in progress.
The work in progress account now contains charges for direct costs and for absorbed
production overheads.
(ix) The sale, on credit, of all goods produced in the month.
Debit Receivables (debtors) control
Credit Sales account
with the sales value achieved.
Debit Finished goods control
Credit Work in progress control
This transfers the cost of the completed goods to the finished goods inventory
(stock) account. This is usually done in stages as production is completed during the
month. For demonstration purposes this has been simplified to show one transfer at
the end of the month.
Debit Cost of sales account
Credit Finished goods control
This transfers the cost of the goods sold from the inventory (stock) account. This
is also usually done in stages as inventory is sold during the month.
(x) The summary income statement ( profit and loss account) is prepared for the month.
Debit Profit and loss account
Credit Cost of sales account
Debit Profit and loss account
Credit Administration overhead control
(Alternatively, the administration overhead control account balance may first be
transferred to the cost of sales account and from there to the profit and loss account.)
This transfers the costs for the month to the income statement (profit and loss
account), to be offset against the sales revenue which is transferred from the sales account:
Debit Sales account
Credit Profit and loss account
This illustration has been simplified to demonstrate the main accounting flows. For
example, in practice there would be more items of production overhead and administration
overhead. There would also be expenditure on other types of overhead such as selling and
distribution costs. Control accounts would be opened for these costs and they would be dealt
with in the same way as the administration overhead in this example.
7.4.2 Accounting for under- or over-absorbed overheads
Take a moment to look back at the production overhead control account in the example
you have just studied.
STUDY MATERIAL C1
176
INTEGRATED ACCOUNTING SYSTEMS
2006.1
You will see that the production overhead control account has acted as a collecting
place for the production overheads incurred during the period. In this simplified example
the account has been debited with the following overhead costs:
Indirect materials issued from stores
The wages cost of indirect workers associated with production
The cost of electricity for production purposes
The depreciation of machinery used for production
At the end of the period the production overhead cost is absorbed into work in progress
costs using the predetermined overhead absorption rate. The amount absorbed is credited in
the production overhead control account and debited in the work in progress account.
The remaining balance on the production overhead control account represents the
amount of production overhead which is under-absorbed (debit balance) or over-absorbed
(credit balance).
If overheads are under-absorbed it effectively means that product costs have been
understated. It is not usually considered necessary to adjust individual unit costs and
therefore inventory values are not altered. However, the cost of units sold will have been
understated and therefore the under-absorption is charged to the income statement
(profit and loss account) for the period.
The reverse is true for any over absorption, which is credited in the income statement
(profit and loss account) for the period.
Some organisations do not charge or credit the under or over absorption to the profit
and loss account every period. Instead, the balance is carried forward in the control
account and at the end of the year the net balance is transferred to the profit and loss
account. This procedure is particularly appropriate when activity fluctuations cause under
and over absorptions which tend to cancel each other out over the course of the year.
Note that under-absorbed or over-absorbed overhead is sometimes referred to as
under-recovered or over-recovered overhead.
7.4.3 Example: integrated accounts
You should now be in a position to tackle a fully worked example on integrated accounts.
Although you would not be required to prepare a full set of ledger accounts in your
assessment, it is still important for you to work carefully through the example. This will
ensure that you have a sound knowledge of how to account for all of the main transactions
in an integrated accounting system.
Exercise 7.1
See if you can complete the relevant ledger accounts yourself before looking at the solution.
IA Ltd produces a product in two processes. Output from process 1 is transferred to
process 2 and from there to finished goods stores.
IA Ltd operates an integrated accounting system and, based on the data given below,
you are required to prepare the relevant ledger accounts for the month ended 31 October,
year 2, close the accounts at the end of the month and draw up the profit and loss account
for the period and the balance sheet as at 31 October year 2.
FUNDAMENTALS OF MANAGEMENT ACCOUNTING
177
INTEGRATED ACCOUNTING SYSTEMS
2006.1
Account balances at 1 October, year 2
£
Debtors/receivables 60,000
Creditors/payables 75,000
Provision for depreciation,
plant and machinery
60,000
Stocks:
Raw materials 350,000
Work in process 1 120,000
Work in process 2 150,000
Finished goods 30,000
Bank 31,000
Sales 500,000
Cost of sales 370,000
Administration overhead 60,000
Selling and distribution overhead 40,000
Production overhead, over-/
under-absorbed (credit balance
brought forward)
10,500
Share capital and reserves 735,500
Plant and machinery at cost 170,000
Transactions for the month ended 31 October, year 2 included:
£
Direct wages incurred:
Process 1 42,400
Process 2 64,600
Direct wages paid 100,000
Production salaries paid 85,000
Production expenses paid 125,000
Paid to creditors 165,000
Received from debtors 570,000
Administration overhead paid 54,000
Selling and distribution overhead paid 42,000
Materials purchased on credit 105,000
Materials returned to suppliers 5,000
Materials issued to:
Process 1 68,000
Process 2 22,000
Goods sold on credit:
At sales prices 550,000
At cost 422,400
Transfer from process 1 to process 2 242,200
Transfer from process 2 448,400
Provision for depreciation of plant and machinery is £4,000 for the month.
The predetermined overhead absorption rates are:
STUDY MATERIAL C1
178
INTEGRATED ACCOUNTING SYSTEMS
2006.1
Process 1 250% of direct wages cost
Process 2 150% of direct wages cost
Solution
The first step is to open a ledger account for each balance listed. Enter the opening balances,
which are all labelled as item 1 in the solution which follows. All of the other transaction
numbers relate to the explanatory notes which you will find at the end of the ledger accounts.
Debtors/receivables
££
1 Balance b/f 60,000 Bank 570,000
7 Sales
550,000 13 Balance c/f 40,000
610,000 610,000
Creditors/payables
££
Bank 165,000 1 Balance b/f 75,000
Raw materials 5,000 Raw materials 105,000
13 Balance c/f
10,000
180,000 180,000
Provision for depreciation
££
13 Balance c/f 64,000 1 Balance b/f 60,000
10 Production o/h control 4,000
64,000 64,000
Raw materials stock
££
1 Balance b/f 350,000 Creditors 5,000
Creditors 105,000 6 Process 1 68,000
6 Process 2 22,000
13 Balance c/f 360,000
455,000 455,000
Finished goods stock
££
1 Balance b/f 30,000 8 Cost of sales 422,400
9 Process 2
448,400 13 Balance c/f 56,000
478,400 478,400
FUNDAMENTALS OF MANAGEMENT ACCOUNTING
179
INTEGRATED ACCOUNTING SYSTEMS
2006.1
Work in process 1
££
1 Balance b/f 120,000 Process 2 242,200
2 Wages control 42,400 13 Balance c/f 94,200
6 Raw materials 68,000
11 Overhead control
106,000
336,400 336,400
Work in process 2
££
1 Balance b/f 150,000 9 Finished goods 448,400
2 Wages control 64,600 13 Balance c/f 127,300
6 Raw materials 22,000
Process 1 242,200
11 Overhead control
96,900
575,700 575,700
Bank
££
1 Balance b/f 31,000 3 Wages control 100,000
Debtors 570,000 4 Production overhead
control
85,000
5 Production overhead
control
125,000
Creditors 165,000
Admin. overhead 54,000
Selling overhead 42,000
13 Balance c/f 30,000
601,000 601,000
Sales
££
13 Profit and loss 1,050,000 1 Balance b/f 500,000
7 Debtors 550,000
1,050,000 1,050,000
Cost of sales
££
1 Balance b/f 370,000 13 Profit and loss 792,400
8 Finished goods
422,400
792,400 792,400
STUDY MATERIAL C1
180
INTEGRATED ACCOUNTING SYSTEMS
2006.1
The control accounts for wages and for production overheads are opened as ‘collecting
places’ for these costs. The wages can then be analysed and charged out as appropriate.
The production overhead can be absorbed into the work in progress accounts.
Administration overhead
££
1 Balance b/f 60,000 13 Profit and loss 114,000
Bank
54,000
114,000 114,000
Selling and distribution overhead
££
1 Balance b/f 40,000 13 Profit and loss 82,000
Bank
42,000
82,000 82,000
Production overhead over-/under-absorbed
££
12 Overhead control 11,100 1 Balance b/f 10,500
13 Profit and loss 600
11,100 11,100
Share capital and reserves
££
Balance c/f 796,500 1 Balance b/f 735,500
Profit for the period 61,000
796,500 796,500
Plant and machinery at cost
££
1 Balance b/f
170,000 13 Balance c/f 170,000
Wages control
££
3 Bank 100,000 2 Process 1 42,400
Balance c/f
7,000 2 Process 2 64,600
107,000 107,000
FUNDAMENTALS OF MANAGEMENT ACCOUNTING
181
INTEGRATED ACCOUNTING SYSTEMS
2006.1
Explanatory notes
1. These are the opening balances as given in the trial balance.
2. Direct wages incurred are credited to the wages control account and debited to the
relevant work in process account. This looks strange at first because there is not yet
any debit entry in the wages control account.
3. Now that the direct wages actually paid have been debited to the control account, you can see
that there is a difference of £7,000 between the wages paid and wages incurred. This
represents a £7,000 accrual for direct wages owing, which is carried down as a credit balance.
4. Production salaries are charged to the production overhead control account for later
absorption into work in process costs.
The production salaries could alternatively have been charged first to the wages
control account. They would then be transferred from there to the production
overhead account, so the net effect is the same.
5. Production expenses are also collected in the production overhead control account
for later absorption into work in process costs.
6. Direct materials issued from inventory (stock) are charged to the relevant work in
process account.
Materials used for indirect production purposes (there are none in this example)
would be debited to the production overhead control account.
7. The sales value of goods sold is credited to the sales account and debited to
receivables (debtors).
8. The cost of the goods sold is transferred from finished goods inventory (stock) to the
cost of sales account.
9. The output from process 2 is transferred to the finished goods inventory (stock) account.
10. The depreciation provision for plant and machinery is a production overhead cost. It
must therefore be collected in the production overhead control account for later
absorption into work in progress costs.
11. Once all of the transactions from the question data have been entered, the next step is
to absorb the production overhead into the two work in process accounts. Use the
predetermined overhead absorption rates that you are given.
Process 1: Wages £42,400 250% ¼ £106,000
Process 2: Wages £64,600 150% ¼ £96,900:
12. The last control account to be dealt with is the one which you opened as a collecting
place for production overhead costs. All of the production overhead costs incurred,
including depreciation, have been debited to this account. The production overheads
have been absorbed into the work in process accounts using the predetermined rates.
Production overhead control
££
4 Bank 85,000 11 Process 1 106,000
5 Bank 125,000 11 Process 2 96,900
10 Depreciation
4,000 12 Under-absorbed 11,100
214,000 214,000
STUDY MATERIAL C1
182
INTEGRATED ACCOUNTING SYSTEMS
2006.1
Therefore, the balance on this account represents the under- or over-absorbed
production overhead for the period. In this example it is transferred to a separate
account and accumulated to be transferred to the profit and loss account.
The debit balance on the production overhead control account means that the
overhead was under-absorbed for this month.
13. Now that all the transactions have been recorded the relevant balances can be transferred
to the profit and loss account (income statement) and balance sheet. Before you read on,
try to complete the final profit and loss account (income statement) and balance sheet for
yourself, using the ledger accounts we have produced.
The layout of your balance sheet might be different from ours; but it should balance!
Balance sheet as at 31 October year 2
££ £
Plant and machinery at cost 170,000
Provision for depreciation
64,000
106,000
Current assets
Raw material stock 360,000
Work in process 1 stock 94,200
Work in process 2 stock 127,300
Finished goods stock 56,000
Debtors 40,000
Bank
30,000
707,500
Current liabilities
Creditors 10,000
Accrued wages
7,000 17,000
690,500
796,500
Share capital and reserves
796,500
Profit and loss account for the period ended 31 October year 2
££
Sales 1,050,000
Cost of sales 792,400
Under absorbed production overhead
600
793,000
Gross profit 257,000
Administration overhead 114,000
Selling and distribution overhead
82,000
196,000
Profit to reserves
61,000
FUNDAMENTALS OF MANAGEMENT ACCOUNTING
183
INTEGRATED ACCOUNTING SYSTEMS
2006.1
How did you get on?
If this is the first time that you have studied integrated accounts, it is important that you
understand all of the entries in this example. Once you have checked each one carefully and
understood it, put the example aside for a few days and then return to try it again without
looking at the solution. You should be able to work all the way through without any errors (!)
7.5 Standard cost bookkeeping
In the remainder of this chapter you will learn how to record standard costs and variances
in the ledger accounts. To be able to study this material effectively you must have a sound
understanding of:
(a) the workings of an integrated accounting system;
(b) the calculation of cost variances in a standard costing system.
If you are not confident that you have a sound understanding of both of these subjects, then
you should return and study them carefully before you begin on this section of the chapter.
7.6 Recording variances in the ledger accounts
A ledger account is usually kept for each cost variance. As a general rule, all variances are
entered in the accounts at the point at which they arise. For example:
(a) labour rate variances arise when the wages are paid. Therefore, they are entered in the
wages control account. An adverse variance is debited in the account for wage rate
variance and credited in the wages control account. For a favourable variance the
entries would be the opposite way round;
(b) labour efficiency variances arise as the employees are working. Therefore, the efficiency
variance is entered in the work in progress account. An adverse variance is debited in
the account for labour efficiency variance and credited in the work in progress account.
For a favourable variance the entries would be the opposite way round.
7.6.1 General rules for recording variances
Although variations do exist, you will find the following general rules useful when you are
recording variances in the ledger accounts:
(a) The materials price variance is recorded in the materials stock account. This is the
procedure if the materials inventory (stock) is held at standard cost. We will learn more
about this later in the chapter.
(b) The labour rate variance is recorded in the wages control account.
(c) The ‘quantity’ variances, that is, material usage, labour efficiency and variable
production overhead efficiency, are recorded in the work in progress account.
(d) The variance for variable production overhead expenditure is usually recorded in the
production overhead control account.
(e) Sales values are usually recorded at actual amounts and the sales variances are not
shown in the ledger accounts.
STUDY MATERIAL C1
184
INTEGRATED ACCOUNTING SYSTEMS
2006.1
Remember that the amount of variance is recorded in the relevant variance account
(a debit for an adverse variance and a credit for a favourable variance). The ‘other
side’ of the entries are those detailed in this list.
7.6.2 The income statement (profit and loss account)
You will see from this list that all of the variances are eliminated before any entries are
made in the finished goods stock account. The finished goods stock is therefore held at
standard cost and the transfer to the cost of sales account and to the profit and loss
account will be made at standard cost.
At the end of the period the variance accounts are totalled and transferred to the profit and loss
account. Adverse variances are debited to profit and loss and favourable variances are credited.
In this way the actual cost (standard cost, plus or minus the variances) is charged against
the sales value in the profit and loss account for the period.
7.7 Standard cost bookkeeping: an example
Work carefully through the following example of integrated standard cost bookkeeping. It
will also give you some useful practice at calculating cost variances.
JC Ltd produces and sells one product only, product J, the standard variable cost of
which is as follows for one unit:
During April, the first month of the financial year, the following were the actual results for
production and sales of 800 units:
The material price variance is extracted at the time of receipt and the raw materials stores
control account is maintained at standard prices. The purchases, bought on credit, during
the month of April were:
X 9,000 kg at £20.50 per kg from K Ltd
Y 5,000 litres at £5.50 per litre from C plc
£
Direct material X: 10 kg at £20 200
Direct material Y: 5 litres at £6 30
Direct wages: 5 hours at £6 30
Variable production overhead
10
Total standard variable cost 270
Standard contribution
130
Standard selling price
400
££
Sales on credit: 800 units at £400 320,000
Direct materials:
X 7,800 kg 159,900
Y 4,300 litres 23,650
Direct wages: 4,200 hours 24,150
Variable production overhead
10,500
218,200
Contribution
101,800
FUNDAMENTALS OF MANAGEMENT ACCOUNTING
185
INTEGRATED ACCOUNTING SYSTEMS
2006.1
Assume no opening stocks, and no opening bank balance.
All wages and production overhead costs were paid from the bank during April.
You are required to:
(a) Calculate the variable cost variances for the month of April.
(b) Show all the accounting entries in T-accounts for the month of April. The work in
progress account should be maintained at standard variable cost and each balance on
the separate variance accounts is to be transferred to a profit and loss account (income
statement) which you are also required to show.
(c) Explain the reason for the difference between the actual contribution given in the
question and the contribution shown in your profit and loss account extract.
Exercise 7.2
See if you can calculate all the variances before you look at the solution. You might also like to try
to complete the bookkeeping entries yourself, using the earlier list of general rules to guide you.
Solution
(a) Direct material price variance
Material X £
9,000 kg purchased should have cost ( £20) 180,000
But did cost (9,000 £20.50)
184,500
Direct material price variance
4,500 adverse
Material Y £
5,000 litres purchased should have cost ( £6) 30,000
But did cost (5,000 £5.50)
27,500
Direct material price variance
2,500 favourable
Direct material usage variance
Material X kg
800 units produced should have used ( 10kg) 8,000
But did use
7,800
Variance in kg
200 favourable
standard price per kg (£20)
Direct material usage variance
£4,000 favourable
Material Y Litres
800 units produced should have used ( 5 litres) 4,000
But did use
4,300
Variance in litres
300 adverse
standard price per litre (£6)
Direct material usage variance
£1,800 adverse
STUDY MATERIAL C1
186
INTEGRATED ACCOUNTING SYSTEMS
2006.1
Direct labour rate variance
£
4,200 hours should have cost ( £6) 25,200
But did cost
24,150
Direct labour rate variance
1,050 favourable
Direct labour efficiency variance
Hours
800 units produced should have taken ( 5 hours) 4,000
But did take
4,200
Variance in hours
200 adverse
standard labour rate per hour (£6)
Direct labour efficiency variance
£1,200 adverse
Variable overhead expenditure variance
£
4,200 hours of variable overhead should cost ( £2) 8,400
But did cost
10,500
Variable overhead expenditure variance
2,100 adverse
Variable overhead efficiency variance
£
Variance in hours (from labour efficiency variance) 200 adverse
standard variable overhead rate per hour £2
Variable overhead efficiency variance
£400 adverse
(b) The easiest way to approach this question is probably to follow the production
through: deal first with the purchase and then the issue of the material; then move on
to deal with the information about the wages. Lastly, prepare the control account for
overheads, before dealing with the transfer from the work in progress account.
Numbers in brackets refer to the notes following the accounts.
Raw materials stores control
££
K Ltd: material X (1) 184,500 Direct material price variance:
C plc: material Y (2) 27,500 material X (1) 4,500
Direct material price variance: Work in progress (3)
material Y (2) 2,500 material X (7,800 £20) 156,000
material Y (4,300 £6) 25,800
Closing stock c/f 28,200
214,500 214,500
FUNDAMENTALS OF MANAGEMENT ACCOUNTING
187
INTEGRATED ACCOUNTING SYSTEMS
2006.1
K Ltd
££
Balance c/f
184,500 Raw materials
stores control (1)
184,500
C plc
££
Balance c/f
27,500 Raw materials
stores control (2)
27,500
Work in progress stock control
££
Raw material stores: (3) Direct material usage
variance: (3)material X 156,000
material Y 1,800material Y
25,800
Direct labour efficiency
variance (6)
1,200
Direct material usage
variance: (3)
Variable overhead efficiency
variance (7)
Finished goods stock: (8)
400
material X 4,000
800 units £270 216,000Wages control (5)
25,200
Production overhead control (7) 8,400
219,400 219,400
Wages control
££
Bank (4) 24,150 Work in progress
(4,200 £6) (5)
25,200
Labour rate variance (5)
1,050
25,200 25,200
Bank
££
Wages control (4) 24,150
Production overhead
control (7)
10,500
STUDY MATERIAL C1
188
INTEGRATED ACCOUNTING SYSTEMS
2006.1
Production overhead control
££
Bank (7) 10,500 Work in progress (7) 8,400
(4,200 £2)
Variable overhead expenditure
variance (7)
2,100
10,500 10,500
Finished goods stock control
££
Work in progress (8) 216,000 Cost of sales (8) 216,000
Cost of sales
££
Finished goods stock (8) 216,000 Profit and loss (8) 216,000
Sales
££
Profit and loss 320,000 Debtors 320,000
Debtors
££
Sales 320,000
Direct material price variance
££
Raw material stores control (1) 4,500 Raw material stores control (2) 2,500
Profit and loss (9) 2,000
4,500 4,500
Direct material usage variance
££
Work in progress: material Y (3) 1,800 Work in progress: material X (3) 4,000
Profit and loss (9)
2,200
4,000 4,000
FUNDAMENTALS OF MANAGEMENT ACCOUNTING
189
INTEGRATED ACCOUNTING SYSTEMS
2006.1
The profit and loss account could also be shown as a T-account. However, a vertical
presentation is probably preferable.
Profit and loss account for April (extract)
Direct labour rate variance
££
Profit and loss (9) 1,050 Wages control (5) 1,050
Direct labour efficiency variance
££
Work in progress control (6) 1,200 Profit and loss (9) 1,200
Variable overhead expenditure variance
££
Production overhead control (7) 2,100 Profit and loss (9) 2,100
Variable overhead efficiency variance
££
Production overhead control (7) 400 Profit and loss (9) 400
££ £
Sales 320,000
Cost of sales (8)
216,000
104,000
Cost variances
Direct material price (2,000)
Direct material usage
2,200
200
Direct labour rate 1,050
Direct labour efficiency
(1,200)
(150)
Variable production overhead expenditure (2,100)
Variable production overhead efficiency
(400)
(2,500)
(2,450)
Contribution
101,550
Note: Variances in brackets are adverse.
STUDY MATERIAL C1
190
INTEGRATED ACCOUNTING SYSTEMS
2006.1
Explanatory notes
1. The actual cost of material X purchases is debited to the raw materials stores control
and credited to K Ltd. The adverse price variance is credited to the raw materials stores
control and debited to the variance account. The net effect of these two entries is that
the material is held in the stores account at standard cost.
2. The actual cost of material Y purchases is debited to the raw materials stores control and
credited to C plc. To bring the stock value of material Y up to standard cost, the favourable
price variance is debited to the stores control account and credited to the variance account.
3. The standard cost of the actual material usage is transferred from the raw materials
stock to work in progress. The usage variances are transferred from work in progress to
the material usage variance account. An adverse variance is debited to the variance
account and credited to work in progress. A favourable variance is credited to the
variance account and debited to work in progress.
The net balance for materials cost in the work in progress account is now equal to
the standard material cost for 800 units. Check this for yourself.
4. The wages paid are collected in the control account.
5. The standard wages cost of the hours worked is debited to work in progress. The
favourable labour rate variance is credited to the variance account.
6. The adverse labour efficiency variance is transferred from work in progress to the
relevant variance account.
The net balance for wages cost in the work in progress account is now equal to the
standard wages cost for 800 units. Check this for yourself.
7. The variable production overhead paid is collected in the production overhead control
account. The standard variable overhead cost of the hours worked is then debited to
work in progress. The adverse variable overhead expenditure variance is debited to the
variance account.
The adverse variable overhead efficiency variance is transferred from work in
progress to the relevant variance account.
Notice the similarity between the accounting entries for labour and for variable
overhead.
8. The standard variable production cost of 800 units (800 £270 ¼£216,000) is
transferred from work in progress to finished goods stock. Since no stocks are held,
this amount is transferred at the end of the month to cost of sales, and from there to
the profit and loss account.
9. At the end of April, the balances on the variance accounts are transferred to the profit
and loss account.
(c) The difference between the actual contribution given in the question and the
contribution shown in the profit and loss account extract in the solution to part (b)
is £250.
This difference is caused by the treatment of the direct material price variance.
£
Actual contribution given in question 101,800
Contribution shown in solution to part (b)
101,550
Difference
250
FUNDAMENTALS OF MANAGEMENT ACCOUNTING
191
INTEGRATED ACCOUNTING SYSTEMS
2006.1
In the actual results given in the question, the material price variance on only the material
actually used has been charged against the sales value. In the bookkeeping entries in part (b),
the material price variances on all of the purchases for the month have been recorded and
transferred to the profit and loss account.
The difference is therefore represented by the price variance on the materials in
inventory (stock) at the end of April.
7.8 Valuing material inventory (stock) at actual cost
In chapter 5 you saw that the material price variance is calculated using a different
method if stock is valued at actual cost. If material stock had been valued at actual cost in
the previous example the material price variance would have been calculated as:
Direct material price variance
The raw materials stores control account would look like this:
Notice that the transfer to the work in progress account is the same as before, therefore
that account will not be altered by the raw material stock valuation method.
Direct material Purchases Usage Stock balance
Price variance
per unit
Price variance
in stock £
X 9,000 kg 7,800 kg 1,200 kg £20 £20.50 ¼(£0.50) (600)
Y 5,000 litres 4,300 litres 700 litres £6 £5.50 ¼£0.50
350
(250)
Note: Variances in brackets are adverse.
Material X £
7,800 kg used should have cost (£20) 156,000
But did cost
159,900
Direct material price variance
3,900 adverse
Material Y £
4,300 litres used should have cost (£6) 25,800
But did cost
23,650
Direct material price variance
2,150 favourable
Raw materials stores control
££
K Ltd: material X Work in progress:
(9,000 £20.50) 184,500 material X (7,800 £20) 156,000
C plc: material Y material Y (4,300 £6) 25,800
(5,000 £5.50) 27,500 Direct material price variance:
Direct material price variance: material X 3,900
material Y 2,150 Closing stock c/f
28,450
214,150 214,150
STUDY MATERIAL C1
192
INTEGRATED ACCOUNTING SYSTEMS
2006.1
Check that the raw material stock balance carried forward into May is correctly valued at
actual cost.
7.8.1 Which inventory (stock) valuation method
is generally preferred?
It is generally accepted that it is better to value the raw material stock at standard cost, for
the following reasons:
(a) The whole of the price variance is eliminated as soon as the raw materials are
purchased. This means that stocks are valued at a uniform rate and that the price
variances are highlighted earlier for management attention.
(b) Raw materials are often purchased in single batches, then broken into several smaller
batches for issue to production. If raw materials stocks are valued at actual cost, then a
separate variance calculation is required for each issue. With valuation at standard cost,
one single calculation is required on purchase.
7.9 Summary
Having read this chapter the main points that you should understand are as follows.
1. An integrated accounting system contains both financial and cost accounts and uses the
same data for all accounting purposes.
2. Overtime premium is the extra hourly rate paid for working above an agreed number of
hours in a period. It is usually an indirect cost unless the overtime has been worked at
the specific request of a customer.
3. The wages control account acts as a collecting place for wage costs. The direct wages
are then transferred to work in progress and the indirect wages are transferred to the
production overhead control account.
4. The production overhead control account act as a collecting place for production
overheads. At the end of the period the production overhead is absorbed into work in
progress using the predetermined overhead absorption rate. The balance on the
production overhead control account represents the under or over absorbed overhead
for the period.
5. As a general rule, in a standard cost bookkeeping system, variances are entered in the
accounts at the point at which they arise. A favourable variance is credited in the
relevant variance account. An adverse variance is debited in the relevant variance
account.
£
Material X: 1,200 kg £20.50 24,600
Material Y: 700 litres £5.50
3,850
Actual cost of material stock
28,450
FUNDAMENTALS OF MANAGEMENT ACCOUNTING
193
INTEGRATED ACCOUNTING SYSTEMS
2006.1
Revision Questions
Question 1 Multiple choice
1.1 A firm operates an integrated cost and financial accounting system. The accounting
entries for an issue of direct materials to production would be:
1.2 During a period £35,750 was incurred for indirect labour. In a typical cost ledger, the
double entry for this is:
1.3 In an integrated cost and financial accounting system, the accounting entries for
factory overhead absorbed would be:
1.4 At the end of a period, in an integrated cost and financial accounting system the
accounting entries for £18,000 overheads under-absorbed would be:
Debit Credit
(A) Work in progress control account Stores control account
( B) Finished goods account Stores control account
(C ) Stores control account Work in progress control account
( D) Cost of sales account Work in progress control account
Debit Credit
(A) Wages control account Overhead control account
( B) WIP control account Wages control account
(C ) Overhead control account Wages control account
( D) Wages control account WIP control account
Debit Credit
(A) Work in progress control account Overhead control account
( B) Overhead control account Work in progress control account
(C ) Overhead control account Cost of sales account
( D) Cost of sales account Overhead control account
Debit Credit
(A) Work in progress control account Overhead control account
( B) Profit and loss account Work in progress control account
(C ) Profit and loss account Overhead control account
( D) Overhead control account Profit and loss account
7
195 2006.1
1.5 In the cost ledger the factory cost of finished production for a period was £873,190.
The double entry for this is
1.6 XYZ Ltd operates an integrated accounting system. The material control account at
31 March shows the following information:
The £125,000 credit entry represents the value of the transfer to the
(A) cost of sales account.
( B) finished goods account.
( C) profit and loss account.
( D) work in progress account.
1.7 In an integrated cost and financial accounting system the correct entries for the
provision for depreciation of production machinery are:
1.8 Data for the finishing department for the last quarter are as follows.
Budgeted cost centre overhead £320,000
Actual cost centre overhead £311,250
Budgeted direct labour hours 40,000
Actual direct labour hours 41,500
The accounting entries to record the under- or over-absorbed overhead for the
quarter would be:
Debit Credit
(A) Overhead control account £20,750 Profit and loss account £20,750
( B) Overhead control account £8,750 Profit and loss account £8,750
(C ) Profit and loss account £20,750 Overhead control account £20,750
(D) Profit and loss account £8,750 Overhead control account £8,750
Debit Credit
(A) Cost of sales account Finished goods control account
( B) Finished goods control account Work in progress control account
(C ) Costing profit and loss account Finished goods control account
( D) Work in progress control account Finished goods control account
Material control account
££
Balance b/d 50,000 Production overhead control account 10,000
Creditors 100,000 ? 125,000
Bank
25,000 Balance c/d 40,000
175,000 175,000
Debit Credit
(A) Provision for depreciation account Work in progress account
( B) Work in progress account Provision for depreciation account
(C ) Overhead control account Provision for depreciation account
(D) Provision for depreciation account Overhead control account.
REVISION QUESTIONS C1
196
INTEGRATED ACCOUNTING SYSTEMS
2006.1
1.9 Q Ltd uses an integrated standard costing system. In October, when 2,400 units of
the finished product were made, the actual material cost details were:
The standard cost details are that two units of the material should be used for each
unit of the completed product, and the standard price of each material unit is £4.70.
The entries made in the variance accounts would be:
1.10 The bookkeeping entries in a standard cost system when the actual price for raw
materials is less than the standard price are:
1.11 A firm uses standard costing and an integrated accounting system. The double entry
for an adverse material usage variance is:
1.12 In a standard cost bookkeeping system, when the actual hourly rate paid for labour
is less than the standard hourly rate, the double entry to record this is:
(A) debit wages control account; credit labour rate variance account.
(B) debit work in progress control account; credit labour rate variance account.
(C) debit labour rate variance account; credit wages control account.
(D) debit labour rate variance account; credit work in progress control account.
Material purchased 5,000 units @ £4.50 each
Material used 4,850 units
Material price
variance account
Material usage
variance account
(A) Debit £970 Debit £225
(B) Debit £1,000 Debit £225
(C) Credit £970 Debit £235
(D) Credit £1,000 Debit £235
Debit Credit
(A) Raw materials control account Raw materials price variance account
(B) WIP control account Raw materials control account
(C) Raw materials price variance account Raw materials control account
(D) WIP control account Raw materials price variance account
Debit Credit
(A) Stores control account Work in progress control account
(B) Material usage variance account Stores control account
(C) Work in progress control account Material usage variance account
(D) Material usage variance account Work in progress control account
FUNDAMENTALS OF MANAGEMENT ACCOUNTING
197
INTEGRATED ACCOUNTING SYSTEMS
2006.1
1.13 Gross wages incurred in department 1 in June were £54,000. The wages analysis
shows the following summary breakdown of the gross pay:
What is the direct wages cost for department 1 in June?
(A) £25,185
(B) £30,625
(C) £34,685
(D) £36,065
1.14 A manufacturing firm is very busy and overtime is being worked.
The amount of overtime premium contained in direct wages would normally be
classed as:
(A) part of prime cost.
( B) production overheads.
(C) direct labour costs.
(D) administrative overheads.
Question 2 Short objective-test questions
2.1 A company purchased materials costing £30,000. Of these, materials worth £1,000
were issued to the maintenance department and materials worth £22,000 were issued
to the production department. Which of the following accounting entries would arise
as a result of these transactions? (Tick all that are correct.)
£
(a) Debit Raw materials control 29,000 &
( b) Debit Raw materials control 30,000 &
(c) Debit Work in progress control 22,000 &
(d) Debit Work in progress control 23,000 &
(e) Debit Work in progress control 30,000 &
(f ) Debit Production overhead control 1,000 &
( g) Credit Raw materials control 23,000 &
(h) Credit Raw materials control 30,000 &
Paid to direct labour Paid to indirect labour
££
Ordinary time 25,185 11,900
Overtime
basic pay 5,440 3,500
premium 1,360 875
Shift allowance 2,700 1,360
Sick pay
1,380 300
36,065 17,935
INTEGRATED ACCOUNTING SYSTEMS
REVISION QUESTIONS C1
198
2006.1
2.2 Look at the following account and then identify whether statements (a) to (c) are true
or false.
2.3 The production overhead absorption rate is £3 per direct labour hour. During the
period 23,000 direct labour hours were worked.
(a) In the production overhead control account for the period shown above, the
value to be inserted at A is £
(b) Production overhead for the period was:
(c) The value of the under/over absorption was £
2.4 Details of the production wages for a company last period are as follows.
Wages control account
££
Bank 82,500 Work in progress control 52,500
PAYE/NI creditor
9,500 Production overhead control 39,500
92,000 92,000
True False
(a ) Gross wages for the period amounted to £82,500. &&
( b) Indirect wages incurred amounted to £39,500. &&
(c ) Direct wages incurred amounted to £92,000. &&
Production overhead control account
££
Wages control 44,000 Work in progress control A
Bank 22,000
Depreciation 8,000
Raw materials control
2,000
76,000
under-absorbed &
over-absorbed &
Gross wages PAYE/NI
£000 £000 £000
Direct wages paid 40 10 30
Indirect wages paid 20 6 14
FUNDAMENTALS OF MANAGEMENT ACCOUNTING
199
INTEGRATED ACCOUNTING SYSTEMS
2006.1
Which of the following accounting entries would be used to record this data? (Tick
all that are correct.)
2.5 Is the following statement true or false?
If material stock is valued at standard cost then the material price variance
calculation should be based on the materials actually used during the period.
True &
False &
2.6 Stocks of material W are valued at their standard price of £ 7 per kilogram. Last
period, 900 kg of W were purchased for £5,400, of which 800 kg were issued to
production. Which of the following accounting entries would arise as a result of
these transactions? ( Tick all that apply.)
Question 3 Cost bookkeeping
D Ltd operates an integrated accounting system, preparing its annual accounts to
31 March each year. The following balances have been extracted from its trial balance at
31 October, year 3:
£000
(a) Debit Wages control 44 &
(b) Debit Work in progress 30 &
(c) Debit Work in progress 40 &
(d) Debit Production overhead control 14 &
(e) Debit Production overhead control 20 &
(f ) Debit Wages control 16 &
(g) Debit Wages control 60 &
(h) Credit Bank 44 &
(i) Credit Wages control 60 &
(j) Credit PAYE/NI creditor 16 &
(k) Credit Bank 60 &
£
Raw material control account 34,789 Dr
Wages control account 5,862 Cr
Production overhead control account 3,674 Cr
Work in progress control account 13,479 Dr
£
(a) Raw material stock 5,400 debit &
(b) Raw material stock 6,300 debit &
(c) Work in progress 4,800 debit &
(d) Work in progress 5,600 debit &
(e) Material price variance 800 credit &
(f ) Material price variance 800 debit &
(g) Material price variance 900 credit &
(h) Material price variance 900 debit &
REVISION QUESTIONS C1
200
INTEGRATED ACCOUNTING SYSTEMS
2006.1
During the first week of November, year 3, the following transactions occurred:
An analysis of the wages incurred shows that £5,200 is direct wages.
Requirements
(a) The balance shown on the production overhead control account means that the
production overhead at 31 October was:
( b) The raw material control account has been prepared for the first week of November:
The values that would be entered as A, B and C would be:
(c) The wages control account has been prepared for the first week of November:
The values that would be entered as A, B and C would be:
£
Purchased materials on credit 4,320
Incurred wages 6,450
Issued direct materials to production 2,890
Issued indirect materials to production 560
Incurred production overheads on credit 1,870
Absorbed production overhead cost 3,800
Cost of units completed 12,480
Paid wages 5,900
under-absorbed &
over-absorbed &
Raw material control account
££
Balance b/d 34,789 Work in progress B
Creditors A Production overhead C
Balance c/d 35,659
Wages control account
££
Bank A Balance b/d 5,862
Work in progress B
Production overhead C
FUNDAMENTALS OF MANAGEMENT ACCOUNTING
201
INTEGRATED ACCOUNTING SYSTEMS
2006.1
(d) At the end of the week, the balance brought down on the production overhead control
account will be a:
The value of the balance will be £
(e) The work in progress control account has been prepared for the first week of
November:
The values shown in the account as A, B, C and D are:
debit balance &
credit balance &
Work in progress control account
££
Balance b/d 13,479 Finished goods D
Raw materials A Balance c/d 12,889
Wages B
Production overhead
C
REVISION QUESTIONS C1
202
INTEGRATED ACCOUNTING SYSTEMS
2006.1
Solutions to
Revision Questions
Solution 1
If you are having trouble identifying the correct entries for each type of transaction, look
back to the flowchart of entries at the beginning of this chapter to refresh your memory.
Take your time and think carefully before selecting the correct option. In many cases,
one of the distractors states the correct accounts but the entries are the ‘wrong way
round’. It is easy to rush into selecting the wrong option.
An adverse variance is always debited in the relevant variance account. A favourable
variance is always credited in the variance account.
1.1 Answer: (A)
Direct costs of production are debited to the work in progress control account.
1.2 Answer: (C)
Indirect costs, including indirect labour, are collected in the debit side of the
overhead control account pending their later absorption into work in progress.
1.3 Answer: (A)
The factory overhead is first collected in the overhead control account. It is then
absorbed into production costs by debiting the work in progress account using the
predetermined overhead absorption rate.
1.4 Answer: (C)
Under-absorbed overhead is transferred from the overhead control account as a
debit to the income statement ( profit and loss account).
1.5 Answer: ( B)
Answer (A) is the double entry for the production cost of goods sold. Answer (C) is
also the entry for the production cost of goods sold, if a cost of sales account is not
used. Answer (D) has entries in the correct accounts but they are reversed.
1.6 Answer: (D)
Materials are issued from stores as either direct materials (to work in progress) or
indirect materials (charged to the production overhead control account). The entry
for the issue of indirect materials is already shown ( £10,000 to production
7
203 2006.1
overhead). Therefore, the £125,000 must be the value of the issue of direct materials
to work in progress.
1.7 Answer: (C)
The provision for depreciation of production machinery is a production overhead
cost. Therefore, it is debited to the production overhead control account to be
accumulated with all other production overheads for the period. At the end of the
period the production overhead will be absorbed into work in progress using the
predetermined overhead absorption rate.
1.8 Answer: (A)
Overhead absorption rate ¼£320,000/40,000 ¼£8 per direct labour hour
£
Overhead absorbed ¼£8 41,500 332,000
Overhead incurred
311,250
Over absorption
20,750
The over absorption is credited to the income statement ( profit and loss account)
and debited to the overhead control account.
1.9 Answer: (D)
5,000 £0.20 ¼£1,000 (F) credited to variance account
50 units @ £4.70 (standard price) ¼£235(A) debited to variance account
1.10 Answer: (A)
If the actual price for raw materials is less than the standard price then the raw
material price variance is favourable. The variance account would therefore be
credited. The corresponding debit entry is made in the raw materials control account.
1.11 Answer: (D)
An adverse variance is debited to the relevant variance account. This leaves us with
options (B) or (D). The usage variance is eliminated where it arises, that is, in the
work in progress account. Therefore, (D) is the correct answer.
1.12 Answer: (A)
The actual hourly rate is less than standard. Therefore, the rate variance is
favourable and is credited to the variance account.
Price variance
5,000 units should cost each 4.70
But actually cost
4.50
Saving
0.20
Usage variance Material units
2,400 finished units should use 4,800
Actual material usage
4,850
Which is an extra
50 units
SOLUTIONS TO REVISION QUESTIONS C1
204
INTEGRATED ACCOUNTING SYSTEMS
2006.1
1.13 Answer: (B)
£25,185 + £5,440 ¼£30,625. The only direct costs are the wages paid to direct
workers for ordinary time, plus the basic pay for overtime. Overtime premium and
shift allowances are usually treated as overheads. However, if and when the
overtime and shiftwork are incurred specifically for a particular cost unit, they are
classified as direct costs of that cost unit. Sick pay is treated as an overhead and is
therefore classified as an indirect cost.
1.14 Answer: (B)
Overtime premium is usually treated as an overhead cost if the overtime cannot be
specifically indentified with a particular cost unit.
Solution 2
2.1 The correct entries are:
( b) The purchased materials are debited in the raw materials control account.
(c) The direct materials are issued to the production department (work in progress).
(f ) Materials issued to maintenance are indirect materials, debited to the production
overhead control account.
( g) The total amount of materials issued is credited in the materials control account.
2.2 (a) False. Gross wages are £92,000.
(b) True. Indirect wages are transferred to the production overhead control account.
(c) False. Direct wages are £52,500: the amount transferred to work in progress.
2.3 (a) The value to be inserted at A is £69,000 323,000 hours)
( b) Production overhead for the period was under-absorbed (see workings in (c)).
2.4 Remember that the wages control account acts as a collecting place for the gross wages
before they are transferred to work in progress or to production overhead control,
according to whether they are direct wages or indirect wages. The gross wages are made up
of two parts: the net wages that are paid from the bank, plus the PAYE/NI deductions.
The correct entries are:
The total gross wages have now been debited to the wages control account.
2.5 False. When material stock is valued at standard, the material price variance is based
on the materials purchased.
£
(c) Overhead incurred 76,000
Overhead absorbed into production
69,000
Overhead under-absorbed
7,000
(a) and ( h) The net wages paid are ‘collected’ in the wages control
account and credited to the bank.
(f ) and ( j) The deductions are ‘collected’ in the wages control account
and credited to the PAYE/NI creditor.
(c), (e) and (i) The gross wages are transferred to work in progress or to
production overhead control according to whether they are
direct or indirect wages.
FUNDAMENTALS OF MANAGEMENT ACCOUNTING
205
INTEGRATED ACCOUNTING SYSTEMS
2006.1
2.6 (b) Standard price of purchases is debited to the stock account (900 £ 7).
(d) Standard price of material issues is debited to work in progress (800 £ 7).
(g) Favourable material price variance is credited to variance account:
£
900 kg purchased should cost
( £7)
6,300
But did cost
5,400
Material price variance
900 favourable
Solution 3
Use the flowchart of entries at the beginning of this chapter if you need help with
remembering the correct double entry for each item.
(a) The credit balance shown on the production overhead control account means that
there was over-absorption of production overhead at 31 October. A debit balance
would have indicated an under-absorption of production overheads at that date.
(b) A £4,320
B £2,890
C £560
(c) A £5,900
B £5,200
C £1,250
Workings:
(d) At the end of the week, the balance on the production overhead control account
will be a credit balance of £3,794.
Working:
* Production overhead absorbed is transferred to work in progress. The over-
absorbed balance is now £3,794, which is carried down to the next week.
(e) A £2,890
B £5,200
C £3,800
D £12,480.
£
Wages incurred 6,450
Direct wages to WIP
5,200
Indirect wages to production overhead
1,250
Production overhead control account
££
Raw materials 560 Balance b/d 3,674
Wages 1,250 Work in progress* 3,800
Creditors 1,870
Balance c/d
3,794
7,474 7,474
INTEGRATED ACCOUNTING SYSTEMS
2006.1
SOLUTIONS TO REVISION QUESTIONS C1
206
8
Specific Order Costing
Specific Order
Costing
8.1 Introduction
Every organisation will have its own costing system with characteristics which are unique
to that particular system. However, although each system might be different, the basic
costing method used by the organisation is likely to depend on the type of activity that the
organisation is engaged in. The costing system would have the same basic characteristics as
the systems of other organisations which are engaged in similar activities.
Specific order costing methods are appropriate for organisations which produce cost
units which are separately identifiable from one another. Job costing, batch costing and
contract costing are all types of specific order costing that you will learn about in this
chapter. In organisations which use these costing methods, each cost unit is different from
all others and each has its own unique characteristics.
8.2 Job costing
Job costing applies where work is undertaken according to specific orders from customers
to meet their own special requirements. Each order is of relatively short duration. For
example, a customer may request the manufacture of a single machine to the customer’s
own specification. Other examples, this time from service organisations, might be the
repair of a vehicle or the preparation of a set of accounts for a client.
The job costing method can also be applied to monitor the costs of internal work done
for the organisation’s own benefit. For example, job cost sheets can be used to collect the
costs of property repairs carried out by the organisation’s own employees, or they may be
used in the costing of internal capital expenditure jobs.
8
LEARNING OUTCOMES
After completing this chapter, you should be able to:
"
compare and contrast job, batch and contract costing;
"
prepare ledger accounts for job and batch costing systems;
"
prepare ledger accounts for contract costs
209 2006.1
8.2.1 Job cost sheets and databases
The main feature of a job costing system is the use of a job cost sheet or job card which is
a detailed record used to collect the costs of each job. In practice this would probably be a
file in a computerised system but the essential feature is that each job would be given a
specific job number which identifies it from all other jobs. Costs would be allocated to this
number as they are incurred on behalf of the job. Since the sales value of each job
can also be separately identified, it is then possible to determine the profit or loss on
each job.
The job cost sheet would record details of the job as it proceeds. The items recorded
would include:
job number;
description of the job; specifications, etc.;
customer details;
estimated cost, analysed by cost element;
selling price, and hence estimated profit;
delivery date promised;
actual costs to date, analysed by cost element;
actual delivery date, once the job is completed;
sales details, for example, delivery note no., invoice no.
An example of a job cost sheet prepared for a plumbing job is shown in Figure 8.1. This
job would have been carried out on the customer’s own premises. The sheet has a separate
section to record the details of each cost element. There is also a summary section where
the actual costs incurred are compared with the original estimate. This helps managers to
control costs and to refine their estimating process.
8.2.2 Collecting the direct costs of each job
(a) Direct labour
The correct analysis of labour costs and their attribution to specific jobs depends on the
existence of an efficient time recording and analysis system. For example, timesheets may
be used to record how each employee’s time is spent, using job numbers where
appropriate to indicate the time spent on each job. The wages cost can then be charged
to specific job numbers (or to overhead costs, if the employee was engaged on
indirect tasks). Figure 8.1 shows that a total of nine direct labour hours were worked by
two different employees on job number 472.
(b) Direct material
All documentation used to record movements of material within the organisation should
indicate the job number to which it relates.
For example a material requisition note, which is a formal request for items to be issued
from stores, should have a space to record the number of the job for which the material is
being requisitioned. If any of this material is returned to stores, then the material returned
note should indicate the original job number which is to be credited with the cost of the
returned material. Figure 8.1 shows that two separate material requisitions were raised for
material used on job number 472.
STUDY MATERIAL C1
210
SPECIFIC ORDER COSTING
2006.1
Sometimes items of material might be purchased specifically for an individual job,
without the material first being delivered to general stores and then requisitioned from
stores for the job. In this situation the job number must be recorded on the supplier’s
invoice or on the relevant cash records. This will ensure that the correct job is charged
with the cost of the material purchased.
(c) Direct expenses
Although direct expenses are not as common as direct material and direct labour costs, it is
still essential to analyse them and ensure that they are charged against the correct job
number.
For example, if a machine is hired to complete a particular job, then this is a direct
expense of the job. The supplier’s invoice should be coded to ensure that the expense is
Estimate no.: Job description:
JOB COST SHEET
MATERIALS PRODUCTION OVERHEADLABOUR
Job no.: 472
897 Instal shower
Model no. 5856
Date started:
15 June 20 × 6
Details: Mrs. P. Johnson
01734 692174
30 Hillside, Whyteham
Price estimate: £330
Date Date Overhead
absorption rate
Cost
ctr
Cost
£
Estimate
£
Actual
£
Hrs Rate £ £HoursValue
£
Price
£
QtyReq.
no.
Emp.
no.
15/6 15/6 3 1 10 10.006.20
134.20
3.10
Total c/f
Total c/f
Job card completed by:
EXPENSES
Description
90.00
JOB COST SUMMARY
Cost element
80.0090.00
Direct labour b/f
50.7052.06
Job profit/loss)
330.00330.00
Price estimate
279.30277.94
Total cost
13.3013.24
Admin. o/h (5%)
266.00264.70
Total production cost
36.0040.50
Production o/h b/f
230.00224.20
Total direct cost
Direct expenses b/f
150.00134.20
Direct materials b/f
Total c/f 40.50Total c/f
2644 12
14/6 15/6 4.504 8 10 80.00 40.509128.00128.001641 17
Figure 8.1 Job cost sheet
FUNDAMENTALS OF MANAGEMENT ACCOUNTING
211
SPECIFIC ORDER COSTING
2006.1
charged to the job. Alternatively, if cash is paid, then the cash book analysis will show the
job number which is to be charged with the cost. We can see from Figure 8.1 that no
direct expenses were incurred on behalf of job number 472.
8.2.3 Attributing overhead costs to jobs
(a) Production overheads
The successful attribution of production overhead costs to cost units depends on the
existence of well-defined cost centres and appropriate absorption bases for the overhead
costs of each cost centre.
It must be possible to record accurately the units of the absorption base which are
applicable to each job. For example if machine hours are to be used as the absorption
base, then the number of machine hours spent on each job must be recorded on the job
cost sheet. The relevant cost centre absorption rate can then be applied to produce a fair
overhead charge for the job.
The production overhead section of the job cost sheet in Figure 8.1 shows that the absorption
rate is £4.50 per labour hour. The labour analysis shows that nine hours were worked on
this job, therefore the amount of production overhead absorbed by the job is £40.50.
(b) Non-production overheads
The level of accuracy achieved in attributing costs such as selling, distribution and
administration overheads to jobs will depend on the level of cost analysis which an
organisation uses.
Many organisations simply use a predetermined percentage to absorb such costs, based
on estimated levels of activity for the forthcoming period. The following example will
demonstrate how this works.
Example
A company uses a predetermined percentage of production cost to absorb distribution costs into the total
cost of its jobs. Based on historical records and an estimate of activity and expenditure levels in the
forthcoming period, they have produced the following estimates:
The plumbing company that has produced the job cost sheet in Figure 8.1 uses a
predetermined percentage of five per cent of total production cost to absorb
administration overhead into job costs. You can see the calculations in the job cost
summary on the sheet.
The use of predetermined rates will lead to the problems of under- or over-absorbed
overhead which we discussed in earlier chapters. The rates should therefore be carefully
monitored throughout the period to check that they do not require adjusting to more
accurately reflect recent trends in costs and activity.
Estimated distribution costs to be incurred £13,300
Estimated production costs to be incurred on all jobs £190,000
Therefore, predetermined overhead absorption rate for
distribution costs ¼£13,300/£190,000 100% ¼7% of production costs
STUDY MATERIAL C1
212
SPECIFIC ORDER COSTING
2006.1
8.2.4 A worked example
The following example will help you to practise presenting a cost analysis for a specific job.
Jobbing Limited manufactures precision tools to its customers’ own specifications. The
manufacturing operations are divided into three cost centres: A, B and C.
An extract from the company’s budget for the forthcoming period shows the following
data:
Cost centre
Budgeted production
overhead
Basis of production overhead
absorption
A £38,500 22,000 machine hours
B £ 75,088 19,760 machine hours
C £40,964 41,800 labour hours
Job number 427 was manufactured during the period and its job cost sheet reveals the
following information relating to the job:
Direct material requisitioned £6,780.10
Direct material returned to stores £39.60
Direct labour recorded against job number 427:
Cost centre A: 146 hours at £4.80 per hour
Cost centre B: 39 hours at £5.70 per hour
Cost centre C: 279 hours at £6.10 per hour
Special machine hired for this job: hire cost £59.00
Machine hours recorded against job number 427:
Cost centre A: 411 hours
Cost centre B: 657 hours
Price quoted and charged to customer,
including delivery
£17,200
Jobbing Limited absorbs non-production overhead using the following predetermined
overhead absorption rates:
You are required to present an analysis of the total cost and profit or loss attributable to
job number 427.
Solution
First, we need to calculate the predetermined overhead absorption rates for each of the
cost centres, using the basis indicated.
Administration and general overhead 10% of production cost
Selling and distribution overhead 12% of selling price
FUNDAMENTALS OF MANAGEMENT ACCOUNTING
213
SPECIFIC ORDER COSTING
2006.1
Now we can prepare the cost and profit analysis, presenting the data as clearly as possible.
8.2.5 Preparing ledger accounts for job costing systems
In job costing systems a separate work in progress account is maintained for each job, as
well as a summary work in progress control account for all jobs worked on in the period.
The best way to see how this is done is to work carefully through the following exercise
and ensure that you understand each entry that is made in every account. You will need to
apply the principles of integrated accounts that you learned in the previous chapter.
Cost and profit analysis: job number 427 ££
Direct material (note 1) 6,740.50
Direct labour:
Cost centre A 146 hours £4.80 700.80
Cost centre B 39 hours £5.70 222.30
Cost centre C 279 hours £6.10
1,701.90
2,625.00
Direct expenses: hire of machine
59.00
Prime cost 9,424.50
Production overhead absorbed:
Cost centre A 411 hours £1.75 719.25
Cost centre B 657 hours £3.80 2,496.60
Cost centre C 279 hours £0.98
273.42
3,489.27
Total production cost 12,913.77
Administration and general overhead
(10% £12,913.77)
1,291.38
Selling and distribution overhead
(12% £17,200)
2,064.00
Total cost 16,269.15
Profit
930.85
Selling price
17,200.00
Note 1
The figure for material requisitioned has been reduced by the amount of returns to give
the correct value of the materials actually used for the job.
Cost centre A ¼
£38;500
22;000
¼ £1:75 per machine hour
Cost centre B ¼
£75;088
19;760
¼ £3:80 per machine hour
Cost centre C ¼
£40;964
41;800
¼ £0:98 per labour hour
STUDY MATERIAL C1
214
SPECIFIC ORDER COSTING
2006.1
Exercise 8.1
JC Limited operates a job costing system. All jobs are carried out on JC’s own premises
and then delivered to customers as soon as they are completed.
Direct employees are paid £10 per hour and production overhead is absorbed into job
costs using a predetermined absorption rate of £24 per hour. General overhead is charged
to profit and loss on completed jobs using a rate of 12 per cent of total production cost.
Details of work done during the latest period are as follows.
Work in progress at beginning of period
Job number 308 was in progress at the beginning of the period.
Activity during the period
Job numbers 309 and 310 were commenced during the period.
The following details are available concerning all work done this period.
Job number: 308 309 310
Direct materials issued from stores £169 £2,153 £452
Excess materials returned to stores £23
Direct labour hours worked 82 53 28
Status of job at the end of the period Completed Completed In progress
Invoice value £9,900 £6,870
Cost of material transferred from job 309 to job 310 £43
Production overhead cost incurred on credit £4,590
General overhead cost incurred on credit £1,312
Requirements
(a) Prepare the ledger account for the period for each job, showing the production cost of
sales transferred on completed jobs.
(b) Prepare the following accounts for the period:
work in progress control
production overhead control
general overhead control
overhead under- or over-absorbed control
profit and loss
(c) Calculate the profit on each of the completed jobs.
Job number 308
Cost incurred up to beginning of period:
£
Direct material 1,790
Direct labour 960
Production overhead absorbed
2,304
Production cost incurred up to beginning
of period
5,054
FUNDAMENTALS OF MANAGEMENT ACCOUNTING
215
SPECIFIC ORDER COSTING
2006.1
Solution
(a) The figures in brackets refer to the explanatory notes below the accounts.
Job 308
££
Balance b/f (1) 5,054 Production cost of sales 8,011
Material stores 169
Wages control (82 £10) 820
Production overhead (82 £24)
1,968
8,011 8,011
Job 309
££
Material stores 2,153 Material stores (2) 23
Wages control (53 £10) 530 Job 310 (3) 43
Production overhead (53 £24)
1,272 Production cost of sales 3,889
3,955 3,955
Job 310
££
Job 309 (3) 43 Balance c/f (4) 1,447
Material stores 452
Wages control (28 £10) 280
Production overhead (28 £24)
672
1,447 1,447
(b)
Work in progress control
££
Balance b/f (1) 5,054 Material stores control (2) 23
Material stores control (5) 2,774 Production cost of sales to
Wages control (163 hours £10) 1,630 profit and loss (6) 11,900
Prod’n o’head control (163 £24)
3,912 Balance c/f (7) 1,447
13,370 13,370
Production overhead control
££
Creditors control (8) 4,590 Work in progress control (9) 3,912
Overhead under- or over-absorbed
control (10) 678
4,590 4,590
STUDY MATERIAL C1
216
SPECIFIC ORDER COSTING
2006.1
General overhead control
££
Creditors control (8) 1,312 General overhead cost to 1,428
Overhead under- or over-absorbed profit and loss (11)
control (10)
116
1,428 1,428
Overhead under/over-absorbed control
££
Production overhead control (10) 678 General overhead control (10) 116
Profit and loss account 562
678 678
Profit and loss account
££
Production cost of sales (6) 11,900 Sales (9,900 + 6,870) 16,770
General overhead control (11) 1,428
Under-absorbed overhead 562
Profit for the period
2,880
16,770 16,770
Notes
1. The cost of the opening work in progress is shown as a brought forward balance in
the individual job account and in the work in progress control account.
2. The cost of materials returned to stores is credited in the individual job account and in
the work in progress control account.
3. The cost of materials transferred between jobs is credited to the job from which the
material is transferred and debited to the job that actually uses the material.
4. Job 310 is incomplete. The production cost incurred this period is carried down as an
opening work in progress balance for next period.
5. The total cost of all materials issued is debited to the work in progress control account.
6. The production cost of both completed jobs (£3,889 + £8,011) is transferred to the
profit and loss account.
7. The balance carried forward to next period is the cost of the work in progress
represented by job 310.
8. The overhead cost incurred is debited in the control account.
9. The production overhead absorbed into work in progress is credited to the overhead
control account.
10. Production overhead is under-absorbed and general overhead is over-absorbed this
period.
11. The general overhead cost charged to profit and loss on completed jobs = 12%
£(3,889 + 8,011) = £1,428
FUNDAMENTALS OF MANAGEMENT ACCOUNTING
217
SPECIFIC ORDER COSTING
2006.1
(c)
Job 308 Job 309
£ £
Production cost 8,011.00 3,889.00
General overhead absorbed at 12%
961.32 466.68
8,972.32 4,355.68
Invoice value
9,900.00 6,870.00
Profit
927.68 2,514.32
The total profit on the two jobs is £3,442. The difference of £562 between this total and
the profit shown in the profit and loss account is the result of the under-absorbed
overhead of £562.
8.3 Batch costing
The CIMA Terminology defines a batch as a ‘group of similar units which maintains its identity
throughout one or more stages of production and is treated as a cost unit’. Examples include
a batch of manufactured shoes or a batch of programmes printed for a local fete.
You can probably see that a batch is very similar in nature to the jobs which we have
been studying so far in this chapter. It is a separately identifiable cost unit for which it is
possible to collect and monitor the costs.
The job costing method can therefore be applied in costing batches. The only
difference is that a number of items are being costed together as a single unit, instead of
a single item or service.
Once the cost of the batch has been determined, the cost per item within the batch can
be calculated by dividing the total cost by the number of items produced.
Batch costing can be applied in many situations, including the manufacture of furniture,
clothing and components. It can also be applied when manufacturing is carried out for the
organisation’s own internal purposes, for example in the production of a batch of
components to be used in production.
8.3.1 Example: batch costing
Needlecraft Limited makes hand embroidered sweat shirts to customer specifications.
The following detail is available from the company’s budget.
Administration, selling and distribution overhead is absorbed into batch costs at a rate
of eight per cent of total production cost. Selling prices are set to achieve a rate of return
of 15 per cent of the selling price.
Cost centre Budgeted overheads Budgeted activity
Cutting and sewing £93,000 37,200 machine hours
Embroidering and packing £64,000 16,000 direct labour hours
STUDY MATERIAL C1
218
SPECIFIC ORDER COSTING
2006.1
An order for 45 shirts, Batch No. 92, has been produced for Shaldene Community
Choir. Details of activity on this batch are as follows.
Required
Calculate the selling price per shirt in Batch No. 92.
Solution
Workings
Calculation of production overhead absorption rates:
Cutting and sewing ¼£93,000/37,200 ¼£2.50 per machine hour
Embroidering and packing ¼£64,000/16,000 ¼£4 per direct labour hour
Batch No. 92
££
Direct material 113.90
Direct labour:
Cutting and sewing 4.50
Embroidering and packing
319.00
323.50
Direct expense: design costs
140.00
Total direct cost 577.40
Production overhead absorbed:
Cutting and sewing (W1) 2 machine hours £2.50 5.00
Embroidering and packing (W1) 29 labour
hours £4
116.00
121.00
Total production cost 698.40
Administration, etc. overhead £698.40 8%
55.87
Total cost 754.27
Profit margin 15/85 £754.27
133.11
Total selling price of batch
887.38
Selling price per shirt £887.38/45
£19.72
Direct materials £113.90
Direct labour
Cutting and sewing 0.5 labour hours at £9 per hour £4.50
Embroidering and packing 29 labour hours at £11 per hour £319.00
Machine hours worked in cutting and sewing 2
Fee paid to designer of logo for sweat shirts £140.00
FUNDAMENTALS OF MANAGEMENT ACCOUNTING
219
SPECIFIC ORDER COSTING
2006.1
8.4 Contract costing
Contract costing is another form of specific order costing. It is usually applied to
construction contracts which are of relatively long duration in comparison with the jobs
and batches which we have so far considered. The contracts are undertaken according to
specific customer requirements and they are usually carried out on sites away from the
organisation’s own premises. Contract costing can be used in bridge-building, tunnel
construction, motorway construction, shipbuilding and similar long-term works.
8.4.1 Architect’s certificates and progress payments
Because of the long-term nature of building work, it is usual for the contract to provide for
the customer to make payments as the contract proceeds. These interim payments are
known as progress payments.
A surveyor or architect will visit the contract at various stages of its completion. Having
inspected the progress of the work, the architect will issue a certificate which states the
sales value of the work which has been completed to date. An appropriate invoice can then
be sent to the customer, with a copy of the architect’s certificate attached to verify the
value of the work certified.
8.4.2 Retention money
The contract will usually provide for the customer to pay only a percentage of the value of
the work certified. The balance which is not paid is called retention money. The retention
percentage varies depending on the terms of the contract, but it is often about 10 per cent of
the certified value. The customer retains this amount until an agreed time after the contract
is completed, to guard against monetary loss due to unforeseen circumstances arising.
8.4.3 Contract accounts
The objective of contract costing is much the same as that of job costing. The costs of
each contract must be systematically collected and monitored. For this purpose a separate
account is maintained for each contract. All of the costs of the contract are collected in the
account, which can then be used to assist in determining the contract profit.
The long-term nature of contracts means that they often span more than one
accounting period. If a contract is still in progress at the end of the company’s financial
year, then it is necessary to value the contract work in progress for balance sheet
purposes. In addition a calculation is performed to determine how much profit has been
earned on the contract during the year and this amount is credited to the total company’s
income statement (profit and loss account) for the year. The profit on a contract is thus
recognised in stages as the contract progresses, instead of waiting until the contract is
completed to recognise any profit.
The reason for this is to attempt to present a true and fair view of the company’s
performance. It avoids the excessive fluctuations in reported profits which may arise if
profits are recognised only when contracts are completed. For example, if several contracts
were completed in one year, then the reported profits would be very high. In the next year
there may be no contracts completed at all and excessive losses would be reported. Anybody
who was trying to use the company’s accounts to assess its performance would find it very
STUDY MATERIAL C1
220
SPECIFIC ORDER COSTING
2006.1
difficult to make any judgements based on such wildly fluctuating reported profits.
Reporting the profits as the contract progresses helps to smooth out these fluctuations.
8.4.4 Accounting for contract materials
Materials delivered to the contract site could come from the organisation’s own stores
or they could be delivered direct to the site by the supplier. In both cases the movement
of the materials must be carefully documented so that the correct contract is charged with
the receipt of the materials. The contract account would be debited with the cost of the
materials delivered. If any material is returned to stores or to the supplier, then the
necessary documentation would be raised and the cost of these materials would be
credited to the contract account.
At the end of the accounting period there will often be some material still on site which is
to be used in the next period. The cost of this material will be credited to the contract account
for the period and carried down as a debit balance at the start of the next period.
8.4.5 Accounting for plant used on the contract
Various types of heavy plant are used on building contracts, for example cranes, bulldozers
and cement mixers. The plant is often transferred from one contract to another as it is
needed. As with the movements of materials, it is important that plant movements are
carefully documented and controlled. The objective is to ensure that the contract receives
a fair charge for the depreciation of the plant while it has been used on the contract. There
are two main ways in which this can be accomplished.
(a) Valuing the plant on transfer
With this method the plant is valued when it is transferred to the contract and this amount is
debited to the contract account. The plant is then valued again when it is transferred from
the contract and the value is credited to the contract account. The difference between these
two amounts represents the depreciation which has been charged to the contract.
If the plant is still in use on the contract at the end of an accounting period, then the
value of the plant remaining on site is credited to the account and carried forward as a
debit balance into the next period. In this way, each accounting period will receive a fair
charge for plant depreciation.
(b) Calculating the depreciation charge
With this method the contract is simply charged a proportion of the annual depreciation
for the plant, depending on the length of time it was used on the contract. This method
would be more appropriate for a plant which is moved frequently and which does not stay
on any one contract for a long time.
8.4.6 Cost classification in contract costing
An important point to appreciate is that, because of the nature of the work undertaken
when contract costing is applied, many costs that would in most circumstances be indirect
costs are, in fact, direct costs of the contract.
Contract work is usually undertaken on a large scale at the customer’s own premises – for
example, when building a hospital or constructing a new road. Each contract will often be
FUNDAMENTALS OF MANAGEMENT ACCOUNTING
221
SPECIFIC ORDER COSTING
2006.1
large enough to merit the employment of a full-time supervisor and perhaps the installation
of its own telephone line and electricity services. This means that costs such as supervisors’
salaries and telephone and electricity expenses would be a direct cost of the contract, because
they can be specifically identified with it. Contrast this with the more common situation, with
other costing methods, where these items are classified as indirect costs and it is necessary to
attribute them as fairly as possible to several different cost centres or cost units.
8.4.7 Calculating contract profit and preparing
balance sheet entries
When calculating the profit to be recognised on uncompleted contracts, it is essential that the
requirements of the prudence concept are adhered to, that is, that profits are not overstated
and a conservative view is taken. Indeed if a loss is foreseen on completion of the project,
then the whole of the future loss should be taken into account as soon as possible.
The best way to see how contract costing works is to study it in the context of the
following example.
8.4.8 Contract costing: a worked example
On 3 January, year 8, B Construction Ltd started work on the construction of an office
block for a contracted price of £750,000 with completion promised by 31 March, year 9.
The construction company’s financial year end was 31 October, year 8, and on that date
the accounts appropriate to the contract contained the following balances:
No difficulties are envisaged during the remaining time to complete the contract.
You are required to:
(a) prepare the contract account for the period ended 31 October, year 8, and show the
amount to be included in the construction company’s income statement ( profit and
loss account) for that period;
(b) show extracts from the construction company’s balance sheet at 31 October, year 8,
so far as the information provided will allow.
£000
Materials issued to site 161
Materials returned from site 14
Wages paid 68
Own plant in use on site, at cost 96
Hire of plant and scaffolding 72
Supervisory staff:
direct 10
indirect 12
Head office charges 63
Cash received related to work certified 330
Estimated cost to complete contract 240
Depreciation on own plant to be provided at the rate of 12.5 per cent per annum on cost.
£2,000 is owing for wages.
Estimated value of materials on site is £24,000.
STUDY MATERIAL C1
222
SPECIFIC ORDER COSTING
2006.1
Solution
The first thing that we need to know is the total cost incurred on the contract in the period.
A contract account is used to collect the costs incurred. Work carefully through the
entries in the account below.
The figures in brackets refer to the explanatory notes which follow the account.
Notice that the cash received from the customer is not entered in the contract account.
This is not an item of cost information.
At the start of the next financial year the account contains all of the brought-forward
balances from the previous year.
Explanatory notes
1. Depreciation of plant. As explained earlier in this chapter, the depreciation charge can be
calculated and charged to the contract, or the remaining value of plant on site can be
carried forward into the next period. The net effect is the same, but in this example it
seems more logical to show the value of the plant carried forward, to reflect the
continuing nature of the contract.
Make sure that you do not make the common mistake of including the value of the
plant and the depreciation charge. This would be double-counting.
Office block contract account to 31 October, year 8
£000 £000
Materials issued 161 Material returned (2) 14
Wages paid 68 Materials on site c/d (4) 24
Plant at cost (1) 96 Plant on site c/d (1) 86
Hire of plant and scaffolding 72 Cost to date c/d (5) 360
Supervisory staff:
direct 10
indirect 12
Head office charges 63
Wages accrued c/d (3)
2
484 484
Office block contract account from 1 November, year 8
£000 £000
Material on site b/d (4) 24 Wages accrued b/d 2
Plant on site b/d (1) 86
Cost to date (5) 360
£000
Value of plant delivered to site 96
Depreciation while in use:
10/12 (£96,000 12.5%)
10
Value of plant carried down to next period
86
FUNDAMENTALS OF MANAGEMENT ACCOUNTING
223
SPECIFIC ORDER COSTING
2006.1
Did you notice that the plant was in use for only ten months of the year, not for the
whole year?
The net effect of the debit of £96,000 and the credit of £86,000, in the contract
account to 31 October, is to charge the correct amount of £10,000 for depreciation.
2. Materials returned. The materials returned from the site are credited to the contract and
debited to the central stores account.
3. Wages accrued. This entry ensures that the correct amount is charged for wages in the
period. The credit entry is carried down into the account for next period. Therefore
when the wages are actually paid next period, the credit entry brought down will be
netted against the payment and there will be no effect on next period’s costs.
4. Materials on site. These materials have not yet been used and their cost is carried down into the
next period. If this was not done, then the cost of the work for the period would be overstated.
5. Cost to date. Now that all of the adjustments have been made to carry forward the costs
that do not relate to this period, the balance on the account must be the cost incurred
to date.
Before any profit can be recognised on a contract, two questions must be asked:
1. Are any losses evident on this contract? If a loss is foreseen on completion of the contract,
then all of the foreseen loss must be recognised now. The answer to this question is ‘no’,
and we can proceed to the second question.
2. Are any difficulties foreseen? It may be possible to foresee difficulties arising during the
remaining time to complete the contract. These difficulties may not actually result in
losses, but any costs should be provided for in full as soon as they are foreseen. In this
example there are no difficulties envisaged.
Since the answer to both these questions is ‘no’, it seems reasonable to proceed and
calculate an amount of profit to be recognised in B Construction Ltd’s accounts for the
year ending 31 October, year 8. Later in this chapter you will see how to deal with the
situations when the answers to these questions are ‘yes’.
The amount of profit to be recognised on the contract will depend on its degree of
completion. Two common formulae that might be used to calculate the profit to be recognised
are based on the cost incurred to date or on the revenue earned to date as follows:
Profit to be recognised ¼estimated final profit on contract
cost incurred to date
estimated final contract cost
or
Profit to be recognised ¼estimated final profit on contract
revenue earned to date
estimated final contract revenue
Many different methods could be used to determine the amount of profit to be
recognised. The most important thing from the point of view of the assessment is
to read the question carefully to check what information is available and follow any
instructions given concerning the calculation of profit.
In this example we will determine the degree of completion by reference to the cost
incurred to date as a proportion of the estimated final contract cost.
STUDY MATERIAL C1
224
SPECIFIC ORDER COSTING
2006.1
Stage of completion ¼ cost incurred to date=estimated final contract cost
¼ 360; 000= 600; 000 ¼ 60%
Profit to be recognised on contract ¼ £150; 000 60% ¼ £90; 000
This is an acceptable solution to the remainder of part (a) in our example: the amount
to be included in the construction company’s income statement (profit and loss account)
for the period ended 31 October, year 8 is £90,000. This would affect the construction
company’s accounts as follows.
Now you need to learn how to deal with part ( b) of the question: showing the relevant
extracts from the company’s balance sheet.
There will be three items in the company’s balance sheet in respect of this contract.
(Figures in brackets refer to the explanatory notes which follow.)
(a) The contract debtor account (account receivable)
The contract debtor account will look like this:
Explanatory notes
1. The revenue of £450,000, as calculated above, will be credited to the sales account and
debited to the debtor account.
2. The cash received related to the work certified, as specified in the question, will be
debited to the bank account and credited to the debtor account.
The balance of £120,000 on the debtor account will be shown within debtors or
receivables on the company’s balance sheet.
The other balance sheet extracts will relate to the remaining balances brought down on
the contract account which you saw earlier, excluding the £360,000 cost which has been
transferred to the income statement/profit and loss account.
£
Revenue to be credited to income statement (£750,000 60%) 450,000
Cost to be charged to income statement (£600,000 60%)
360,000
Profit recognised
90,000
££
Contracted price 750,000
Cost incurred to date 360,000
Estimated cost to complete contract
240,000
Estimated final contract cost
600,000
Estimated final profit on contract
150,000
Office block contract debtor
£000 £000
Sales (1) 450 Bank (2) 330
Balance c/d 120
450 450
FUNDAMENTALS OF MANAGEMENT ACCOUNTING
225
SPECIFIC ORDER COSTING
2006.1
(b) The plant on site
The £86,000 book value of the plant on site will be shown under non-current assets
(fixed assets) on the balance sheet.
(c) The other contract balances
The remaining balances of material stock (inventory) £24,000 and wages accrued
£2,000 will be shown on the company’s balance sheet as an asset and a liability,
respectively.
8.4.9 Accounting for a loss-making contract
If a loss is foreseen on the contract, then the whole of the loss should be recognised
immediately, even if revenues received exceed the costs to date.
Suppose that because of problems envisaged before completion, the estimated costs to
complete the office block contract are £410,000. A loss can be foreseen on the contract as
follows.
££
Contracted price 750,000
Cost incurred to date 360,000
Estimated cost to complete contract
410,000
Estimated final contract cost
770,000
Estimated final loss on contract
(20,000)
The whole of the loss would be recognised immediately and the effect on the income
statement (profit and loss account) would be as follows. For demonstration purposes we
will use the same degree of completion as before.
£
Revenue to be credited to income statement (£750,000 60%) 450,000
Cost to be charged to income statement (£600,000 60%) (360,000)
Provision for future losses (balancing figure)
(110,000)
Contract loss
(20,000)
The relevant ledger accounts would look like this:
Office block contract account from 1 November, year 8
£000 £000
Material on site b/d 24 Wages accrued b/d 2
Plant on site b/d 86
Office block contract debtor
£000 £000
Sales 450 Bank 330
Balance c/d 120
450 450
STUDY MATERIAL C1
226
SPECIFIC ORDER COSTING
2006.1
8.4.10 Contract costing: a second example
Work carefully through this next example, checking that you understand all the
workings.
E Ltd, a construction company, has two sites on which it is building residential homes.
Site A was started on 1 November year 4 and is expected to be completed by 30 June
year 6. Site B was started on 1 October year 5 and is not due for completion until 30 April
year 7.
The company’s financial year ends on 31 December.
The following details relate to the contracts as at 31 December year 5.
Notes:
1. The plant was sent to site at the commencement of the contract. For site A, the value
shown is its net book value at 1 January year 5 and for site B, the value shown is that at
the commencement of the contract. Depreciation is to be provided using the reducing
balance method at an annual rate of 20 per cent.
2. At 31 December year 5 there were wages outstanding of £2,000 at site A and £1,000 at
site B.
3. The cash received from clients represents the value of work certified and invoiced less
an agreed retention of 5 per cent.
4. The total contract prices are £600,000 for site A and £400,000 for site B.
Site A Site B
£000 £000
Work in progress (1 January year 5) 51
Materials sent to site 193 63
Materials returned from site 11 3
Plant sent to site 75 40
Material on site (31 December year 5) 6 25
Direct wages paid 142 48
Other site expenses paid 46 13
Cash received from clients 475 38
Company cost of sales account (extract)
£000 £000
Office block contract:
Cost of work completed 360
Provision for losses
110
470
Provision for contract losses
£000 £000
Cost of sales 110
FUNDAMENTALS OF MANAGEMENT ACCOUNTING
227
SPECIFIC ORDER COSTING
2006.1
5. The estimated costs to complete the work at the sites is £110,000 at site A and
£240,000 at site B.
6. No profit was recognised in respect of site A in the financial year ended 31 December
year 4.
Solution
The first step is to prepare a contract account for each of the sites. For ease of
presentation our solution shows the accounts side by side in a columnar format.
Note: Depreciation of plant
Site A ¼ £75,000 20% ¼ £15,000
Value of plant on site c/d ¼ £75,000 £15,000 ¼ £60,000
Site B ¼ £40,000 20%
3
12
¼ £2,000
Value of plant on site c/d ¼ £40,000 £2,000 ¼ £38,000
Note that the plant is in use at site B for only three months.
The next step is to calculate the profit to be taken on each contract.
The degree of completion can be measured using either sales values or costs.
Contract accounts to 31 December year 5
AB AB
£000 £000 £000 £000
Work in progress b/d 51 Materials returned from site 11 3
Materials sent to site 193 63 Material on site c/d 6 25
Plant sent to site 75 40 Plant on site c/d (see note) 60 38
Direct wages paid 142 48 Cost incurred to date 432 99
Other site expenses paid 46 13
Wages accrued c/d
2 1
509 165 509 165
Site A Site B
Using sales values
Value certified (note 1):
£475,000
100
95
£500,000
£38,000
100
95
£40,000
Contract price £600,000 £400,000
Degree of completion:
500
600
83.3%
40
400
10.0%
Using cost values
Cost incurred/estimated total cost (note 2):
£432;000
£542;000
79.7%
£99;000
£339;000
29.2%
STUDY MATERIAL C1
228
SPECIFIC ORDER COSTING
2006.1
Notes
1. The agreed retention is 5 per cent. Therefore, the cash received from clients is
multiplied by
100
95
to determine the value certified.
2.
You can see that there is a difference in the estimated degree of completion calculated
using each method. Whichever method is used it must be applied consistently.
You can also see that the degree of completion at site B is small. Therefore, it is not
prudent to recognise any profit on this contract at this stage. As a general guide, no profit
should be recognised until a contract is at least 30 per cent complete.
For contract A, the profit to be recognised is as follows:
The most prudent figure is taken for degree of completion (i.e. the lowest figure).
This would affect E Ltd’s accounts as follows:
Site A Site B
£000 £000
Estimated total costs:
costs incurred to date 432 99
estimated costs to complete
110 240
542 339
Site A
£000
Contract price 600
Estimated total cost
542
Estimated final profit on contract
58
Degree of completion
*
: 79.7%
Profit to be recognised ¼
£46,000 (to the nearest £000)
Site A Site B
£000 £000
Cost to be charged to P&L a/c (542 79.7%) 432 44
Profit to be recognised
46
Revenue to be credited to P&L a/c (600 79.7%)
478 44
Contract debtor accounts
Site A Site B Site A Site B
£000 £000 £000 £000
Sales 478 44 Bank 475 38
Balance c/d 3 6
478 44 478 44
FUNDAMENTALS OF MANAGEMENT ACCOUNTING
229
SPECIFIC ORDER COSTING
2006.1
Balance sheet extracts
8.4.11 Contract costing: a final example
Try to produce your own answer to this example before you read the solution.
S Ltd is building an extension to a local factory. The agreed contract price is £300,000.
The contract commenced on 1 March year 2 and is scheduled for completion on
30 June year 3.
S Ltd’s financial year ends on 31 December.
The following details are available concerning the factory contract as at 31 December
year 2.
You are required to prepare the contract account for the period ended 31 December
year 2, and to show the amount to be included in S Ltd’s profit and loss account (income
statement) in respect of the contract for that period.
Site A Site B Total
£000 £000 £000
Material on site 6 25 31
Receivables (debtors) 3 6 9
Contract in progress (site B = 99 cost incurred
less 44 transferred to profit and loss)
55 55
Plant on site 60 38 98
£000
Materials sent to site from central stores 15
Materials delivered to site direct from suppliers 70
Plant delivered to site (net book value) 40
Direct wages paid 85
Direct site expenses paid 38
Head office charges 12
Material returned from site to central stores 6
Net book value of plant on site, 31 December year 2 32
Materials on site, 31 December year 2 4
Direct wages owing at 31 December year 2 3
Cash received from customer 207
Estimated cost to complete the contract 119
STUDY MATERIAL C1
230
SPECIFIC ORDER COSTING
2006.1
Solution
In order to decide whether a profit should be recognised on the contract we will refer to
the questions detailed in Section 8.4.8.
1. Are any losses evident on the contract? Yes, the following calculation shows that a loss is
foreseen, therefore the whole of the future loss should be taken into account now.
The charge to cost of sales must allow for the full amount of the loss.
The degree of completion of the contract, based on the costs incurred to date as a
percentage of the final contract cost, is 221/340 ¼65%.
The sales revenue and cost of sales in the profit and loss account (income statement) are
therefore as follows.
Factory extension contract account to 31 December, year 2
£000 £000
Materials from stores 15 Materials returned to stores 6
Materials from suppliers 70 Plant on site c/d 32
Plant delivered to site 40 Material on site c/d 4
Direct wages paid 85 Cost of work to date
(balancing figure)
221
Direct site expenses paid 38
Head office charges 12
Wages accrued c/d
3
263 263
Plant on site b/d 32 Wages accured b/d 3
Materials on site b/d 4
£000
Cost of work to date (from contract account) 221
Estimated cost to complete the contract
119
Total cost of contract 340
Agreed contract price
300
Expected loss on contract
(40)
£
Revenue to be credited to income statement (£300,000 65%) 195,000
Cost to be charged to income statement (£340,000 65%) (221,000)
Provision for future losses (balancing figure)
(14,000)
Contract loss
(40,000)
FUNDAMENTALS OF MANAGEMENT ACCOUNTING
231
SPECIFIC ORDER COSTING
8.5 Summary
Having read this chapter the main points that you should understand are as follows.
1. Specific order costing methods are appropriate for organisations that produce cost
units which are separately identifiable from each other. Job costing, batch costing and
contract costing are all specific order costing methods.
2. Job costing applies where work is undertaken according to individual customer
requirements. Each job is of relatively short duration and may be undertaken on the
customer’s premises or on the contractor’s premises.
3. Contract costing also applies where work is undertaken according to individual customer
requirements, but each contract is usually of longer duration. Contracts frequently span
more than one accounting period and are often constructional in nature.
4. Batch costing is a form of job costing where each batch of similar items is a separately
identifiable cost unit.
5. In a job costing system, each job is given a unique number and the costs of each job are
collected and analysed on a job cost sheet.
6. As a contract progresses the work completed is certified at various stages by an
architect and the customer will make progress payments to the contractor. The
customer might not pay the full amount of the value certified because retention monies
are often held in case unforeseen circumstances arise.
7. In order to avoid wide fluctuations in reported profits an estimate may be made of the
profit earned on an incomplete contract to date and this profit may be recognised in the
contractor’s income statement ( profit and loss account).
8. Profit may be recognised on an incomplete contract as long as its outcome can be
reasonably foreseen and no adverse circumstances are expected. If a loss is expected on
a contract then the whole of the loss must be provided for immediately.
STUDY MATERIAL C1
232
SPECIFIC ORDER COSTING
Revision Questions
Question 1 Multiple choice
1.1 Which of the following are characteristics of job costing?
(i) Customer-driven production.
(ii) Complete production possible within a single accounting period.
(iii) Homogeneous products.
(A) (i) and (ii) only.
(B) (i) and (iii) only.
(C) (ii) and (iii) only.
(D) All of them.
1.2 Which of the following are characteristics of contract costing?
(i) Homogeneous products.
(ii) Customer-driven production.
(iii) Short timescale from commencement to completion of the cost unit.
(A) (i) and (ii) only.
(B) (ii) and (iii) only.
(C) (i) and (iii) only.
(D) (ii) only.
1.3 The following items may be used in costing jobs:
(i) Actual material cost.
(ii) Actual manufacturing overheads.
(iii) Absorbed manufacturing overheads.
(iv) Actual labour cost.
Which of the above are contained in a typical job cost?
(A) (i), (ii) and (iv) only.
(B) (i) and (iv) only.
(C) (i), (iii) and (iv) only.
(D) All four of them.
8
233 2006.1
Data for questions 1.4 and 1.5
A firm uses job costing and recovers overheads on direct labour cost.
Three jobs were worked on during a period, the details of which were:
The overheads for the period were exactly as budgeted: £140,000.
1.4 Jobs 1 and 2 were the only incomplete jobs. What was the value of closing work in
progress?
(A) £81,900
(B) £90,175
(C) £140,675
(D) £214,425
1.5 Job 3 was completed during the period and consisted of a batch of 2,400 identical
circuit boards. The firm adds 50 per cent to total production costs to arrive at a
selling price. What is the selling price of a circuit board?
(A) It cannot be calculated without more information.
(B) £31.56
(C) £41.41
(D) £58.33
1.6 BH Ltd is currently undertaking a contract to build an apartment block. The
contract commenced on 1 January year 2 and is expected to take 13 months to
complete. The contract value is £54m. The contractor’s financial year ends on
30 September.
The contract account for the building of the apartment block indicates the
following situation at 30 September year 2:
The amount of profits to be recognised is based on the cost incurred to date. It is
company policy not to recognise profit on contracts unless the cost incurred is at
least 30 per cent of the total contract cost.
The maximum amount of profit=loss for the contract that can be taken to the
profit and loss account for the year ended 30 September year 2 is:
(A) Nil
(B) £5 m
(C) £7 m
(D) £10 m.
Job 1 Job 2 Job 3
£££
Opening work-in-progress 8,500 0 46,000
Material in period 17,150 29,025 0
Labour for period 12,500 23,000 4,500
Value of work certified £30 m
Costs incurred to date £20 m
Future costs to completion £20 m
REVISION QUESTIONS C1
234
SPECIFIC ORDER COSTING
2006.1
Question 2 Short objective-test questions
2.1 Match the organisational activities below to the most appropriate costing method by
writing (a), (b) or (c) in the box provided.
Costing methods
(a) Job costing
(b) Batch costing
(c) Contract costing
Organisational activities
Accounting and taxation services &
Shoe manufacturing &
Plumbing and heating repairs &
Road building &
Building maintenance and repairs &
2.2 Calculate the selling price for each job (a) to (c) (to the nearest penny), and write the
correct answer in the box provided.
(a) Total cost of job ¼£45. Profit mark-up ¼25 per cent of cost. Job selling
price ¼£
.
(b) Production cost of job ¼£38. Percentage to be added to production cost to
absorb general overheads ¼10 per cent. Profit mark-up ¼20 per cent of total
cost. Job selling price ¼£
.
(c) Total cost of job ¼£ 75. Profit margin ¼15 per cent of selling price. Job selling
price ¼£
.
2.3 Is the following sentence true or false? Tick the correct box.
Interim payments that are received from a customer as a contract progresses are
known as retention monies.
True &
False &
2.4 A plant with a net book value of £40,000 is delivered to contract ZX on 31 March.
The plant is still in use on the contract at the company’s year end, 31 December.
Company policy is to depreciate all contract plant on a reducing balance basis, at a
rate of 25 per cent per annum.
Complete the box in the contract account to show how the plant would be
accounted for.
2.5 The cost incurred on contract D372 to date is £465,000. The cost to be incurred to
complete the contract is £116,250 and no problems are foreseen before its
completion. The value of work certified is £545,000 and the cash received from the
Contract ZX [extract]
££
31 Mar. Plant delivered to contract 40,000 31 Dec. Plant c=d
FUNDAMENTALS OF MANAGEMENT ACCOUNTING
235
SPECIFIC ORDER COSTING
2006.1
customer is £517,750. The final contract value is £640,000. The profit to be
recognised on the contract is to be calculated as follows:
Profit to be recognised ¼ Final contract profit
cost incurred to date
final contract cost
The revenue to be credited to the company profit and loss account (income
statement) in respect of contract D372 is £
.
2.6 A company calculates the prices of jobs by adding overheads to the prime cost and
adding 30 per cent to total costs as a profit margin. Complete the following job cost
summary information:
2.7 A particular contract has earned a nominal profit to date but the contract overall is
expected to incur a loss by the time it is completed. The loss should not be
recognised in the accounts until the period when the loss actually occurs.
True &
False &
2.8 A commercial decorating organisation budgets for four per cent idle time on all its
jobs.
The estimated number of active labour hours required to complete decorating job
no. D47 is 120 hours. The hourly labour rate is £11.
The estimated labour cost of job no. D47 is (to the nearest £ ) £
.
Question 3 Batch costing
Jetprint Ltd specialises in printing advertising leaflets and is in the process of preparing its
price list. The most popular requirement is for a folded leaflet made from a single sheet of
A4 paper. From past records and budgeted figures, the following data has been estimated
for a typical batch of 10,000 leaflets.
General fixed overheads are £15,000 per period, during which a total of 600 labour
hours are expected to be worked.
The firm wishes to achieve 30 per cent profit on sales.
Job Y256 £
Prime cost
Overheads 694
Total cost
Profit margin
Selling price 1,690
Artwork £65
Machine setting 4 hours at £22 per hour
Paper £12.50 per 1,000 sheets
Ink and consumables £40
Printers’ wages 4 hours at £8 per hour (Note: Printers’
wages vary with volume.)
REVISION QUESTIONS C1
236
SPECIFIC ORDER COSTING
2006.1
Requirements
(a) The selling prices (to the nearest pound ) per thousand leaflets for quantities of:
(i) 10,000 leaflets is £
(ii) 20,000 leaflets is £
(b) During the period, the firm printed and sold 64 batches of 10,000 leaflets and
36 batches of 20,000 leaflets. All costs were as expected.
(i) General fixed overhead for the period was (tick the correct box):
& under-absorbed
& over-absorbed
(ii) The value of the under-/over-absorption of general fixed overhead was £
.
Question 4 Contract costing
HR Construction plc makes up its accounts to 31 March each year. The following details
have been extracted in relation to two of its contracts as at 31 March 20X5:
Depreciation is charged on plant using the straight-line method at the rate of 12 per cent p.a.
Requirements
(a) The net book value of the plant on site at 31 March 20X5 is:
(i) Contract A: £
(ii) Contract B: £
(b) The total cost of materials for the contracts to 31 March 20X5 is:
(i) Contract A: £
(ii) Contract B: £
Contract A Contract B
Commencement date 1 April 20X4 1 December 20X4
Target completion date 31 May 20X5 30 June 20X5
£000 £000
Contract price 2,000 550
Materials sent to site 700 150
Materials returned to stores 80 30
Plant sent to site 1,000 150
Materials transferred to contract B 40
Materials transferred from contract A 40
Materials on site 31 March 20X5 75 15
Cost incurred to date 1,200 406
Estimated additional cost to completion 400 174
FUNDAMENTALS OF MANAGEMENT ACCOUNTING
237
SPECIFIC ORDER COSTING
2006.1
(c) HR’s policy is to recognise profit on uncompleted contracts as:
Estimated total contract profit
Cost incurred
Estimated total contract cost
(i) The profit to be recognised on contract A to date is £
(ii) The charge to the profit and loss account/income statement as a provision for
future losses in respect of contract B is £
SPECIFIC ORDER COSTING
2006.1
REVISION QUESTIONS C1
238
Solutions to
Revision Questions
Solution 1
If you are reduced to guessing the answer to a multiple-choice question, remember to
eliminate first those answers that you know to be incorrect. Then, select an answer from
the remaining options. This technique would be particularly useful for questions 1.1
and 1.2.
In question 1.5 read the information you are given carefully to determine whether the
profit percentage is calculated as a percentage of cost or as a percentage of selling price.
1.1 Answer: (A)
Job costing applies to situations where work is carried out to customer specifications,
and each order is of relatively short duration. Each job is separately identifiable,
therefore characteristic (iii) is incorrect.
1.2 Answer: (D)
Contract costing applies to situations where work is carried out to customer
specifications, and typically each contract takes more than one year to complete.
Thus, only (ii) is correct.
1.3 Answer: (C)
Overheads are absorbed into the cost of each job as the period progresses, using a
predetermined overhead absorption rate. It is not usually possible to identify the
actual overhead cost for each individual job therefore option A is incorrect.
Option (B) is incorrect because it does not include any overhead cost. Option (D) is
incorrect because it includes a double charge for overhead.
1.4 Answer: (D)
Overhead absorption rate ¼
£140,000
£40,000
100% ¼ 350% of direct labour:
8
239 2006.1
1.5 Answer: (C)
1.6 Answer: (C)
The cost incurred is more than 30 per cent of the total contract cost therefore a profit
can be recognised on this contract. The maximum amount of profit that might be
recognised at 30 September is as follows:
Solution 2
2.1 Accounting and taxation services (a)
Shoe manufacturing (b)
Plumbing and heating repairs (a)
Road building (c)
Building maintenance and repairs (a) (the cost units are probably of relatively
short duration)
£
Costs given in question 50,500
Overhead absorbed: £4,500 350%
15,750
Total production cost 66,250
Mark up 50%
33,125
Sales value of batch
99,375
Selling price per circuit board
99,375
2,400

¼
£41.41.
£m
Contract value 54
Less:
Costs to date (20)
Future costs
(20)
Expected profit
14
Profit to be recognised ¼£14 m (£20 m/£40 m) ¼£7 m.
Work in progress valuation ££
Costs given in question:
Job 1 38,150
Job 2
52,025
90,175
Overhead absorbed:
Job 1 £12,500 350% 43,750
Job 2 £23,000 350%
80,500
124,250
214,425
SOLUTIONS TO REVISION QUESTIONS C1
240
SPECIFIC ORDER COSTING
2006.1
2.2 (a) £45 + 25% ¼£56.25
(b) £38 + 10% ¼£41.80 total cost + 20% ¼£50.16
(c) Note that the margin is expressed as a percentage of selling price:
£75
100
85
¼ £88:24
2.3 False. This is a description of progress payments.
2.4
2.5 The revenue to be credited to the company profit and loss account in respect of
contract D372 is £512,000. The contract is 80 per cent complete and no problems
are foreseen, therefore it is acceptable to recognise a profit on the contract.
Stage of completion ¼cost incurred to date/estimated final contract cost
¼465,000/581,250 ¼80%
Revenue to be credited to income statement ¼£640,000 80% ¼£512,000
2.6
In this question the profit is calculated as a percentage of cost. Sometimes the
profit is expressed as a percentage of selling price so be sure to read the question
carefully.
Calculate the total cost first, then the remaining answers can be slotted in as
balancing figures.
2.7 False. A contract loss should be allowed for in the accounts as soon as it is foreseen.
2.8 The estimated labour cost of job no. D47 is £1,375.
Workings:
The idle time would be stated as a percentage of the paid labour hours.
Hours
Active labour hours required 120
Idle time ( 4=96)
5
Total paid hours required
125
Labour cost @ £11 per hour
£1,375
CONTRACT ZX (extract)
££
31 Mar. Plant delivered to contract
40,000 31 Dec. Plant c=d* 32,500
* Depreciation for 9 months ¼£40,000 25%
9
12
¼£ 7,500
; Net book value of plant at 31 December ¼£40,000 £ 7,500 ¼£32,500
Job Y256 £
Prime cost 606
Overheads
694
Total cost ð1;690
100
130
Þ 1,300
Profit margin
390
Selling price
1,690
FUNDAMENTALS OF MANAGEMENT ACCOUNTING
241
SPECIFIC ORDER COSTING
2006.1
Solution 3
You will need to recognise that some costs are fixed and others are variable note that
you cannot simply double the cost of 10,000 leaflets to obtain the cost for 20,000.
In part (b), not all the capacity is utilised and consequently there is an under-absorption
of fixed overheads.
(a) (i) £64
(ii) £53
(b) (i) General fixed overhead for the period was under-absorbed.
Actual labour hours worked ¼(64 4 hours) + (36 8 hours) ¼544 hours.
This is less than the budgeted labour hours of 600 and all costs were as expected
therefore the overhead would be under-absorbed.
Solution 4
You will need to produce a lot of workings. These will be for your own benefit because
workings do not earn marks in the assessment.
Note that contract B has been in operation for only four months.
(a) (i) £880,000
(ii) £144,000
Workings: Cost of batch 10,000 leaflets Cost of batch 20,000 leaflets
££
Artwork
1
65.00 65.00
Machine setting
1
88.00 88.00
Paper 125.00 250.00
Ink and consumables 40.00 80.00
Printers’ wages
32.00 64.00
350.00 547.00
General fixed overheads
2
100.00 200.00
Total cost 450.00 747.00
Profit
30
70
cost

192.86 320.14
Sales revenue required
642.86 1,067.14
Selling price per 1,000 £64.00 £53.00
Notes:
1. Machine setting and artwork costs are not affected by the size of the batch.
2. General fixed overhead ¼£15,000=600 ¼£25 per hour.
(ii) Overhead absorbed ¼544 hours £25 ¼ £13,600
Overhead incurred
£15,000
Under-absorbed overhead
£1,400
SOLUTIONS TO REVISION QUESTIONS C1
242
SPECIFIC ORDER COSTING
2006.1
Workings:
(b) (i) £505,000
(ii) £145,000
Workings:
(c) (i) £300,000
(ii) £9,000
Workings:
Contract B
Degree of completion, based on cost incurred ¼406/580 total cost ¼70% £
Revenue to be credited to income statement (£550,000 70%) 385,000
Cost to be charged to income statement (£580,000 70%) (406,000)
Provision for future losses (balancing figure)
(9,000)
Contract loss
(30,000)
Contract A Contract B
£000 £000
Plant sent to site 1,000 150
Depreciation
(12%) 120
ð12%
4
12
Þ 6
Net book value
880 144
Contract A Contract B
£000 £000
Materials sent to site 700 150
Materials returned to stores (80) (30)
Materials transferred (40) 40
Materials on site at 31 March
(75) (15)
505 145
Contract A Contract B
£000 £000
Contract price 2,000 550
Cost incurred to date (1,200) (406)
Cost to completion
(400) (174)
Estimated total contract profit/(loss)
400 (30)
Recognised
300
1
(30)
2
Notes:
1. 400 ð
1200
1600
Þ
2. The full amount of loss is allowed for.
FUNDAMENTALS OF MANAGEMENT ACCOUNTING
243
SPECIFIC ORDER COSTING
2006.1
Process Costing
9
Process Costing
9.1 Introduction
In this chapter you will learn about another costing method: process costing. Process
costing is used by organisations where a number of production processes are involved and
the output of one process is the input to a later process, this continuing until the final
product is completed. Examples of industries where process costing might be applied are
food processing, chemicals and brewing. The final product is said to be homogeneous (i.e.,
each unit is identical and cannot be distinguished from another unit), and is usually
manufactured for inventory (stock) from which sales are made to customers. Unlike job
costing the product is not customer specific and the range of products available is likely to
be limited, but it is likely that the customer base will be large.
9.2 Process accounts
When using process costing the process is the collection point for costs incurred. This
means that materials and labour costs will be identified with the particular process to
which they relate. The method is best explained by a simple example.
9
LEARNING OUTCOMES
After completing this chapter you should be able to:
"
compare and contrast job, batch, contract and process costing;
"
prepare ledger accounts for process costing systems.
247 2006.1
Example
During August a processing company incurred the following costs in its three processes:
The quantities of input and output were as follows:
The input quantities shown above do not include the output from the previous process. The output
from process 1 is transferred to process 2, which in turn transfers its output to process 3 which after
further processing results in the final product.
The process accounts will appear as follows:
You should note the layout of the process account. It is a ledger account with debit and
credit entries, but it is different from financial accounting ledger accounts because it includes
other columns. On the debit side there is a column for the quantity as well as the values, and on
Process 1 Process 2 Process 3
£££
Direct materials 6,000 4,000 9,000
Direct labour 1,000 2,000 3,000
Direct expenses 2,000 3,000 4,000
Production overhead 1,000 2,000 3,000
Process 1 Process 2 Process 3
kg kg kg
Input 500 200 300
Output 500 700 1,000
Process 1
kg £ kg £/kg £
Materials 500 6,000 Output 500 20.00 10,000
Labour 1,000
Expenses 2,000
Overheads
1,000
500 10,000 500 10,000
Process 2
kg £ kg £/kg £
Process 1 500 10,000 Output 700 30.00 21,000
Materials 200 4,000
Labour 2,000
Expenses 3,000
Overheads
2,000
700 21,000 700 21,000
Process 3
kg £ kg £/kg £
Process 2 700 21,000 Output 1,000 40.00 40,000
Materials 300 9,000
Labour 3,000
Expenses 4,000
Overheads
3,000
1,000 40,000 1,000 40,000
STUDY MATERIAL C1
248
PROCESS COSTING
2006.1
the credit side as well as the quantity column there is a column showing the cost per unit. The
value per unit of output is calculated by dividing the cost by the number of units.
When preparing process accounts it is important that the quantity columns are
completed first and balanced before attempting to value the units. This example was a
simple one, but as this chapter progresses and introduces more complications you will see
why this technique is recommended.
Note too that the total cost of process 1 is attributed to its output and that this is then
transferred to process 2. This procedure is repeated in process 2. The output from
process 3 is finished goods.
9.3 Losses in process
The majority of process industries expect there to be a loss in the production process.
A certain amount of loss is expected and therefore unavoidable and this is referred
to in cost accounting terminology as a normal loss.
This loss may occur through evaporation or may be a form of defective production.
The extent of the normal loss may be estimated using past records and experience. As a
loss, the only value that the organisation can derive from it is its scrap value (if it has any).
It is therefore considered good practice to regard the net cost (after deducting any scrap
sale proceeds if applicable) of producing the normal loss as a cost of the process and to
attribute it to the remaining units. The following example of a single process shows how
this is achieved.
The costs of the process are as follows:
Process 1
£
Direct materials 6,000
Direct labour 1,000
Direct expenses 2,000
Production overhead 1,000
The input quantity was 500 kg and the expected or normal loss was 10 per cent of input.
Actual output was 450 kg. The process account would appear as follows:
The total costs of the process (£10,000) have been attributed to the output of 450 kg.
This has the effect of increasing the cost per kg of good output to compensate for the cost
of producing the unavoidable normal loss.
Process 1
kg £ kg £/kg £
Materials 500 6,000 Output 450 22.22 10,000
Labour 1,000 Normal loss 50
Expenses 2,000
Overheads
1,000
500 10,000 500 10,000
FUNDAMENTALS OF MANAGEMENT ACCOUNTING
249
PROCESS COSTING
2006.1
If the normal loss could be sold for scrap at a value of £5 per kg, then this would reduce
the net cost of producing the normal loss. The effect of this on the entries in the process
account is as follows:
Note now the credit side of the process account shows the scrap value of the normal
loss. The net cost of the process is reduced by the £250 scrap value to £9,750 and this is
attributed to the output. The effect is to reduce the cost per kg of the output to £21.67.
The double entry for the normal loss is usually made in a scrap stock or scrap sales account.
9.4 Abnormal losses and gains
We have seen that the normal loss is an estimate of the loss expected to occur in a particular
process. This estimate may be incorrect and a different amount of loss may occur.
If the actual loss is greater than the normal loss then the excess loss is referred to as
an abnormal loss.
If the actual loss is less than the normal loss then the difference is referred to as an
abnormal gain.
The following example illustrates the calculations and entries in the process account
when an abnormal loss occurs.
Example
Input 500 kg of materials costing £6,000
Labour cost £1,000
Expenses cost £2,000
Overhead cost £1,000
Normal loss is estimated to be 10 per cent of input.
Losses may be sold as scrap for £5 per kg.
Actual output was 430 kg.
The process account is shown below.
Remember that, earlier in the chapter, we recommended that you should insert the units into the
process account first, and then balance them off. In this example, this results in a balancing value on the
credit side of 20 kg, which is the abnormal loss.
Scrap stock account
kg ££
Process 1 normal loss
50 250 Debtor/cash 250
Process 1
kg £ kg £/kg £
Materials 500 6,000 Output 450 21.67 9,750
Labour 1,000
Expenses 2,000 Normal loss 50 5.00 250
Overheads
1,000
500 10,000 500 10,000
STUDY MATERIAL C1
250
PROCESS COSTING
2006.1
The valuation per kg of £21.67 is calculated as follows:
Cost incurred scrap value of normal loss
Expected output
¼
£10;000 £250
450
¼ £21:67
The abnormal loss units are valued at the same rate per unit as the good output units. The normal loss
is valued at its scrap value only.
The next step is to prepare the scrap stock and abnormal loss accounts. These are shown below.
Scrap stock account
££
Process normal loss 250 Debtor/cash: (50 + 20) £5 350
Abnormal loss transfer
100
350 350
The scrap stock balance now represents the total of 70 kg scrapped, with a total scrap value of £350.
The resulting balance on the abnormal loss account is the net cost of producing an excess loss (i.e.,
after deducting the scrap sale proceeds). It has now been highlighted separately for management
attention, and the balance is transferred to the income statement (profit and loss account).
If the actual loss is smaller than the amount expected, then an abnormal gain is said to have occurred.
The abnormal gain is the extent to which the loss is smaller than expected. If we consider the same
example again, except that the actual output achieved was 470 kg, we can see that the following process
account results. Remember to balance the units column first. The normal loss is the same, because the
input is the same.
Process account
kg £ kg £/kg £
Materials 500 6,000 Output 430 21.67 9,317
Labour 1,000 Normal loss 50 5.00 250
Expenses 2,000 Abnormal loss 20 21.67 433
Overheads 1,000
500 10,000 500 10,000
Abnormal loss account
££
Process 433 Scrap stock: 20 £5 100
Profit and loss
333
433 433
Process account
kg £ kg £/kg £
Materials 500 6,000 Output 470 21.67 10,183
Labour 1,000
Expenses 2,000 Normal
loss
50 5.00 250
Overheads 1,000
Abnormal gain 20
433
520 10,433 520 10,433
FUNDAMENTALS OF MANAGEMENT ACCOUNTING
251
PROCESS COSTING
2006.1
Note that the balancing value in the quantity column is now on the debit side. It represents the abnormal
gain. The calculation of the cost per unit remains the same, but now there is an additional entry on the
debit side.
Exercise 9.1
Following the principles that you have learned so far, attempt to produce the scrap stock
and abnormal gain accounts yourself, before you look at the accounts which follow.
Solution
Note that the balance carried down in the scrap stock account is only £150. This represents
the cash available from the sale of the loss. The loss which actually occurred was only 30 kg.
In the abnormal gain account the balance of £333 represents the net benefit of
producing a smaller loss than expected (this is after deducting the scrap sale proceeds
which would have been received if the normal loss had occurred).
9.5 Closing work in progress: the concept
of equivalent units
To calculate a unit cost of production it is necessary to know how many units were
produced in the period. If some units were only partly processed at the end of the period,
then these must be taken into account in the calculation of production output. The
concept of equivalent units provides a basis for doing this. The work in progress (the
partly finished units) is expressed in terms of how many equivalent complete units it
represents. For example, if there are 500 units in progress which are 25 per cent complete,
these units would be treated as the equivalent of 500 25% ¼125 complete units.
A further complication arises if the work in progress has reached different degrees of
completion in respect of each cost element. For example, you might stop the process of
cooking a casserole just as you were about to put the dish in the oven. The casserole would
probably be complete in respect of ingredients, almost complete in respect of labour, but
Scrap stock account
££
Process normal loss 250 Bank/debtors: (50 20) £5 150
Abnormal gain
100
250 250
Abnormal gain account
££
Scrap stock 100 Process 433
Profit and loss/income statement
333
433 433
STUDY MATERIAL C1
252
PROCESS COSTING
2006.1
most of the overhead cost would be still to come in terms of the cost of the power to cook
the casserole.
It is common in many processes for the materials to be added in full at the start of
processing and for them to be converted into the final product by the actions of labour
and related overhead costs. For this reason, labour and overhead costs are often referred
to as conversion costs.
Conversion cost is the ‘cost of converting material into finished product, typically
including direct labour, direct expense and production overhead.’ CIMA Terminology
To overcome the problem of costs being incurred at different stages in the process,
a separate equivalent units calculation is performed for each cost element. An example will
help to make this clear. For simplicity, losses have been ignored. These will be introduced
in the next example.
Example
Input materials 1,000 kg @ £9 per kg
Labour cost £4,800
Overhead cost £5,580
Outputs Finished goods: 900 kg
Closing work in progress: 100 kg
The work in progress is completed:
100% as to material
60% as to labour
30% as to overhead
Now that you are beginning to learn about more complications in process costing, this is a good point
to get into the habit of producing an input/output reconciliation as the first stage in your workings. This
could be done within the process account, by balancing off the quantity columns in the way that we have
done so far in this chapter. However, with more complex examples it is better to have total quantity
columns in your working paper and do the ‘balancing off’ there.
In the workings table which follows, the first stage is to balance the input and output quantities, that is, check
that the total kg input is equal to the total kg output. Then, each part of the output can be analysed to show how
many equivalent kg of each cost element it represents.
Equivalent kg to absorb cost
Input kg Output kg Materials Labour Overhead
Materials 1,000 Finished goods 900 900 900 900
Closing WIP 100
(100%) 100 (60%) 60 (30%) 30
1,000 1,000 1,000 960 930
Costs
£9,000 £4,800 £5,580
Cost/eq. unit £9 £5 £6
For the equivalent unit calculations there is a separate column for each cost element. The number of
equivalent units is found by multiplying the percentage completion by the number of kg in progress. For
example, equivalent kg of labour in progress is 100 kg 60% ¼60 equivalent kg.
The number of equivalent units is then totalled for each cost element and a cost per equivalent unit is
calculated.
FUNDAMENTALS OF MANAGEMENT ACCOUNTING
253
PROCESS COSTING
2006.1
These costs per equivalent unit are then used to value the finished output and the closing work in
progress.
The process account is shown below, together with the calculation of the value of the closing work in
progress. Note that this method may be used to value the finished output, but it is easier to total the equivalent
unit costs (£9 + £5 + £6) and use the total cost of £20 multiplied by the finished output of 900 kg.
Closing WIP valuation £
Materials 100 equivalent units £9 900
Labour 60 equivalent units £5 300
Overheads 30 equivalent units £6 180
1,380
Process account
kg £ kg £/kg £
Materials 1,000 9,000 Finished goods 900 20.00 18,000
Labour 4,800 WIP 100 13.80 1,380
O/Hds
5,580
1,000 19,380 1,000 19,380
The next example follows the same principles but it includes process losses. Work
through the equivalent units table carefully and ensure that you understand where each
figure comes from.
Example: Closing work in progress
Data concerning process 2 last month was as follows:
Transfer from process 1 400 kg at a cost of £2,150
Materials added 3,000 kg £6,120
Conversion costs £2,344
Output to finished goods 2,800 kg
Output scrapped 400 kg
Normal loss 10 per cent of materials
added in the period
The scrapped units were complete in materials added but only 50 per cent complete in respect of
conversion costs. All scrapped units have a value of £2 each.
There was no opening work in progress, but 200 kg were in progress at the end of the month, at the
following stages of completion:
80% complete in materials added
40% complete in conversion costs
You are required to write up the accounts for the process.
Solution
The first step is to produce an input/output reconciliation as in the last example. Notice that the losses
are not complete. You will need to take account of this in the equivalent units columns. And remember
that the normal loss units do not absorb any of the process costs. They are valued at their scrap value
only, so they must not be included as part of the output to absorb costs.
STUDY MATERIAL C1
254
PROCESS COSTING
2006.1
Notes:
1. The abnormal loss is inserted in the output column as a balancing figure. Losses are 50 per cent
complete in conversion costs. Therefore, the 100 kg of abnormal loss represents 50 equivalent
complete kg in respect of conversion costs.
2. By convention, the scrap value of normal loss is usually deducted from the first cost element.
For each cost element the costs incurred are divided by the figure for equivalent kg produced. For
example: the cost per kg for materials added ¼£6;120=3;060 ¼ £2 per kg.
The unit rates can now be used to value each part of the output. For example the 160 equivalent kg of
materials added in the work in progress are valued at 160 £2 ¼£320. The 80 equivalent kg of
conversion costs in work in progress are valued at 80 £0.80 ¼£64.
It is now possible to draw up the relevant accounts using these valuations of each part of the process output.
Exercise 9.2
See if you can complete the process accounts before looking at the rest of the solution.
Remember that the normal loss is valued at its scrap value.
Solution
Process 2 account
kg £ kg £
Process 1 400 2,150 Finished goods 2,800 9,240
Materials added 3,000 6,120 Normal loss 300 600
Conversion costs 2,344 Abnormal loss 100 290
Work in progress 200 484
3,400 10,614 3,400 10,614
Equivalent kg to absorb cost
Input kg Output kg
Process 1
transfer
Materials
added
Conversion
costs
Process 1 transfer 400 Finished goods 2,800 2,800 2,800 2,800
Material added 3,000 Normal loss 300
Abnormal loss
1
100 100 100 50
Work in progress 200 200 160 80
3,400 3,400 3,100 3,060 2,930
Costs ££ £
Incurred in period 2,150 6,120 2,344
Scrap value of
normal loss
2
(600)
1,550 6,120 2,344
Cost per unit £3.30 0.50 2.00 0.80
Valuation Total
Process 1
transfer
Materials
added
Conversion
costs
££ £ £
Finished goods 9,240 1,400 5,600 2,240
Abnormal loss 290 50 200 40
Work in progress 484 100 320 64
FUNDAMENTALS OF MANAGEMENT ACCOUNTING
255
PROCESS COSTING
2006.1
Abnormal loss account
££
Process 2 290 Scrap stock 200
Profit and loss 90
290 290
9.6 Previous process costs
A common problem that students experience when studying process costing is
understanding how to deal with previous process costs. An important point that you
should have grasped by now is that production passes through a number of sequential
processes. Unless the process is the last in the series, the output of one process becomes
the input of the next. A common mistake is to forget to include the previous process cost
as an input cost in the subsequent process.
You should also realise that all of the costs of the previous process (materials, labour
and overhead) are combined together as a single cost of ‘input material’ or ‘previous
process costs’ in the subsequent process.
In the workings for the example in Section 9.5 we assumed that the work in progress
must be 100 per cent complete in respect of Process 1 costs. This is also an important
point to grasp. Even if the Process 2 work had only just begun on these units, there cannot
now be any more cost to add in respect of Process 1. Otherwise the units would not yet
have been transferred out of Process 1 into Process 2.
9.7 Opening work in progress
Opening work in progress consists of incomplete units in process at the beginning of the
period. Your syllabus requires you to know how to value work in progress using the
average cost method. With this method, opening work in progress is treated as follows:
1. The opening work in progress is listed as an additional part of the input to the process
for the period.
2. The cost of the opening WIP is added to the costs incurred in the period.
3. The cost per equivalent unit of each cost element is calculated as before, and this is
used to value each part of the output. The output value is based on the average cost per
equivalent unit, hence the name of this method.
The best way to see how this is done is to work through some examples. The last
two examples in this chapter include some opening work in progress. Work through
them carefully, and try to learn the layout of the working paper so that you can use it quickly to
do any workings that you need in the assessment. It will save you valuable time!
Scrap account
££
Process 2 600 Bank/debtors: (300 +100) £2 800
Abnormal loss account
200
800 800
STUDY MATERIAL C1
256
PROCESS COSTING
2006.1
Example: Opening work in progress
The following information is available for Process 3 in June:
Normal loss is 10 per cent of input from process 2; 70 units were scrapped in the month, and all
scrap units realise £0.20 each.
Output to the next process was 850 units.
You are required to complete the account for process 3 in June.
Solution
As before, the first step is to complete an input/output reconciliation and then to extend this to calculate
the number of equivalent units for each cost element.
Equivalent units to absorb cost
Input Units Output Units
Process 2
input
Materials
added
Conversion
costs
Opening stock
1
100 To process 4 850 850 850 850
Process 2
2
900 Normal loss 90
Abnormal gain
3
(20) (20) (20) (20)
Closing stock
4
80 80 56 44
1,000 1,000 910 886 874
Costs £ £ £
Opening stock
5
176 300 216
Input costs 1,600 3,294 4,190
Normal loss value (18)
1,758 3,594 4,406
££ £ £
Cost per unit 11.029 1.932 4.056 5.041
Evaluation
6
To process 4 9,375 1,642 3,448 4,285
Abnormal gain (221) (39) (81) (101)
Closing stock 604 155 227 222
Notes:
1. The opening stock is included as part of the input in the input/output reconciliation. The degree of
completion of the opening stock is not relevant, because we are going to average its cost over all
units produced in the period.
2. Note that we are not told the quantity of material added because it does not affect the number of
basic units processed.
3. The number of units scrapped is less than the normal loss. There is thus an abnormal gain.
Degree of completion and cost
Units Cost
Process 2
input
Materials added
in Process 3
Conversion
costs
£%£% £%£
Opening stock 100 692 100 176 60 300 30 216
Closing stock 80 100 70 55
Input costs:
Input from process 2 900 1,600
Materials added in process 3 3,294
Conversion costs 4,190
FUNDAMENTALS OF MANAGEMENT ACCOUNTING
257
PROCESS COSTING
2006.1
4. The equivalent units of closing stock takes account of the degree of completion for each cost
element.
5. The opening stock is included in the statement of costs, so that its value is averaged over the
equivalent units produced in the period.
6. In the evaluation section, the unit rate for each cost element is multiplied by the number of equivalent units in
each part of the output. These values can then be used to complete the process account.
Process 3 account
Units £ Units £
Opening stock 100 692 Process 4 850 9,375
Process 2 900 1,600 Normal loss 90 18
Materials added 3,294 Closing stock 80 604
Conversion costs 4,190
Abnormal gain 20 221
1,020 9,997 1,020 9,997
Exercise 9.3
To give yourself some extra practice, draw up the abnormal gain account and the scrap
account.
Solution
Abnormal gain account
££
Scrap stock (20 £0.20) 4 Process 3 221
Profit and loss account
217
221 221
Scrap account
££
Normal loss 18 Bank/debtor: (90 20) £0.20 14
Abnormal gain 4
18 18
9.8 Process costing: a further example
You must try to get as much practice as possible in preparing process cost accounts and
you will find it much easier if you use a standard format for the working papers. Although
you will not be required to reproduce the workings in the assessment, for your own benefit
you need to work quickly through the available data to produce the required answer.
Work carefully through the next example or better still try it for yourself before
looking at the suggested solution. Notice that the scrapped units are not complete. You
will need to take account of this in the equivalent units calculations.
STUDY MATERIAL C1
258
PROCESS COSTING
2006.1
Example
The following information is available for process 2 in October:
Degree of completion and cost
Units Cost
Process 1
input
Materials added
in process 2
Conversion
costs
£%£% £%£
Opening stock 600 1,480 100 810 80 450 40 220
Closing stock 350 100 90 30
Input costs:
Input from process 1 4,000 6,280
Materials added in process 2 3,109
Conversion costs 4,698
Normal loss is 5 per cent of input from process 1.
300 units were scrapped in the month. The scrapped units had reached the following degrees of
completion.
Materials added 90%
Conversion cost 60%
All scrapped units realised £1 each.
Output to the next process was 3,950 units.
You are required to complete the account for process 2 and for the abnormal loss or gain in October.
Solution
The first step is to prepare an input/output reconciliation to see if there was an abnormal loss or
abnormal gain. This is found as a balancing figure in the output column.
Equivalent units to absorb cost
Input Units Output Units
Process 1
input
Materials
added
Conversion
costs
Opening stock 600 To process 3 3,950 3,950 3,950 3,950
Process 1 4,000 Normal loss 200
Abnormal loss 100 100 90 60
Closing stock 350 350 315 105
4,600 4,600 4,400 4,355 4,115
Costs ££ £
Opening stock 810 450 220
Input costs 6,280 3,109 4,698
Normal loss value
(200)
6,890 3,559 4,918
££ £ £
Cost per unit 3.578 1.566 0.817 1.195
Evaluation
To process 3 14,133 6,186 3,227 4,720
Abnormal loss 303 157 74 72
Closing stock 931 548 257 126
Process 2 account
Units £ Units £
Opening stock 600 1,480 Process 3 3,950 14,133
Process 1 4,000 6,280 Normal loss 200 200
Materials added 3,109 Abnormal loss 100 303
Conversion costs
4,698 Closing stock 350 931
4,600 15,567 4,600 15,567
FUNDAMENTALS OF MANAGEMENT ACCOUNTING
259
PROCESS COSTING
2006.1
Abnormal loss account
££
Process 2 303 Scrap stock 100
Profit and loss
203
303 303
Scrap account
££
Normal loss 200 Bank/debtors:
(200 + 100) £1
300
Abnormal loss
100
300 300
9.9 Contrasting process costing and specific
order costing
Now that you have a clear picture of how process costing works you are in a position to
think about the differences between process costing and specific order costing methods.
Remember that specific order costing is the collective term for the costing methods that
you learned about in the last chapter: job, batch and contract costing.
Process costing can be contrasted with specific order costing methods such as job, batch
and contract costing in a number of ways:
since there is a continuous flow of identical units, individual cost units cannot be
separately identified in a process costing environment. In a specific order costing
environment, each cost unit is different from all others;
costs incurred are averaged over the units produced in a process costing system. In
contrast to a specific order costing system, it is not possible to allocate costs to specific
cost units;
each cost unit usually undergoes the same process or sequence of processes. In specific
order costing environments, each cost unit often involves different operations or
processes, depending on the customer’s requirements;
in process costing environments, items are usually produced to replenish inventory,
rather than for a specific customer’s requirements;
9.10 Summary
Having read this chapter the main points that you should understand are as follows.
1. The process costing method is appropriate for organisations that produce a
continuous flow of identical units. The costs incurred are averaged over the number
of units produced in the period in order to determine the cost per unit.
STUDY MATERIAL C1
260
PROCESS COSTING
2006.1
2. There may be more than one process involved in process costing. The output of one
process becomes the input of the next process in the sequence.
3. A normal loss is the expected level of loss for the period. The normal loss does not
absorb any process costs. If it is saleable it is valued at its scrap value, otherwise the
normal loss will have zero value.
4. The scrap value of the normal loss is conventionally deducted from the cost of the
first cost element in the analysis, which is usually either materials cost or previous
process cost.
5. If losses are greater than the normal loss, the extra loss is called an abnormal loss. If
losses are lower than the normal loss the difference is called an abnormal gain.
6. Abnormal losses and gains are valued at the same unit rate as good output. Their
scrap values do not affect the main process account but are accounted for in a
separate abnormal loss or abnormal gain account.
7. Where there are incomplete units in the process at the end of the period, that is,
when there is closing work in progress, it is necessary to determine the number of
equivalent units of production in order to calculate the production cost per unit.
8. There are a number of ways in which process costing can be contrasted with specific
order costing methods such as job, batch and contract costing.
FUNDAMENTALS OF MANAGEMENT ACCOUNTING
261
PROCESS COSTING
2006.1
Revision Questions
Question 1 Multiple choice
1.1 Process B had no opening stock. 13,500 units of raw material were transferred in at
£4.50 per unit. Additional material at £1.25 per unit was added in process. Labour
and overheads were £6.25 per completed unit and £2.50 per unit incomplete. If
11,750 completed units were transferred out, what was the closing stock in
process B?
(A) £ 77,625
(B) £14,437.50
(C) £141,000
(D) £21,000
1.2 In a process account, abnormal losses are valued:
(A) at their scrap value.
(B) at the same rate as good production.
(C) at the cost of raw materials.
(D) at good production cost less scrap value.
1.3 A chemical process has a normal wastage of 10 per cent of input. In a period,
2,500 kg of material were input and there was an abnormal loss of 75 kg.
What quantity of good production was achieved?
(A) 2,175 kg
(B) 2,250 kg
(C) 2,325 kg
(D) 2,475 kg
1.4 In process costing, where losses have a positive scrap value, when an abnormal gain
arises the abnormal gain account is:
(A) credited with the normal production cost of the abnormal gain units.
(B) debited with the normal production cost of the abnormal gain units and credited
with the scrap value of the abnormal gain units.
(C) credited with the normal production cost of the abnormal gain units and debited
with the scrap value of the abnormal gain units.
(D) credited with the normal production cost of the abnormal gain units and credited
with the scrap value of the abnormal gain units.
9
263 2006.1
Data for questions 1.5–1.7
X plc makes one product, which passes through a single process. Details of the process are
as follows:
Materials: 5,000 kg at 50p per kg
Labour: £800
Production overheads 200% of labour
Normal losses are 20 per cent of input in the process, and without further processing
any losses can be sold as scrap for 30 p per kg.
The output for the period was 3,800 kg from the process.
There was no work in progress at the beginning or end of the period.
1.5 What value will be credited to the process account for the scrap value of the normal loss?
(A) £300
(B) £530
(C) £980
(D) £1,021
1.6 What is the value of the abnormal loss?
(A) £60
(B) £196
(C) £230
(D) £245
1.7 What is the value of the output?
(A) £3,724
(B) £4,370
(C) £4,655
(D) £4,900
Data for questions 1.8–1.10
A product is manufactured as a result of two processes, A and B. Details of process B for
the month of August were as follows:
Materials transferred from process A 10,000 kg valued at £40,500
Labour costs 1,000 hours @ £5.616 per hour
Overheads 50% of labour costs
Output transferred to finished goods 8,000 kg
Closing work in progress 900 kg
Normal loss is 10 per cent of input and losses do not have a scrap value.
Closing work in progress is 100 per cent complete for material, and 75 per cent
complete for both labour and overheads.
1.8 What is the value of the abnormal loss (to the nearest £ )?
(A) Nil
(B) £489
(C) £544
(D) £546
REVISION QUESTIONS C1
264
PROCESS COSTING
2006.1
1.9 What is the value of the output (to the nearest £ )?
(A) £39,139
(B) £43,488
(C) £43,680
(D) £43,977
1.10 What is the value of the closing work in progress (to the nearest £ )?
(A) £4,403
(B) £4,698
(C) £4,892
(D) £4,947
Data for questions 1.11 and 1.12
The following data relates to a process for the latest period:
Opening work in process 1,000 litres valued at £1,500
Input 30,000 litres costing £15,000
Conversion costs £10,000
Output 24,000 litres
Closing work in process 3,500 litres
Losses in process are expected to be 10 per cent of period input. They are complete as
to input material costs but are discovered after 60 per cent conversion. Losses have a scrap
value of £0.20 per litre.
Closing work in process is complete as to input materials and 80 per cent complete as to
conversion.
1.11 The number of material-equivalent units was:
(A) 24,000
(B) 28,000
(C) 30,000
(D) 31,000
1.12 The number of conversion-equivalent units was:
(A) 27,100
(B) 27,300
(C) 28,000
(D) 30,100
Data for questions 1.13 and 1.14
PP Ltd makes one product, which passes through a single process. The details of the
process for period 2 were as follows.
There were 400 units of opening work in progress, valued as follows:
Material £49,000
Labour £23,000
Production overheads £3,800
No losses are expected in the process.
FUNDAMENTALS OF MANAGEMENT ACCOUNTING
265
PROCESS COSTING
2006.1
During the period, 900 units were added to the process, and the following costs occurred:
Material £198,000 (900 units)
Labour £139,500
Production overheads £79,200
There were 500 units of closing work in progress, which were 100 per cent complete for
material, 90 per cent complete for labour and 40 per cent complete for overheads. No
losses were incurred in the process.
PP Ltd uses weighted average costing.
1.13 How many equivalent units are used when calculatingthecostperunitinrelationtolabour?
(A) 450
(B) 850
(C) 1,250
(D) 1,300
1.14 The value of completed output for the period was
(A) £171,555
(B) £201,500
(C) £274,488
(D) £322,400
Question 2 Short objective-test questions
2.1 When the actual loss in a process is less than the expected loss for the period, there is an:
abnormal loss &
abnormal gain &
2.2 Input to a process last period was 5,000 kg. There was no opening work in progress
but 800 kg were in process at the end of the period. Normal loss is 20 per cent of
input. During the period 4,100 kg were transferred to the next process.
(a) During the period there was an:
abnormal loss &
abnormal gain &
(b) The abnormal loss/gain amounted to
kg
2.3 Last period, an abnormal gain of 50 kg arose in process 1. Normal loss was 400 kg.
The cost of good output from process 1, after allowing for the abnormal gain, was
£3.50 per kg. Scrap from process 1 can be sold for £0.20 per kg.
The scrap account in respect of process 1 for the period is shown below.
The values to be entered as A, B and C in the scrap account are:
A
B C
Scrap account
££
Process 1 A Abnormal gain B
Balance c/d C
REVISION QUESTIONS C1
266
PROCESS COSTING
2006.1
2.4 In process 2 at the end of a period, 200 units are in progress. They are 100 per cent
complete in respect of materials, 50 per cent complete in respect of labour and 20
percent complete in respect of overhead. The cost of an equivalent complete unit for
the period was £4 for materials, £3 for labour and £2 for overhead. Complete the
following table to show the value of the work in progress at the end of the period.
Equivalent units
in progress
Valuation
£
Materials
Labour
Overhead
2.5 In the following process, all losses were fully processed and scrap units from the
process can be sold for £3 per unit.
The values to be entered as A and B in the process account below are:
A
B
Process account [extract]
Units £
Finished goods 4,000 88,000
Normal loss 90 A
Abnormal loss 50 B
Question 3 Process costing
A firm operates a process, the details of which for the period were as follows:
There was no opening work in progress.
During the period 8,250 units were received from the previous process at a value of
£453,750, labour and overheads were £350,060 and material introduced was £24,750.
At the end of the period the closing work-in-progress was 1,600 units, which were
100 per cent complete in respect of materials, and 60 per cent complete in respect of
labour and overheads.
The balance of units were transferred to finished goods.
Requirements
(a) The number of equivalent units of labour and overheads produced during the period
was
(b) In the process account for the period, the following values will be credited:
(i) finished goods value: £
(ii) closing work in progress value: £
FUNDAMENTALS OF MANAGEMENT ACCOUNTING
267
PROCESS COSTING
2006.1
Question 4 Process costing with abnormal losses
Chemical Processors manufacture Wonderchem using two processes mixing and
distillation. The following details relate to the distillation process for a period:
No opening work in progress
Input from mixing 36,000 kg at a cost of £166,000
Labour for period £43,800
Overheads for period £29,200
Closing WIP of 8,000 kg, which was 100 per cent complete for materials and 50 per cent
complete for labour and overheads.
The normal loss in distillation is 10 per cent of fully complete production. Actual loss in
the period was 3,600 kg, fully complete, which was scrapped.
Requirements
(a) The abnormal loss for the period was
kg.
(b) The number of equivalent kg produced during the period was:
materials:
equivalent kg.
labour and overhead:
equivalent kg.
(c) (i) The value of the abnormal loss is £
(ii) (Tick the correct box): This value is entered in the process account as a:
debit &
credit &
(d) The values to be credited in the process account in respect of the following outputs
for the period are:
finished goods £
normal loss £
closing work in progress £
Question 5 Process costing with opening work in progress
A company operates an expensive processing plant to produce a single product from one
process. At the beginning of October, 3,400 completed units were still in the processing
plant awaiting transfer to finished stock. They were valued as follows:
£
Direct material 25,500
Direct wages 10,200
Production overhead 20,400 (200% of direct wages)
During October, 37,000 further units were put into process and the following costs
charged to the process:
£
Direct materials 276,340
Direct wages 112,000
Production overhead 224,000
REVISION QUESTIONS C1
268
PROCESS COSTING
2006.1
A total of 36,000 units were transferred to finished stock and 3,200 units remained in
work-in-progress at the end of October, which were complete as to material and half
complete as to labour and production overhead. The normal level of scrap (1,200 units)
occurred during the process.
Requirements
(a) The number of equivalent units produced during the period was:
materials
labour and overhead
(b) The value of the outputs from the process during the period was:
finished goods £
closing work in progress £
Question 6 Process account
Complete the following account for process 3 last period. The work in progress was
complete as to materials and 50 per cent complete as to labour and overhead.
Process 3 account
Units £ Units £
Process 2 input 2,000 8,000 Finished goods 1,800
Labour and overhead 3,800 Work in progress 200
2,000 11,800 2,000 11,800
FUNDAMENTALS OF MANAGEMENT ACCOUNTING
269
PROCESS COSTING
2006.1
Solutions to
Revision Questions
Solution 1
For some of these multiple-choice questions you will need to use some fairly extensive
workings. In the assessment you will not be awarded marks for the workings, but do not
be tempted to rush them: they are an important part of answering the question, and they
will be of no use to you if you cannot read them!
1.1 Answer: (B)
Closing stock in process B ¼(13,500 11,750) units ¼1,750 units
Unit value ¼£4.50 + £1.25 + £2.50 ¼£8.25
Closing stock value ¼£8.25 1,750 ¼£14,437.50
1.2 Answer: (B)
Abnormal losses are valued at the same rate as good production, so that their
occurrence does not affect the cost of good production.
1.3 Answer: (A)
kg
Input 2,500
Normal loss (10%) (250)
Abnormal loss
(75)
Good production
2,175
1.4 Answer: (C)
The abnormal gain account shows the net benefit of the abnormal gain. The scrap
value must be debited to the abnormal gain account to allow for the ‘forgone’ scrap
value of the normal loss units which did not arise.
1.5 Answer: (A)
Normal loss 5,000 kg 20% ¼1,000 kg @ 30p ¼£300
9
271 2006.1
1.6 Answer: (C)
Cost per kg ¼
£4,900 £300
4,000
¼ £1:15 per kg
Output 3,800 þ abnormal loss 200 ¼ 4,000 kg
Abnormal loss £1:15 200 kg ¼ £230:
1.7 Answer: (B)
Value of output ¼£1.15 3,800 kg ¼£4,370.
Equivalent unit table for 1.8–1.10
1.8 Answer: (D)
Value of abnormal loss ¼100 (£4.50 + £0.96) ¼£546.
1.9 Answer: (C)
Value of output ¼8,000 (£4.50 + £0.96) ¼£43,680.
Abnormal loss kg
Input 5,000
Normal loss (1,000)
Output
(3,800)
Abnormal loss
200
Production costs £
Materials 2,500
Labour 800
Production overheads
1,600
4,900
Units Materials Labour/overheads
% EU % EU
Output 8,000 100 8,000 100 8,000
Normal loss 1,000
Abnormal loss 100 100 100 100 100
Closing work in progress
900 100 900 75 675
Total equivalent units
10,000 9,000 8,775
Costs £40,500 £8,424
Equivalent unit cost £4.50 £0.96
SOLUTIONS TO REVISION QUESTIONS C1
272
PROCESS COSTING
2006.1
1.10 Answer: (B)
1.11 Answer: (B)
Workings for 1.11 are shown as part of solution 1.12.
1.12 Answer: (A)
1.13 Answer: (C)
Workings are shown as part of solution 1.14.
1.14 Answer: (D)
Equivalent units table
Value of completed output ¼800 (£190 + £130 + £83) ¼£322,400.
Solution 2
2.1 When the actual loss in a process is less than the expected loss for the period, there is
an abnormal gain.
Equivalent litres
Input Litres Output Litres
Input
material
Conversion
costs
Opening WIP 1,000 Finished output 24,000 24,000 24,000
Input 30,000 Normal loss 3,000
Abnormal loss 500 500 (60%) 300
Closing WIP 3,500 3,500 (80%) 2,800
31,000 31,000 28,000 27,100
Materials Labour Production o/h
Description Units % EU % EU % EU
Output 800 100 800 100 800 100 800
Closing WIP 500 100
500 90 450 40 200
EU
1,300 1,250 1,000
£££
Costs Period 198,000 139,500 79,200
Closing WIP
49,000 23,000 3,800
Total cost
247,000 162,500 83,000
Cost per equivalent unit
190 130 83
Closing work in progress
900 £4.50 4,050
675 £0.96 648
4,698
FUNDAMENTALS OF MANAGEMENT ACCOUNTING
273
PROCESS COSTING
2006.1
2.2
2.3 A £80; B £10; C £70.
2.4
2.5 A £270; B £1,100
*Abnormal loss units are valued at the same rate as good output (£88,000/
4,000 ¼£22).
Solution 3
You can use the standard layout for the working paper that you should have become
accustomed to when working through this chapter. You can then pick out the relevant
parts that you need for your answers.
There are no losses, therefore the question is quite straightforward.
The transfer to finished goods is calculated as follows: 8,250 units input, less 1,600 units
in progress, equals 6,650 units to finished stock.
(a) 7,610
(b) (i) £691,600
(ii) £136,960
kg
Transferred to next process 4,100
Normal loss (20% 5,000) 1,000
Closing work in process 800
Abnormal gain
(900)
5,000
Scrap account
££
Process 1 normal loss
(400 kg £0.20) 80
Abnormal gain
(50 £0.20)
10
Balance c/d
70
80 80
Equivalent units in progress Valuation £
Materials 200 ( £4) 800
Labour 100 ( £3) 300
Overhead 40 ( £2) 80
1,180
Process account [extract]
Units £
Finished goods 4,000 88,000
Normal loss 90 ( £3) 270
Abnormal loss 50 ( £22*) 1,100
SOLUTIONS TO REVISION QUESTIONS C1
274
PROCESS COSTING
2006.1
Workings:
Solution 4
Read the question carefully. The normal loss calculation is based on the completed
production rather than on the more usual basis of input to the process.
The losses are completely processed, therefore you can use the total cost per unit to
value the abnormal loss.
(a) The abnormal loss for the period was 800 kg.
Workings:
Equivalent units produced
Input Units Output Units
Previous
process
Materials
added
Labour
and o/h
Previous
process
8,250 Finished
goods
6,650 6,650 6,650 6,650
Closing
WIP
1,600 1,600 1,600 960(60%)
8,250 Equiv.
units
produced
8,250 8,250 8,250 7,610
Costs £ £ £ £
Period
costs
453,750 24,750 350,060
Cost per
equiv. unit
Valuation
104 55 3 46
Finished
goods
691,600
Closing
WIP
136,960 88,000
(1,600 £55)
4,800
(1,600 £3)
44,160
(960 £46)
kg
Input 36,000
Less: Closing WIP
(8,000)
Production
28,000
Normal loss:
10% 28,000 kg
2,800
Actual loss
3,600
\ Abnormal loss
800
FUNDAMENTALS OF MANAGEMENT ACCOUNTING
275
PROCESS COSTING
2006.1
(b) Materials: 33,200 equivalent kg.
Labour and overhead: 29,200 equivalent kg.
(c) (i) £6,000
(ii) Credit.
(d) Finished goods: £183,000
Normal loss: £0
Closing work in progress: £50,000
Workings:
Solution 5
There is opening work in progress to deal with in this question, so you will probably
find it easiest to use the full working schedule, beginning with an input/output
reconciliation. Although you will not be awarded any marks for these workings, they will
help you to achieve the required 100 per cent accuracy.
Do not be confused by the fact that the opening work in progress consists of complete
units. Simply deal with it using the method that you learned in this chapter, that is
include it as part of the input and include its value in the cost section of your working
schedule.
(a) Materials: 39,200
Labour and overhead: 37,600.
(b) Finished goods: £628,200
Closing work in progress: £40,240.
Equivalent units
Total Material Labour Overhead
Input kg Output kg kg kg kg
From
mixing
36,000 Finished goods 24,400 24,400 24,400 24,400
Abnormal loss
800 800 800 800
25,200 25,200 25,200 25,200
Normal loss 2,800
Closing WIP:
Materials (100%) 8,000 8,000
Labour (50%) 4,000
Overheads (50%)
4,000
36,000 33,200 29,200 29,200
Cost (£) 239,000 166,000 43,800 29,200
Cost per unit (£ ) 7.50 5.00 1.50 1.00
Evaluation )
Finished goods 183,000
Abnormal loss 6,000
Closing WIP 50,000 40,000 6,000 4,000
SOLUTIONS TO REVISION QUESTIONS C1
276
PROCESS COSTING
2006.1
Workings:
Solution 6
You will need to prepare a statement of equivalent units and calculate the cost per
equivalent unit.
There are no losses to be accounted for, so all of the cost incurred is to be divided over
the completed units and the units in progress.
Be accurate with your workings. Although they will not be awarded marks, they will help
you to achieve the necessary 100 per cent accuracy.
Equivalent units to absorb cost
Input Units Output Units Materials Labour/OH
Process 2 input 2,000 Finished goods 1,800 1,800 1,800
Closing WIP
200 200 (50%) 100
2,000 2,000 2,000 1,900
Costs £ £ £
Incurred in
period
8,000 3,800
Cost per unit 6 4 2
Evaluation
Finished
goods
(1,800 £6) 10,800
Closing WIP 1,000 800 200
Equivalent units produced
Input Units Output Units Materials Labour Overhead
Opening
WIP
3,400 Finished goods 36,000 36,000 36,000 36,000
Further
units
37,000 Normal loss 1,200
Closing WIP 3,200 3,200 1,600 1,600
40,400 40,400 39,200 37,600 37,600
Cost ££££
Opening WIP 56,100 25,500 10,200 20,400
Period costs 612,340 276,340 112,000 224,000
668,440 301,840 122,200 244,400
Cost per unit 17.45 7.70 3.25 6.50
Evaluation
Finished goods 628,200
Closing WIP 40,240 24,640 5,200 10,400
FUNDAMENTALS OF MANAGEMENT ACCOUNTING
277
PROCESS COSTING
2006.1
Process 3 account
Units £ Units £
Process 2 input 2,000 8,000 Finished goods 1,800 10,800
Labour and overhead
3,800 Work in progress 200 1,000
2,000 11,800 2,000 11,800
SOLUTIONS TO REVISION QUESTIONS C1
278
PROCESS COSTING
2006.1
10
Presenting Management
Information
Presenting
Management
Information
10.1 Introduction
In this chapter you will be learning about the effective presentation of management
accounting reports to managers so that they have the information they need to be able to
manage their area of responsibility in the most effective way.
You will be learning how these reports might be structured and about a range of
different performance measures that might be highlighted in management reports in a
variety of different types of organisation.
10.2 Subjective and objective classification
In chapter 1 of this text you learned that the classification of costs involved arranging the
costs into logical groups by nature, purpose or responsibility.
You saw that classification by nature involves grouping costs according to whether they
are material cost, labour cost or expenses. This classification is referred to as subjective
classification.
10
LEARNING OUTCOMES
After completing this chapter you should be able to:
"
explain the difference between subjective and objective classifications of expenditure
and the importance of tracing costs both to products/services and to responsibility
centres;
"
construct coding systems that facilitate both subjective and objective classification of
costs;
"
prepare financial statements that inform management;
"
explain why gross revenue, value added, contribution, gross margin, marketing
expense, general and administration expense, etc. might be highlighted in
management reporting;
"
compare and contrast management reports in a range of organisations including
commercial enterprises, charities and public sector undertakings.
281 2006.1
Classification by purpose involves grouping costs according to the reason they are
incurred, for example whether they are a direct or indirect cost of a particular cost object.
This classification is referred to as objective classification.
10.2.1 Responsibility centres
Classification by responsibility involves grouping costs according to which individual
manager or management team is responsible for the control of the cost. A responsibility
accounting system divides an organisation into several parts, or responsibility centres,
with an individual manager responsible for the operation and performance of each
responsibility centre.
10.2.2 Reporting management accounting information
The different systems of classification allow expenditure to be reported in different ways,
according to the reason why the information is being provided.
For example managers might be interested in assessing the profitability of a particular
product or service, in which case costs might be classified by purpose (objective
classification) so that they can be traced to individual products or services.
Alternatively managers might be interested in assessing the expenditure incurred by a
particular responsibility centre within the organisation. In this situation it would be more
useful to trace expenditure to individual responsibility centres rather than to particular
products or services.
10.3 Coding of costs
The CIMA Terminology defines a code as a ‘brief, accurate reference designed to assist
classification of items by facilitating entry, collation and analysis.’
The coding system is based on the selected cost classifications. It provides a way of
expressing the classification of each cost or item of expenditure in a shortened symbolised form.
10.3.1 Composite codes
The CIMA Terminology describes the use of composite symbols in codes. For example, let
us consider the hypothetical composite symbol 298.311.
The first three digits might indicate the nature of the expenditure.
Remember this is the subjective classification of the expenditure.
2 labour
9 semi-skilled
8 grade 8
Anyone who is familiar with the coding system would be able to identify that the
expenditure was incurred on grade 8 semi-skilled labour.
The last three digits might indicate the cost object to be charged, for example a
particular cost unit or cost centre.
STUDY MATERIAL C1
282
PRESENTING MANAGEMENT INFORMATION
2006.1
Remember this is the objective classification of the expenditure.
3 indirect cost
1 north east factory
1 machining department
The code can indicate that the expenditure is to be charged as indirect labour to the
machining department in the north east factory.
The code number 298.311 is much clearer than this lengthy description of where the
cost is to be charged.
10.3.2 The advantages of a coding system
Some of the advantages of a well-designed coding system are as follows.
A code is usually briefer than a description. The example in the previous section
demonstrates this advantage very clearly. This saves time in a manual system and
reduces the data storage capacity required in a computerised system.
A coding system enables costs to be accumulated for each code number so that they can
be logically grouped for reporting to managers.
A code reduces ambiguity. Two people might each use a quite different description for
the same item of expenditure but a code will be more precise.
A code is more suitable than a description in computerised systems so that data
processing is facilitated.
Exercise 10.1
The XY Manufacturing Company uses a four digit code to classify its expenditure items.
The first digit indicates the responsibility centre to be charged.
1 ¼responsibility centre 1, 2 ¼responsibility centre 2 and so on.
The second digit indicates the machine group within the responsibility centre that has
incurred the cost.
1 ¼machine group 1, 2 ¼machine group 2 and so on.
The last two digits indicate the nature of the expenditure.
For example 01 ¼direct materials, 02 ¼indirect materials, 03 ¼direct labour,
11 ¼depreciation, 12 ¼power cost and so on.
(a) State the code number that would be used for the following two expenditure items.
(i) Direct labour cost incurred in machine group 4 within responsibility centre 2
(ii) Power cost incurred on machine group 1 within responsibility centre 1
(b) State the expenditure that is represented by the code number 2202.
Solution
(a) (i) 2403
(ii) 1112
(b) Indirect material cost incurred in machine group 2 within responsibility centre 2
FUNDAMENTALS OF MANAGEMENT ACCOUNTING
283
PRESENTING MANAGEMENT INFORMATION
2006.1
10.3.3 The requirements for an efficient coding system
(a) The code should be unique and certain, that is, each item should have only one
possible code number which can easily be identified from the structure of the code.
(b) The coding system should be comprehensive and elastic, that is, it should be possible
to identify a code for every item and the coding system should be capable of
expanding to accommodate new items.
(c) The code should be as brief as possible, having regard to the amount of detail which is
needed in the analysis of the items being coded.
(d) To minimise errors, the code should incorporate check digits so that a computerised
system can detect coding errors.
(e) The maintenance of the coding system should be centrally controlled. It should not
be possible for individuals to independently add new codes to the existing coding
system.
(f) Wherever possible, all codes should be of the same length. This makes errors easier to
spot and it assists computerised data processing.
10.4 Preparing financial statements that inform
management
The usefulness of a financial statement is greatly enhanced if it highlights subtotals, totals
and performance measures that are relevant to the recipient. This enables the manager
who receives the information to focus on the most relevant information from a point of
view of management action.
A performance measure will be particularly relevant if it is controllable by the manager
for whom the report is prepared, that is if the manager is able to take action to influence
the measure, and if an improvement in the performance measure would improve the
performance of the responsibility centre or the organisation overall.
Let us look now at a number of performance measures that you might see highlighted in
management reports.
10.4.1 Value added
Value added is a performance measure which is sometimes used as an alternative to profit.
Traditionally, value added is calculated as follows.
Value added ¼ sales revenue cost of materials and bought-in services
Since value added excludes all bought-in costs paid to people from outside the organisation,
it effectively focuses on the additional revenue created by the organisation’s own internal
efforts. For this reason it is sometimes used as the basis for labour incentive schemes.
You might sometimes see value added calculated by ‘working backwards’ from the
profit figure:
Value added ¼ profit þinterest þall conversion costs
You should remember from the last chapter that conversion costs are the costs of
converting raw material into the finished product. Conversion costs include direct labour
and production overhead costs.
STUDY MATERIAL C1
284
PRESENTING MANAGEMENT INFORMATION
2006.1
This calculation method is intended to give the same result for the value added. However,
it will only do so if bought-in overhead costs are treated as non-conversion costs.
If you have to calculate value added in an assessment question then you should use
the traditional method of calculation, i.e. sales revenue cost of materials and
bought-in services.
10.4.2 Contribution
You should remember from earlier in this text that contribution is calculated as follows.
Contribution ¼ sales revenue variable costs
Contribution is often highlighted in management reports when it is important for
managers to be able to see whether individual cost objects are generating sufficient
revenue to cover the variable costs they incur.
Highlighting contribution can also help managers to see the potential effect on profit of
an increase or decrease in activity. For example, if it is assumed that variable costs are
linear and that the selling price per unit is constant, then the contribution earned will
change in direct proportion to the change in activity.
Example: a product contribution analysis
This example will demonstrate why it might be important to highlight the contribution
earned by each product.
Product A Product B Product C Total
£000 £000 £000 £000
Gross revenue
931 244 954 2,129
Variable costs:
Direct material and labour 547 87 432 1,066
Variable production overhead 54 58 179 291
Variable marketing expense
9 3 7 19
Total variable cost
610 148 618 1,376
Contribution
321 96 336 753
Fixed production overhead 43 35 34 112
Fixed marketing expense 38 10 40 88
Fixed general and administration
expense
60 56 60 176
Profit/(loss)
180 (5) 202 377
Contribution to sales (PV) ratio
34.5% 39.3% 35.2%
This product contribution analysis reveals the following:
Product B appears to be incurring a loss. Its contribution is not sufficient to cover the
fixed production, marketing, general and administration expenses attributed to it.
However the product is earning a contribution. If the fixed costs attributed to product B
are costs that would be incurred anyway, even if product B was discontinued, then it
may be worth continuing the sale and production of product B since it does earn
FUNDAMENTALS OF MANAGEMENT ACCOUNTING
285
PRESENTING MANAGEMENT INFORMATION
2006.1
a contribution of £96,000 towards these fixed costs. If product B was discontinued then
this £96,000 contribution would be forgone.
Although product B is earning a contribution, it does not at present generate sufficient
contribution to cover its fair share of support costs such as marketing, general and
administration overhead. The profitability of product B does require management
attention.
Product B earns the highest contribution to sales ratio. This means that if gross sales
revenue of product B can be increased without affecting the fixed costs, the resulting
increase in contribution will be higher than with the same sales increase on products A
and C. Thus the key to product B’s profitability might be to increase the volume sold.
10.4.3 Gross margin
Gross margin is the difference between the sales revenue and the direct production or
purchasing costs incurred. Indirect costs or overheads are then deducted from the gross
margin to determine the net profit.
The gross margin percentage is also useful. It is the gross margin calculated as a
percentage of the sales revenue and it helps to highlight the relationship between sales
revenues and production/purchasing costs.
Look at the following example.
Example: a gross margin analysis
The following extract is taken from the monthly managerial report of the DD
Organisation.
Month 1 Month 2 Month 3 Month 4
£000 £000 £000 £000
Gross sales revenue 896 911 919 935
Direct cost of goods sold
699 713 722 737
Gross margin
197 198 197 198
Gross margin percentage
22.0% 21.7% 21.4% 21.2%
This gross margin analysis focuses managers’ attention on the relationship between the
sales value and the direct cost of sales, before indirect costs or overheads are taken into
account. This analysis reveals the following:
Although the gross sales revenue is steadily increasing, the gross margin is relatively
constant each month.
The gross margin percentage is steadily decreasing each month. If the gross margin
percentage could have been maintained at 22% the total gross margin earned would
have been higher.
Perhaps selling prices are being increased but the reduction in the gross margin
percentage might be the result of a failure to increase selling prices sufficiently in line
with increasing direct costs.
Alternatively the sales volume might be increasing but direct costs are not being
contained as the sales increase.
STUDY MATERIAL C1
286
PRESENTING MANAGEMENT INFORMATION
2006.1
10.5 Managerial reports in a service organisation
There is a very wide variety of service organisations, ranging from private sector
organisations such as hotels and courier services, to public sector organisations such as
hospitals and schools.
One aspect of services that can present difficulties for the information provider is
establishing a suitable cost unit.
10.5.1 Establishing a suitable cost unit
Many service organisations produce an intangible ‘output’, that is, their output has no
physical substance and it cannot be physically seen and touched. In order to maintain
effective cost control it is essential to establish a measurable cost unit for which we can
ascertain and monitor the costs.
In Chapter 1 we saw how composite cost units are often used to monitor and control
the costs in service operations. Any cost unit can be used as long as it can be objectively
measured and its cost can be determined and compared from one period to another and if
possible from one organisation to another.
Exercise 10.2
Suggest a composite cost unit that could be used in each of these service organisations:
(i) hotel; (ii) hospital; (iii) haulage contractor.
Solution
(i) Hotel: bed-night or room-night.
(ii) Hospital: in-patient day.
(iii) Haulage contractor: tonne-kilometre.
10.5.2 Establishing the cost per unit
Once a suitable cost unit has been selected, the cost for each unit can be determined using
an averaging method:
Average cost per unit of service ¼
Total costs incurred in period
Number of units of service supplied in the period
10.5.3 The instantaneous and perishable
nature of services
Many services are provided instantaneously rather than for inventory (stock); for example,
a restaurant meal is cooked as it is ordered by the customer. This brings with it particular
management problems of planning and control but it does mean that the incidence of
work in progress is very low, that is, it is rarely necessary to value part-finished units of
service at the end of an accounting period.
FUNDAMENTALS OF MANAGEMENT ACCOUNTING
287
PRESENTING MANAGEMENT INFORMATION
2006.1
Many services also ‘perish’ immediately; for example, if a cinema seat is vacant when a
film is showing it cannot be stored in inventory (stock) for a later sale. The opportunity to
gain revenue from that seat at that particular showing of the film has been lost forever.
Therefore, capacity utilisation becomes a very important issue for managers in many
service organisations.
Example: managerial reporting in a consultancy business
As you read through this example, notice that we are applying all of the principles of cost analysis
that you have already learned about in this Learning System. The only difference is that the
principles are being applied to determine the cost of intangible services, rather than of tangible
products.
Mr G and Mrs H have recently formed a consultancy business and they wish to establish the following
rates to charge clients:
an hourly rate for productive client work;
an hourly rate for time spent travelling to/from the clients’ premises;
a rate per mile for expenses incurred in travelling to/from the clients’ premises.
Pricing policy
Mr G and Mrs H have decided that their pricing policy will be based on the cost per hour plus a
5 per cent profit mark-up. Travelling time will be charged to clients at one-third of the normal hourly rate.
Travelling expenses will be charged to clients at cost.
Activity estimates
Mr G and Mrs H each expect to work for 8 hours per day, 5 days per week, 45 weeks per year. They
refer to this as ‘available time’.
Twenty-five per cent of the available time will be spent dealing with administrative matters relating to
the general running of the business.
In the first year, 22.5 per cent of the available time will be idle, that is, no work will be done in
this time.
The remainder of the available time is expected to be chargeable to clients.
Travelling time will amount to 25 per cent of the chargeable time, during which a total of
18,000 miles will be travelled.
Cost estimates
Mr G and Mrs H each wish to receive a salary of £20,000 in the first year of trading.
Other costs to be incurred in the first year of trading:
Requirement
Prepare a summary report for Mr G and Mrs H which states the client charge rates that they wish to
establish.
£
Electricity 1,200
Fuel for vehicles 1,800
Depreciation of vehicles 6,000
Insurance professional liability and office 600
Vehicle insurance and road tax 1,080
Office rent and rates 8,400
Telephone expenses 3,000
General office expenses 8,900
Servicing and repair of vehicles 1,200
STUDY MATERIAL C1
288
PRESENTING MANAGEMENT INFORMATION
2006.1
Solution
If you look back to Section 10.5.2 you will be reminded that we need to know two things in order to
establish the cost per unit of service:
the total costs incurred in the period;
the number of units of service supplied in the period.
We need to classify the costs provided to determine the total cost associated with travelling, and that
associated with providing consultancy services.
Now we need to determine the number of units of service by which each of these cost totals is to be
divided.
The calculation of the rate per mile for travelling expenses is relatively straightforward:
Rate per mile ¼
Total travelling expenses
Miles travelled
¼
£10,080
18,000
¼ £0:56 per mile
The calculation of the hourly rate for productive work and travelling time is a little more complicated.
The first step is to determine the number of units of service supplied, that is, the chargeable hours. We
need to look at the activity estimates provided in order to analyse the available time.
Travelling time will be charged at one-third of the normal hourly rate, therefore we need to calculate a
‘weighted’ figure for chargeable time.
Weighted chargeable time ¼ 1,417:5 þ
472:5
3
¼ 1,575 hours
Now we can combine the consultancy services costs and the weighted chargeable time to determine an
hourly rate for each type of work.
Consultancy Travelling
££
Salaries 40,000
Electricity 1,200
Fuel 1,800
Depreciation 6,000
Insurance 600
Vehicle insurance, etc. 1,080
Office rent and rates 8,400
Telephone expenses 3,000
General office expenses 8,900
Servicing vehicles, etc. 1,200
62,100 10,080
Hours
Total available hours for the first year ¼2 people 8 hours 5 days 45 weeks 3,600
Less: administration time 25.0%
idle time
22.5%
47.5% 3,600
(1,710)
Time chargeable to clients
1,890
Productive time spent with clients (75%) 1,417.5
Travelling time (25%) 472.5
FUNDAMENTALS OF MANAGEMENT ACCOUNTING
289
PRESENTING MANAGEMENT INFORMATION
2006.1
Cost per chargeable hour ¼
£62,100
1,575
¼ £39:43
Hourly rate for productive client work ¼ £39:43 þ 5% profit mark-up ¼ £41:40 per hour
Hourly rate for travelling time ¼
£41:40
3
¼ £13:80 per hour
We can perform a final check to ensure that these hourly rates will produce the required revenue.
Required revenue ¼consultancy costs £62,100 + 5% mark-up ¼£65,205
Summary report: client charge rates
To: Mr G and Mrs H
From: AN Other
Date: xx.xx.xx
Subject: Client charge rates
REPORT
In response to your request, in accordance with the cost and activity data provided, I detail below the required
charge rates to clients.
Hourly rate for productive client work £41.40
Hourly rate for travelling time £13.80
Rate per mile for travelling expenses £0.56
Exercise 10.3
The following data is available for the Central Hospital for the latest period.
Use this data to calculate the following cost control measures for the monthly
management report, to the nearest penny.
(a) Operating theatre cost per hour
(b) Admission costs per patient
(c) Patient care cost per night
Activity data
Number of patients 1,040
Number of patient nights 4,750
Number of operating theatres 5
Number of days theatres in use during month 26
Number of hours theatres used per day 15
Cost data £
Operating theatre costs in total 510,000
Updating patient records on admission 33,900
Bed scheduling costs 20,833
Nursing 1,077,000
Patient catering costs 244,200
Medical supplies 120,000
Patient laundry costs 100,000
Other patient care costs 60,900
Revenue at calculated hourly rates: £
Productive client work ¼1,417.5 hours £41.40 58,684.50
Travelling time ¼472.5 hours £13.80 6,520.50
Total revenue
65,205.00
STUDY MATERIAL C1
290
PRESENTING MANAGEMENT INFORMATION
2006.1
Solution
(a) Number of theatre hours ¼5 theatres 26 days 15 hours ¼1,950
Operating theatre cost per hour ¼£510,000/1,950 ¼£261.54
(b) Admission costs £
Updating patient records 33,900
Bed scheduling
20,833
Total admission costs
54,733
Admission costs per patient ¼£54,733/1,040 ¼£52.63
(c) Patient care costs £
Nursing 1,077,000
Patient catering costs 244,200
Medical supplies 120,000
Patient laundry costs 100,000
Other patient care costs
60,900
Total patient care costs
1,602,100
Patient care cost per patient night ¼£1,602,100/4,750 ¼£337.28
10.5.4 Managerial reporting in a charity: example
The TW Care Charity has just completed an overseas aid programme to assist homeless
orphans. Cost and revenue data concerning the programme are as follows.
£
Income from donations 157,750
Grants received from government and others 62,000
Fundraising costs 23,900
Direct staff costs, including travel and insurance 68,800
Medical supplies and temporary accommodation 78,120
Food, blankets and clothes 17,100
Transport costs 24,300
Other direct costs 9,800
Apportioned administrative support costs 13,200
Requirement
Prepare a statement to enable managers to monitor the total net cost of the aid
programme, highlighting any subtotals that you think may be useful to the managers.
FUNDAMENTALS OF MANAGEMENT ACCOUNTING
291
PRESENTING MANAGEMENT INFORMATION
2006.1
Solution
TW Care Charity
Report on overseas aid programme
££
Income from donations 157,750
Grants received from government and others
62,000
Gross revenue 219,750
Less fundraising costs
23,900
Net revenue 195,850
Direct staff costs, including travel and insurance 68,800
Medical supplies and temporary accommodation 78,120
Food, blankets, clothes 17,100
Transport costs 24,300
Other direct costs
9,800
Total direct cost
198,120
Net direct cost of programme (2,270)
Apportioned administrative support costs
13,200
Total net cost of programme
15,470
Points to note about the statement are as follows.
The fundraising costs are netted off against the gross revenue. Managers can use the
resulting net revenue to monitor the effectiveness of the fundraising activities undertaken.
Direct costs of the programme are highlighted separately. Managers are able to see
whether the net revenue from the fundraising efforts was sufficient to cover the directly
identifiable costs of undertaking the programme. In this case the direct costs exceeded
the net fundraising revenue by £2,270.
Administrative support costs are apportioned so that managers can see the final net
impact of this programme on the charity’s resources.
10.6 Summary
Having read this chapter the main points you should understand are as follows.
1. Costs can be classified according to their nature, purpose or responsibility.
2. Classification by nature is known as subjective classification.
3. Classification by purpose is known as objective classification.
4. A coding system provides a means of expressing the classification of expenditure in a
shortened symbolised form, and a means of accumulating data for analysis purposes.
5. Value added focuses on the value created by an organisation’s own efforts. It can be
calculated as: sales revenue less cost of materials and bought-in services, or as profit
plus interest plus all conversion costs.
6. The output of service organisations is often intangible and ‘instantly perishable’. With
many services it is impossible to produce the service to hold in inventory for sale at a
later date. Capacity utilisation is therefore important.
7. Composite cost units are often used to monitor and control costs in a service
organisation.
STUDY MATERIAL C1
292
PRESENTING MANAGEMENT INFORMATION
2006.1
Revision Questions
Question 1 Multiple choice
1.1 State which of the following are characteristics of managerial reports prepared in a
service organisation:
(i) a low incidence of work in progress at the end of a period
(ii) the use of composite cost units
(iii) the use of equivalent units
(A) (i) only
(B) (i) and (ii) only
(C) (ii) only
(D) (i), (ii) and (iii)
1.2 Which of the following is a correct calculation of value added:
(A) Sales revenue variable production costs
(B) Sales revenue direct labour costs
(C) Sales revenue all bought-in costs
(D) Sales revenue all variable costs.
Question 2 Short objective-test questions
2.1 Match the organisations with the most appropriate cost unit by writing (a), (b), (c),
(d) or (e) in the box provided.
Organisations
Hotel &
Transport service &
College &
Restaurant &
Accountancy service &
Cost units
(a) Enrolled student
(b) Meal served
(c) Chargeable hour
(d) Room night
(e) Tonne-kilometre
10
293 2006.1
2.2 Happy Stays hotel has 345 rooms. During the latest week the following data was
collected concerning unoccupied rooms.
Day Number of unoccupied rooms
Monday 77
Tuesday 43
Wednesday 26
Thursday 31
Friday 17
Saturday 12
Sunday 88
(a) The number of occupied room nights during the week was
(b) The overall percentage room occupancy rate during the week was %
(to the nearest whole number)
Question 3 Managerial reporting for a service organisation
Happy Returns Ltd operates a haulage business with three vehicles. The following
estimated operating costs and performance data are available:
Petrol £0.50 per km on average
Repairs £0.30 per km
Depreciation £1.00 per km, plus £50 per week per vehicle
Drivers’ wages £300.00 per week per vehicle
Supervision costs £550.00 per week
Loading costs £6.00 per tonne
During week 26 it is expected that all three vehicles will be used, 280 tonnes will be
loaded and a total of 3,950 km travelled (including return journeys when empty) as shown
in the following table:
Journey Tonnes carried Kilometres
(one way) (one way)
1 34 180
2 28 265
3 40 390
4 32 115
5 26 220
6 40 480
729 90
8 26 100
9
25 135
280 1,975
REVISION QUESTIONS C1
294
PRESENTING MANAGEMENT INFORMATION
2006.1
Requirements
(a) The total variable operating cost incurred in week 26 was £
(b) The total fixed operating cost incurred in week 26 was £
(c) The total cost for week 26, including administration cost, amounted to £13,265.
To the nearest penny, the average total cost per tonne-kilometre for week 26
was £
Question 4 Managerial reporting for a service organisation
The Ludford Hotel and Conference Centre is used for conference bookings and private
guest bookings. Conference bookings use some bedrooms each week, the balance being
available for private guests.
Data has been collected relating to private guest bookings (i.e., non-conference
bookings) which are summarised below for a 10-week period.
Week
Double rooms available for
private guest bookings Number of guests
Average stay
(nights)
1 55 198 2.1
2 60 170 2.6
3 72 462 1.4
4 80 381 3.2
544 835.6
6 62 164 3.4
7 80 348 2.6
8 54 205 1.7
9 80 442 1.8
10 24 84 3.2
Some of the costs for private guest bookings vary with the number of guests, regardless of
the length of their stay, while others vary with the number of rooms available in any week.
Variable cost per guest £17.50
Variable cost per week per room available £56.00
The general fixed cost for private guest bookings per week is £8,100.
Requirements
(a) To the nearest penny, the total costs for private guests’ bookings for the 10-week
period is £
(b) To the nearest whole number, the number of private guest-nights achieved in the
10-week period is
(c) The number of private guest-nights available for the 10-week period is
FUNDAMENTALS OF MANAGEMENT ACCOUNTING
295
PRESENTING MANAGEMENT INFORMATION
2006.1
Solutions to
Revision Questions
Solution 1
Do not rush the narrative multiple choice questions. Take the time to read each question
carefully because some of the distractors seem very similar when they are read in a hurry.
1.1 Answer: (B)
Many services are consumed as soon as they are made available to the customer.
They cannot be held in stock for sale at a later date. Therefore there is a low
incidence of work in progress at the end of a period.
Composite cost units are often used because they are more useful for control
purposes, for example in a haulage company a cost per tonne mile might be more
useful for planning and control purposes than a simple cost per tonne.
Equivalent units are more likely to be used in process costing.
1.2 Answer: (C)
Direct labour is not a bought-in cost therefore options A, B and D are incorrect.
Solution 2
2.1
Hotel (d)
Transport service (e)
College (a)
Restaurant (b)
Accountancy service (c)
2.2 (a) The number of occupied room nights during the week was 2,121.
(b) The overall percentage room occupancy rate during the week was 88 per cent.
Workings :
Number of room nights available ¼345 7 nights ¼2,415 room nights
Total number of unoccupied room nights ¼294
Number of occupied room nights ¼2,415 294 ¼2,121
Percentage occupancy ¼2,121/2,415 ¼88%
10
297 2006.1
Solution 3
This question provides an example of the use of a composite cost unit. The cost per
tonne-kilometre is the cost of transporting 1 tonne for 1 km.
(a) £8,790
(b) £1,600
(c) £0.20
Workings:
Tonne-kilometres
£
Variable operating costs
Loading: 280 £6 ¼ 1,680
£ per km
Running costs: Petrol 0.50
Repairs 0.30
Depreciation
1.00
1.80 3,950 7,110
8,790
Fixed operating costs £
Depreciation (3 £50) 150
Supervision 550
Drivers’ wages (3 £300)
900
1,600
Total operating cost
10,390
Average total cost per tonne-kilometre ¼
£13,265
66,325
¼£0.20
Journey Tonnes carried km tonne-km
1 34 180 6,120
2 28 265 7,420
3 40 390 15,600
4 32 115 3,680
5 26 220 5,720
6 40 480 19,200
7 29 90 2,610
8 26 100 2,600
9
25 135 3,375
280 1,975 66,325
SOLUTIONS TO REVISION QUESTIONS C1
298
PRESENTING MANAGEMENT INFORMATION
2006.1
Solution 4
You will be using a composite cost unit in this question as well: a guest night. The cost
per guest night is the cost incurred by the hotel for one guest to stay for one night. In
this example, the number of guest nights is calculated as:
No. of guest nights ¼ no. of guests average no. of nights stayed
You will need to prepare some preliminary workings in part (a). The totals to be
calculated for the 10-week period are:
(i) the number of rooms available (you need this in order to calculate the costs incurred);
(ii) the number of guests (this is also needed for the cost calculation);
(a) £159,613.50
(b) 6,064
(c) 8,554
Total costs for private guests’ bookings ¼(611 £56) + (2,537 £17.50)
+ (10 £8,100) ¼£159,613.50
Guest nights available ¼611 rooms 7 nights 2 guests ¼8,554.
Workings:
Week Rooms Guests Average stay Guest nights
1 55 198 2.1 415.8
2 60 170 2.6 442.0
3 72 462 1.4 646.8
4 80 381 3.2 1,219.2
5 44 83 5.6 464.8
6 62 164 3.4 557.6
7 80 348 2.6 904.8
8 54 205 1.7 348.5
9 80 442 1.8 795.6
10
24 84 3.2 268.8
611 2,537 6,063.9
FUNDAMENTALS OF MANAGEMENT ACCOUNTING
299
PRESENTING MANAGEMENT INFORMATION
2006.1
11
Financial Planning
and Control
Financial Planning
and Control
11.1 Introduction
In this chapter you will learn about budgets: what they are for, how they are prepared, and
their use in planning and controlling the activities of an organisation.
11.2 The purposes of budgeting
Budgets have two main roles:
(a) they act as authorities to spend, that is, they give authority to budget managers to incur
expenditure in their part of the organisation;
(b) they act as comparators for current performance, by providing a yardstick against
which current activities can be monitored.
These two roles are combined in a system of budgetary planning and control.
11
LEARNING OUTCOMES
After completing this chapter, you should be able to:
"
explain why organisations set out financial plans in the form of budgets, typically
for a financial year;
"
prepare functional budgets for material usage and purchase, labour and
overheads, including budgets for capital expenditure and depreciation;
"
prepare a master budget: income statement balance sheet and cash flow
statement, based on the functional budgets;
"
interpret budget statements and advise managers on financing projected cash
shortfalls and/or investing projected cash surpluses;
"
prepare a flexed budget based on the actual levels of sales and production and
calculate appropriate variances;
"
compare and contrast fixed and flexed budgets;
"
explain the use of budgets in designing reward strategies for managers.
303 2006.1
11.2.1 Budgetary planning and control
Planning the activities of an organisation ensures that the organisation sets out in the right
direction. Individuals within the organisation will have definite targets which they will aim
to achieve. Without a formalised plan the organisation will lack direction and managers will
not be aware of their own targets and responsibilities. Neither will they appreciate how
their activities relate to those of other managers within the organisation.
A formalised plan will help to ensure a coordinated approach and the planning process
itself will force managers to continually think ahead, planning and reviewing their activities
in advance.
However, the budgetary process should not stop with the plan. The organisation has
started out in the right direction but to ensure that it continues on course it is the
management’s responsibility to exercise control.
Control is best achieved by comparison of the actual results with the original plan.
Appropriate action can then be taken to correct any deviations from the plan.
The comparison of actual results with a budgetary plan, and the taking of action to
correct deviations, is known as feedback control.
The two activities of planning and control must go hand in hand. Carrying out the
budgetary planning exercise without using the plan for control purposes is performing
only part of the task.
11.2.2 What is a budget?
A budget could be defined as ‘a quantified plan of action relating to a given period
of time’.
For a budget to be useful it must be quantified. For example, it would not be particularly
useful for the purposes of planning and control if a budget was set as follows:
‘We plan to spend as little as possible in running the printing department this year’; or
‘We plan to produce as many units as we can possibly sell this quarter’.
These are merely vague indicators of intended direction; they are not quantified plans.
They will not provide much assistance in management’s task of planning and controlling
the organisation.
These ‘budgets’ could perhaps be modified as follows:
‘Budgeted revenue expenditure for the printing department this year is £60,000’; and
‘Budgeted production for the quarter is 4,700 units’.
The quantification of the budgets has provided:
(a) a definite target for planning purposes; and
(b) a yardstick for control purposes.
11.2.3 The budget period
You may have noticed that in each of these ‘budgets’ the time period was different. The
first budget was prepared for a year and the second budget was for a quarter. The time
period for which a budget is prepared and used is called the budget period. It can be any
length to suit management purposes but it is usually one year.
STUDY MATERIAL C1
304
FINANCIAL PLANNING AND CONTROL
2006.1
The length of time chosen for the budget period will depend on many factors, including
the nature of the organisation and the type of expenditure being considered. Each budget
period can be subdivided into control periods, also of varying lengths, depending on the
level of control which management wishes to exercise. The usual length of a control
period is one month.
11.2.4 Strategic planning, budgetary planning
and operational planning
It will be useful at this stage to distinguish in broad terms between three different types of
planning:
strategic planning;
budgetary planning;
operational planning.
These three forms of planning are interrelated. The main distinction between them
relates to their timespan which may be short term, medium term or long term.
The short term for one organisation may be the medium or long term for another,
depending on the type of activity in which it is involved.
Strategic planning
Strategic planning is concerned with preparing long-term action plans to attain the
organisation’s objectives.
Strategic planning is also known as corporate planning or long-range planning.
Budgetary planning
Budgetary planning is concerned with preparing the short- to medium-term plans of
the organisation. It will be carried out within the framework of the strategic plan. An
organisation’s annual budget could be seen as an interim step towards achieving the long-
term or strategic plan.
Operational planning
Operational planning refers to the short-term or day-to-day planning process. It is
concerned with planning the utilisation of resources and will be carried out within the
framework set by the budgetary plan. Each stage in the operational planning process can
be seen as an interim step towards achieving the budget for the period.
Operational planning is also known as tactical planning.
Remember that the full benefit of any planning exercise is not realised unless the plan is
also used for control purposes. Each of these types of planning should be accompanied by
the appropriate control exercise covering the same time span.
11.3 The preparation of budgets
The process of preparing and using budgets will differ from organisation to organisation.
However there are a number of key requirements in the design of a budgetary planning
and control process.
FUNDAMENTALS OF MANAGEMENT ACCOUNTING
305
FINANCIAL PLANNING AND CONTROL
2006.1
11.3.1 Coordination: the budget committee
The need for coordination in the planning process is paramount. The interrelationship
between the functional budgets (e.g. sales, production, purchasing) means that one budget
cannot be completed without reference to several others.
For example, the purchasing budget cannot be prepared without reference to the
production budget, and it may be necessary to prepare the sales budget before the
production budget can be prepared. The best way to achieve this coordination is to set up
a budget committee. The budget committee should comprise representatives from all
functions in the organisation. There should be a representative from sales, a representative
from marketing, a representative from personnel, and so on.
The budget committee should meet regularly to review the progress of the budgetary
planning process and to resolve problems that have arisen. These meetings will effectively
bring together the whole organisation in one room, to ensure a coordinated approach to
budget preparation.
11.3.2 Participative budgeting
The CIMA Terminology defines participative budgeting as a ‘budgeting process where all
budget holders have the opportunity to participate in setting their own budgets’. This
may also be referred to as ‘bottom-up budgeting’. It contrasts with imposed or top-
down budgets where the ultimate budget holder does not have the opportunity to
participate in the budgeting process. The advantages of participative budgeting are as
follows:
Improved quality of forecasts to use as the basis for the budget. Managers who are doing a
job on a day-to-day basis are likely to have a better idea of what is achievable, what is
likely to happen in the forthcoming period, local trading conditions, etc.
Improved motivation. Budget holders are more likely to want to work to achieve a budget
that they have been involved in setting themselves, rather than one that has been
imposed on them by more senior managers. They will own the budget and accept
responsibility for the achievement of the targets contained therein.
Detail on the behavioural aspects of budgeting is outside the scope of the
Fundamentals of Management Accounting syllabus.
The main disadvantage of participative budgeting is that it tends to result in a more
extended and complex budgetary process. However, the advantages are generally accepted
to outweigh this.
11.3.3 Information: the budget manual
Effective budgetary planning relies on the provision of adequate information to the
individuals involved in the planning process.
Many of these information needs are contained in the budget manual.
STUDY MATERIAL C1
306
FINANCIAL PLANNING AND CONTROL
2006.1
A budget manual is a collection of documents which contains key information for those
involved in the planning process. Typical contents could include the following:
(a) An introductory explanation of the budgetary planning and control process including a
statement of the budgetary objective and desired results.
Participants should be made aware of the advantages to them and to the
organisation of an efficient planning and control process. This introduction should
give participants an understanding of the workings of the planning process, and of the
sort of information that they can expect to receive as part of the control process.
(b) A form of organisation chart to show who is responsible for the preparation of each
functional budget and the way in which the budgets are interrelated.
(c) A timetable for the preparation of each budget. This will prevent the formation of a
‘bottleneck’, with the late preparation of one budget holding up the preparation of all others.
(d) Copies of all forms to be completed by those responsible for preparing budgets, with
explanations concerning their completion.
(e) A list of the organisation’s account codes, with full explanations of how to use them.
(f ) Information concerning key assumptions to be made by managers in their budgets, for
example, the rate of inflation, key exchange rates, etc.
(g) The name and location of the person to be contacted concerning any problems
encountered in preparing budgetary plans. This will usually be the coordinator of the
budget committee (the budget officer) and will probably be a senior accountant.
11.3.4 Early identification of the principal budget factor
The principal budget (key budget) factor is the factor which limits the activities of the
organisation. The early identification of this factor is important in the budgetary planning
process because it indicates which budget should be prepared first.
The principal budget factor was referred to in Chapter 4 as the limiting factor.
For example, if sales volume is the principal budget factor, then the sales budget must
be prepared first, based on the available sales forecasts. All other budgets should then be
linked to this.
Alternatively machine capacity may be limited for the forthcoming period and therefore
machine capacity is the principal budget factor. In this case the production budget must be
prepared first and all other budgets must be linked to this.
Failure to identify the principal budget factor at an early stage could lead to delays at a
later stage when managers realise that the targets they have been working with are not
feasible.
11.3.5 The interrelationship of budgets
The critical importance of the principal budget factor stems from the fact that all budgets
are interrelated. For example, if sales is the principal budget factor this is the first budget to
be prepared. This will then provide the basis for the preparation of several other budgets
including the selling expenses budget and the production budget.
However, the production budget cannot be prepared directly from the sales budget
without a consideration of stockholding policy. For example, management may plan to
increase finished goods inventory (stock) in anticipation of a sales drive. Production
quantities would then have to be higher than the budgeted sales level. Similarly, if a
FUNDAMENTALS OF MANAGEMENT ACCOUNTING
307
FINANCIAL PLANNING AND CONTROL
2006.1
decision is taken to reduce the level of material inventories held, it would not be necessary
to purchase all of the materials required for production.
11.3.6 Using computers in budget preparation
A vast amount of data is involved in the budgetary planning process and managing this
volume of data in a manual system is an onerous and cumbersome task.
A computerised budgetary planning system will have the following advantages over a
manual system:
computers can easily handle the volume of data involved;
a computerised system can process the data more rapidly than a manual system;
a computerised system can process the data more accurately than a manual system;
computers can quickly and accurately access and manipulate the data in the system.
Organisations may use specially designed budgeting software. Alternatively, a well-
designed spreadsheet model can take account of all of the budget interrelationships
described above.
The model will contain variables for all of the factors about which decisions must be
made in the planning process, for example, sales volume, unit costs, credit periods and
stock volumes.
If managers wish to assess the effect on the budget results of a change in one of the
decision variables, this can be accommodated easily by amending the relevant variable in
the spreadsheet model. The effect of the change on all of the budgets will be calculated
instantly so that managers can make better informed planning decisions.
This process of reviewing the effect of changes in the decision variables is called
‘what-if ?’ analysis. For example, managers can rapidly obtain the answer to the
question, ‘What if sales volumes are 10 per cent lower than expected?’.
Budgetary planning is an iterative process. Once the first set of budgets has been
prepared, those budgets will be considered by senior managers. The criteria used to assess
the suitability of budgets may include adherence to the organisation’s long-term objectives,
profitability and liquidity. Computerised spreadsheet models then provide managers with
the ability to amend the budgets rapidly, and adjust decision variables until they feel that
they have achieved the optimum plan for the organisation for the forthcoming period.
11.3.7 The master budget
The master budget is a summary of all the functional budgets. It usually comprises
the budgeted income statement (profit and loss account), budgeted balance sheet
and budgeted cash flow statement.
It is this master budget which is submitted to senior managers for approval because they
should not be burdened with an excessive amount of detail. The master budget is designed
to give the summarised information that they need to determine whether the budget is an
acceptable plan for the forthcoming period.
STUDY MATERIAL C1
308
FINANCIAL PLANNING AND CONTROL
2006.1
11.4 Preparation of functional budgets
The best way to see how budgets are prepared is to work through an example.
Example: Preparing a functional budget
A company manufactures two products, Aye and Bee. Standard cost data for the products for next year
are as follows:
Budgeted inventories (stocks) for next year are as follows:
Budgeted sales for next year: product Aye 2,400 units; product Bee 3,200 units.
You are required to prepare the following budgets for next year:
(a) production budget, in units;
(b) material usage budget, in kilos;
(c) material purchases budget, in kilos and £;
(d) direct labour budget, in hours and £.
Solution
(a) Production budget for next year
Product Aye
units
Product Bee
units
1 January 400 800
31 December 500 1,100
Material X Material Y Material Z
kg kg kg
1 January 30,000 25,000 12,000
31 December 35,000 27,000 12,500
Product Aye
units
Product Bee
units
Sales units required 2,400 3,200
Closing stock at end of year
500 1,100
2,900 4,300
Less opening stock
400 800
Production units required
2,500 3,500
Product Aye
per unit
Product Bee
per unit
Direct materials:
X at £2 per kg 24 kg 30 kg
Y at £5 per kg 10 kg 8 kg
Z at £6 per kg 5 kg 10 kg
Direct wages:
Unskilled at £3 per hour 10 hours 5 hours
Skilled at £5 per hour 6 hours 5 hours
FUNDAMENTALS OF MANAGEMENT ACCOUNTING
309
FINANCIAL PLANNING AND CONTROL
2006.1
(b) Material usage budget for next year
(c) Material purchases budget for next year
(d) Direct labour budget for next year
Material X Material Y Material Z
kg kg kg
Requirements for production:
Product Aye
1
60,000 25,000 12,500
Product Bee
105,000 28,000 35,000
Total material usage
165,000 53,000 47,500
Note 1: Material X for product Aye:
2,500 units produced 24 kg ¼60,000 kg
The other material requirements are calculated in the same way.
Material X Material Y Material Z
kg kg kg Total
Material required for production 165,000 53,000 47,500
Closing stock at end of year
35,000 27,000 12,500
200,000 80,000 60,000
Less opening stock
30,000 25,000 12,000
Material purchases required
170,000 55,000 48,000
Standard price per kg £2 £5 £6
Material purchases value
£340,000 £275,000 £288,000 £903,000
Unskilled labour
hours
Skilled labour
hours Total
Requirements for production:
Product Aye
1
25,000 15,000
Product Bee
17,500 17,500
Total hours required
42,500 32,500
Standard rate per hour £3 £5
Direct labour cost
£127,500 £162,500 £290,000
Note 1: Unskilled labour for product Aye:
2,500 units produced 10 hours ¼25,000 hours
The other labour requirements are calculated in the same way.
STUDY MATERIAL C1
310
FINANCIAL PLANNING AND CONTROL
2006.1
11.4.1 Budget interrelationships
This example has demonstrated how the data from one functional budget becomes an
input in the preparation of another budget. The last budget in the sequence, the direct
labour budget, would now be used as an input to other budgets. The material purchases
budget will also provide input data for other budgets.
For example, the material purchases budget would probably be used in preparing the
payables (creditors) budget, taking account of the company’s intended policy on the
payment of suppliers. The creditors budget would indicate the payments to be made to
creditors, which would then become an input for the cash budget, and so on.
The cash budget is the subject of the next section of this chapter.
11.5 The cash budget
The cash budget is one of the most vital planning documents in an organisation. It will
show the cash effect of all of the decisions taken in the planning process.
Management decisions will have been taken concerning such factors as stockholding
policy, credit policy, selling price policy and so on. All of these plans will be designed
to meet the objectives of the organisation. However, if there are insufficient cash resources
to finance the plans they may need to be modified or perhaps action might be taken to
alleviate the cash restraint.
A cash budget can give forewarning of potential problems that could arise so that
managers can be prepared for the situation or take action to avoid it.
The use of forecasts to modify actions so that potential threats are avoided or
opportunities exploited is known as feedforward control.
There are four possible cash positions that could arise:
Notice that the type of action taken by management will depend not only on whether a
deficit or a surplus is expected, but also on how long the situation is expected to last.
For example, management would not wish to use surplus cash to purchase non-current
(fixed) assets, if the surplus was only short term and the cash would soon be required again
for day-to-day operations.
Cash budgets therefore forewarn managers of whether there will be cash surpluses or
cash deficits, and how long the surpluses or deficits are expected to last.
Cash position Possible management action
Short-term deficit Arrange a bank overdraft, reduce receivables (debtors) and
inventories (stocks), increase payables (creditors)
Long-term deficit Raise long-term finance, such as long-term loan capital or
share capital
Short-term surplus Invest short term, increase debtors and stocks to
boost sales, pay creditors early to obtain cash discount
Long-term surplus Expand or diversify operations, replace or update
non-current (fixed) assets
FUNDAMENTALS OF MANAGEMENT ACCOUNTING
311
FINANCIAL PLANNING AND CONTROL
2006.1
11.5.1 Preparing cash budgets
Before we work through a full example of the preparation of a cash budget, it will be
useful to discuss a few basic principles.
(a) The format for cash budgets
There is no definitive format which should be used for a cash budget. However, whichever
format you decide to use it should include the following:
(i) A clear distinction between the cash receipts and cash payments for each control period. Your budget
should not consist of a jumble of cash flows. It should be logically arranged with a
subtotal for receipts and a subtotal for payments.
(ii) A figure for the net cash flow for each period. It could be argued that this is not an essential
feature of a cash budget. However, you will find it easier to prepare and use a cash
budget if you include the net cash flow. Also, managers find in practice that a figure
for the net cash flow helps to draw attention to the cash flow implications of their
actions during the period.
(iii) The closing cash balance for each control period. The closing balance for each period will be
the opening balance for the following period.
(b) Depreciation is not included in cash budgets
Remember that depreciation is not a cash flow. It may be included in your data for overheads
and must therefore be excluded before the overheads are inserted into the cash budget.
(c) Allowance must be made for bad and doubtful debts
Bad debts will never be received in cash and doubtful debts may not be received. When
you are forecasting the cash receipts from customers you must remember to adjust for
these items, if necessary.
Example: cash budget
Watson Ltd is preparing its budgets for the next quarter. The following information has been drawn from
the budgets prepared in the planning exercise so far:
Other information
Watson sells 10 per cent of its goods for cash. The remainder of customers receive one month’s credit.
Payments to creditors are made in the month following purchase.
Wages are paid as they are incurred.
Watson takes one month’s credit on all overheads.
Sales value June (estimate) £12,500
July (budget) £13,600
August £17,000
September £16,800
Direct wages £1,300 per month
Direct material purchases June (estimate) £3,450
July (budget) £3,780
August £2,890
September £3,150
STUDY MATERIAL C1
312
FINANCIAL PLANNING AND CONTROL
2006.1
Production overheads are £3,200 per month.
Selling, distribution and administration overheads amount to £1,890 per month.
Included in the amounts for overhead given above are depreciation charges of £300 and £190,
respectively.
Watson expects to purchase a delivery vehicle in August for a cash payment of £9,870.
The cash balance at the end of June is forecast to be £1,235.
You are required to prepare a cash budget for each of the months July to September.
Solution
Watson Ltd cash budget for July to September
11.5.2 Interpretation of the cash budget
This cash budget forewarns the management of Watson Limited that their plans will lead
to a cash deficit of £1,115 at the end of August. They can also see that it will be a short-
term deficit and can take appropriate action.
They may decide to delay the purchase of the delivery vehicle or perhaps negotiate a
period of credit before the payment will be due. Alternatively overdraft facilities may be
arranged for the appropriate period.
The important point to appreciate is that management should take appropriate action
for a forecast short-term deficit. For example it would not be appropriate to arrange a five
year loan to manage a cash deficit that is expended to last for only one month.
If it is decided that overdraft facilities are to be arranged, it is important that due
account is taken of the timing of the receipts and payments within each month.
July August September
££ £
Sales receipts:
10% in cash 1,360 1,700 1,680
90% in one month
11,250 12,240 15,300
Total receipts
12,610 13,940 16,980
Payments
Material purchases (one month credit) 3,450 3,780 2,890
Direct wages 1,300 1,300 1,300
Production overheads
1
2,900 2,900 2,900
Selling, distribution and administration
overhead
1
1,700 1,700 1,700
Delivery vehicle
9,870
Total payments
9,350 19,550 8,790
Net cash inflow/(outflow) 3,260 (5,610) 8,190
Opening cash balance
1,235 4,495 (1,115)
Closing cash balance at the end of
the month
4,495 (1,115) 7,075
Note 1: Depreciation has been excluded from the overhead payment figures because it is
not a cash item.
FUNDAMENTALS OF MANAGEMENT ACCOUNTING
313
FINANCIAL PLANNING AND CONTROL
2006.1
For example, all of the payments in August may be made at the beginning of the month
but receipts may not be expected until nearer the end of the month. The cash deficit could
then be considerably greater than it appears from looking only at the month-end balance.
If the worst possible situation arose, the overdrawn balance during August could become
as large as £4,495 £19,550 ¼£15,055. If management had used the month-end balances as
a guide to the overdraft requirement during the period then they would not have arranged a
large enough overdraft facility with the bank. It is important therefore that they look in
detail at the information revealed by the cash budget, and not simply at the closing cash
balances.
Exercise 11.1
Practise what you have just learned about cash budgets by attempting this exercise before
you look at the solution.
The following information relates to XY Ltd:
(a) It is expected that the cash balance on 31 May will be £22,000.
(b) The wages may be assumed to be paid within the month they are incurred.
(c) It is company policy to pay suppliers for materials three months after receipt.
(d) Credit customers are expected to pay two months after delivery.
(e) Included in the overhead figure is £2,000 per month which represents depreciation on
two cars and one delivery van.
(f ) There is a one-month delay in paying the overhead expenses.
(g) Ten per cent of the monthly sales are for cash and 90 per cent are sold on credit.
(h) A commission of 5 per cent is paid to agents on all the sales on credit but this is not
paid until the month following the sales to which it relates; this expense is not included
in the overhead figures shown.
(i) It is intended to repay a loan of £25,000 on 30 June.
(j) Delivery is expected in July of a new machine costing £45,000 of which £15,000 will
be paid on delivery and £15,000 in each of the following two months.
(k) Assume that overdraft facilities are available if required.
You are required to prepare a cash budget for each of June, July and August.
Wages incurred Materials purchases Overhead Sales
Month £000 £000 £000 £000
February 6 20 10 30
March 8 30 12 40
April 10 25 16 60
May 9 35 14 50
June 12 30 18 70
July 10 25 16 60
August 9 25 14 50
September 9 30 14 50
STUDY MATERIAL C1
314
FINANCIAL PLANNING AND CONTROL
2006.1
Solution
Cash budget for June, July and August
Explanatory notes
1. The cash received from credit sales is 90 per cent of the sales made two months before,
that is, for June, 90 per cent of April sales ¼90 per cent £60,000.
2. Cash sales are 10 per cent of the sales made in the month.
3. March purchases are paid for three months later in June, and so on.
4. May overheads, less depreciation ¼£14,000 £2,000 ¼£12,000. These are paid in
cash in June, and so on.
5.
These amounts for commission are paid one month later, that is, in June, July and
August.
11.6 A complete exercise
Now that you have seen how to prepare functional budgets and cash budgets, have a go at
the following exercise. It requires you to work from basic data to produce a number of
functional budgets, as well as the master budget, that is, budgeted cash flow, income
statement (profit and loss account) and balance sheet.
June
July August
£££
Receipts
Receipts from credit sales
1
54,000 45,000 63,000
Cash sales
2
7,000 6,000 5,000
61,000 51,000 68,000
Payments
Wages 12,000 10,000 9,000
Materials
3
30,000 25,000 35,000
Overhead
4
12,000 16,000 14,000
Commission
5
2,250 3,150 2,700
Loan repayment 25,000
Payments for new machine
15,000 15,000
81,250 69,150 75,700
Net cash inflow/(outflow) (20,250) (18,150) (7,700)
Opening balance
22,000 1,750 (16,400)
Closing balance
1,750 (16,400) (24,100)
May June July
Credit sales (90%) £45,000 £63,000 £54,000
5% commission £2,250 £3,150 £2,700
FUNDAMENTALS OF MANAGEMENT ACCOUNTING
315
FINANCIAL PLANNING AND CONTROL
2006.1
Exercise 11.2
C Ltd make two products, Alpha and Beta. The following data is relevant for year 3:
Direct labour is paid £5 per hour.
Production overhead cost is estimated to be £200,000, which includes £25,000 for
depreciation of property and equipment. Production overhead cost is absorbed into
product costs using a direct labour hour absorption rate.
Each unit of finished product requires:
The sales director has forecast that sales of Alpha and Beta will be 5,000 and 1,000
units, respectively, during year 3. The selling prices will be:
She estimates that the inventory (stock) at 1 January, year 3, will be 100 units of Alpha and 200
units of Beta. At the end of year 3 she requires the stock level to be 150 units of each product.
The production director estimates that the raw material stocks on 1 January, year 3, will
be 3,000 units of material M and 4,000 units of material N. At the end of year 3 the stocks
of these raw materials are to be:
The finance director advises that the rate of tax to be paid on profits during year 3 is
likely to be 30 per cent. Selling and administration overhead is budgeted to be £75,000 in
year 3, which includes £5,000 for depreciation of equipment.
A quarterly cash-flow forecast has already been completed and is set out below:
Material prices: Material M £2 per unit
Material N £3 per unit
Alpha Beta
Material M 12 units 12 units
Material N 6 units 8 units
Direct labour 7 hours 10 hours
Alpha £130 per unit
Beta £115 per unit
M: 4,000 units
N: 2,000 units
1234
Quarter, year 3 ££££
Receipts 140,000 160,000 170,000 240,000
Payments:
Materials 22,000 37,000 40,000 60,000
Direct wages 50,000 55,250 60,500 58,500
Overhead 45,000 50,000 70,000 65,000
Taxation 5,000
Machinery purchase 20,000
STUDY MATERIAL C1
316
FINANCIAL PLANNING AND CONTROL
2006.1
The company’s balance sheet at 1 January, year 3, is expected to be as follows:
You are required to prepare the company’s budgets for year 3 including a budgeted income
statement and loss account for the year and a balance sheet at 31 December, year 3.
Solution
Note the order in which the budgets are prepared. The sales budget determines production
requirements, which in turn determines materials usage, which in turn determines
materials purchases and then payments to suppliers. Since the sales budget is prepared
first, sales are termed the principal (key) budget factor.
Sales budget for the year ended 31 December, year 3
£££
Cost Depreciation Net
Fixed assets
Land 50,000 50,000
Buildings and equipment
400,000 75,000 325,000
450,000 75,000 375,000
Current assets
Stocks
raw materials 20,000
finished goods
15,000
35,000
Debtors 25,000
Cash at bank
10,000
70,000
Current liabilities
Creditors 9,000
Taxation
5,000
14,000
56,000
431,000
Financed by
Ordinary shares 350,000
Retained profits
81,000
431,000
Alpha Beta Total
Sales volume 5,000 1,000
Selling price £130 £115
Sales revenue £650,000 £115,000 £765,000
FUNDAMENTALS OF MANAGEMENT ACCOUNTING
317
FINANCIAL PLANNING AND CONTROL
2006.1
Production budget for the year ended 31 December, year 3
Raw materials usage budget for the year ended 31 December, year 3
Raw materials purchases budget for the year ended 31 December, year 3
Direct labour budget for the year ended 31 December, year 3
Alpha Beta
units units
Required by sales 5,000 1,000
Required closing stock
150 150
5,150 1,150
Less expected opening stock
100 200
Production required
5,050 950
Material M Material N
units units
Required by production of Alpha
1
60,600 30,300
Required by production of Beta
11,400 7,600
Total raw material usage
72,000 37,900
Note 1: The material usage for Alpha is determined as follows:
Units
Material M: 5,050 12 60,600
Material N: 5,050 6 30,300
The material requirements for Beta are calculated in the same way.
Material M Material N
units units Total
Raw materials required by production 72,000 37,900
Required closing stock
4,000 2,000
76,000 39,900
Less expected opening stock
3,000 4,000
Quantity to be purchased
73,000 35,900
Price per unit £2 £3
Value of purchases
£146,000 £107,700 £253,700
Labour
hours
Rate
per hour
Labour
cost
££
Product Alpha 5,050 units 35,350 5 176,750
Product Beta 950 units
9,500 5 47,500
44,850 224,250
STUDY MATERIAL C1
318
FINANCIAL PLANNING AND CONTROL
2006.1
Production cost budget: preliminary workings
Production overhead absorption rate ¼
£200,000
44,850
¼ £4:459 per labour hour
Overhead absorbed by Alpha ¼ 35,350 hours £4:459 ¼ £157,626
Overhead absorbed by Beta ¼ 9,500 hours £4:459 ¼ £42,361
Production cost budget for the year ended 31 December, year 3
Cash budget for the year ended 31 December, year 3
Quarter 1 2 3 4
££££
Receipts
140,000 160,000 170,000 240,000
Payments:
Materials 22,000 37,000 40,000 60,000
Direct wages 50,000 55,250 60,500 58,500
Overhead 45,000 50,000 70,000 65,000
Taxation 5,000
Machinery purchase
20,000
Total payments 122,000 142,250 190,500 183,500
Net cash inflow/(outflow) 18,000 17,750 (20,500) 56,500
Balance b/fwd
3
10,000 28,000 45,750 25,250
Balance c/fwd
28,000 45,750 25,250 81,750
Note 3: The balance b/fwd in quarter 1 is the cash at bank on the forecast balance
sheet for 1 January, year 3.
Alpha Beta
££
Direct materials
–M
2
121,200 22,800
N 90,900 22,800
Direct wages 176,750 47,500
Production overhead
157,626 42,361
546,476 135,461
Cost per unit (used for closing stock valuation)
£108.21 £142.59
Note 2 : The direct material cost for Alpha is determined as follows:
Material Usage (units) £
M 60,600 @ £2 121,200
N 30,300 @ £3 90,900
The material cost for Beta is calculated in the same way.
FUNDAMENTALS OF MANAGEMENT ACCOUNTING
319
FINANCIAL PLANNING AND CONTROL
2006.1
Budgeted income statement for the year ended 31 December, year 3
££
Sales 765,000
Opening stock of raw materials
4
20,000
Purchases of raw materials
253,700
273,700
Closing stock of raw materials
5
14,000
259,700
Direct wages 224,250
Production overhead
200,000
Production cost of goods completed 683,950
Opening stock of finished goods
4
15,000
698,950
Closing stock of finished goods
5
37,620
Production cost of goods sold
661,330
Gross profit 103,670
Selling and administration overhead
75,000
Net profit before taxation 28,670
Taxation
8,601
20,069
Retained profit b/f
81,000
Retained profit c/f
101,069
Note 4: The opening stock figures for raw materials and finished
goods are taken from the opening balance sheet.
Note 5: The closing stocks are calculated as follows:
£
Raw materials:
M: 4,000 £2 8,000
N: 2,000 £3
6,000
14,000
Finished goods:
Alpha: 150 £108.21 16,231.50
Beta: 150 £142.59
21,388.50
37,620.00
STUDY MATERIAL C1
320
FINANCIAL PLANNING AND CONTROL
2006.1
Budgeted balance sheet at 31 December, year 3
Cost Depreciation Net
£££
Fixed assets
Land 50,000 50,000
Buildings and equipment
6
420,000 105,000 315,000
470,000 105,000 365,000
Current assets
Stocks
raw materials 14,000
finished goods
37,620
51,620
Debtors
7
80,000
Cash at bank
81,750
213,370
Current liabilities
Creditors
8
118,700
Taxation
8,601
127,301
86,069
451,069
Financed by
Ordinary shares 350,000
Retained profits
101,069
451,069
£000
Note 6: Property and equipment
Opening cost balance 400
Purchases during year
20
420
Opening depreciation balance 75
Production depreciation 25
Selling depreciation
5
105
Note 7: Debtors
Opening balance 25
Sales 765
Receipts (cash budget)
(710)
80
FUNDAMENTALS OF MANAGEMENT ACCOUNTING
321
FINANCIAL PLANNING AND CONTROL
2006.1
££
Note 8: Closing creditor balance
Opening balance of creditors 9,000
Material purchases from budget 253,700
Overhead, excluding depreciation:*
Production 175,000
Selling and administration
70,000
507,700
Less payments (from cash budget):
Materials 159,000
Overhead
230,000
389,000
Closing balance of creditors
118,700
* The depreciation must be excluded from the overhead because it is not a cash item, i.e. it
is not a payment which must be made to suppliers.
11.7 Rolling budgets
The CIMA Terminology defines a rolling budget as a ‘budget continuously updated
by adding a further accounting period (month or quarter) when the earliest
accounting period has expired. Its use is particularly beneficial where future costs and/or
activities cannot be forecast accurately’.
For example, a budget may initially be prepared for January to December, year 1. At the
end of the first quarter, that is, at the end of March, year 1, the first quarter’s budget is
deleted. A further quarter is then added to the end of the remaining budget, for January to
March, year 2. The remaining portion of the original budget is updated in the light of
current conditions. This means that managers have a full year’s budget always available and
the rolling process forces them continually to plan ahead.
A system of rolling budgets is also known as continuous budgeting. Rolling budgets can be
particularly useful when future events cannot be forecast reliably.
It is not necessary for all of the budgets in a system to be prepared on a rolling basis.
For example, many organisations will use a rolling system for the cash budget only.
In practice, most organisations carry out some form of updating process on all their
budgets, so that the budgets represent a realistic target for planning and control purposes.
The formalised budgetary planning process will still be performed on a regular basis to
ensure a coordinated approach to budgetary planning.
11.8 Budgets for non-operating functions
So far in this chapter we have been concentrating mainly on budgets for operating
functions. You have seen that once the principal budget factor has been identified and
budgeted, most of the operating budgets can be linked to and coordinated with this one.
The level of expenditure is thus directly linked to the level of activity.
STUDY MATERIAL C1
322
FINANCIAL PLANNING AND CONTROL
2006.1
Budgets for non-operating functions such as computer services and research and
development are only indirectly linked to activity levels. Determining the level of
expenditure to be included in these non-operating budgets is not quite so straightforward.
11.8.1 Incremental budgeting
Many non-operating budgets are set using an incremental approach. This means that the
budget for each period is based on the budget or actual results for the previous period,
adjusting for any expected changes and inflation.
This approach is unlikely to result in the optimum allocation of resources. It tends to
perpetuate inefficient and unnecessary practices, and may result in budget slack, which is
unnecessary expenditure built into the budget.
11.8.2 Zero-based budgeting
Zero-based budgeting (ZBB) was developed as an alternative to the incremental approach.
The CIMA Terminology defines ZBB as a ‘method of budgeting that requires all
costs to be specifically justified by the benefits expected.’
Zero-based budgeting is so called because it requires each budget to be prepared and
justified from zero, instead of simply using last year’s budget or actual results as a base.
Incremental levels of expenditure on each activity are evaluated according to the resulting
incremental benefits. Available resources are then allocated where they can be used most
effectively.
The major advantage of ZBB exercises is that managers are forced to consider
alternative ways of achieving the objectives for their activity and they are required to justify
the activities which they currently undertake. This helps to eliminate or reduce the
incidence of budget slack, which is the intentional overestimation of expenses and/or
underestimation of revenues in the budgeting process.
A detailed discussion of ZBB is outside the scope of your Fundamentals of Management
Accounting syllabus, but you should be aware that there are a number of different
approaches to budgetary planning.
11.9 Budgetary control information
You have now learned about the basic principles underlying the budgetary planning process.
You have seen how budgets are created to guide and coordinate the activities of individuals
within the organisation, to ensure that the organisation starts out in the right direction.
In the remainder of this chapter you will see how budgets are used for control purposes
to ensure that the organisation continues in the right direction.
Budgetary control is achieved by comparing the actual results with the budget. The
differences are calculated as variances and management action may be taken to investigate
and correct the variances if necessary or appropriate.
FUNDAMENTALS OF MANAGEMENT ACCOUNTING
323
FINANCIAL PLANNING AND CONTROL
2006.1
If costs are higher or revenues are lower than the budget, then the difference is an
adverse variance.
If costs are lower or revenues are higher than the budget, then the difference is a
favourable variance.
11.9.1 Budget centres
The CIMA Terminology defines a budget centre as a ‘section of an entity for which control
may be exercised through prepared budgets’. Each budget centre is often a responsibility
centre. Each centre will have its own budget and a manager will be responsible for
managing the centre and controlling the budget. This manager is often referred to as the
budget holder. Regular budgetary control reports will be sent to each budget holder so that
they may monitor their centre’s activities and take control action if necessary.
11.9.2 Budgetary control reports
If managers are to use the budgets to control effectively, they must receive regular control
information.
The budgetary control reports should be:
(a) Timely. The information should be made available as soon as possible after the end of
the control period. Corrective action will be much more effective if it is taken soon
after the event, and adverse trends could continue unchecked if budgetary reporting
systems are slow.
(b) Accurate. Inaccurate control information could lead to inappropriate management
action. There is often a conflict between the need for timeliness and the need
for accuracy. More accurate information might take longer to produce. The design of
budgetary reporting systems should allow for sufficient accuracy for the purpose to be
fulfilled.
(c) Relevant to the recipient. Busy managers should not be swamped with information that is
not relevant to them. They should not need to search through a lot of irrelevant
information to reach the part which relates to their area of responsibility. The natural
reaction of managers in this situation could be to ignore the information altogether.
The budgetary reporting system should ideally be based on the exception principle
which means that management attention is focused on those areas where performance
is significantly different from budget. Subsidiary information could be provided on
those items which are in line with the budget.
Many control reports also segregate controllable and non-controllable costs and
revenues, that is, the costs and revenues over which managers can exercise control are
highlighted separately in the reports from those over which they have no control.
A number of accounting packages have the facility to record actual and budget
details against each account code for each budget centre. These may then be printed in
the form of a report.
(d) Communicated to the correct manager. Control information should be directed to the
manager who has the responsibility and authority to act upon it. If the information is
communicated to the wrong manager its value will be immediately lost and any
adverse trends may continue uncorrected. Individual budget holders’ responsibilities
must be clearly defined and kept up to date in respect of any changes.
STUDY MATERIAL C1
324
FINANCIAL PLANNING AND CONTROL
2006.1
11.10 Fixed and flexible budgets
When managers are comparing the actual results with the budget for a period, it is
important to ensure that they are making a valid comparison. The use of flexible budgets
can help to ensure that actual results are monitored against realistic targets.
11.10.1 Flexible budgets: an example
An example will demonstrate how flexible budgets may be used.
A company manufactures a single product and the following data show the actual
results for costs for the month of April compared with the budgeted figures.
Operating statement for April
Note : Variances in brackets are adverse.
Looking at the costs incurred in April, a cost saving of £3,010 has been made compared
with the budget. However, the number of units produced was 200 less than budget so
some savings in expenditure might be expected. It is not possible to tell from this
comparison how much of the saving is due to efficient cost control, and how much is the
result of the reduction in activity.
The type of budget being used here is a fixed budget. A fixed budget is one which
remains unchanged regardless of the actual level of activity. In situations where activity
levels are likely to change, and there is a significant proportion of variable costs, it is
difficult to control expenditure satisfactorily with a fixed budget.
If costs are mostly fixed, then changes in activity levels will not cause problems for cost
comparisons with fixed budgets.
A flexible budget can help managers to make more valid comparisons. It is designed
to show the allowed expenditure for the actual number of units produced and sold.
Comparing this flexible budget with the actual expenditure it is possible to distinguish genuine
efficiencies.
11.10.2 Preparing a flexible budget
Before a flexible budget can be prepared managers must identify which costs are fixed and
which are variable. The allowed expenditure on variable costs can then be increased or
decreased as the level of activity changes. You will recall that fixed costs are those costs
Actual Budget Variance
Units produced 1,000 1,200 (200)
££ £
Direct material 16,490 19,200 2,710
Direct labour 12,380 13,200 820
Production overhead 24,120 24,000 (120)
Administration overhead 21,600 21,000 (600)
Selling and distribution o/head
16,200 16,400 200
Total cost
90,790 93,800 3,010
FUNDAMENTALS OF MANAGEMENT ACCOUNTING
325
FINANCIAL PLANNING AND CONTROL
2006.1
which will not increase or decrease over the relevant range of activity. The allowance for
these items will therefore remain constant.
We can now continue with the example.
Management has identified that the following budgeted costs are fixed:
It is now possible to identify the expected variable cost per unit produced.
Now that managers are aware of the fixed costs and the variable costs per unit it is
possible to ‘flex’ the original budget to produce a budget cost allowance for 1,000 units
produced.
The budget cost allowance for each item is calculated as follows:
Cost allowance ¼ Budgeted fixed cost
þ (number of units produced variable cost per unit)
For the costs that are wholly fixed or wholly variable, the calculation of the budget cost
allowance is fairly straightforward. The remaining costs are semi-variable, which you will
recall means that they are partly fixed and partly variable. For example, the budget cost
allowance for direct labour is calculated as follows:
Cost allowance for direct labour ¼ £8,400 þð1,000 £4Þ¼£12,400
A full flexible budget can now be produced.
£
Direct labour 8,400
Production overhead 18,000
Administration overhead 21,000
Selling and distribution overhead 14,000
Original
budget
Fixed
cost
Variable
cost
V ’ble cost
per unit
(a) (b) (c) ¼(a) (b) ¼(c)/1,200
Units produced 1,200
££ £ £
Direct material 19,200 19,200 16
Direct labour 13,200 8,400 4,800 4
Production overhead 24,000 18,000 6,000 5
Administration overhead 21,000 21,000
Selling and distribution o/head
16,400 14,000 2,400 2
93,800 61,400 32,400 27
STUDY MATERIAL C1
326
FINANCIAL PLANNING AND CONTROL
2006.1
Flexible budget comparison for April
This revised analysis shows that in fact the cost was £2,390 higher than would have
been expected from a production volume of 1,000 units.
The cost variances in the flexible budget comparison are almost all adverse.
These overspendings were not revealed when a fixed budget was used and managers
may have been under the false impression that costs were being adequately
controlled.
11.10.3 The total budget variance
If we now produce a statement showing the fixed budget, the flexible budget and the
actual results together, it is possible to analyse the total variance between the original
budget and the actual results.
Cost allowances
Fixed Variable Total Actual cost Variance
£££££
Direct material 16,000 16,000 16,490 (490)
Direct labour 8,400 4,000 12,400 12,380 20
Production overhead 18,000 5,000 23,000 24,120 (1,120)
Administration overhead 21,000 21,000 21,600 (600)
Selling and distn. o/h
14,000 2,000 16,000 16,200 (200)
Total cost
61,400 27,000 88,400 90,790 (2,390)
Note: Variances in brackets are adverse.
Fixed budget Flexible budget Actual results
Expenditure
variances
££ ££
Direct material 19,200 16,000 16,490 (490)
Direct labour 13,200 12,400 12,380 20
Production overhead 24,000 23,000 24,120 (1,120)
Administrative overhead 21,000 21,000 21,600 (600)
Selling and distribution overhead
16,400 16,000 16,200 (200)
93,800 88,400 90,790 (2,390)
5,400 (2,390)
Volume
variance
Expenditure
variance
3,010
Total variance
FUNDAMENTALS OF MANAGEMENT ACCOUNTING
327
FINANCIAL PLANNING AND CONTROL
2006.1
The total variance is therefore made up of two parts:
the volume variance of £5,400 favourable, which is the expected cost saving resulting
from producing 200 units less than budgeted;
the expenditure variance of £2,390 adverse, which is the net total of the over- and
under-expenditure on each of the costs for the actual output of 1,000 units.
Notice that the volume variance is the saving in standard variable cost:
200 units £27 per unit ¼£5,400.
In Chapter 5 you learned how some of the expenditure variances can be analysed
between their price and usage elements – for example, how much of the variance is caused
by paying the wrong price per hour of labour (the labour rate variance), or per kilogram of
material (the material price variance), and how much is caused by using the wrong quantity
of material or labour (the usage and efficiency variances).
11.10.4 Using flexible budgets for planning
You should appreciate that while flexible budgets can be useful for control purposes they
are not particularly useful for planning. The original budget must contain a single target
level of activity so that managers can plan such factors as the resource requirements and
the product pricing policy. This would not be possible if they were faced with a range of
possible activity levels although managers will of course consider a range of possible
activity levels before they select the target budgeted activity level.
The budget can be designed so that the fixed costs are distinguished from the variable
costs. This will facilitate the preparation of a budget cost allowance for control purposes at
the end of each period, when the actual activity is known.
11.10.5 Flexible budgets: another example
Now that you have got the idea of how a flexible budget can be prepared, work through
the following example to consolidate your understanding.
In this example, as in practice, you will need to investigate the cost behaviour patterns
to determine which costs are fixed, which are variable and which are semi-variable.
The first step in investigating cost behaviour patterns is to look at the cost data. You
should be able to easily spot any fixed costs because they remain constant when activity
levels change.
The easiest way to identify the behaviour patterns of non-fixed costs is to divide each
cost figure by the related activity level. If the cost is a linear variable cost, then the cost per
unit will remain constant. For a semi-variable cost the unit rate will reduce as the activity
level increases, because the same basic amount of fixed costs is being spread over a greater
number of units.
You will then need to recall how to use the high–low method to determine the fixed and
variable elements of any semi-variable costs. Look back to Chapter 1 if you have forgotten
how the high–low method works.
STUDY MATERIAL C1
328
FINANCIAL PLANNING AND CONTROL
2006.1
Example
Lawrence Ltd operates a system of flexible budgets and the flexed budgets for expenditure for the first two
quarters of year 3 were as follows:
Flexed budgets quarters 1 and 2
Despite a projected increase in activity, the cost structures in quarters 1 and 2 are expected to
continue during quarter 3 as follows:
(a) The variable cost elements behave in a linear fashion in direct proportion to volume. However, for
production output in excess of 14,000 units the unit variable cost for production labour increases by
50 per cent. This is due to a requirement for overtime working and the extra amount is payable only on
the production above 14,000 units.
(b) The fixed cost elements are not affected by changes in activity levels.
(c) The variable elements of production costs are directly related to production volume.
(d) The variable element of selling and distribution overhead is directly related to sales volume.
You are required to prepare a statement of the budget cost allowances for quarter 3, when sales were
14,500 units and production was 15,000 units.
Solution
If you divide each cost figure by the relevant activity figure, you will find that the only wholly variable cost
is direct material, at £13 per unit.
You can also see that the only wholly fixed cost is administration overhead since this is a constant
amount for both activity levels, £26,000.
For the remaining costs you will need to use the high–low method to determine the fixed and variable
elements.
Production labour
Variable cost per unit ¼
£7,500
3,000
¼ £2:50 per unit
Fixed cost ¼ £81,500 ð£2:50 13,000Þ¼£49,000
Quarter 1 Quarter 2
Activity
Sales units 9,000 14,000
Production units 10,000 13,000
Budget cost allowances ££
Direct materials 130,000 169,000
Production labour 74,000 81,500
Production overhead 88,000 109,000
Administration overhead 26,000 26,000
Selling and distribution overhead
29,700 36,200
Total budget cost allowance
347,700 421,700
Production, units £
Quarter 2 13,000 81,500
Quarter 1
10,000 74,000
Change
3,000 7,500
FUNDAMENTALS OF MANAGEMENT ACCOUNTING
329
FINANCIAL PLANNING AND CONTROL
2006.1
Production overhead
Variable cost per unit ¼
£21,000
3,000
¼ £7 per unit
Fixed cost ¼ £109,000 ð£7 13,000Þ¼£18,000
Selling and distribution overhead
Note that the example data says that selling and distribution overhead is related to sales volume.
Variable cost per unit sold ¼
£6,500
5,000
¼ £1:30 per unit
Fixed cost ¼ £36,200 ð£1:30 14,000Þ¼£18,000:
We can now prepare a statement of the budget cost allowances for quarter 3.
Note 1: The unit variable cost for production labour increases by 50 per cent for production over
14,000 units.
Note 2: The flexible budget allowance for selling and distribution overhead must be based on the
sales volume of 14,500 units.
Production, units £
Quarter 2 13,000 109,000
Quarter 1
10,000 88,000
Change
3,000 21,000
Sales, units £
Quarter 2 14,000 36,200
Quarter 1
9,000 29,700
5,000 6,500
Quarter 3
Budget cost allowance
££
Direct material (15,000 units £13) 195,000
Production labour:
1
Fixed 49,000
Variable up to 14,000 units (14,000 £2.50) 35,000
Variable above 14,000 units (1,000 £3.75)
3,750
87,750
Production overhead:
Fixed 18,000
Variable (15,000 £7)
105,000
123,000
Administration overhead: fixed 26,000
Selling and distribution overhead:
Fixed 18,000
Variable (14,500 £1.30)
2
18,850
36,850
Total budget cost allowance
468,600
STUDY MATERIAL C1
330
FINANCIAL PLANNING AND CONTROL
2006.1
11.10.6 Extrapolating outside the relevant range
In the preceding example you were told that the cost structures would remain unaltered
despite the increase in activity. In practice, if you need to do a similar extrapolation outside
the range for which you have available data, you should always state the assumption that
the observed behaviour patterns will still be applicable.
11.10.7 Example: producing a flexible budget control
statement
G Limited produces and sells a single product. The budget for the latest period is as
follows.
The actual results for the period were as follows.
Required
Prepare a flexible budget control statement and comment on the results.
£
Sales revenue (12,600 units)
277,200
Variable costs
Direct material 75,600
Direct labour 50,400
Production overhead 12,600
Fixed costs
Production overhead 13,450
Other overhead
10,220
162,270
Budget profit
114,930
£
Sales revenue (13,200 units)
303,600
Variable costs
Direct material 78,350
Direct labour 51,700
Production overhead 14,160
Fixed costs
Production overhead 13,710
Other overhead
10,160
168,080
Actual profit
135,520
FUNDAMENTALS OF MANAGEMENT ACCOUNTING
331
FINANCIAL PLANNING AND CONTROL
2006.1
Solution
The budgeted sales revenue and the budget cost allowances for the variable costs are
increased by a factor of 13,200/12,600 to derive the flexed budget for the actual activity
achieved during the period. The budget cost allowance for the fixed costs remains unaltered.
Flexible budget control statement for the latest period
Note: variances in brackets are adverse
Comments
1. The total budget variance can be analysed as follows.
2. The favourable sales price variance indicates that a higher selling price than standard
was charged for the units sold. Despite the higher price the sales volume achieved was
higher than budgeted.
3. Expenditure on direct material, direct labour and other overhead costs was lower than
the budget cost allowance for the activity level achieved. It is not possible to tell from
the data provided whether the savings were achieved as a result of a lower price or a
lower usage of resources.
4. Expenditure on production overhead costs, both fixed and variable, was higher than
the budget cost allowance for the activity level achieved.
Original
budget
Flexed
budget
Actual
results Variance
Activity (units) 12,600 13,200 13,200
££££
Sales revenue
277,200 290,400 303,600 13,200
Variable costs
Direct material 75,600 79,200 78,350 850
Direct labour 50,400 52,800 51,700 1,100
Production overhead 12,600 13,200 14,160 (960)
Fixed costs
Production overhead 13,450 13,450 13,710 (260)
Other overhead
10,220 10,220 10,160 60
162,270 168,870 168,080 790
Profit
114,930 121,530 135,520 13,990
££
Sales volume variance* (£121,530 £114,930) 6,600
Sales price variance 13,200
Expenditure variance
790
13,990
Total budget variance (£135,520 £114,930)
20,590
* You can calculate the sales volume variance separately as a check on the budget
figures: increase in sales volume above budget standard contribution per
unit ¼(13,200 – 12,600) £((277,200 – 75,600 – 50,400 – 12,600)/
12,600) ¼600 units £11 standard contribution ¼£6,600 favourable.
STUDY MATERIAL C1
332
FINANCIAL PLANNING AND CONTROL
2006.1
11.11 Using budgets as a basis for rewards
Budgets may be used as a basis for reward strategies for managers. In this situation the
budget acts as a target for achievement and the budget holder’s success in meeting the
budget might be rewarded by the payment of a bonus.
11.11.1 Example
The manager of Department T had a total budget expenditure allowance of £210,000 for
the latest period.
The manager is paid a bonus of 10% of any savings he achieves against the budgeted
expenditure.
Actual expenditure for the latest period was £207,800.
Requirement
Calculate the amount of any bonus payable to the manager.
Bonus payable ¼10% £(210,000 207,800) ¼£220.
11.11.2 Factors to consider in the design of budget
reward schemes
The potential to earn a bonus by the achievement of a budget target can create a powerful
incentive for budget holders. However, a number of factors should be considered in the
design of reward strategies.
A flexible budget system should be used where appropriate so that the budget holder’s
performance is monitored against a realistic revenue and expenditure target for the
actual level of activity achieved.
Managers who are responsible for setting their own budgets in a participative budgeting
system might set easy targets for themselves and build in budgetary slack in order to
improve their chances of earning a bonus. If the targets are not realistic as a result of this
budget padding then the budget will not be useful as a planning or control document.
Managers who are aiming to achieve a bonus based on their short-term budget
performance might be tempted to cut back on expenditure which is necessary for the
longer-term strategy of the organisation, for example training and development
expenditure, that is, the budget reward system might encourage short termism.
Managers might become demotivated if they fail to achieve their budget targets, and
thus do not earn a bonus, due to factors which are outside their control.
Therefore a reward system based on the achievement of budget targets should be
designed and operated with due regard for the possible impact on managers’ behaviour.
FUNDAMENTALS OF MANAGEMENT ACCOUNTING
333
FINANCIAL PLANNING AND CONTROL
2006.1
11.12 Summary
Having read this chapter the main points that you should understand are as follows.
1. A budget is a quantified plan of action relating to a given period of time. An
organisation’s annual budget is set within the framework of the long-term strategic
plans.
2. The budget committee coordinates the preparation of budgets and issues the budget
manual which provides information to those involved in the planning and control
process.
3. The principal (key) budget factor is the factor which limits the activities of the
organisation. The budget for the principal factor should be prepared first.
4. The master budget is the summary of all the functional budgets, usually including a
budgeted income statement (profit and loss account), balance sheet and cash flow
statement.
5. Cash budgets allow for feedforward control by forewarning managers of the cash effect
of all their planning decisions.
6. Rolling or continuous budgets are continuously updated by adding a further period
when the earliest period has expired.
7. Incremental budgeting involves using the prior period’s budget or actual results as a
basis for the next year’s budget. Zero-based budgeting begins each year’s budget from
scratch.
8. A fixed budget is prepared for a single activity level. A flexible budget is more useful for
control because it recognises cost and revenue behaviour patterns and the budget cost
allowance for each cost and revenue is designed to change as the volume of activity
changes.
9. Budgets may be used as a basis for reward strategies for managers.
STUDY MATERIAL C1
334
FINANCIAL PLANNING AND CONTROL
2006.1
Revision Questions
Question 1 Multiple choice
1.1 When preparing a production budget, the quantity to be produced equals:
(A) sales quantity + opening stock + closing stock.
(B) sales quantity opening stock + closing stock.
(C) sales quantity opening stock closing stock.
(D) sales quantity + opening stock closing stock.
1.2 A job requires 2,400 actual labour hours for completion and it is anticipated that
there will be 20 per cent idle time. If the wage rate is £10 per hour, what is the
budgeted labour cost for the job?
(A) £19,200
(B) £24,000
(C) £28,800
(D) £30,000.
1.3 The term ‘budget slack’ refers to:
(A) the extended lead time between the preparation of the functional budgets and
the master budget.
(B) the difference between the budgeted output and the breakeven output.
(C) the additional capacity available which can be budgeted for.
(D) the deliberate overestimation of costs and underestimation of revenues in a
budget.
1.4 Of the four costs shown below, which would not be included in the cash budget of
an insurance firm?
(A) depreciation of fixed assets
(B) commission paid to agents
(C) office salaries
(D) capital cost of a new computer.
1.5 The following details have been extracted from the debtor collection records of C
Limited:
Invoices are issued on the last day of each month.
Invoice paid in the month after sale 60%
Invoice paid in the second month after sale 25%
Invoice paid in the third month after sale 12%
Bad debts 3%
11
335 2006.1
Customers paying in the month after sale are entitled to deduct a 2 per cent
settlement discount.
Credit sales values for June to September are budgeted as follows:
June July August September
£35,000 £40,000 £60,000 £45,000
The amount budgeted to be received from credit sales in September is
(A) £47,280
(B) £47,680
(C) £48,850
(D) £49,480.
1.6 A flexible budget is:
(A) a budget which, by recognising different cost behaviour patterns, is designed to
change as the volume of activity changes.
(B) a budget for a defined period of time which includes planned revenues,
expenses, assets, liabilities and cash flow.
(C) a budget which is prepared for a period of one year which is reviewed monthly,
whereby each time actual results are reported, a further forecast period is added
and the intermediate period forecasts are updated.
(D) a budget of semi-variable production costs only.
1.7 The following extract is taken from the production cost budget of S Limited:
The budget cost allowance for an activity level of 4,000 units is
(A) £7,200
(B) £14,700
(C) £17,200
(D) £22,200.
1.8 A master budget comprises:
(A) the budgeted income statement.
(B) the budgeted cash flow, budgeted income statement and budgeted balance sheet.
(C) the budgeted cash flow.
(D) the entire set of budgets prepared.
Question 2 Short objective-test questions
2.1 Tick the correct box.
A participative budgeting system may also be described as a:
bottom-up budget &
top-down budget &
Production (units) 2,000 3,000
Production cost ) 11,100 12,900
REVISION QUESTIONS C1
336
FINANCIAL PLANNING AND CONTROL
2006.1
2.2 Which of the following items of information would be contained in the budget
manual? (Tick all that are correct.)
(a) An organisation chart. &
(b) The timetable for budget preparation. &
(c) The master budget. &
(d) A list of account codes. &
(e) Sample forms to be completed during the budgetary process. &
2.3 Is the following statement true or false?
The principal budget factor is always the forecast sales volume.
True &
False &
2.4 Assuming that sales volume is the principal budget factor, place the following
budgets in the order that they would be prepared in the budgetary planning process.
Indicate the correct order by writing 1, 2, 3, etc. in the boxes provided.
& Sales budget
& Materials purchases budget
& Materials stock budget
& Production budget
& Finished goods stock budget
& Materials usage budget.
2.5 PR Ltd’s cash budget forewarns of a short-term surplus. Which of the
following would be appropriate actions to take in this situation? (Select all that
are correct).
(a) Increase debtors and stock to boost sales. &
(b) Purchase new fixed assets. &
(c) Repay long-term loans. &
(d) Pay creditors early to obtain a cash discount. &
2.6 Each finished unit of product H contains 3 litres of liquid L. Ten per cent of
the input of liquid L is lost through evaporation in the production process.
Budgeted output of product H for June is 3,000 units. Budgeted stocks of
liquid L are:
Opening stock, 1 June 1,200 litres
Closing stock, 30 June 900 litres
The required purchases of liquid L for June are
litres.
2.7 Tick the correct box.
A system of budgeting whereby the budget is continuously updated by adding a
further accounting period when the earliest accounting period has expired, is known
as a system of:
rolling budgets &
incremental budgets &
FUNDAMENTALS OF MANAGEMENT ACCOUNTING
337
FINANCIAL PLANNING AND CONTROL
2006.1
2.8 The totals from KM Ltd’s budgetary control report for February are as follows;
Complete the following table, ticking the box to indicate whether the variance is
adverse or favourable.
2.9 Which of the following best describes the principle of reporting by exception?
Sending budget reports only to those exceptional managers who
are able to understand their content.
Providing detailed reports only on those areas of the business that
are performing exceptionally well and providing only subsidiary
information about other areas of the business.
Providing detailed reports only on those areas of the business that
are not performing according to budget and providing only subsidiary
information about aspects that are in line with budget.
2.10 F Limited uses a flexible budgeting system to control the costs incurred in its staff
canteen.
The budget cost allowance for consumable materials is flexed according to the
average number of employees during the period.
Complete the following equation by inserting ‘+’, ’or‘ as appropriate in the
boxes:
2.11 The following extract is taken from the catering costs budget of a company that
provides training courses.
In a flexible budget for 185 delegates, the budget cost allowance for catering costs
will be £
Fixed budget Flexible budget Actual results
££ £
Total sales revenue 124,310 135,490 134,580
Total expenditure
93,480 98,450 97,920
Total profit
30,830 37,040 36,660
£ Adverse Favourable
Sales price variance
&&
Sales volume variance
&&
Expenditure variance
&&
Total budget variance
&&
Flexible budget
cost allowance
for consumable
materials
¼ budgeted fixed
cost
& ( budgeted variable
(cost per employee
& average no.)
of employees)
Number of delegates 120 170
Catering cost £1,470 £2,020
FINANCIAL PLANNING AND CONTROL
REVISION QUESTIONS C1
338
2006.1
Question 3 Functional budgets
An ice cream manufacturer is in the process of preparing budgets for the next few months,
and the following draft figures are available:
Each case uses 2.5 kg of ingredients and it is policy to have stocks of ingredients at the
end of each month to cover 50 per cent of next month’s production.
There are 750 cases of finished ice cream in stock on 1 June and it is policy to have
stocks at the end of each month to cover 10 per cent of the next month’s sales.
Requirements
(a) The production budget (in cases) for June and July will be:
June
July
(b) The ingredient purchases budget (in kg) for August will be
Question 4 Cash budget
A small manufacturing firm is to commence operations on 1 July. The following estimates
have been prepared:
It is planned to have raw material stocks of £10,000 at the end of July, and to maintain
stocks at that level thereafter.
Selling prices, costs and other information:
Fixed overheads are expected to be £5,000 per month, including £1,000 depreciation.
Settlement terms on sales: 10 per cent cash, the balance payable the month following
sale. Labour is paid in the month incurred, and all other expenditures the following
month.
Sales forecast
June 6,000 cases
July 7,500 cases
August 8,500 cases
September 7,000 cases
October 6,500 cases
July August September
Sales (units) 10 36 60
Production (units) 40 50 50
Opening stock (units) NIL
Per unit
£
Selling price 900
Material cost 280
Labour cost 160
Variable overheads 40
FUNDAMENTALS OF MANAGEMENT ACCOUNTING
339
FINANCIAL PLANNING AND CONTROL
2006.1
Requirements
(a) The budgeted cash receipts from sales are:
July £
August £
September £
(b) The budgeted cash payments for raw materials are:
July £
August £
September £
(c) The total of the budgeted cash payments for labour and overhead in August is £ .
(d) A cash budget can be used to give forewarning of potential cash problems that could
arise so that managers can take action to avoid them. This is known as:
feedforward control &
feedback control &
(e) A cash budget is continuously updated to reflect recent events and changes to forecast
events. This type of budget is known as a:
flexible budget &
rolling budget &
Question 5 Flexible budget
The Arcadian Hotel operates a budgeting system and budgets expenditure over eight
budget centres as shown below. Analysis of past expenditure patterns indicates that
variable costs in some budget centres vary according to occupied room nights (ORN),
while in others the variable proportion of costs varies according to the number of visitors ( V ).
The budgeted expenditures for a period with 2,000 ORN and 4,300 V were as follows:
Variable costs
vary with:
Budgeted
expenditure
Partial cost anlaysis
Budget centre £
Budget expenditure
includes:
Cleaning ORN 13,250 £2.50 per ORN
Laundry V 15,025 £1.75 per V
Reception ORN 13,100 £12,100 fixed
Maintenance ORN 11,100 £0.80 per ORN
Housekeeping V 19,600 £11,000 fixed
Administration ORN 7,700 £0.20 per ORN
Catering V 21,460 £2.20 per V
General overheads
11,250 all fixed
112,485
REVISION QUESTIONS C1
340
FINANCIAL PLANNING AND CONTROL
2006.1
In period 9, with 1,850 ORN and 4,575 V, actual expenditures were as follows:
Requirements
(a) The total budget cost allowances for the following costs in the flexible budget for
period 9 are:
(b) The total budget cost allowance in the flexible budget for period 9 is £113,521.
The total expenditure variance for period 9 is £
. The variance is:
adverse &
favourable &
Budget centre Actual expenditure
£
Cleaning 13,292
Laundry 14,574
Reception 13,855
Maintenance 10,462
Housekeeping 19,580
Administration 7,930
Catering 23,053
General overheads
11,325
114,071
£
Cleaning
Laundry
Reception
Maintenance
Housekeeping
General overheads
FINANCIAL PLANNING AND CONTROL
2006.1
FUNDAMENTALS OF MANAGEMENT ACCOUNTING
341
Solutions to
Revision Questions
Solution 1
In question 1.2 you cannot simply add 20 per cent to the actual labour hours to allow
for the idle time. The idle time is 20 per cent of the hours to be paid for, so you will
need to think more carefully about how to make the adjustment.
In question 1.5 remember that the 3 per cent bad debts will never be received in cash.
1.1 Answer: (B)
Requirements for closing stock increase the amount to be produced, so these must
be added. Stock available in opening stock reduces production requirements, so this
must be deducted.
1.2 Answer: (D)
Idle time is 20 per cent of the total hours to be paid for. Therefore, hours to be paid
for ¼ 2,400=0:8 ¼ 3,000. Budgeted labour cost ¼3,000 £10 ¼£30,000.
1.3 Answer: (D)
A manager might build some slack into a budget to provide some ‘leeway’ to disguise
unnecessary spending.
1.4 Answer: (A)
Depreciation is not a cash flow.
1.5 Answer: (D)
Amount to be received in September is:
60% of August sales less 2% discount: £
£60,000 60% 98% 35,280
25% of July sales: £40,000 25% 10,000
12% of June sales: £35,000 12%
4,200
49,480
1.6 Answer: (A)
A flexible budget is designed to show the budgeted costs and revenues at different
levels of activity.
11
343 2006.1
1.7 Answer: (B)
Increase in cost £1,800
Increase in production 1,000 units
Variable costs: £1,800/1,000 £1.80/unit
£
Variable cost of 2,000 units 3,600
Total cost of 2,000 units
11,100
Fixed cost
7,500
Variable cost of 4,000 units 7,200
Fixed cost
7,500
14,700
1.8 Answer: (B).
Solution 2
2.1 A participative budgeting system may also be described as a bottom-up budget.
2.2 (a), (b), (d) and (e) would be contained in a budget manual. The master budget (c) is
the end result of the budgetary planning process.
2.3 False. The forecast sales volume will often be the principal budget factor or limiting
factor, but this is not always the case.
2.4 1. Sales budget
2. Finished goods stock budget
3. Production budget
4. Materials usage budget
5. Materials stock budget
6. Materials purchases budget
2.5 (a) and (d) would be appropriate actions in this situation. Actions (b) and (c) would
not be appropriate because they would involve investing the surplus funds for
too long.
2.6
Litres
Liquid L required for finished output
(3,000 units 3 litres)
9,000
Evaporation loss ð
10
90
Þ* 1,000
Total required input of liquid L 10,000
Less: reduction in stock
300
Required purchases of liquid L
9,700
* evaporation loss is 10 per cent of input
2.7 A system of budgeting whereby the budget is continuously updated by adding a
further accounting period when the earliest accounting period has expired is known
as a system of rolling budgets. It is also known as a continuous budgeting system.
SOLUTIONS TO REVISION QUESTIONS C1
344
FINANCIAL PLANNING AND CONTROL
2006.1
2.8
£
Sales price variance
1
£(134,580 135,490) 910 Adverse
Sales volume variance
2
£(37,040 30,830) 6,210 Favourable
Expenditure variance
3
£(98,450 97,920) 530 Favourable
Total budget variance
4
£(36,660 30,830) 5,830 Favourable
Notes:
1. The sales price variance is the difference between the sales revenue that was achieved
and the sales revenue that would be expected for the actual activity level that
occurred (that is, the sales revenue in the flexible budget).
2. The sales volume variance is the additional standard contribution that arose as a
result of the change in the sales volume from the original budget.
3. The expenditure variance is the difference between the actual expenditure and the
expenditure that would be expected for the actual activity achieved.
4. The total budget variance is the difference between the original budget profit and the
actual profit achieved.
2.9 Exception reporting involves providing detailed reports only on those areas of the
business that are not performing according to budget and providing only subsidiary
information about aspects that are in line with budget. This ensures that
management do not receive too much information and that their attention is
focused where control action is most needed.
2.10
Flexible budget
cost allowance
for consumable
materials
¼ budgeted fixed
cost
&
þ
( budgeted variable
(cost per employee
&
þ
average no.)
of employees)
2.11 In a flexible budget for 185 delegates the budget cost allowance for catering costs
will be £2,185
Delegates £
170 2,020
120 1,470
50 550
Variable catering cost per delegate ¼£550/50 ¼£11
Fixed catering cost ¼£2,020 £(170 11) ¼£150
Budget cost allowance for 185 delegates ¼£150 + £(185 11) ¼£2,185.
FUNDAMENTALS OF MANAGEMENT ACCOUNTING
345
FINANCIAL PLANNING AND CONTROL
2006.1
Solution 3
Use a clear columnar layout for your budget workings. Although your workings will not
earn marks, clear workings help you to avoid arithmetical errors because 100 per cent
accuracy is vital.
Do not forget to adjust for the budgeted movement in stock in parts (a) and (b). A common
error is to get the opening and closing stock calculations the wrong way round.
(a) June: 6,000
July: 7,600
(b) August: 19,125
Workings:
Production budget (in cases)
June July August September
Cases to be
sold
6,000 7,500 8,500 7,000
Closing stock (7,500 10%) 750 (8,500 10%) 850 (7,000 10%) 700 (6,500 10%) 650
Opening stock
(750) (750) (850) (700)
Production
budget
6,000 7,600 8,350 6,950
Ingredients purchases budget (in kg)
August
Quantity to be used in production (8,350 2.5) 20,875
Quantity in closing stock (6,950 2.5 50%) 8,687.5
Quantity in opening stock (8,350 2.5 50%)
(10,437.5)
Ingredients purchases budget
19,125.0
Solution 4
Remember to exclude depreciation from the fixed overhead figures. Depreciation is not
a cash flow.
Read the wording of the question carefully to determine the timing of each cash flow.
(a) July £900
August £11,340
September £34,560
Workings:
££
July: 10% (10 £900) 900
August: 90% July sales (10 £900) 8,100
10% August sales (36 £900)
3,240
11,340
September: 90% August sales (36 £900) 29,160
10% September sales (60 £900)
5,400
34,560
SOLUTIONS TO REVISION QUESTIONS C1
346
FINANCIAL PLANNING AND CONTROL
2006.1
(b) July £0
August £21,200
September £14,000
Workings:
Cash payments each month are for the previous month’s purchases. Therefore, no
payments are made in July.
££
August: payment for July closing stock 10,000
payment for July usage (40 £280)
11,200
21,200
September: payment for August usage (50 £280) 14,000
(c) £13,600
Workings:
£
August labour cost paid in month incurred (50 £160) 8,000
July variable overhead cost paid in August (40 £40) 1,600
Fixed overhead cash cost (£5,000 £1,000 depreciation)
4,000
13,600
(d) This is known as feedforward control.
(e) This type of budget is known as a rolling budget.
Solution 5
A common error in this type of question is to calculate the expenditure variance
( part (b)) by comparing the actual results with the budget supplied in the question. This is
the budget for quite different activity levels, so the flexed budget should be used instead.
(a)
£
Cleaning 12,875
Laundry 15,506
Reception 13,025
Maintenance 10,980
Housekeeping 20,150
General overheads 11,250
Workings:
Activity
ðORN= VÞ
Variable cost
per unit
Variable cost
allowance
Fixed cost
allowance
Total budget
cost allowance
££££
Cleaning 1,850 2.50 4,625 8,250
1
12,875
Laundry 4,575 1.75 8,006 7,500
2
15,506
Reception 1,850 0.50
3
925 12,100 13,025
Maintenance 1,850 0.80 1,480 9,500
4
10,980
Housekeeping 4,575 2.00
5
9,150 11,000 20,150
General o/heads 11,250 11,250
FUNDAMENTALS OF MANAGEMENT ACCOUNTING
347
FINANCIAL PLANNING AND CONTROL
2006.1
1. £
Total budget cost allowance for 2,000 ORN 13,250
Less variable allowance (2,000 £2.50)
5,000
Fixed cost allowance
8,250
2. £15,025 (4,300 £1.75) ¼£7,500
3. £
Total budget cost allowance for 2,000 ORN 13,100
Less fixed allowance
12,100
Variable cost allowance for 2,000 ORN
1,000
Variable cost allowance per ORN:
£1;000
2;000
£0.50
4. £11,100 (2,000 £0.80) ¼£9,500
5.
ð£19;600£11;000 Þ
4;300
¼ £2 per visitor
(b) £550 adverse
Workings:
£
Flexible budget expenditure 113,521
Actual expenditure
114,071
Expenditure variance
550 adverse
SOLUTIONS TO REVISION QUESTIONS C1
348
FINANCIAL PLANNING AND CONTROL
2006.1
Preparing for the
Assessment
Preparing for the
Assessment
Format of the assessment
The assessment for Fundamentals of Management Accounting is a two hour computer-based
assessment (CBA) comprising 50 objective test questions with one or more parts. There
will be no choice of questions and all questions should be attempted if time permits. There
is no penalty for incorrect answers.
Objective questions are used. The most common type is multiple choice, where the
candidate is required to select the correct answer from a list of possible options. Other
types of objective question that may be used include true/false questions, matching pairs
of text and graphic, sequencing and ranking, labelling diagrams and single and multiple
numeric entry. Candidates answer the questions by pointing and clicking the mouse,
moving objects around the screen, typing numbers, or a combination of these responses.
CIMA are continuously developing the question styles within the cba system and you
are strongly advised to try the online demo at www.cimaglobal.com/cba. This will enable
you to gain familiarity with the assessment software and to keep track of the latest style of
questions being used. You are also advised to keep an eye on the articles in the ‘Study
Notes’ section of Financial Management magazine which will forewarn of any changes in
question styles.
Revision technique
Planning
The first thing to say about revision is that it is an addition to your initial studies, not a
substitute for them. In other words, don’t coast along early in your course in the hope of
This chapter is intended for use when you are ready to start revising for your assessment.
It contains:
"
Details of the format of the assessment.
"
A summary of useful revision techniques.
"
Guidance on how to tackle the assessment.
"
A bank of assessment-standard revision questions and suggested solutions.
"
Two mock assessments. These should be attempted when you consider yourself to
be ready for the assessment, and you should simulate assessment conditions when
you attempt them.
351 2006.1
catching up during the revision phase. On the contrary, you should be studying and
revising concurrently from the outset. At the end of each week, and at the end of each
month, get into the habit of summarising the material you have covered to refresh your
memory of it.
As with your initial studies, planning is important to maximise the value of your revision
work. You need to balance the demands for study, professional work, family life and other
commitments. To make this work, you will need to think carefully about how to make best
use of your time.
Begin by comparing the estimated hours you will need to devote to revision with the
hours available to you in the weeks leading up to the assessment. Prepare a written
schedule setting out the areas you intend to cover during particular weeks, and break that
down further into topics for each day’s revision. To help focus on the key areas try to
establish which areas you are weakest on, so that you can concentrate on the topics where
effort is particularly needed.
Do not forget the need for relaxation, and for family commitments. Sustained
intellectual effort is only possible for limited periods, and must be broken up at intervals
by lighter activities. And do not continue your revision timetable right up to the moment
when you enter the assessment room; you should aim to stop work a day or even two days
before the assessment. Beyond this point, the most you should attempt is an occasional
brief look at your notes to refresh your memory.
Getting down to work
By the time you begin your revision you should already have settled into a fixed work
pattern: a regular time of day for doing the work, a particular location where you sit,
particular equipment that you assemble before you begin and so on. If this is not already a
matter of routine for you, think carefully about it now in the last vital weeks before the
assessment.
You should have notes summarising the main points of each topic you have covered.
Begin each session by reading through the relevant notes and trying to commit the
important points to memory.
Usually this will be just your starting point. Unless the area is one where you already feel
very confident, you will need to track back from your notes to the relevant chapter(s) in
the Learning System. This will refresh your memory on points not covered by your notes and
fill in the detail that inevitably gets lost in the process of summarisation.
When you think you have understood and memorised the main principles and
techniques, attempt some assessment questions. At this stage of your studies, you should
normally be expecting to complete the questions in something close to the actual time
allocation allowed in the assessment. After completing your effort, check the solution
provided and add to your notes any extra points it reveals.
Tips for the final revision phase
As the assessment looms closer, consider the following list of techniques and make use of
those that work for you:
Summarise your notes into a more concise form, perhaps on index cards that you can
carry with you for revision on the way to work.
Go through your notes with a highlighter pen, marking key concepts and definitions.
STUDY MATERIAL C1
352
PREPARING FOR THE ASSESSMENT
2006.1
Summarise the main points in a key area by producing a wordlist, mind map or other
mnemonic device.
On areas that you find difficult, rework questions that you have already attempted, and
compare your answers with those provided in the Learning System.
Rework questions you attempted earlier in your studies with a view to completing them
within the time limits.
In the week preceding the assessment, quickly go through any recent articles in the
‘Study Notes’ section of Financial Management magazine, paying particular attention to
those relevant to your subject.
Avoid late-night study, as your assessment is based on daytime performance, not night-
time performance.
Make sure that you cover the whole syllabus in your revision, as all questions in the
assessment are compulsory.
How to tackle the assessment
Assessment day
Before leaving for the assessment you should ensure that you know where you are
going: plan your route and ensure that you have the necessary documentation and your
calculator with you. It is advisable to bring a second calculator and some spare batteries!
Arrive early and settle into your assessment environment. You will have enough nerves
on the day without compounding them by arriving late.
The assessment
Multiple-choice questions
Multiple-choice questions (MCQs) are broken down into two parts; the problem or task to
be solved, and the options you must choose from. There is only ever one correct answer:
the other options are known as distractors.
Your approach to MCQs should be as follows:
For numerical MCQs, in the majority of cases you will need to do some rough workings.
Never rush to select your answer; some options might initially look plausible, but on
closer scrutiny turn out to be distractors. Unless you are certain of the answer, look
carefully at all the options before choosing.
If you are finding the MCQ difficult and you are taking up too much time, move on to
the next one.
Time permitting, revisit those MCQs which you left unanswered and refer to your
original workings.
Remember: you must never omit to answer any question in the assessment as there is no
penalty for an incorrect answer.
Other types of question
Prepare neat workings where necessary for your own benefit. Only your final answers will be
marked, not workings, methods or justifications. However, your workings will help you
to achieve the necessary 100 per cent accuracy.
Check your answer carefully. If you have typed in your answer, check the figures are
typed correctly.
Never omit to answer a question. There is no penalty for an incorrect answer.
FUNDAMENTALS OF MANAGEMENT ACCOUNTING
353
PREPARING FOR THE ASSESSMENT
2006.1
Revision Questions
The following table indicates the main learning outcome covered by each question in the
bank that follows. Once you have revised each topic you can attempt the relevant
question(s). However you should be aware that some questions relate to more than one
learning outcome.
Learning outcome Question
number(s)
Explain why organisations need to know how much products
and processes cost and why they need costing systems;
5
Explain the idea of a cost object; 5
Explain the concept of a direct cost and an indirect cost; 6, 13
Explain why the concept of cost needs to be qualified as
direct, full, marginal, etc. in order to be meaningful;
7
Distinguish between the historical cost of an asset and the
economic value of an asset to an organisation;
11
Apply first-in-first-out (FIFO), last-in-first-out (LIFO) and
average cost (AVCO) methods of accounting for stock,
calculating stock values and related gross profit;
8, 9, 10, 12
Explain why FIFO is essentially a historical cost
method, while LIFO approximates economic
cost;
11
Prepare cost statements for allocation and apportionment of
overheads, including between reciprocal service departments;
14, 18, 19
Calculate direct, variable and full costs of products, services
and activities using overhead absorption rates to trace
indirect costs to cost units;
14, 15, 16, 17,
18, 19, 20, 53
Explain the use of cost information in pricing decisions,
including marginal cost pricing and the calculation of ‘full
cost’ based prices to generate a specified return on sales
or investment;
21, 53, 54
Explain how costs behave as product, service or activity
levels increase or decrease;
1, 2, 4
Distinguish between fixed, variable and semi-variable costs; 1, 4
355 2006.1
Explain step costs and the importance of time-scales in their
treatment as either variable or fixed;
2
Compute the fixed and variable elements of a semi-variable
cost using the high-low method and ‘line of best fit’ method;
3
Explain the concept of contribution concept and its use in
Cost–Volume–Profit (CVP) analysis;
22, 23, 24
Calculate and interpret the breakeven point, profit target,
margin of safety and profit/volume ratio for a single
product or service;
22, 23, 24, 27,
28
Prepare breakeven charts and profit/volume graphs for a
single product or service;
25, 26, 29
Calculate the profit maximising sales mix for a multi-
product company that has limited demand for each
product and one other constraint or limiting factor;
30, 31
Explain the difference between ascertaining costs after the
event and planning by establishing standard costs in
advance;
32
Explain why planned standard costs, prices and volumes are
useful in setting a benchmark for comparison and so
allowing managers’ attention to be directed to areas of the
businessthat are performing below or above expectation;
32
Calculate standard costs for the material, labour and variable
overhead cost elements of cost of a product or service;
33
Calculate variances for material, labour, variable overhead,
sales prices and sales volumes;
34, 35, 36, 37,
38, 39, 40
Prepare a statement that reconciles budgeted contribution
with actual contribution;
41
Interpret statements of variances for variable costs, sales
prices and sales volumes including possible inter-relations
between cost variances, sales price and volume variances,
and cost and sales variances;
42, 43
Discuss the possible use of standard labour costs in
designing incentive schemes for factory and office workers;
44
Explain the principles of manufacturing accounts and the
integration of the cost accounts with the financial
accounting system;
45, 46, 47, 48
Prepare a set of integrated accounts, given opening balances
and appropriate transactional information, and show
standard cost variances;
49, 50
Compare and contrast job, batch, contract and process
costing;
18, 51
Prepare ledger accounts for job, batch and process costing
systems;
52, 53, 54, 56,
57, 58, 59
Prepare ledger accounts for contract costs; 55
Explain the difference between subjective and objective
classifications of expenditure and the importance of tracing
costs both to products/services and to responsibility
centres;
60
REVISION QUESTIONS C1
356
2006.1
PREPARING FOR THE ASSESSMENT
Question 1 Cost behaviour
The following data have been collected for four cost types W, X, Y, Z at two activity levels:
Cost Cost
100 units 140 units
Cost type £ £
W 8,000 10,560
X 5,000 5,000
Y 6,500 9,100
Z 6,700 8,580
Where V ¼variable, SV ¼semi-variable and F ¼fixed, assuming linearity, the four cost
types W, X, Y and Z are, respectively:
WXY Z
(A) V F SV V
(B) SV F V SV
(C)VFVV
(D) SV F SV SV
Construct coding systems that facilitate both subjective and
objective classification of costs;
61
Prepare financial statements that inform management; 62
Explain why gross revenue, value added, contribution, gross
margin, marketing expense, general and administration
expense, etc. might be highlighted in management reporting;
63
Compare and contrast management reports in a range of
organisations including commercial enterprises, charities
and public sector undertakings;
6, 62, 64
Explain why organisations set out financial plans in the
form of budgets, typically for a financial year;
65
Prepare functional budgets for material usage and purchase,
labour and overheads, including budgets for capital
expenditure and depreciation;
68, 69, 70, 73
Prepare a master budget: income statement, balance sheet
and cash flow statement, based on the functional budgets;
66, 67, 71, 74
Interpret budget statements and advise managers on
financing projected cash shortfalls and/or investing pro-
jected cash surpluses;
76
Prepare a flexed budget based on the actual levels of sales
and production and calculate appropriate variances;
75
Compare and contrast fixed and flexed budgets; 72
Explain the use of budgets in designing reward strategies for
managers.
77
FUNDAMENTALS OF MANAGEMENT ACCOUNTING
357
2006.1
PREPARING FOR THE ASSESSMENT
Question 2 Step fixed costs
Which of the following costs would be classified as step costs (tick all that apply)?
(i) The cost of materials is £3 per kg for purchases up to 10,000 kg. From 10,001 kg
to 15,000 kg the cost is £2.80 per kg.
Thereafter the cost is £2.60 per kg.
&
(ii) The cost of supervisory labour is £18,000 per period for output up to 10,000
units. From 10,001 units to 15,000 units the cost is £37,000 per period.
Thereafter the cost is £58,000 per period.
&
(iii) The cost of machine rental is £4,500 per period for output up to 3,000 units.
From 3,001 units to 6,000 units the cost is £8,700 per period.
Thereafter the cost is £12,200 per period.
&
(iv) The mileage charge for a rental car is £0.05 per mile up to 400 miles. From 401
miles to 700 miles the charge is £0.07 per mile.
Thereafter the cost is £0.08 per mile.
&
Question 3 High–low method
The following data relate to the overhead expenditure of a contract cleaners at two activity levels:
Square metres cleaned 12,750 15,100
Overheads £73,950 £83,585
What is the estimate of the overheads if 16,200 square metres are to be cleaned?
(A) £88,095
(B) £89,674
(C) £93,960
(D) £98,095.
Question 4 Cost behaviour patterns
Select the correct equation below.
AG Ltd rents an office photocopier for £300 per month. In addition, the cost incurred
per copy taken is 2 pence. If £ y ¼total photocopying cost for the month and x ¼the
number of photocopies taken, the total photocopying cost for a month can be expressed as:
y ¼300 + 2x &
y ¼300x +2 &
y ¼300 + 0.02x. &
Question 5 Cost object
Which of the following could be used as a cost object in an organisation‘s costing system
(tick all that apply)?
(i) Customer number 879 &
(ii) Department A &
(iii) The finishing process in department A &
REVISION QUESTIONS C1
358
2006.1
PREPARING FOR THE ASSESSMENT
(iv) Product H &
(v) Employee number 776 &
(vi) Order processing activity &
Question 6 Direct cost and indirect cost
Which of the following costs would a local council classify as a direct cost of providing a
door-to-door refuse collection service (tick all that apply)?
(i) Depreciation of the refuse collection vehicle &
(ii) Wages paid to refuse collectors &
(iii) Cost of leaflets sent to customers to advertise refuse collection times and dates &
(iv) Employer’s liability insurance premium to cover all council employees &
Question 7 Full cost
Is the following statement true or false?
‘The only cost that is really useful in setting a selling price for a particular service to be
provided is the full cost’.
True & False &
Question 8 Stock valuation
ABC Ltd had an opening stock value of £880 (275 units valued at £3.20 each) on 1 April.
The following receipts and issues were recorded during April:
Using the FIFO or LIFO method, what was the total value of the issues on 30 April?
Question 9 Stock valuation
The effect of using the last in, first out (LIFO) method of stock valuation rather than the
first in, first out (FIFO) method in a period of rising prices is
(A) to report lower profits and a lower value of closing stock.
(B) to report higher profits and a higher value of closing stock.
(C) to report lower profits and a higher value of closing stock.
(D) to report higher profits and a lower value of closing stock.
8 April Receipts 600 units £3.00 per unit
12 April Issues 200 units
15 April Receipts 400 units £3.40 per unit
30 April Issues 925 units
FIFO LIFO
A £2,850 £2,935
B £2,850 £2,960
C £2,890 £2,935
D £2,890 £2,960
FUNDAMENTALS OF MANAGEMENT ACCOUNTING
359
2006.1
PREPARING FOR THE ASSESSMENT
Question 10 Stock valuation
Is the following statement true or false?
With all average price systems where it is required to keep prices up to date, the average
price must be recalculated each time an issue is made from stock. True & False &
Question 11 Economic value
The R Organisation is experiencing rapid inflation in its raw material prices. Which of the
following inventory valuation methods is most likely to ensure that the prices at which
material issues are charged to cost of production approximate the economic cost of the
materials?
First In, First Out (FIFO) &
Last In, Last Out (LIFO) &
Average cost (AVCO) &
Question 12 Stock valuation methods
The following extract is taken from the stores ledger record for material M:
Date Receipts Issues Balance
September Qty Price £ Qty Price £ Qty Price £
1 12 18.00
3 6 2.10 12.60 18 30.60
7 8 2.35 18.80 26 49.40
12 5 A
14 8 BC
The values that would be entered on the stores ledger record as A, B and C are:
(a) Using FIFO: (b) Using LIFO: (c) Using weighted average (AVCO):
Question 13 Direct cost
Wages paid to which of the following would be classified as direct labour costs of the
organisation’s product or service (tick all that apply):
A driver in a taxi company &
A carpenter in a construction company &
An assistant in a factory canteen &
A hair stylist in a beauty salon. &
REVISION QUESTIONS C1
360
2006.1
PREPARING FOR THE ASSESSMENT
Question 14 Cost attribution
A method of accounting for overheads involves attributing them to cost units using
predetermined rates. This is known as
(A) overhead allocation.
(B) overhead apportionment.
(C) overhead absorption.
(D) overhead analysis.
Question 15 Overhead absorption
A company absorbs overheads on standard machine hours which were budgeted at
11,250 with overheads of £258,750. Actual results were 10,980 standard machine hours
with overheads of £254,692.
Overheads were:
(A) under-absorbed by £2,152.
(B) over-absorbed by £4,058.
(C) under-absorbed by £4,058.
(D) over-absorbed by £2,152.
Question 16 Overhead absorption rates
XX Ltd absorbs overheads based on units produced. In one period, 23,000 units were
produced, actual overheads were £276,000 and there was £46,000 under absorption.
The budgeted overhead absorption rate per unit was:
(A) £10
(B) £12
(C) £13
(D) £14.
Question 17 Overhead absorption
Tick the box to indicate whether the overhead was over- or under-absorbed, and insert the value of the
under or over absorption.
XY operates a standard absorption costing system. Data for last period are as follows:
To the nearest whole number, the overhead for the period was £
under-absorbed
over-absorbed.
Budgeted labour hours 48,500
Actual standard labour hours 49,775
Budgeted overheads £691,125
Actual overheads £746,625
FUNDAMENTALS OF MANAGEMENT ACCOUNTING
361
2006.1
PREPARING FOR THE ASSESSMENT
Question 18 Overhead analysis
TRI-D Ltd has three production departments Extrusion, Machining and Finishing – and
a service department known as Production Services which works for the production
departments in the ratio of 3:2:1.
The following data, which represent normal activity levels, have been budgeted for the
period ending 31 December 20X6:
Requirements
(a) The template being used by the management accountant to analyse the overheads for
the period is shown below:
The values that would be entered on the overhead analysis sheet in the boxes A to
E are:
A
B
C
D
E
(b) After completion of the allocation, apportionment and reapportionment exercise, the
total departmental overheads are:
Extrusion Machining Finishing
£ 206,350 £ 213,730 £ 75,920
Extrusion Machining Finishing
Production
Services Total
Direct labour hours 7,250 9,000 15,000 31,250
Machine hours 15,500 20,000 2,500 2,000 40,000
Floor area (m
2
) 800 1,200 1,000 1,400 4,400
Fixed assets £160,000 £140,000 £30,000 £70,000 £400,000
Employees 40 56 94 50 240
Extrusion Machining Finishing
Production
Services Total
Cost allocated Basis £££££
Indirect wages Allocated 102,000
Apportioned
Depreciation Fixed asset val’n A 84,000
Rates Floor area B 22,000
Power C 180,000
Personnel D 60,000
Other
48,000
109,600
Production
services
E (109,600)
496,000
REVISION QUESTIONS C1
362
2006.1
PREPARING FOR THE ASSESSMENT
Calculate appropriate overhead absorption rates (to two decimal places) for the period
ending 31 December 20X6 and tick the box to indicate in each case whether labour
hours or machine hours are to be used as the absorption basis:
(c) Which of the following are specific order costing systems:
(i) Contract costing &
(ii) Batch costing &
(iii) Process costing &
(iv) Job costing. &
Question 19 Overhead analysis
(a) The management accountant of X Ltd is preparing the budgeted overhead analysis
sheet for the year 20X2/X3. The company has two production cost centres
(Machining and Assembly) and two service departments (Stores and Maintenance).
The directly attributable production overheads have already been allocated to the cost
centres but other costs need to be apportioned. A section of the template being used
by the management accountant and other information are shown below:
Overhead analysis sheet 20X2/3
Other information
The values that would be entered on the overhead analysis sheet in the boxes A, B and
C are:
(i) Extrusion department: £ for each: labour hour &
machine hour &
(ii) Machining department: £
for each: labour hour &
machine hour &
(iii) Finishing department: £
for each: labour hour &
machine hour &
Basis of Machining Assembly Stores Maintenance Total
Costs apportionment ££££ £
Various Allocated 1,105,000 800,000 90,000 350,000 2,345,000
Rent Area occupied A 750,000
Personnel dept B 60,000
Equipment dep’n C 200,000
Departments
Machining Assembly Stores Maintenance
Employees 75 210 25 40
Area occupied (square metres) 10,000 6,000 3,000 1,000
Cost of equipment £ 1,200,000 150,000 50,000 200,000
Machine hours 500,000 50,000
Direct labour hours 30,000 120,000
FUNDAMENTALS OF MANAGEMENT ACCOUNTING
363
2006.1
PREPARING FOR THE ASSESSMENT
(b) When the allocation and apportionment exercise had been completed by the
management accountant, the analysis showed:
Machining Assembly Stores Maintenance Total
££££ £
Total 2,250,000 1,900,000 250,000 800,000 5,200,000
The management accountant has now established the workloads of the service
departments. The service departments provide services to each other as well as to the
production departments as shown below:
Machining Assembly Stores Maintenance
Stores 30% 30% 40%
Maintenance 45% 30% 25%
After the apportionment of the service department overheads to the production
departments (and acknowledging the reciprocal servicing), the total overhead for the
machining department will be £
(to the nearest £000).
Question 20 Elements of cost
Data concerning one unit of product B produced last period are as follows.
Direct material 3kg @ £9 per kg
Direct labour: department A 4 hours @ £14 per hour
department B 6 hours @ £11 per hour
Machine hours: department A 3 hours
department B 2 hours
Production overhead is absorbed at a rate of £7 per direct labour hour in department A
and £6 per machine hour in department B.
(a) The direct cost per unit of product B is £
(b) The full production cost per unit of product B is £
Question 21 Pricing to achieve a specified return on investment
Data for product Q are as follows.
Direct material cost per unit £54
Direct labour cost per unit £87
Direct labour hours per unit 11 hours
Production overhead absorption rate £7 per direct labour hour
Mark-up for non-production overhead costs 3%
10,000 units of product Q are budgeted to be sold each year. Product Q requires an
investment of £220,000 and the target rate of return on investment is 14% per annum.
The selling price for one unit of product Q, to the nearest penny is £
REVISION QUESTIONS C1
364
2006.1
PREPARING FOR THE ASSESSMENT
Question 22 Breakeven analysis
Data for questions 22 and 23
JJ Ltd manufactures a product which has a selling price of £14, a variable cost of £6 per unit.
The company incurs annual fixed costs of £24,400. Annual sales demand is 8,000 units.
New production methods are under consideration, which would cause a 30 per cent
increase in fixed costs and a reduction in variable cost to £5 per unit. The new production
methods would result in a superior product and would enable sales to be increased to
8,500 units per annum at a price of £15 each.
If the change in production methods were to takeplace, the breakeven output level would be:
(A) 122 units higher
(B) 372 units higher
(C) 610 units lower
(D) 915 units higher
Question 23 Breakeven analysis
If the organisation implements the new production methods and wishes to achieve the
same profit as that under the existing method, how many units would need to be produced
and sold annually to achieve this?
(A) 7,132 units
(B) 8,000 units
(C) 8,500 units
(D) 9,710 units
Question 24 Breakeven analysis
X Ltd produces and sells a single product, which has a contribution to sales ratio of 30 per
cent. Fixed costs amount to £120,000 each year.
The number of units of sale required each year to break even:
(A) is 156,000.
(B) is 171,428.
(C) is 400,000.
(D) cannot be calculated from the data supplied.
Question 25 Breakeven graph
The following graph relates to questions 25 and 26
Level of activit
y
Profit
£
0
K
FUNDAMENTALS OF MANAGEMENT ACCOUNTING
365
2006.1
PREPARING FOR THE ASSESSMENT
Point K on the graph indicates the value of:
(A) semi-variable cost.
(B) total cost.
(C) variable cost.
(D) fixed cost.
Question 26 Breakeven graph
This graph is known as a:
(A) conventional breakeven chart.
(B) contribution breakeven chart.
(C) semi-variable-cost chart.
(D) profit–volume chart.
Question 27 Cost analysis
A company makes a single product which generates a contribution to sales ratio of 30 per cent.
In a period when fixed costs were £30,000 the net profit was £56,400. Direct wages are
20 per cent of variable costs.
The direct wages cost for the period was £
.
Question 28 Breakeven analysis
Tick the correct boxes.
A company makes and sells a single product. If the fixed costs incurred in making and
selling the product increase:
Question 29 Cost behaviour/breakeven chart
Z plc operates a single retail outlet selling direct to the public. Profit statements for August
and September are as follows:
Increase Decrease Stay the same
(a) the breakeven point will && &
(b) the contribution to sales ratio will && &
(c) the margin of safety will && &
August September
££
Sales 80,000 90,000
Cost of sales
50,000 55,000
Gross profit 30,000 35,000
Less:
Selling and distribution 8,000 9,000
Administration
15,000 15,000
Net profit
7,000 11,000
REVISION QUESTIONS C1
366
2006.1
PREPARING FOR THE ASSESSMENT
The data for August has been used to draw the following breakeven chart:
Contribution breakeven chart
Activity:
sales value,
£000
£000
80
B
C
D
0
0
A
80
variable cos
t
total cost
sales
Requirements
The values of A–D read from the chart would be:
.
Question 30 Limiting factor
The following budgeted information is available for a company that manufactures four
types of specialist paints:
All four products use the same machine.
In a period when machine hours are in short supply, the product that makes the most
profitable use of machine hours is:
(A) Product W
(B) Product X
(C) Product Y
(D) Product Z.
Product W
per litre
Product X
per litre
Product Y
per litre
Product Z
per litre
££££
Selling price 20.00 15.00 15.00 17.50
Variable overhead 9.60 6.00 9.60 8.50
Fixed overhead
3.60 3.00 2.10 2.10
Profit
6.80 6.00 3.30 6.90
Machine hours per litre 12 9 6 11
FUNDAMENTALS OF MANAGEMENT ACCOUNTING
367
2006.1
PREPARING FOR THE ASSESSMENT
Question 31 Limiting factor decision-making
Triproduct Ltd makes and sells three types of electronic security systems for which the
following information is available:
Standard cost and selling prices per unit
Day scan
Product
Night scan Omni scan
££ £
Materials 70 110 155
Manufacturing labour 40 55 70
Installation labour 24 32 44
Variable overheads 16 20 28
Selling price 250 320 460
Fixed costs for the period are £450,000 and the installation labour, which is highly
skilled, is available for 25,000 hours only in a period and is paid £8 per hour.
Both manufacturing and installation labour are variable costs.
The maximum demand for the products is:
Requirements
(a) The shortfall in hours of installation labour each period is
hours.
(b) In order to maximise profits for the next period, the optimum production plan is:
Question 32 Standard costing
Which of the following are not provided by a system of standard costing and variance analysis?
(A) Unit standard costs as a benchmark for comparison.
(B) Variances to direct managers’ attention where control action will be most worthwhile.
(C) Actual unit costs to be incurred in the future.
(D) Unit standard costs for budgetary planning.
Question 33 Standard cost
JR Limited produces product H. The standard cost card indicates that each unit of H
requires 4 kg of material W and 2 kg of material X at a standard price of £1 and £5 per kg,
respectively.
Standard direct labour hours required per unit are 14 at a standard rate of £8 per hour.
Variable production overheads are absorbed at a rate of £4 per direct labour hour.
The standard variable production cost of one unit of product H is £
.
Day scan 2,000 units
Night scan 3,000 units
Omni scan 1,800 units
Day scan
units
Night scan
units
Omni scan
units.
REVISION QUESTIONS C1
368
2006.1
PREPARING FOR THE ASSESSMENT
Question 34 Variance analysis
Data for questions 34–36
PP Ltd has prepared the following standard cost information for one unit of product X:
Actual results for the period were recorded as follows:
All of the materials were purchased and used during the period.
The direct material price and usage variances are:
Question 35 Variance analysis
The direct labour rate and efficiency variances are:
Question 36 Variance analysis
The variable overhead expenditure and efficiency variances are:
Expenditure Efficiency
(A) £6,750A £1,500A
(B) £6,750A £1,500F
(C) £8,250A £1,500A
(D) £8,250F £1,500F
Direct materials 2 kg @ £13/kg £26.00
Direct labour 3.3 hours @ £4/hour £13.20
Variable overheads 3.3 hours @ £2.50 £8.25
Production 12,000 units
Materials 26,400 kg £336,600
Labour 40,200 hours £168,840
Variable overheads £107,250
Material price Material usage
(A) £6,600F £31,200A
(B) £6,600F £31,200F
(C) £31,200F £6,600A
(D) £31,200A £6,600A
Labour rate Labour efficiency
(A) £8,040A £2,400A
(B) £8,040A £2,400F
(C) £8,040F £2,400A
(D) £8,040F £2,400F
FUNDAMENTALS OF MANAGEMENT ACCOUNTING
369
2006.1
PREPARING FOR THE ASSESSMENT
Question 37 Materials variances
In a period, 11,280 kg of material were used at a total standard cost of £46,248. The
material usage variance was £492 adverse. What was the standard allowed weight of
material for the period?
(A) 11,520 kg
(B) 11,280 kg
(C) 11,394 kg
(D) 11,160 kg.
Question 38 Labour variances
In a period, 6,500 units were made and there was an adverse labour efficiency variance of
£26,000. Workers were paid £8 per hour, total wages were £182,000 and there was a nil
rate variance.
How many standard labour hours were allowed per unit?
(A) 3
(B) 3.5
(C) 4
(D) They cannot be calculated without more information.
Question 39 Variance analysis
During a period, 25,600 labour hours were worked at a standard rate of £7.50 per hour.
The direct labour efficiency variance was £8,250 adverse.
The number of standard hours produced was
.
Question 40 Sales variances
Budgeted sales of product V are 4,800 units per month. The standard selling price and
variable cost of product V are £45 per unit and £22 per unit respectively.
During June the sales revenue achieved from actual sales of 4,390 units of product V
amounted to £231,900.
(a) The sales price variance for product V for June was £
adverse/favourable
(delete as appropriate)
(b) The sales volume contribution variance for product V for June was
£
adverse/favourable (delete as appropriate)
REVISION QUESTIONS C1
370
2006.1
PREPARING FOR THE ASSESSMENT
Question 41 Profit reconciliation
The following variances have been calculated for the latest period:
£
Sales volume contribution variance 11,245 (F)
Material usage variance 6,025 (F)
Labour rate variance 3,100 (A)
Variable overhead expenditure variance 2,415 (A)
All other variances were zero. The budgeted contribution for the period was £48,000.
The actual contribution reported for the period was £
.
Question 42 Variance interpretation I
The direct labour efficiency variance for the latest period was adverse. Which of the
following reasons could have contributed to this variance? (tick all that apply).
(a) Output was higher than budgeted &
(b) The purchasing department bought poor quality material
which was difficult to process &
(c) The original standard time for the output was set too low &
(d) The hourly labour rate was higher than had been expected
when the standard was set &
(e) Employees were more skilled than specified in the standard &
Question 43 Variance interpretation II
The sales volume contribution variance for the latest period was favourable. Which of the
following reasons could have contributed to this variance? (tick all that apply).
(a) A lower selling price was charged than standard &
(b) The variable cost per unit was lower than standard, which led to a higher
actual contribution per unit than standard &
(c) Demand for the product was greater than had been expected &
Question 44 Labour incentive scheme
A company operates a premium bonus system by which employees receive a bonus of
75 per cent of the time saved compared with a standard time allowance (at the normal
hourly rate).
Details relating to employee X are shown below:
Actual hours worked 42
Hourly rate of pay £10
Output achieved 400 units of product Y
Standard time allowed per unit of Y 7 minutes
FUNDAMENTALS OF MANAGEMENT ACCOUNTING
371
2006.1
PREPARING FOR THE ASSESSMENT
The bonus payable to employee X (to the nearest £ ) is:
(A) £35
(B) £47
(C) £70
(D) £82.
Question 45 Integrated accounts
A company operates an integrated cost and financial accounting system. The accounting
entries for the return to stores of unused direct materials from production would be:
Question 46 Integrated accounts
ABC Ltd operates an integrated cost accounting system. The production overhead control
account at 31 July, which is ABC Ltd’s year end, showed the following information:
The £5,000 credit entry represents the value of the transfer to:
(A) the income statement for the under-recovery of production overheads.
(B) the income statement for the over-recovery of production overheads.
(C) the work in progress account for the under-recovery of production overheads.
(D) the following period.
Question 47 Integrated accounts
Wages incurred last period amounted to £33,400, of which £27,400 were direct wages and
£6,000 were indirect production wages. Wages paid in cash were £31,700.
Which of the following entries would arise as a result of these transactions? (Tick all
that are correct.)
Debit Credit
(A) Work in progress account Stores control account
(B) Stores control account Work in progress account
(C) Stores control account Finished goods account
(D) Cost of sales account Work in progress account
Production overhead control account
££
Trade creditors 50,000 Work in progress 120,000
Bank 20,000 ? 5,000
Depreciation 5,000
Salaries 40,000
Materials
10,000
125,000 125,000
REVISION QUESTIONS C1
372
2006.1
PREPARING FOR THE ASSESSMENT
Question 48 Integrated accounting system
WYZ Limited operates an integrated accounting system.
The following information was available for period 7:
At the beginning of the period, the relevant account balances were:
Production overheads are absorbed on the basis of 280 per cent of direct wages cost.
Any production overheads under- or over-absorbed for the period are transferred to the
income statement at the end of the period.
Requirements
ðaÞ
The values that would be entered as A–D in the above account extract are:
£
(a) Debit Wages control account 33,400 &
(b) Debit Wages control account 31,700 &
(c) Debit Work in progress account 27,400 &
(d) Debit Production overhead control account 6,000 &
(e) Credit Wages control account 33,400 &
(f ) Credit Wages control account 31,700 &
(g) Credit Work in progress account 27,400 &
(h) Credit Production overhead control account 6,000 &
£
Cost of finished goods produced 1,241,500
Direct wages 173,400
Direct material issues 598,050
Indirect material issues 32,800
Direct material purchases on credit 617,300
Production overheads (actual expenditure) 359,725
Depreciation of production machinery 35,000
Account £
Work in progress control 125,750
Direct material stores control 48,250
Direct material stores control account (extract)
££
Balance b/f A Work in progress B
Creditors C Production overhead control D
FUNDAMENTALS OF MANAGEMENT ACCOUNTING
373
2006.1
PREPARING FOR THE ASSESSMENT
(b) (i) The production overheads for the period were:
under-absorbed &
over-absorbed &
(ii) The value of the under-/over-absorption was £
(iii) This amount will be transferred as a:
credit &
debit &
in the income statement at the end of the period.
Question 49 Standard cost bookkeeping
A company uses standard costing and an integrated accounting system. The double entry
to record a favourable labour rate variance is:
Question 50 Standard cost bookkeeping
STD Ltd operates an integrated standard costing system for its single product. All stocks
are valued at standard price.
During a period the following variances were recorded:
(a) Tick the correct boxes to show the entries that will be made to record the material
price variance.
(b) Tick the correct boxes to show the entries that will be made to record the material
usage variance.
Debit Credit
(A) Work in progress account Labour rate variance account
(B) Labour rate variance account Work in progress account
(C) Wages control account Labour rate variance account
(D) Labour rate variance account Wages control account
Favourable Adverse
££
Material price 3,950
Material usage 1,925
Labour rate 1,325
Labour efficiency 1,750
Debit Credit
No entry
in this account
Materials control account && &
Material price variance account && &
Work in progress account && &
Debit Credit
No entry
in this account
Materials control account && &
Material usage variance account && &
Work in progress account && &
REVISION QUESTIONS C1
374
2006.1
PREPARING FOR THE ASSESSMENT
(c) The labour force was paid at a:
higher hourly rate than standard &
lower hourly rate than standard &.
(d) Tick the correct boxes to show the entries that will be made to record the labour
efficiency variance.
Debit Credit
No entry
in this account
Wages control account && &
Labour efficiency variance account && &
Work in progress account && &
Question 51 Specific order costing
PZ Ltd undertakes work to repair, maintain and construct roads. When a customer
requests the company to do work, PZ Ltd supplies a fixed price to the customer, and
allocates a works order number to the customer’s request. This works order number is
used as a reference number on all documentation to enable the costs of doing the work to
be collected.
PZ Ltd’s financial year ends on 31 December. At the end of December 20X1, the data
shown against four of PZ Ltd’s works orders were as follows:
The most appropriate costing method for accounting for each of the four works order
numbers is:
Question 52 Job costing
An accountant is to set up in private practice. She anticipates working a 35-hour week and
taking four weeks’ holiday per year. General expenses of the practice are expected to be
£20,000 per year, and she has set herself a target of £30,000 a year salary.
Assuming that only 75 per cent of her time worked will be chargeable to clients, what
should she quote (to the nearest £ ) for a job anticipated to take 50 hours?
(A) £794
(B) £1,831
(C) £1,984
(D) £2,172.
Works order number 488 517 518 519
Date started 1.11.X0 1.10.X1 14.11.X1 20.11.X1
Estimated completion date 28.2.X2 30.7.X2 31.1.X2 31.1.X2
Selling price £450,000 £135,000 £18,000 £9,000
Works order number Contract costing Job costing
(i) 488 &&
(ii) 517 &&
(iii) 518 &&
(iv) 519 &&
FUNDAMENTALS OF MANAGEMENT ACCOUNTING
375
2006.1
PREPARING FOR THE ASSESSMENT
Question 53 Job costing
A company has been asked to quote for a job. The company aims to make a net profit of
30 per cent on sales. The estimated cost for the job is as follows:
Variable production overheads are recovered at the rate of £2 per labour hour.
Fixed production overheads for the company are budgeted to be £100,000 each year and
are recovered on the basis of labour hours. There are 10,000 budgeted labour hours each year.
Other costs in relation to selling, distribution and administration are recovered at the
rate of £50 per job.
The company quote for the job should be:
(A) £572
(B) £637
(C) £700
(D) £833.
Question 54 Job/batch costing
Acme Electronics Ltd makes specialist electronic equipment to order. There are three
main departments: Preparation, Etching and Assembly. Preparation and Etching are
departments which use a considerable amount of machinery while Assembly is mainly a
manual operation using simple hand tools.
For period 7, the following budgets have been prepared:
During the period, an enquiry is received for a batch of 200 control units for which the
following estimates have been made:
Requirements
(a) (i) The prime cost of the batch of 200 control units is £
(ii) The production overhead cost of the batch of 200 control units is:
Preparation Department overheads: £
Etching Department overheads: £
Assembly Department overheads: £
Direct materials 10 kg @ £10 per kg
Direct labour 20 hours @ £5 per hour
Production overheads
Department £ Activity
Preparation 165,000 3,000 machine hours
Etching 98,000 1,400 machine hours
Assembly 48,600 1,800 labour hours
Total direct materials £26,500
Preparation 260 machine hours
90 labour hours at £8 per hour
Etching 84 machine hours
130 labour hours at £7 per hour
Assembly 180 labour hours at £6 per hour
REVISION QUESTIONS C1
376
2006.1
PREPARING FOR THE ASSESSMENT
(b) After an addition has been made to the batch cost to cover administrative overheads,
the total cost of the batch of 200 control units is £65,100.
If the company wishes to achieve a 30 per cent profit margin on sales, the price per
control unit which should be quoted is £
.
Question 55 Specific order costing
SS Developments Ltd is currently converting a former hospital into residential apartments.
The contract commenced on 1 March 20X0 and is expected to take a year and a half to
complete. The contract value is £10 million. SS Developments Ltd’s financial year runs
from 1 January to 31 December.
The contract account for the building of the apartments includes the following data at
31 December 20X0:
Depreciation on plant is to be provided at the rate of 20 per cent on cost each year. The
estimated value of the materials on site at 31 December 20X0 was £180,000.
The company recognises profit on contracts according to the proportion of the total
estimated contract cost that is represented by the cost incurred to date.
Requirements
(a) The total materials cost of the contract to 31 December 20X0 is £
.
(b) The net book value of the plant on site at 31 December 20X0 is £
.
(c) The total cost incurred to date, including the cost of materials and plant
depreciation, is £3,360,000. The profit to be recognised on the contract is
£
.
Question 56 Process costing
Data for sub-questions 56.1–56.3
A company produces a single product that passes through two processes. The details for
process 1 are as follows:
Normal losses are 15 per cent of input in process 1 and without further processing any
losses can be sold as scrap for £1 per kg.
The output for the period was 18,500 kg from process 1.
There was no work in progress at the beginning or at the end of the period.
£000
Materials delivered direct to site 1,200
Materials issued from head office stores 200
Plant delivered to site at cost 900
Estimated costs to completion 2,640
Materials input 20,000 kg at £2.50 per kg
Direct labour £15,000
Production overheads 150 per cent of direct labour
FUNDAMENTALS OF MANAGEMENT ACCOUNTING
377
2006.1
PREPARING FOR THE ASSESSMENT
56.1 What value (to the nearest £ ) will be credited to the process 1 account in respect of
the normal loss?
(A) Nil
(B) £3,000
(C) £4,070
(D) £5,250.
56.2 What is the value (to the nearest £ ) of the abnormal loss/gain for the period in
process 1?
(A) £6,104
(B) £6,563
(C) £7,257
(D) £7,456.
56.3 What is the value (to the nearest £ ) of the output to process 2?
(A) £88,813
(B) £90,604
(C) £91,956
(D) £94,063.
Question 57 Process costing
A cleansing detergent is manufactured by passing raw material through two processes. The
details of the process costs for Process 1 for April were as follows:
Normal loss is 3% of the input during the period and has a scrap value of £0.20 per litre. It
is company policy to deduct the income from the sale of normal loss from that period’s
materials cost.
A template that could be used to calculate the cost of the output from Process 1 is shown
below. The template has been partially completed.
Costs Equivalent litres
Cost per
equivalent litre
OWIP + Period = Total Transfer out + Abnormal loss + CWIP = Total
Materials £2,925 + A 500 £0.75
Conversion = £68,985 = 51,100 £1.35
OWIP = Opening work-in-progress
CWIP = Closing work-in-progress
Opening work-in-progress 5,000 litres valued as follows:
Material cost £2,925
Conversion costs £6,600
Raw material input 50,000 litres valued at a cost of £37,500
Conversion costs £62,385
Actual output to Process 2 49,000 litres
Closing work-in-progress 4,000 litres, which were 100% complete for
materials and 40% complete for conversion costs.
REVISION QUESTIONS C1
378
2006.1
PREPARING FOR THE ASSESSMENT
(a) The value to be inserted in the table at A is £
(b) The total value of the transfers to process 2 is £
(c) The value of the abnormal loss is £
(d) The value of the closing work in progress is £ .
Question 58 Process costing
Industrial Solvents Ltd mixes together three chemicals A, B and C in the ratio 3:2:1 to
produce Allklean, a specialised anti-static fluid. The chemicals cost £8, £6 and £3.90 per
litre, respectively.
In a period, 12,000 litres in total were input to the mixing process. The normal process
loss is 5 per cent of input and in the period there was an abnormal loss of 100 litres, while
the completed production was 9,500 litres. There was no opening work in progress (WIP)
and the closing WIP was 100 per cent complete for materials and 40 per cent complete for
labour and overheads. Labour and overheads were £41,280 in total for the period.
Materials lost in production are scrapped.
Requirements
(a) The number of equivalent litres of labour and overhead produced during the period
was
equivalent litres.
(b) The cost per equivalent litre of materials produced was £
.
Question 59 Process costing
A company manufactures a variety of liquids which pass through a number of processes.
One of these products, P, passes through processes 1, 2 and 3 before being transferred to
the finished goods warehouse.
The following process 3 data is available for October:
£
Work in process at 1 October is 6,000 units, valued as:
Transfer from process 2 14,400
Materials added 2,160
Wages and overhead
2,880
19,440
Transfer from process 2 during October:
48,000 units 110,400
Transferred to finished goods: 46,500 units
Costs incurred:
Materials added 27,180
Wages and overhead 54,720
Work in process at 31 October: 4,000 units
Degree of completion:
Materials added: 50%
Wages and overhead: 30%
Normal loss in process: 6% of units in opening stock plus transfers from
process 2 less closing stock
FUNDAMENTALS OF MANAGEMENT ACCOUNTING
379
2006.1
PREPARING FOR THE ASSESSMENT
At a certain stage in the process, it is convenient for the quality control inspector to
examine the product and, where necessary, to reject it. Rejected products are sold for
£0.80 per unit. During October an actual loss of 7 per cent was incurred, with product P
having reached the following stage of production:
Direct materials added: 80%
Wages and overhead: 60%
Requirements
The cost per equivalent unit produced was:
Question 60 Subjective classification
Tick the relevant box to indicate whether the following statement is true or false.
‘The classification of expenditure items according to the cost centre or cost unit to be
charged is known as subjective classification’.
True & False &
Question 61 Code number
In the composite code number 544.221 the first three digits indicate the nature of the
expenditure and the last three digits indicate the cost object to be charged with the
expenditure.
Delete as appropriate in the following statements.
(a) 544 indicates the objective/subjective classification of the expenditure item.
(b) 221 indicates the objective/subjective classification of the expenditure item.
Question 62 Managerial reporting in a service organisation
Speedee Ltd has three main divisions a motor-cycle courier service, a domestic parcel
delivery service, and a bulk parcel service for industry.
The following information is available for a period:
Variable costs vary both with the distance travelled and also the type of vehicle used,
and are £307,800 for the company as a whole. A technical estimate shows that the various
vehicles used for the three services incur variable costs per kilometre in the ratio of 1:3:5,
respectively, for the courier service, domestic parcels and bulk parcels.
(a) process 2 input: £
per equivalent unit
(b) material added: £
per equivalent unit
(c) wages and overhead: £
per equivalent unit.
Courier
service
Domestic
parcels
Bulk
parcels
Sales (£000) 205 316 262
Distance travelled (000 km) 168 82 54
REVISION QUESTIONS C1
380
2006.1
PREPARING FOR THE ASSESSMENT
Requirements
The contribution for each service for the period is:
(a) courier service: £
(b) domestic parcels: £
(c) bulk parcels: £ .
Question 63 Value added
An extract from the performance report of the F Division for the latest period is as follows.
££
Sales revenue 289,500
Cost of goods sold
Material costs 89,790
Labour costs 72,340
Production overhead
54,030
216,160
Gross margin 73,340
Marketing overhead 21,890
General and administration overhead
38,120
60,010
Net profit
13,330
The following salary costs are included within the overhead costs.
Salary cost included
Production overhead £10,710
Marketing overhead £14,560
General and administration overhead £21,330
For the F Division for the latest period, the value added was £
Question 64 Managerial reporting in a charity
As part of its fundraising and awareness-raising activities a charity operates a number of
retail shops, selling new and donated second-hand goods.
Data for the latest period for the Southmere shop are as follows.
£
Sales income
New goods 6,790
Donated goods sold to customers 4,880
Purchase cost of new goods 3,332
Cost of laundering and cleaning selected donated goods 120
Delivery cost paid for new goods 290
Other income: low quality donated goods sold for recycling 88
Salary costs 810
Amount paid to valuer to assess selected donated items 30
General overhead costs 1,220
FUNDAMENTALS OF MANAGEMENT ACCOUNTING
381
2006.1
PREPARING FOR THE ASSESSMENT
(a) The gross margin generated by second-hand donated goods sold was £
(b) The gross margin generated by new goods sold was £
Question 65 The role of budgets
Which of the following is not a main role of a budget?
(A) A budget gives authority to budget managers to incur expenditure in their area of
responsibility.
(B) A budget provides a means for an organisation to expand its activities.
(C) A budget coordinates the activities of various parts of the organisation.
(D) A budget acts as a comparator for current performance.
Question 66 Principal budget factor
A principal budget factor is:
(A) the highest value item of cost.
(B) a factor which limits the activities of an undertaking.
(C) a factor common to all budget centres.
(D) a factor controllable by the manager of the budget centre.
Question 67 IT in the budget process
Which of the following are benefits of using a computerised budget system as opposed to
a manual one (tick all that are correct):
(a) & data used in drawing up the budget can be processed more quickly.
(b) & budget targets will be more acceptable to the managers responsible for their
achievement.
(c) & changes in variables can be incorporated into the budget more quickly.
(d) & the principal budget factor can be identified before budget preparation begins.
(e) & continuous budgeting is only possible using a computerised system.
Question 68 Production budget
AB Ltd is currently preparing its production budget for product Z for the forthcoming year.
The sales director has confirmed that he requires 120,000 units of product Z. Opening stock
is estimated to be 13,000 units and the company wishes to reduce stock at the end of the year
by 50 per cent. How many units of product Z will need to be produced?
(A) 113,500 units
(B) 120,000 units
(C) 126,500 units
(D) 133,000 units.
REVISION QUESTIONS C1
382
2006.1
PREPARING FOR THE ASSESSMENT
Question 69 Material budget
A company is currently preparing a material usage budget for the forthcoming year for
material Z that will be used in product XX. The production director has confirmed that
the production budget for product XX will be 10,000 units.
Each unit of product XX requires 4 kg of material Z. Opening stock of material Z is budgeted
to be 3,000 kg and the company wishes to reduce stock at the end of the year by 25 per cent.
What is the usage budget for material Z for the forthcoming year?
(A) 34,750 kg
(B) 39,250 kg
(C) 40,000 kg
(D) 40,750 kg.
Question 70 Functional budgets
Budgeted sales of product P for next month are 4,000 units. Each unit of P requires 2 kg
of raw material. Other budget information for next month is as follows:
The budgeted purchases of raw material for next month should be:
(A) 8,000 kg
(B) 8,300 kg
(C) 9,500 kg
(D) 12,500 kg.
Question 71 Cash budget
The following details have been extracted from the debtor collection records of X Ltd:
Credit sales for June to August are budgeted as follows:
Raw materials
Opening stocks 3,000 kg
Closing stocks 4,500 kg
Finished product P
Opening stocks 2,400 units
Closing stocks 1,800 units
Invoices paid in the month after sale 60%
Invoices paid in the second month after sale 20%
Invoices paid in the third month after sale 15%
Bad debts 5%
June £100,000
July £150,000
August £130,000
FUNDAMENTALS OF MANAGEMENT ACCOUNTING
383
2006.1
PREPARING FOR THE ASSESSMENT
Customers paying in the month after sale are entitled to deduct a 2 per cent settlement
discount. Invoices are issued on the last day of the month. The amount budgeted to be
received in September from credit sales is
(A) £115,190
(B) £116,750
(C) £121,440
(D) £123,000.
Question 72 Budgetary control
Tick the correct box.
A budget which is designed to show the allowed expenditure for the actual level of activity
achieved is known as
a rolling budget &
a flexible budget &
a fixed budget &
Question 73 Functional budgets
RD Ltd is in the process of preparing its budgets for 20X2. The company produces and
sells a single product, Z, which currently has a selling price of £100 for each unit.
The budgeted sales units for 20X2 are expected to be as follows:
The company expects to sell 7,000 units in January 20X3.
The selling price for each unit will be increased by 15 per cent with effect from 1 March 20X2.
A total of 1,000 units of finished goods are expected to be in stock at the end of 20X1.
It is company policy to hold a closing stock balance of finished goods equal to 20 per cent
of the following month’s sales.
Each unit of Z produced requires 3 kg of material X, which currently costs £5 per kg.
This price is expected to increase by 10 per cent on 1 June 20X2.
Stock of raw material at the end of 20X1 is expected to be 3,750 kg. The company
wishes to avoid any stock-outs and requires the closing stock of raw materials to be set at
20 per cent of the following month’s production requirements.
The production of each unit of Z requires 4 hours of skilled labour and 2 hours of
unskilled labour.
Requirements
(a) The sales budget for quarter 1 is £
.
(b) The production budget for quarter 4 is
units.
(c) The material usage budget for quarter 2 is
kg.
(d) The material purchase budget for quarter 1 is £
.
(e) The direct labour budget for quarter 3 is
hours.
JFMAMJ JASOND
5,000 5,500 6,000 6,000 6,250 6,500 6,250 7,000 7,500 7,750 8,000 7,500
REVISION QUESTIONS C1
384
2006.1
PREPARING FOR THE ASSESSMENT
Question 74 Cash budget
The following data and estimates are available for ABC Limited for June, July and August:
The following information is available regarding direct materials:
Notes
1. Ten per cent of sales are for cash: the balance is received the following month.
2. Wages are paid in the month in which they are incurred.
3. Overheads include £1,500 per month for depreciation. Overheads are settled in the
month following.
4. Purchases of direct materials are paid for in the month purchased.
Requirements
(a) The budget value of direct materials purchases is:
June: £
July: £
August: £ .
(b) The budgeted cash receivable from customers in August is £
.
(c) The budgeted cash payable for wages and overhead in July is £
.
Question 75 Flexible budgets
S Ltd makes a single product for which the budgeted costs and activity for a typical month
are as follows:
Budgeted production and sales 15,000 units
Budgeted unit costs £
Direct labour 46
Direct materials 30
Variable overheads 24
Fixed overheads
80
180
The standard selling price of the product is £220 per unit.
June July August
£££
Sales 45,000 50,000 60,000
Wages 12,000 13,000 14,500
Overheads 8,500 9,500 9,000
June July August September
££ £ £
Opening stock 5,000 3,500 6,000 4,000
Material usage 8,000 9,000 10,000
FUNDAMENTALS OF MANAGEMENT ACCOUNTING
385
2006.1
PREPARING FOR THE ASSESSMENT
Requirements
(a) During October, only 13,600 units were produced. The total budget cost allowance
contained in the flexed budget for October is £
.
(b) During November, 14,500 units were produced and sold at the standard selling price,
and the following actual costs were incurred:
£
Direct labour 658,000
Direct materials 481,400
Variable overheads 334,600
Fixed overheads
1,340,000
2,814,000
(i) The sales volume contribution variance for November was £
adverse &
favourable &.
(ii) The total expenditure variance for November was £
adverse &
favourable &.
Question 76 Interpreting cash budgets
CB Ltd’s cash budget forewarns of a short-term cash deficit. Which of the following
would be appropriate actions to take in this situation (tick all that apply)?
(a) Arrange a bank overdraft &
(b) Reduce receivables (debtors) &
(c) Increase inventories (stocks) &
(d) Sell more shares in the company &
Question 77 Using budgets in management reward strategies
The maintenance manager is paid a bonus of 5 per cent of any savings he achieves against
the flexible budget cost allowance for maintenance costs in the period.
The flexible budget cost allowance for the latest two periods was as follows.
Activity level Budget cost allowance
Machine hours £
Period 4 3,800 11,040
Period 5 4,320 11,976
During period 6 the actual maintenance expenditure was £10,990 and 4,090 machine hours
were worked.
The bonus paid to the maintenance manager in period 6 is (to the nearest penny)
£
.
REVISION QUESTIONS C1
386
2006.1
PREPARING FOR THE ASSESSMENT
Solutions to
Revision Questions
Solution 1
Cost type X is clearly fixed for this range of activity levels.
For the other costs, divide the total cost by the number of units at the head of the
column. Variable costs result in a constant amount per unit of output. Semi-variable
costs result in a varying amount per unit of output.
Answer: (B)
Cost type
Cost per unit
@ 100 units
Cost per unit
@ 140 units
££
W 80 75.43 semi-variable
Y 65 65.00 variable
Z 67 61.29 semi-variable
Solution 2
It might help to actually draw a rough sketch graph of each cost described. Then tick the
boxes of the costs whose behaviour patterns resemble a flight of stairs.
Costs (ii) and (iii) are step costs. The total expenditure on these costs remains constant for
a range of activity levels until a critical activity level is reached. At this point the cost
increases to a new level and then remains constant for a further range of activity levels.
Costs (i) and (iv) are non-linear variable costs. The gradient of a graph of cost (i) will
become less steep as activity increases. The gradient of a graph of cost (iv) will become
more steep as activity increases.
Solution 3
You need to be completely familiar with the high–low method. The data shows that the
technique can be applied in service situations as well as in manufacturing.
Answer: (A)
1. Find the variable overheads per square metre:
Extra m
2
cleaned ¼15,100 12,750 ¼2,350
Extra overhead cost ¼£83,585 £ 73,950 ¼£9,635
Variable overhead per m
2
¼£9,635/2,350 ¼£4.10
387 2006.1
2. Find the fixed overhead:
£
Total overheads of cleaning 12,750 m
2
¼ 73,950
Variable overheads ¼12,750 £4.10 ¼
52,275
Fixed overhead ¼
21,675
3. Total overheads for 16,200 m
2
:
£
Variable overhead ¼16,200 £4.10 ¼ 66,420
Fixed overhead ¼
21,675
88,095
Solution 4
Do not be confused by the use of the y and x notation. You simply need to think
through how to calculate the total cost of a semi-variable cost.
Total semi-variable cost ¼fixed cost + (variable cost per unit no. of units)
Answer: y ¼300 + 0.02x
Solution 5
An organisation’s costing system is designed to provide the basic cost information that
managers need in order to make operational and strategic planning and control
decisions. For this purpose they may need to know the cost of a wide variety of items,
ranging from a particular product or service to an individual customer.
All of the items described could be used as a cost object.
The CIMA Terminology provides the following description of a cost object: ‘for example
a product, service, centre, activity, customer or distribution channel in relation to which
costs are ascertained’.
Solution 6
A direct cost is a cost which can be specifically attributed to a single cost object without
the need for any potentially arbitrary apportionments.
Costs (i), (ii) and (iii) are direct costs of the service because they can be specifically
attributed to the service provided.
Cost (iv) is an indirect cost of the service because it applies to all council employees, not
only to those who are providing the refuse collection service.
SOLUTIONS TO REVISION QUESTIONS C1
388
PREPARING FOR THE ASSESSMENT
2006.1
Solution 7
Think carefully before you answer a true/false question like this. For a statement to be
true it must apply in all circumstances.
The statement is false.
Although the full cost, which includes absorbed overhead, shows the long run average
cost that will be incurred per unit of service provided, it might be necessary to consider the
marginal or incremental cost when making a special, one-off pricing decision.
Solution 8
You will need to produce some clear workings for your own benefit, since workings will
not be awarded marks. You might like to draw up full stock records to ensure greater
accuracy.
Answer: (D)
Under FIFO the 200 units issued on 12 April would have been priced at £3.20 from the
opening stock. Therefore the remaining 75 units from the opening stock make up the first
part of the batch issued on 30 April:
£
75 units at £3.20 240
600 units at £3.00 1,800
250 units at £3.40 850
925 2,890 (FIFO)
Under LIFO the 200 units issued on 12 April would have been priced at £3.00 from
the latest batch in stock at that date (received 8 April). Therefore, the 400 units
remaining from the £3.00 batch received on 8 April will be a part of the issues made
on 30 April:
£
400 units at £3.40 1,360
400 units at £3.00 1,200
125 units at £3.20 400
925 2,960 (LIFO)
Solution 9
Read the question carefully to ascertain whether prices are rising or falling
Answer: (A)
The issues to production will have been charged at the most recent, higher prices–lower
profit figure.
The remaining stock will be valued at the earlier, cheaper prices–lower stock value.
FUNDAMENTALS OF MANAGEMENT ACCOUNTING
389
PREPARING FOR THE ASSESSMENT
2006.1
Solution 10
Think before you answer! An issue of stock at the average price will not alter the average
price of the items remaining in stock.
False. The average price must be recalculated each time a purchase is made at a
different price.
Solution 11
You should appreciate that, although the LIFO method results in issues from stores
being valued at the most recent prices paid, the costs used are still historical costs.
Since the LIFO method uses the most recent prices to value issues from stock it is most
likely to ensure that the prices charged to cost of production approximate the economic
cost of the materials.
Solution 12
Remember the need for total accuracy. Prepare as many workings as you need to help
you, but note that workings will not be awarded any marks. Our workings will help you
to detect any errors you may have made.
£
(a) A 5 units £1.50 (from opening stock)
7.50
B 7 units £1.50 (remaining units from opening stock) 10.50
1 unit £2.10 (from September 3 receipts) 2.10
8 12.60
C 5 units £2.10 (remaining units from September 3) 10.50
8 units £2.35 (September 7 receipts)
18.80
29.30
£
(b) A 5 units £2.35 (from September 7 receipts)
11.75
B 3 units £2.35 (remaining units from September 7) 7.05
5 units £2.10 (from September 3 receipts) 10.50
8 17.55
C 1 unit £2.10 (remaining unit from September 3) 2.10
12 units £1.50 (opening stock)
18.00
20.10
(c) Weighted average price of stock on 7 September ¼ £
49:40
26
¼ £1:90
A 5 units £1.90 ¼£9.50
B 8 units £1.90 ¼£15.20
C 13 units £1.90 ¼£24.70.
SOLUTIONS TO REVISION QUESTIONS C1
390
PREPARING FOR THE ASSESSMENT
2006.1
Solution 13
Direct wages are those paid to employees working directly on the organisation’s output.
Their wages can be traced to specific cost units.
The wages paid to the driver, carpenter and hair stylist are all direct labour costs. The
wages paid to the canteen assistant are indirect wages because the assistant is not working
directly on the organisation’s output.
Solution 14
Take your time and read all the options. This is not a difficult question but it would be
easy to rush and select the wrong answer.
Answer: (C)
Overhead allocation is the allotment of whole items of cost to cost units or cost
centres. Overhead apportionment is the sharing out of costs over a number of cost centres
according to the benefit used. Overhead analysis refers to the whole process of recording
and accounting for overheads.
Solution 15
First you need to calculate the overhead absorption rate per standard machine hour.
Remember that this is always based on the budgeted data.
Next you must use the absorption rate to calculate the overhead absorbed, and then
compare this with the overhead incurred to determine the over or under absorption.
Answer: (A)
Overhead absorption rate ¼
£258,750
11,250
¼ £23 per standard machine hour
£
Overhead absorbed ¼10,980 std. hours £23 252,540
Overhead incurred
254,692
Under absorption
2,152
Solution 16
Use the under absorption to adjust the actual overhead incurred, to determine the
overhead absorbed. Since there was an under absorption, the actual overhead incurred
must be greater than the overhead absorbed.
Lastly, divide the overhead absorbed by the number of units produced.
FUNDAMENTALS OF MANAGEMENT ACCOUNTING
391
PREPARING FOR THE ASSESSMENT
2006.1
Answer: (A)
£000
Actual incurred 276
Under absorption
46
Absorbed
230
No. of units 23,000
Rate per unit ¼£230,000/23,000 ¼£10.
Solution 17
Remember that the overhead absorption rate (OAR) is based on the budgeted data.
Overheads absorbed for the period ¼OAR actual standard labour hours achieved.
OAR ¼
£691,125
48,500
¼ £14:25
£
Overhead absorbed during period
49,775 £14.25 709,293.75
Overhead incurred
746,625.00
Overhead under-absorbed
37,331 (to nearest whole number).
Solution 18
The basic data on labour and machine hours seems to indicate that the Extrusion and
Machining departments are machine-intensive, so a machine hour rate would be most
appropriate. The Finishing department appears to be labour-intensive, so a labour hour
rate would be more suitable.
(a) A:
£140,000
£400,000
£84,000 ¼ £29,400
B:
800
4,400
£22,000 ¼ £4,000
C:
2,500
40,000
£180,000 ¼ £11,250
D:
50
240
£60,000 ¼ £12,500
E:
3
ð3 þ2 þ 1Þ
£109,600 ¼ £54,800:
SOLUTIONS TO REVISION QUESTIONS C1
392
PREPARING FOR THE ASSESSMENT
2006.1
(b) Overhead absorption rates
Extrusion
£206,350
15,500 machine hours
¼ £13:31 for each machine hour
Machining
£213,730
20,000 machine hours
¼ £10:69 for each machine hour
Finishing
£75,920
15,000 labour hours
¼ £5:06 for each labour hour:
(c) (i) Contract costing
(ii) Batch costing
(iv) Job costing
Solution 19
You will need a systematic approach in part (b), with neat workings to help you achieve
100 per cent accuracy.
(b) £2,850,000 (to the nearest 000)
Workings:
Machining Assembly Stores Maintenance
£££ £
Initial allocation 2,250,000 1,900,000 250,000 800,000
Apportion stores 75,000 75,000 (250,000) 100,000
Apportion maintenance 405,000 270,000 225,000 (900,000)
Apportion stores 67,500 67,500 (225,000) 90,000
Apportion maintenance 40,500 27,000 22,500 (90,000)
Apportion stores 6,750 6,750 (22,500) 9,000
Apportion maintenance 4,050 2,700 2,250 (9,000)
Apportion stores 675 675 (2,250) 900
Apportion maintenance 405 270 225 (900)
Apportion stores 68 67 (225) 90
Apportion maintenance
40 27 23 (90)
Total apportioned
2,849,988
(a) A: Rent cost apportioned to Assembly Department ¼
6,000
20,000
£750,000 ¼£225,000
B: Personnel cost apportioned to Assembly Department ¼
210
350
£60,000 ¼£36,000
C: Using an apportionment basis of cost of equipment, depreciation cost
apportioned to Machining Department ¼
1,200,000
1,600,000
£200,000 ¼ £150,000
FUNDAMENTALS OF MANAGEMENT ACCOUNTING
393
PREPARING FOR THE ASSESSMENT
2006.1
Solution 20
The full production cost includes production overheads absorbed using the
predetermined rates provided in the question
(a) The direct cost per unit of product B is £149
(b) The full production cost per unit of product B is £189
Working £ £
Direct material (3kg £9) 27
Direct labour
Department A (4 hours £14) 56
Department B (6 hours £11)
66
122
Total direct cost 149
Production overhead
Department A (4 labour hours £7) 28
Department B (2 machine hours £6)
12
40
Full production cost
189
Solution 21
Remember to add on the target return for one unit of Q and not the total required
return from all 10,000 units.
The selling price for one unit of product Q, to the nearest penny, is £227.62
£ per unit
Direct material cost 54.00
Direct labour cost
87.00
Total direct cost 141.00
Production overhead absorbed ¼11 hours £7
77.00
Total production cost 218.00
Mark-up for non-production costs ¼3% £218.00
6.54
Full cost 224.54
Profit mark-up (see working)
3.08
Selling price
227.62
Working:
Target return on investment in product Q ¼£220,000 14% ¼£30,800
Target return per unit of product Q ¼£30,800/10,000 units ¼£3.08.
SOLUTIONS TO REVISION QUESTIONS C1
394
PREPARING FOR THE ASSESSMENT
2006.1
Solution 22
Calculate the breakeven point before and after the change in production methods, using
the formula:
Breakeven point in units ¼
Fixed costs
Contribution per unit
Answer: (A)
Existing situation:
Breakeven point ¼
£24,400
£8
¼ 3,050 units
Working:
New production methods:
Breakeven point ¼
£ 24,400 1:3
£10
¼ 3,172 units
Working:
Increase in number of units: 3,172 3,050 ¼122.
Solution 23
First calculate the existing profit level.
Using the new cost and selling price, calculate the required sales volume using the
formula:
Required sales volume ¼
ðFixed costs þrequired profitÞ
Contribution per unit
Answer: (A)
£31,720 þ£39,600
£15 £5
¼ 7,132 units
(Working for existing profit: 8,000 units £8 ¼£64,000 contribution less fixed costs
£24,400 ¼£39,600.)
Contribution per unit £
Selling price 14
Variable cost
(6)
Contribution
8
Contribution per unit £
Selling price 15
Variable cost
(5)
Contribution
10
FUNDAMENTALS OF MANAGEMENT ACCOUNTING
395
PREPARING FOR THE ASSESSMENT
2006.1
Solution 24
Do not rush this question. You can probably easily calculate the breakeven point in
terms of sales value, but then you will need to stop and think carefully.
Answer: (D)
Breakeven point in terms of sales value ¼
£120,000
0:3
¼ £400,000:
This must now be divided by the selling price.
The breakeven point in terms of units cannot be derived because we do not know the
unit selling price.
Solution 25
The single line drawn on the graph represents profits or losses earned for a range of
activity levels.
Answer: (D)
Point K indicates the loss incurred at zero activity. At this point, the loss incurred is equal
to the fixed cost.
Solution 26
Profit–volume chart is the name given to a graph which indicates the profits or losses
earned for a range of activity levels.
Answer: (D)
Charts A and B would include lines for costs and revenues. Chart C would be depicted by
a single line, starting at a point above the origin on the vertical axis. This point represents
the total fixed cost incurred at zero activity.
Solution 27
Remember that contribution for a period is equal to the fixed costs plus the profit for
the period.
Once you have calculated the contribution you can use the C/S ratio to derive the sales
value, and that will lead you to the variable costs and thus the direct wages for the period.
Contribution ¼ £30,000 þ£56,400 ¼ £86,400
Contribution
Sales
¼ 0:3
Sales ¼
£86,400
0:3
¼ £288,000
Variable costs ¼ sales value contribution ¼ £288,000 £86,400 ¼ £201,600
Direct wages ¼ 20% £201,600 ¼ £40,320:
SOLUTIONS TO REVISION QUESTIONS C1
396
PREPARING FOR THE ASSESSMENT
2006.1
Solution 28
For a given level of sales, the margin of safety and the breakeven point will always move
in the ‘opposite direction’ to each other. If one increases then the other decreases.
The contribution to sales ratio is not affected by the level of fixed costs incurred.
(a) The breakeven point will increase.
(b) The contribution to sales ratio will stay the same.
(c) The margin of safety will decrease.
Solution 29
You will need to use the contribution to sales (C/S) ratio in this question, in calculating
the breakeven sales value. Once you have calculated the variable costs as a percentage of
sales value you should be able to use this to determine the C/S ratio.
The contribution breakeven chart that has been drawn shows the variable cost line
instead of the fixed cost line. This means that contribution can be read directly from the
chart.
Workings:
August September Change
£££
Sales 80,000 90,000 10,000
Cost of sales 50,000 55,000 5,000
Selling and distribution 8,000 9,000 1,000
Administration 15,000 15,000 nil
(i) Cost of sales:
Variable £5,000/£10,000 ¼50p/£1 of sales (50% of sales)
Fixed £50,000 (50% £80,000) ¼£10,000
(ii) Selling and distribution:
Fixed nil
Variable £1,000/£10,000 ¼10p/£1 of sales (10% of sales)
(iii) Administration:
Fixed £15,000
Variable nil
A: Breakeven sales value ¼
Fixed costs
C/S ratio
¼
£25,000
0:4
¼ £ 62,500
* Variable costs have been calculated to be 60 per cent of sales. Therefore, the C/S
ratio is 40 per cent.
B: Total cost of £80,000 sales value ¼£73,000 (from original data).
C: Total variable cost for £80,000 sales value ¼£80,000 0.6 ¼£48,000.
D: Total fixed cost ¼cost of sales £10,000 + administration £15,000 ¼£25,000.
FUNDAMENTALS OF MANAGEMENT ACCOUNTING
397
PREPARING FOR THE ASSESSMENT
2006.1
Solution 30
The products must be ranked in order of their contribution per machine hour used.
Answer: (B)
Product W Product X Product Y Product Z
££ ££
Contribution per litre 10.40 9.00 5.40 9.00
Contribution per
machine hour
10:40=12 9:00=95:40=69:00=11
¼ 0.8667 1.0000 0.9000 0.8182
Ranking 3rd 1st 2nd 4th
Solution 31
The best production plan in part (b) is that which will maximise the contribution from
the installation labour. The products must therefore be ranked in order of their
contribution per hour.
(a) 2,900 hours.
Workings:
Hours of installation labour required to satisfy maximum demand
Hours
Day scan*: 2,000 units 3 hours/unit 6,000
Night scan: 3,000 units 4 hours/unit 12,000
Omni scan: 1,800 units 5.5 hours/unit
9,900
27,900
Available hours
25,000
Shortfall
2,900
* Hours of installation labour for Day scan ¼
£24
£8
¼ 3 hours.
(b) Day scan 2,000 units
Night scan 2,275 units
Omni scan 1,800 units
SOLUTIONS TO REVISION QUESTIONS C1
398
PREPARING FOR THE ASSESSMENT
2006.1
Workings:
Day scan Night scan Omni scan
££ £
Selling price 250 320 460
Variable costs
Material (70) (110) (155)
Manufacturing labour (40) (55) (70)
Installation labour (24) (32) (44)
Variable overheads
(16) (20) (28)
Contribution per unit
100 103 163
Installation hours required 3 4 5.5
Contribution per installation hour £33.33 £25.75 £29.64
Production priority 1st 3rd 2nd
Best production plan
Units Hours used
Day scan to maximum demand 2,000 ( 3.0) 6,000
Omni scan to maximum demand 1,800 ( 5.5) 9,900
This leaves (25,000 6,000 9,900) ¼9,100 installation labour hours for Night scan.
Therefore, production of Night scan ¼
9,100
4
¼ 2,275 units.
Solution 32
Answer: (C)
Although standard costs are based on estimates of what might happen in the future, a
standard costing system does not provide actual future costs.
Solution 33
This is a straightforward exercise in accumulating costs using the data provided.
The standard variable production cost of one unit of product H is £182
£ per unit
Direct material W (4 kg £1) 4
Direct material X (2 kg £5) 10
Direct labour (14 £8) 112
Variable Production overhead (14 £4)
56
Total variable production cost
182
FUNDAMENTALS OF MANAGEMENT ACCOUNTING
399
PREPARING FOR THE ASSESSMENT
2006.1
Solution 34
Remember that all the ‘quantity’ variances (material usage, labour efficiency and variable
overhead efficiency) are valued at the standard rate rather than at the actual rate.
Therefore the material usage variance in kg should be multiplied by the standard price
per kg to determine the monetary value of the material usage variance.
Answer: (A)
Material price variance £
Standard cost of materials used 26,400 kg £13 343,200
Actual cost
336,600
6,600 F
Material usage variance Kg
Standard usage 12,000 units 2 kg 24,000
Actual usage
26,400
2,400 £13/kg ¼£31,200 A
Solution 35
Once again, remember to use the standard rate per hour to evaluate the labour efficiency
variance.
Answer: (A)
Labour rate variance £
Standard cost of hours used 40,200 £4 160,800
Actual labour cost
168,840
8,040 A
Labour efficiency variance Hours
Standard time 12,000 units 3.3 hours 39,600
Actual time
40,200
600 £4/hour ¼£2,400 A
Solution 36
Remember that the variable overhead efficiency variance is directly related to the labour
efficiency variance
Answer: (A)
Variable overhead expenditure variance £
40,200 hours of variable overhead should cost ( £2.50) 100,500
But did cost
107,250
6,750 adverse
SOLUTIONS TO REVISION QUESTIONS C1
400
PREPARING FOR THE ASSESSMENT
2006.1
Variable overhead efficiency variance
Efficiency variance in hours, from labour efficiency variance
standard variable overhead rate per hour
600
£2.50
adverse
£1,500 adverse
Solution 37
The usage must have been higher than standard because the usage variance is adverse.
Remember that the usage variance is equal to the excess usage multiplied by the
standard price per kg of material.
Answer: (D)
Standard price per kilogram of material:
£46,248
11,280
¼ £4:10
Number of kilograms excess usage:
£492
£4:10
¼ 120 kg:
Standard usage: 11,280 kg 120 kg ¼11,160 kg.
Solution 38
The adverse efficiency variance means that the actual time taken was higher than the
standard allowance.
Notice that there was a nil rate variance. This means that the actual rate per hour was the
same as the standard rate per hour.
There are a number of ways of calculating the correct solution. You might have used a
different method it does not matter as long as you arrive at the correct answer!
Answer: (A)
Excess hours above standard time ¼efficiency variance standard rate per hour
¼
£26,000
8
¼3,250 hours
Actual hours worked ¼
£ 182,000
£8
¼22,750 hours
Standard hours for actual output ¼22,750 3,250
¼19,500 hours
Standard hours for one unit ¼
19,500
6,500
¼3 hours.
Solution 39
‘Backwards’ variance questions are a good way of testing whether you really understand
the logic of the variance calculations.
If you got this question wrong, go back and study variance analysis again to ensure that
you can calculate all the required variances quickly and accurately.
Actual labour hours worked 25,600
Adverse efficiency variance in hours
£8,250
£7:50
1,100
Standard hours expected for production achieved
24,500
FUNDAMENTALS OF MANAGEMENT ACCOUNTING
401
PREPARING FOR THE ASSESSMENT
2006.1
Solution 40
Remember that the sales volume contribution variance is evaluated using the standard
contribution per unit.
(a) The sales price variance for product V for June was £34,350 favourable
(b) The sales volume contribution variance for product V for June was £9,430 adverse
£
4,390 units should sell for ( £45) 197,550
But did sell for
231,900
Sales price variance
34,350 favourable
Actual sales volume 4,390 units
Budget sales volume
4,800 units
Sales volume variance in units 410 adverse
standard contribution per unit £(45 22) £23
Sales volume contribution variance
£9,430 adverse
Solution 41
Adverse variances are deducted from the budgeted contribution to derive the actual
contribution. Favourable variances are added because they would increase the
contribution above the budgeted level.
The actual contribution reported for the period was £59,755.
£48,000 þ£ð11,245 þ 6,025 3,100 2,415Þ¼£59,755:
Solution 42
Only (b) and (c) could have contributed to an adverse direct labour efficiency variance.
(a) Higher output would not in itself cause an adverse efficiency variance. In calculating
the efficiency variance the expected labour hours would be flexed according to the
actual output achieved.
(b) If material was difficult to process the number of labour hours taken might have been
higher than standard. This would result in an adverse labour efficiency variance.
(c) If the original standard time was set too low then actual times are likely to be higher
than standard, thus resulting in an adverse labour efficiency variance.
(d) A higher hourly labour rate would cause an adverse labour rate variance, not an
adverse efficiency variance.
(e) Using employees who are more skilled than specified in the standard is more likely to
result in a favourable direct labour efficiency variance.
SOLUTIONS TO REVISION QUESTIONS C1
402
PREPARING FOR THE ASSESSMENT
2006.1
Solution 43
Reason (a) is an example of an inter-relationship between variances; the adverse sales
price variance could have resulted in the favourable sales volume contribution variance.
Reasons (a) and (c) could have contributed to a favourable sales volume contribution
variance. A lower sales price might encourage more customers to buy which, as with (c),
might increase sales volumes above budget and a favourable sales volume contribution
variance would result.
A higher actual contribution than the standard per unit (reason (b)) would not result in a
favourable sales volume contribution variance, since the variance is evaluated at the
standard contribution per unit.
Solution 44
First calculate the standard time allowed and compare this with the time taken. You can
convert all times to minutes or to hours, whichever you find easier.
Next read the question carefully to determine the bonus. Every bonus scheme is different,
so do not assume that this one is calculated in the same way as the last scheme you met!
Answer: (A)
Solution 45
Remember: if you are reduced to guessing, then eliminate first the options that are
obviously incorrect. For example, option D must be incorrect because direct materials
returned to stores unused cannot yet have become part of cost of sales.
Answer: (B)
This is the reverse of the entries that would have been made when the direct materials
were first issued to production.
Solution 46
Ensure that you read the introduction to the question carefully. We need to know that
this is the company’s year end, in order to be able to select the correct entry.
Answer: (A)
Since this is the year end, the balance on the overhead control account would be transferred
to the income statement, rather than carried forward to the following period.
The debit side of the account (the overhead incurred) is greater than the credit side of
the account (the overhead absorbed into work in progress). Therefore, the overhead is
under-recovered or under-absorbed.
Standard time allowed ¼400 units 7 minutes 2,800 minutes
Actual time taken ¼42 hours 60 minutes
2,520 minutes
Time saved against standard allowance
280 minutes
Bonus payable ¼ 75% 280=60ðÞ£10 ¼ £35:
FUNDAMENTALS OF MANAGEMENT ACCOUNTING
403
PREPARING FOR THE ASSESSMENT
2006.1
Solution 47
You might find it easiest to quickly sketch the T-accounts from the data provided, then
you can simply pick out the correct journal entries and tick them.
Wages incurred are higher than the wages paid, so there must be an accrual for the period,
but you are not asked about the accounting entries for this element of the transactions.
The correct choices are:
Solution 48
In part (b), you might like to draw up your own production overhead control account.
Although you would not earn marks for this, it might help you to collect together all the
information you need to calculate the under- or over-absorption.
(a) A £48,250
B £598,050 (direct materials issued to work in progress)
C £617,300
D £32,800 (indirect materials issued).
(b) The production overheads for the period were over-absorbed by £57,995. This amount
will be transferred as a credit in the income statement at the end of the period.
Working:
Production overhead control account
££
Creditors 359,725 Work in progress (280% £173,400) 485,520
Provision for depreciation 35,000
Indirect materials 32,800
Over-absorption
57,995
485,520 485,520
Solution 49
A favourable variance is always credited to the relevant variance account, so you can
easily eliminate options ( B) and (D) as incorrect.
Answer: (C)
As a general rule, all variances are entered in the accounts at the point at which they arise.
The labour rate variance is therefore recorded in the wages control account.
(b) Debit wages control account: £31,700
Wages actually paid are debited to the wages control account and credited to the bank or
cash account.
(c) Debit work in progress account: £27,400
(d) Debit production overhead control account: £6,000
(e) Credit wages control account: £33,400
Direct wages incurred are credited to the wages control account and debited to work in
progress. Indirect wages incurred are credited to the wages control account and debited to
the production overhead control account, pending their later absorption into work
in progress.
SOLUTIONS TO REVISION QUESTIONS C1
404
PREPARING FOR THE ASSESSMENT
2006.1
Solution 50
Remember that adverse variances are always debited in the relevant variance account,
and favourable variances are always credited in the variance account.
(a) Debit Credit No entry in this account
Materials control account 4
Material price variance account 4
Work in progress account 4
(b) Debit Credit No entry in this account
Materials control account 4
Material usage variance account 4
Work in progress account 4
(c) The labour force was paid at a higher hourly rate than standard. (Because the labour rate
variance is adverse.)
(d) Debit Credit No entry in this account
Wages control account 4
Labour efficiency variance account 4
Work in progress account 4
Solution 51
Do not confuse the term ‘works order number’ with ‘job number’. A reference number
(in this case the works order number) is used in the recording of costs in any specific
order costing system, whether it is a job, batch or contract system.
After our answer we have included some discussion, so that you can understand the
reasoning behind the answers, for revision purposes. You would not add any such
discussion or workings in the actual assessment.
(i) Number 488 contract costing
(ii) Number 517 contract costing
(iii) Number 518 job costing
(iv) Number 519 job costing
Discussion
Works order 488. This should be accounted for as a long-term contract since it spans three
accounting years, and because the sums of money involved in the contract are large.
Works order 517. This work spans a financial year-end with a significant sales value, so
although the case for ‘contract’ status would not be as strong as for works order 488, this
nevertheless would be appropriate.
Works orders 518 and 519. Both of these are of small value, and both have durations of
approximately two months, although spanning a financial year-end. In neither case would the
apportionment of profit over the two financial years be worthwhile, any profit being most
likely to be taken at the end of the work. Should a loss be expected, however, this should be
brought forward into the accounts of the first financial period covered. Long-term contract
status would not be appropriate, however, so they should be accounted for using job costing.
FUNDAMENTALS OF MANAGEMENT ACCOUNTING
405
PREPARING FOR THE ASSESSMENT
2006.1
Solution 52
Use the information provided to determine the number of chargeable hours each year.
Calculate the hourly rate that the accountant needs to charge to cover her expenses and
salary, based on the number of chargeable hours.
Apply the hourly rate to the job in question.
Answer: (C)
Chargeable hours each year will be (52 4weeks¼) 48 weeks 35 hours per
week ¼1,680 hours 75% ¼1,260 hours.
In these 1,260 hours, she must make £50,000 to cover her salary and general expenses.
Therefore, her charge rate should be
£50,000
1,260
¼ £39:68254 per hour
Thus, the quote for a 50-hour job should be £39.68254/hour 50 ¼£1,984.
Solution 53
Read the question carefully. Profit is calculated as a percentage of sales, not as a
percentage of cost.
Answer: (C)
Solution 54
Be careful when you are adding the profit percentage to the total cost in part (b). The
question states that the company wishes to achieve 30 per cent profit margin on sales.
Do not make the common mistake of simply adding 30 per cent to cost. This will not
produce 30 per cent profit margin on sales.
£
Direct materials 10 £10 100
Direct labour 20 £5
100
Prime cost 200
Variable production overheads 20 £2 40
Fixed production overheads 20 £10*
200
Total production cost 440
Other costs
50
Total cost 490 70%
Profit
210 30%
Quote for the job
700 100%
* £100,000 overheads/10,000 hours ¼£10 per hour.
SOLUTIONS TO REVISION QUESTIONS C1
406
PREPARING FOR THE ASSESSMENT
2006.1
(a) (i) £29,210
Workings:
££
Direct materials 26,500
Labour
Preparation: 90 £8 720
Etching: 130 £7 910
Assembly: 180 £6
1,080
2,710
29,210
(ii) Preparation Department overheads £14,300
Etching Department overheads £5,880
Assembly Department overheads £4,860
Workings:
Overhead absorption rates:
Preparation:
£165,000
3,000
¼ £55 per machine hour
Etching:
£98,000
1,400
¼ £70 per machine hour
Assembly:
£48,600
1,800
¼ £27 per labour hour
Overheads charged to batch
Preparation: 260 £55 £14,300
Etching: 84 £70 £5,880
Assembly: 180 £27 £4,860
(b) £465
Workings:
£
Batch cost 65,100
Profit ( 30/70)
27,900
Sales value of batch
93,000
Selling price per unit 93,000=200ðÞ
£465
FUNDAMENTALS OF MANAGEMENT ACCOUNTING
407
PREPARING FOR THE ASSESSMENT
2006.1
Solution 55
When you are calculating the depreciation charge in part (b), don’t forget that the
contract has been in operation for only ten months.
(a) £1,220,000
(b) £750,000
(c) £2,240,000
Workings:
£000
(a) Materials delivered direct to site 1,200
Materials issued from head office stores
200
1,400
Materials on site at 31 December 20X0
(180)
Cost of material used on contract
1,220
(b) Plant delivered to site at cost 900
Depreciation 900 20%
10
12

150
Net book value of plant at 31 December
750
(c) Final contract value 10,000
Cost incurred to date 3,360
Cost to completion
2,640
Estimated contract profit
4,000
Profit to be recognised on contract ¼£4,000,000 (3,360,000/6,000,000 total cost)
¼£2,240,000.
Solution 56
You may be able to solve this question without producing a reconciliation of the input and
output volumes. We have shown a reconciliation so that you can use it for revision purposes.
56.1 Answer: (B)
20,000 kg input 15% ¼ 3,000 kg normal loss £1 ¼ £3,000
56.2 Answer: (D)
Input Kg Output Kg Kg to absorb cost
Materials 20,000 To process 2 18,500 18,500
Normal loss 3,000
Abnormal gain (1,500) (1,500)
20,000 20,000 17,000
SOLUTIONS TO REVISION QUESTIONS C1
408
PREPARING FOR THE ASSESSMENT
2006.1
Costs £
Materials input 50,000
Direct labour 15,000
Production overheads 22,500
Scrap value normal loss
(3,000)
84,500
Cost per kg £84,500/17,000 4.9706
Value of abnormal gain ¼1,500 kg £4.9706 ¼£7,456.
56.3 Answer: (C)
Value of output ¼ 18,500 kg £4:9706 ¼ £91,956:
Solution 57
Do not be put off by the slightly different layout of the working template.
The question is in fact very straightforward. To perform the necessary valuations you
simply need to use the unit rates supplied in the template.
(a) £37,200
Working:
£
Raw material input 37,500
Less scrap value of normal loss (50,000 litres 3% £0.20)
(300)
Material cost for the period
37,200
(b) £102,900
Working:
Value of transfer to process 2 ¼ 49,000 litres ð£0:75 þ£1: 35Þ¼£102,900:
(c) £1,050
Working:
Value of abnormal loss ¼ 500 litres £ð0:75 þ 1:35Þ¼£1,050:
(d) £5,160
Working:
Value of closing work in progress ¼ð4,000 litres £0:75Þ
þ½ð4,000 40%Þ litres £1:35
¼ £3,000 þ£2,160
¼ £5,160:
FUNDAMENTALS OF MANAGEMENT ACCOUNTING
409
PREPARING FOR THE ASSESSMENT
2006.1
Solution 58
The materials lost in production are scrapped. Therefore, no value is allocated to the normal
loss. A common error would be to attempt to allocate a monetary value to the normal loss.
(a) 10,320 equivalent litres
(b) £7.00 per equivalent litre
Workings:
Material cost: £
A 3/6 12,000 £8.00 48,000
B 2/6 12,000 £6.00 24,000
C 1/6 12,000 £3.90
7,800
79,800
Statement of equivalent litres
Total Materials
Labour and
overheads
Completed production 9,500 9,500 9,500
Abnormal loss 100 100 100
Normal loss 600
Closing WIP:
Material 1,800 1,800
Labour and overheads
(40% 1,800)
720
Equivalent litres
12,000 11,400 10,320
Cost £ 79,800 £41,280
Cost per equivalent litre £11.00 £ 7.00 £4.00
Solution 59
You will find process costing questions much quicker and easier to answer if you learn a
pro-forma layout for your working papers, but remember that you will earn no marks
for your workings.
When you are carrying out your equivalent units calculation, remember that any units
that are now in process 3 must be complete as regards process 2 input.
The cost per equivalent unit produced was:
(a) process 2 input: £2.40 per equivalent unit
(b) material added: £0.60 per equivalent unit
(c) wages and overhead: £1.20 per equivalent unit
SOLUTIONS TO REVISION QUESTIONS C1
410
PREPARING FOR THE ASSESSMENT
2006.1
Workings:
Input Units Output Units
Process
2 input
Material
added
Wages and
overhead
Opening stock 6,000 Finished goods 46,500 46,500 46,500 46,500
Process 2 48,000 Normal loss 3,000
1
––
Abnormal loss 500
2
500 400 300
Closing WIP 4,000 4,000 2,000 1,200
54,000 54,000 51,000 48,900 48,000
Costs £ £ £ £
Opening stock 14,400 2,160 2,880
Input costs 110,400 27,180 54,720
Normal loss value
(2,400)
122,400 29,340 57,600
Cost per unit 4.20 2.40 0.60 1.20
Notes:
1. Normal loss ¼6% (6,000 + 48,000 4,000) ¼3,000 units.
2. The abnormal loss is found as a balancing figure in the input/output reconciliation.
Solution 60
The statement is false. It describes objective classification.
Subjective classification is the classification of expenditure items according to the nature
of the expenditure.
Solution 61
(a) 544 indicates the subjective classification of the expenditure item.
(b) 221 indicates the objective classification of the expenditure item.
Solution 62
We are told that the various vehicles incur variable costs per kilometre in the ratio 1:3:5.
Therefore, we need to calculate a weighted total number of kilometres travelled, in order
to fairly share out the total variable costs incurred. We cannot simply calculate the variable
cost per kilometre as (costs incurred kilometres travelled), because a kilometre travelled
by a motor-cycle costs less than a kilometre travelled by a bulk parcel van or lorry.
(a) Courier service £129,400
(b) Domestic parcels £205,300
(c) Bulk parcels £140,500
FUNDAMENTALS OF MANAGEMENT ACCOUNTING
411
PREPARING FOR THE ASSESSMENT
2006.1
Workings:
Weighted total kilometres travelled
Distance Weighted
Weight km km
1 168,000 168,000
3 82,000 246,000
5 54,000
270,000
684,000
Total variable costs ¼£307,800
Therefore, variable costs per weighted km ¼ £
307,800
684,000
¼ £0:45 per weighted km
Variable cost per service is therefore:
£
Courier service: 168,000 0.45 75,600
Domestic parcels: 246,000 0.45 110,700
Bulk parcels: 270,000 0.45
121,500
307,800
Thus, contribution per service is:
Courier Domestic Bulk
££ £
Sales 205,000 316,000 262,000
Variable costs
(75,600) (110,700) (121,500)
Contribution
129,400 205,300 140,500
Solution 63
Value added = sales revenue less materials costs and the cost of bought-in goods and
services. Wages and salary costs are not bought-in costs and must be excluded from the
overhead cost figures when calculating the value added.
For the F Division for the latest period, the value added was £132,270
££ £
Sales revenue 289,500
Less materials cost 89,790
Production overhead cost 54,030
Less salaries included
10,710
Bought-in production overhead cost 43,320
Marketing overhead cost 21,890
Less salaries included
14,560
7,330
General and admin. overhead 38,120
Less salaries included
21,330
16,790
Total bought in goods and services
157,230
Value added
132,270
SOLUTIONS TO REVISION QUESTIONS C1
412
PREPARING FOR THE ASSESSMENT
2006.1
Solution 64
The salary costs and general overhead costs cannot be specifically attributed to either type of
goods therefore these costs should not be included in the calculation of gross margin.
(a) The gross margin generated by second-hand donated goods sold was £4,818
(b) The gross margin generated by new goods sold was £3,168
Second-hand donated goods ££
Sales income
Sold to customers 4,880
Sold for recycling
88
4,968
Cost of laundering, etc. 120
Valuation costs
30
150
Gross margin
4,818
New goods ££
Sales income 6,790
Less: purchase cost 3,332
delivery cost
290
3,622
Gross margin
3,168
Solution 65
Only three of the budget roles are correct here, but there are others that are not
mentioned including communication, planning, resource allocation and motivation.
Answer: (B)
A budget does not provide a means for expansion. In fact, an organisation can budget to
reduce its level of activity.
Solution 66
If you remember that the principal budget factor is sometimes referred to as the limiting
factor, then you should not have too many problems in selecting the correct answer!
Answer: (B)
The principal budget factor is important because it must be identified at the start of the
budgeting process. Once the budget for the limiting factor has been prepared, all other
budgets must be coordinated with it.
FUNDAMENTALS OF MANAGEMENT ACCOUNTING
413
PREPARING FOR THE ASSESSMENT
2006.1
Solution 67
Although continuous budgeting is quicker and easier using a computerised system it can
be accomplished with a manual system.
Options (a) and (c) are correct.
Solution 68
Remember the formula to calculate budgeted production:
Budgeted sales þBudgeted closing stock Budgeted opening stock
Answer: (A)
Units
Required by sales 120,000
Required closing stock 6,500
Less opening stock anticipated
(13,000)
Production level
113,500
Solution 69
Did you read the question carefully and note that the material usage budget was required,
not the material purchases budget?
Answer: (C)
10,000 units 4kg¼ 40,000 kg
Solution 70
The first step is to calculate the required production volume, taking account of the
budgeted change in finished goods stocks.
Convert the production volume into material usage requirements, then adjust for the
budgeted change in raw materials stocks to determine the budgeted purchases.
Answer: (B)
Units
Budgeted sales of product P 4,000
Required decrease in finished goods stock
600
Required production
3,400
Kg
Raw materials usage budget ( 2 kg) 6,800
Increase in raw materials stocks
1,500
Budgeted purchases of raw material
8,300
SOLUTIONS TO REVISION QUESTIONS C1
414
PREPARING FOR THE ASSESSMENT
2006.1
Solution 71
Note that the 5 per cent bad debts will never be received in cash.
Do not forget to allow for the 2 per cent settlement discount for those customers paying
in September for August sales.
Answer: (C)
£
Receipts in September from:
June sales £100,000 15% 15,000
July sales £150,000 20% 30,000
August sales £130,000 60% less
2% settlement discount
76,440
Total receipts in September
121,440
Solution 72
Make sure that you are completely familiar with the descriptions of all of the types of
budgeting covered in this Learning System.
A budget which is designed to show the allowed expenditure for the actual level of activity
achieved is known as a flexible budget. A fixed budget is prepared for a single level of activity
and a rolling budget is a continuously updated budget.
Solution 73
You will need to produce quite a few workings for your own benefit.
(a) £1,740,000
Quarter 1 Sales Budget
January February March Total
Sales (units) 5,000 5,500 6,000
Selling price for each unit £100 £100 £115
Sales (£) £500,000 £550,000 £690,000 £1,740,000
(b) 23,100 units
Quarter 4 Production Budget units
Required sales units 23,250 (7,750 + 8,000 + 7,500)
Add:
Required closing stock 1,400 (20% 7,000 January 20X3 sales units)
Less:
Opening stock
(1,550) (20% 7,750 October sales units)
Production budget
23,100
FUNDAMENTALS OF MANAGEMENT ACCOUNTING
415
PREPARING FOR THE ASSESSMENT
2006.1
(c) 56,400 kg
Quarter 2 Production Budget units
Required sales units 18,750 (6,000 + 6,250 + 6,500)
Add:
Required closing stock 1,250 (20% 6,250 July sales units)
Less:
Opening stock
(1,200) (20% 6,000 April sales units)
Production budget
18,800*
(d) £249,900
Quarter 1 Material Purchases Budget
Quarter 1 material usage 50,100 kg (16,700* 3 kg)
Add:
Required closing stock 3,630 kg (6,050* 3kg20%)
Less:
Opening stock
(3,750) kg
Purchases 49,980 kg
Price of each kg £5.00
Total material purchases budget £249,900
Quarter 1 Production Budget units
Required sales units 16,500 (5,000 + 5,500 + 6,000)
Add:
Required closing stock 1,200 (20% 6,000 April sales units)
Less:
Opening stock
(1,000)
Production budget
16,700*
April Production Budget units
Required sales units 6,000
Add:
Required closing stock 1,250 (20% 6,250 May sales units)
Less:
Opening stock
(1,200) (20% 6,000 April sales units)
Production budget
6,050*
Quarter 2 Material Usage Budget
Quarter 2 production units 18,800*
Material usage for each unit 3 kg
Total quarter 2 material usage 56,400 kg
*It is calculated below as a result of the production budget.
SOLUTIONS TO REVISION QUESTIONS C1
416
PREPARING FOR THE ASSESSMENT
2006.1
(e) 126,300 hours
Quarter 3 Production Budget units
Required sales units 20,750
Add:
Required closing stock 1,550 (20% 7,750 October sales units)
Less:
Opening stock
(1,250) (20% 6,250 July sales units)
Production budget
21,050
Total skilled labour
hours required 84,200 hours (21,050 4 hours)
Total unskilled labour
hours required 42,100 hours (21,050 2 hours)
Total hours required 126,300 hours.
Solution 74
Remember to exclude depreciation from your calculations of overhead cash payments.
It is not a cash flow.
(a) June: £6,500
July: £11,500
August: £8,000
Workings:
June July August
£££
Closing stock 3,500 6,000 4,000
Material usage
8,000 9,000 10,000
11,500 15,000 14,000
Less: opening stock
5,000 3,500 6,000
Direct material purchases
6,500 11,500 8,000
(b) £51,000
(c) £20,000
Workings:
(b) £
Sales receipts in August:
Cash sales (10% £60,000) 6,000
Credit sales from July (90% £50,000)
45,000
51,000
(c) Cash payments in July:
Wages 13,000
Overheads ( June £8,500 less depreciation)
7,000
20,000
FUNDAMENTALS OF MANAGEMENT ACCOUNTING
417
PREPARING FOR THE ASSESSMENT
2006.1
Solution 75
The flexed budgets are reasonably straightforward to produce: all variable costs are
multiplied by a factor of 13,600 and 14,500, respectively, and fixed overheads remain
unaltered by the change in activity.
(a) £2,560,000
Workings:
£
Direct labour: £46 13,600 units 625,600
Direct material: £30 13,600 units 408,000
Variable overheads: £24 13,600 units 326,400
Fixed overheads: original budget
(£80 15,000 units)
1,200,000
2,560,000
(b) (i) £60,000 adverse
(ii) £164,000 adverse
Workings:
££
Actual cost 2,814,000
Budget cost allowance:
Labour, materials and variable o/h
£(46 + 30 + 24) 14,500 1,450,000
Fixed overhead original budget
1,200,000
2,650,000
Expenditure variance
164,000 (A)
Sales volume contribution variance ¼volume shortfall standard contribution per unit
¼500 units £(220 100)
¼£60,000 adverse
Solution 76
Be careful to select actions that are appropriate both for a deficit and for the short term.
Actions (a) and (b) would be appropriate actions.
Action (c) would not be appropriate because increasing debtors would drain the cash
balance still further. Action (d) is more suited to a long-term deficit, since share capital is a
long term source of finance.
SOLUTIONS TO REVISION QUESTIONS C1
418
PREPARING FOR THE ASSESSMENT
2006.1
Solution 77
The first step is to use the high-low method to determine the fixed and variable
elements of the maintenance costs.
Use the result to determine the budget cost allowance for period 6 and then compare
this allowance with the actual expenditure during the period.
The bonus to be paid to the maintenance manager in period 6 is £28.60.
Using the high-low method: Activity £
Hours
Period 4 3,800 11,040
Period 5
4,320 11,976
Change
520 936
Variable maintenance budgeted cost per hour ¼£936/520 ¼£1.80
Fixed maintenance cost budgeted per period ¼£11,976 (4,320 £1.80) ¼£4,200
£
Fixed cost allowance for period 6 4,200
Variable cost (4,090 machine hours £1.80)
7,362
Total budget cost allowance for period 6 11,562
Actual maintenance expenditure
10,990
Saving
572
Bonus payable ¼5% £572 ¼£28.60
FUNDAMENTALS OF MANAGEMENT ACCOUNTING
419
PREPARING FOR THE ASSESSMENT
2006.1
Mock Assessment 1
Mock Assessment 1
Certificate in Business Accounting
Fundamentals of Management Accounting
You are allowed two hours to complete this assessment.
The assessment contains 50 questions.
All questions are compulsory.
Do not turn the page until you are ready to attempt the assessment under timed
conditions.
423 2006.1
Mock Assessment Questions
Question 1
Which ONE of the following would be classified as direct labour?
& Personnel manager in a company servicing cars.
& Bricklayer in a construction company.
& General manager in a DIY shop.
& Maintenance manager in a company producing cameras.
Question 2
The principal budget factor is the
& factor which limits the activities of the organisation and is often the starting point in
budget preparation.
& budgeted revenue expected in a forthcoming period.
& main budget into which all subsidiary budgets are consolidated.
& overestimation of revenue budgets and underestimation of cost budgets, which
operates as a safety factor against risk.
Question 3
R Ltd absorbs overheads based on units produced. In one period 110,000 units were
produced and the actual overheads were £500,000. Overheads were £50,000 over-
absorbed in the period.
The overhead absorption rate was £
per unit.
Question 4
X Ltd operates an integrated cost accounting system. The Work-in-Progress Account at
the end of the period showed the following information:
The £200,000 credit entry represents the value of the transfer to the
& Cost of sales account.
& Material control account.
& Sales account.
& Finished goods stock account.
Work-in-Progress Account
££
Stores ledger a/c 100,000 ? 200,000
Wage control a/c 75,000
Factory overhead a/c
50,000 Balance c/d 25,000
225,000 225,000
MOCK ASSESSMENT C1
424
MOCK ASSESSMENT 1
2006.1
Question 5
X Ltd absorbs overheads on the basis of machine hours. Details of budgeted and actual
figures are as follows:
(a) Overheads for the period were:
under-absorbed &
over-absorbed &
(b) The value of the under/over absorption for the period was £
.
Question 6
In an integrated bookkeeping system, when the actual production overheads exceed the
absorbed production overheads, the accounting entries to close off the production
overhead account at the end of the period would be:
Question 7
A retailer uses a Last In First Out (LIFO) stock valuation system. Movements of item M
for February are as follows.
Units £ per unit
1st February Opening stock balance 230 7.80
3rd February Receipts 430 7.95
8th February Issues 370
14th February Issues 110
22nd February Receipts 400 8.01
No other movements of item M occurred during the month.
(a) The value of the closing stock of item M at the end of February is £
(b) All units of item M were sold for £14 each. The gross profit achieved on item M
during February was £
Budget Actual
Overheads £1,250,000 £1,005,000
Machine hours 250,000 hours 220,000 hours
Debit Credit
No entry in
this account
Production overhead account && &
Work in progress account && &
Profit and loss account && &
FUNDAMENTALS OF MANAGEMENT ACCOUNTING
425
MOCK ASSESSMENT 1
2006.1
Question 8
A Limited has completed the initial allocation and apportionment of its overhead costs to
cost centres as follows.
Cost centre
Initial
allocation
£000
Machining 190
Finishing 175
Stores 30
Maintenance
25
420
The stores and maintenance costs must now be reapportioned taking account of the
service they provide to each other as follows.
Machining Finishing Stores Maintenance
Stores to be apportioned 60% 30% 10%
Maintenance to be apportioned 75% 20% 5%
After the apportionment of the service department costs, the total overhead cost of the
production departments will be (to the nearest £000):
Machining £
Finishing £
Question 9
The budgeted contribution for R Limited last month was £32,000. The following variances
were reported.
Variance £
Sales volume contribution 800 adverse
Material price 880 adverse
Material usage 822 favourable
Labour efficiency 129 favourable
Variable overhead efficiency 89 favourable
No other variances were reported for the month.
The actual contribution earned by R Limited last month was £
MOCK ASSESSMENT C1
426
MOCK ASSESSMENT 1
2006.1
Question 10
The following scattergraph has been prepared for the costs incurred by an organisation
that delivers hot meals to the elderly in their homes.
6000
5000
4000
3000
2000
200 400
Number of meals delivered
1000
£
Based on the scattergraph:
(a) the period fixed cost is £
(b) the variable cost per meal delivered is £
Question 11
A company operates a differential piece-rate system and the following weekly rates have
been set:
Details relating to employee A for the latest week are shown below:
Employee A
There is a guaranteed minimum wage of £5 per hour for a 40-hour week paid to all
employees.
The amount payable (to the nearest £ ) to employee A is £
.
Question 12
Overtime premium is
& the additional amount paid for hours worked in excess of the basic working week.
& the additional amount paid over and above the normal hourly rate for hours worked
in excess of the basic working week.
1–500 units £0.20 per unit in this band
501–600 units £0.25 per unit in this band
601 units and above £0.55 per unit in this band
Actual output achieved 800 units
Actual hours worked 45
FUNDAMENTALS OF MANAGEMENT ACCOUNTING
427
MOCK ASSESSMENT 1
2006.1
& the additional amount paid over and above the overtime rate for hours worked in
excess of the basic working week.
& the overtime rate.
The following information is required for Questions 13 and 14
X Ltd has two production departments, Assembly and Finishing, and one service
department, Stores.
Stores provide the following service to the production departments: 60% to Assembly
and 40% to Finishing.
The budgeted information for the year is as follows:
Budgeted production overheads:
Question 13
The budgeted production overhead absorption rate for the Assembly Department will
be £
per unit.
Question 14
At the end of the year, the total of all of the production overheads debited to the Finishing
Department Production Overhead Control Account was £130,000, and the actual output
achieved was 100,000 units.
(a) The overheads for the Finishing Department were:
under-absorbed
over-absorbed
(b) The value of the under/over absorption was £ .
Question 15
R Ltd has been asked to quote for a job. The company aims to make a profit margin of
20% on sales. The estimated total variable production cost for the job is £125.
Fixed production overheads for the company are budgeted to be £250,000 and are
recovered on the basis of labour hours. There are 12,500 budgeted labour hours and this
job is expected to take 3 labour hours.
Other costs in relation to selling, distribution and administration are recovered at the
rate of £15 per job.
The company quote for the job should be £
.
Assembly £100,000
Finishing £150,000
Stores £50,000
Budgeted output 100,000 units
MOCK ASSESSMENT C1
428
MOCK ASSESSMENT 1
2006.1
Question 16
Which of the following would NOT be included in a cash budget? Tick all that would
NOT be included.
& Depreciation
& Provisions for doubtful debts
& Wages and salaries
The following information is required for Questions 17 and 18
X Ltd is preparing its budgets for the forthcoming year.
The estimated sales for the first four months of the forthcoming year are as follows:
40% of each month’s sales units are to be produced in the month of sale and the
balance is to be produced in the previous month.
50% of the direct materials required for each month’s production will be purchased in
the previous month and the balance in the month of production.
The direct material cost is budgeted to be £5 per unit.
Question 17
The production budget for Month 1 will be units.
Question 18
The material cost budget for Month 2 will be £ .
Question 19
When calculating the material purchases budget, the quantity to be purchased equals
& material usage + materials closing stock materials opening stock
& material usage materials closing stock + materials opening stock
& material usage materials closing stock materials opening stock
& material usage + materials closing stock + materials opening stock
Question 20
The following extract is taken from the overhead budget of X Ltd:
The overhead budget for an activity level of 80% would be £
.
Month 1 6,000 units
Month 2 7,000 units
Month 3 5,500 units
Month 4 6,000 units
Budgeted activity 50% 75%
Budgeted overhead £100,000 £112,500
FUNDAMENTALS OF MANAGEMENT ACCOUNTING
429
MOCK ASSESSMENT 1
2006.1
Question 21
Which of the following would be included in the cash budget, but would not be included
in the budgeted profit and loss account? Tick all that are correct.
& Repayment of a bank loan.
& Proceeds from the sale of a fixed asset.
& Bad debts write off.
Question 22
Sales
£
Total cost
Fixed cost
Level of activit
y
This graph is known as a
& semi-variable cost chart.
& conventional breakeven chart.
& contribution breakeven chart.
& profit volume chart.
Question 23
The following details have been extracted from the creditors’ records of X Limited:
Purchases for July to September are budgeted as follows:
For suppliers paid in the month of purchase, a settlement discount of 5% is received.
The amount budgeted to be paid to suppliers in September is £
.
Invoices paid in the month of purchase 25%
Invoices paid in the first month after purchase 70%
Invoices paid in the second month after purchase 5%
July £250,000
August £300,000
September £280,000
MOCK ASSESSMENT C1
430
MOCK ASSESSMENT 1
2006.1
Question 24
Level of activit
y
£
Y
X
The difference in the values ) between point X and point Y on the profit volume chart
shown above represents:
& contribution
& profit
& breakeven
& loss
Question 25
Level of activit
y
£
The shaded area on the breakeven chart shown above represents:
& loss
& fixed cost
& variable cost
& profit
FUNDAMENTALS OF MANAGEMENT ACCOUNTING
431
MOCK ASSESSMENT 1
2006.1
Question 26
In a standard cost bookkeeping system, when the actual material usage has been greater
than the standard material usage, the entries to record this is in the accounts are:
Question 27
R Ltd makes one product, which passes through a single process.
Details of the process for period 1 were as follows:
There was no work-in-progress at the beginning or end of the period. Process losses
have no value.
The cost of the abnormal loss (to the nearest £ ) is £
.
The following information is required for Questions 28–35
X Ltd operates a standard costing system. The following budgeted and standard cost
information is available:
Actual results for the period were as follows:
For all calculated variances, tick the correct box to indicate whether the variance is
adverse or favourable.
Debit Credit
No entry in
this account
Material usage variance account && &
Raw material control account && &
Work in progress account && &
£
Material cost 20,000 kg 26,000
Labour cost 12,000
Production overhead cost 5,700
Output 18,800 kg
Normal losses 5% of input
Budgeted production and sales 10,000 units
£ per unit
Selling price 250
Direct material cost 3 kg £10 30
Direct labour cost 5 hours £8 40
Variable production overheads 5 hours £4 20
Production and sales 11,500 units
£
Sales value 2,817,500
Direct material 36,000 kg 342,000
Direct labour 52,000 hours 468,000
Variable production overheads 195,000
MOCK ASSESSMENT C1
432
MOCK ASSESSMENT 1
2006.1
Question 28
The direct material price variance is £
adverse &
favourable &
Question 29
The direct material usage variance is £
adverse &
favourable &
Question 30
The direct labour rate variance is £
adverse &
favourable &
Question 31
The direct labour efficiency variance is £
adverse &
favourable &
Question 32
The variable production overhead expenditure variance is £
adverse &
favourable &
Question 33
The variable production overhead efficiency variance is £
adverse &
favourable &
Question 34
The sales volume contribution variance is £
adverse &
favourable &
FUNDAMENTALS OF MANAGEMENT ACCOUNTING
433
MOCK ASSESSMENT 1
2006.1
Question 35
The sales price variance is £
adverse &
favourable &
Question 36
X Ltd uses the FIFO method to charge material issue costs to production. Opening stock
of material M at the beginning of April was 270 units valued at £4 per unit.
Movements of material M during April were as follows.
(a) The total value of the issues to production during April was £
.
(b) The value of the closing stock at the end of April was £
.
Question 37
X Ltd manufactures a product called the ‘ZT’. The budget for next year was:
If the selling price of the ZT were reduced by 10 per cent, the sales revenue that would
be needed to generate the original budgeted profit would be £
.
Question 38
A company is faced with a shortage of skilled labour next period.
When determining the production plan that will maximise the company’s profit next
period, the company’s products should be ranked according to their:
& profit per hour of skilled labour
& profit per unit of product sold
& contribution per hour of skilled labour
& contribution per unit of product sold
4 April Received 30 units at £4.10 per unit
9 April Issued 210 units
14 April Issued 80 units
22 April Received 90 units at £4.20 per unit
24 April Issued 40 units
Annual sales 10,000 units
£ per unit
Selling price 20
Variable cost 14
Fixed costs
3
Profit
3
MOCK ASSESSMENT C1
434
MOCK ASSESSMENT 1
2006.1
Question 39
Which of the following would contribute towards a favourable sales price variance (tick all
that apply)?
(a) The standard sales price per unit was set too high
.
(b) Price competition in the market was not as fierce as expected
.
(c) Sales volume was higher than budgeted and therefore sales revenue was higher than
budgeted
.
Question 40
R Ltd has the following year-end information regarding one of its long-term contracts:
(a) The cost charged to the income statement in respect of this contract was £
.
(b) The value of the contract debtor is £
.
Question 41
The following data relate to a process for the latest period.
Opening work in progress 300 kg valued as follows
Input material £1,000
Conversion cost £200
Input during period 8,000 kg at a cost of £29,475
Conversion costs £11,977
Output 7,000 kg
Closing work in progress 400 kg
Closing work in progress is complete as to input materials and 70 per cent complete as to
conversion costs.
Losses are expected to be 10 per cent of input during the period and they occur at the
end of the process. Losses have a scrap value of £2 per kg.
The value of the completed output (to the nearest £ ) is £
Question 42
Which of the following stock valuation methods results in charges to cost of sales which
are close to the economic cost?
First In, First Out (FIFO) &
Last In, First Out (LIFO) &
Average Cost (AVCO) &
£
Revenue credited to income statement 2,500,000
Profit recognised 750,000
Cash received 1,875,000
Costs to date 2,200,000
Future costs 220,000
FUNDAMENTALS OF MANAGEMENT ACCOUNTING
435
MOCK ASSESSMENT 1
2006.1
Data for questions 43 and 44
A company makes a single product T and budgets to produce and sell 7,200 units each
period. Cost and revenue data for the product at this level of activity are as follows.
$ per unit
Selling price 53
Direct material cost 24
Direct labour cost 8
Other variable cost 3
Fixed cost
7
Profit
11
Question 43
The contribution to sales ratio (P/V ratio) of product T (to the nearest whole number)
is
%.
Question 44
The margin of safety of product T (to the nearest whole number) is % of budgeted
sales volume.
Data for questions 45 and 46
The total figures from TY Division’s budgetary control report are as follows.
Fixed budget
Flexed budget
allowances Actual results
$$ $
Total sales revenue 520,000 447,000 466,500
Total expenditure
389,000 348,000 329,400
Total profit
131,000 99,000 137,100
Question 45
(a) The sales price variance for the period is $ adverse/favourable
(b) The sales volume variance for the period is $
adverse/favourable
Question 46
(a) The total expenditure variance for the period is $ adverse/favourable
(b) The total budget variance for the period is $
adverse/favourable
MOCK ASSESSMENT C1
436
MOCK ASSESSMENT 1
2006.1
Question 47
In an integrated bookkeeping system, the correct entries to record the depreciation of
production machinery are:
Debit Credit
No entry in
this account
Depreciation of production machinery && &
Work in progress account && &
Production overhead control account && &
Question 48
In an integrated bookkeeping system, the correct entries to record the issue of indirect
materials for production purposes are:
Debit Credit
No entry in
this account
Materials control account && &
Work in progress account && &
Production overhead control account && &
Question 49
H Limited budgets to produce and sell 4,000 units of product H next year. The amount of
capital investment required to support product H will be £290,000 and H Limited requires
a rate of return of 14 per cent on all capital invested.
The full cost per unit of product H is £45.90.
To the nearest penny, the selling price per unit of product H that will achieve the
specified return on investment is £
.
Question 50
The Drop In Cafe´ sells specialist coffees to customers to drink on the premises or to take
away.
The proprietors have established that the cost of ingredients is a wholly variable cost in
relation to the number of cups of coffee sold whereas staff costs are semi-variable and rent
costs are fixed.
Within the relevant range, as the number of cups of coffee sold increases (delete as
appropriate):
(a) The ingredients cost per cup sold will increase/decrease/stay the same.
(b) The staff cost per cup sold will increase/decrease/stay the same.
(c) The rent cost per cup sold will increase/decrease/stay the same.
FUNDAMENTALS OF MANAGEMENT ACCOUNTING
437
MOCK ASSESSMENT 1
2006.1
First Mock Assessment Solutions
Solution 1
Bricklayer in a construction company.
The bricklayer’s wages can be identified with a specific cost unit therefore this is a direct
cost. The wages paid to the other three people cannot be identified with specific cost units.
Therefore they would be indirect costs.
Solution 2
The principal budget factor is the factor which limits the activities of the organisation and
is often the starting point in budget preparation.
Solution 3
The overhead absorption rate was £5 per unit.
Workings:
Overhead absorption rate ¼ £550,000/110,000 units ¼ £5.
Solution 4
Finished goods stock account.
Solution 5
Overheads for the period were over-absorbed by £95,000.
Workings:
Overhead absorption rate ¼ £1,250,000/250,000 ¼ £5 per hour
Solution 6
£
Actual overheads 500,000
Over absorption
50,000
Overhead absorbed
550,000
£
Absorbed overhead ¼ 220,000 hours £5 1,100,000
Actual overhead incurred
1,005,000
Over-absorbed overhead
95,000
Debit Credit No entry in this account
Production overhead account 4
Work in progress account 4
Profit and loss account 4
MOCK ASSESSMENT C1
438
MOCK ASSESSMENT 1
2006.1
Solution 7
(a) The value of the closing stock of item M at the end of February is £4,608
(b) All units of item M were sold for £14 each. The gross profit achieved on item M
during February was £2,911.50
Workings
Receipts Sales Balance
Date Qty Price £ Qty Price £ Qty Price £
1 Feb
230 7.80 1,794.00
3 Feb 430 7.95 3,418.50 230 7.80 1,794.00
430 7.95 3,418.50
660 5,212.50
8 Feb 370 7.95 2,941.50 230 7.80 1,794.00
60 7.95 477.00
290 2,271.00
14 Feb 60 7.95 477.00
50 7.80 390.00
110 867.00 180 7.80 1,404.00
22 Feb 400 8.01 3,204.00 180 7.80 1,404.00
400 8.01 3,204.00
580 4,608.00
(b)
£
Sales revenue (480 units £14) 6,720.00
Cost of goods sold (2,941.50 + 867.00)
3,808.50
Gross profit
2,911.50
Solution 8
After the apportionment of the service department costs, the total overhead cost of the
production departments will be (to the nearest £000):
Machining £230,000
Finishing £190,000
Workings
Machining Finishing Stores Maintenance
£000 £000 £000 £000
Apportioned costs 190.00 175.00 30.0 25.0
Stores apportionment 18.00 9.00 (30.0) 3.0
Maintenance apportionment 21.00 5.60 1.4 (28.0)
Stores apportionment 0.84 0.42 (1.4) 0.14
Maintenance apportionment
0.11 0.03 (0.14)
Total
229.95 190.05
FUNDAMENTALS OF MANAGEMENT ACCOUNTING
439
MOCK ASSESSMENT 1
2006.1
Solution 9
The actual contribution earned by R Limited last month was £31,360.
£ð32,000 800 880 þ 822 þ129 þ89Þ¼ £31;360:
Solution 10
(a) The period fixed cost is £3,000
(b) The variable cost per meal delivered is £5
Workings :
Variable cost per meal ¼
£5,000 £3,000
400 meals
¼ £5
Solution 11
The amount payable to employee A is £235.
Solution 12
Overtime premium is the additional amount paid over and above the normal hourly rate
for hours worked in excess of the basic working week.
Solution 13
The budgeted production overhead absorption rate for the Assembly Department will
be £1.30 per unit.
Workings:
Units £
500 20p 100
100 25p 25
200 55p 110
800 235
Workings:
Assembly
£
Budgeted overheads 100,000
Reapportioned stores overhead 60% £50,000
30,000
Total budgeted overhead
130,000
Budgeted output 100,000
OAR ¼
£130,000
100,000
¼ £1.30 per unit
MOCK ASSESSMENT C1
440
MOCK ASSESSMENT 1
2006.1
Solution 14
The overheads for the Finishing Department were over-absorbed by £40,000.
Workings:
Solution 15
The company quote for the job should be £250.
Workings:
Solution 16
Depreciation and provisions for doubtful debts are not cash flows and would not be
included in a cash budget.
Finishing
£
Budgeted overheads 150,000
Reapportioned stores overhead 40% £50,000
20,000
Total budgeted overhead
170,000
Budgeted output 100,000
OAR ¼
£170,000
100,000
¼ £1.70 per unit
£
Absorbed overhead £1.70 100,000 170,000
Actual overhead incurred
130,000
Over absorption
40,000
Job quote
£
Variable production costs 125
Fixed production overheads
£250,000
12,500
3
60
Selling, distribution and administration
15
Total cost 200
Profit margin 20%
50
Quote
250
FUNDAMENTALS OF MANAGEMENT ACCOUNTING
441
MOCK ASSESSMENT 1
2006.1
Solution 17
The production budget for month 1 will be 6,600 units.
Workings:
Solution 18
The material cost budget for Month 2 will be £30,500.
Workings:
Solution 19
The quantity to be purchased equals material usage + materials closing stock materials
opening stock.
Solution 20
The overhead budget for an activity level of 80% would be £115,000.
Workings:
Using the high/low method
Substitute into 75% activity £
Month 1 Month 2 Month 3 Month 4
Units Units Units Units
Sales 6,000 7,000 5,500 6,000
Production
40% in the month 2,400 2,800 2,200 2,400
60% in the previous month
4,200 3,300 3,600
Production
6,600 6,100 5,800
Month 2 6,100 units produced @ £5 per unit ¼£30,500.
£
High 75% 112,500
Low
50% 100,000
Change
25% 12,500 variable cost of 25%
1% 500 variable cost of 1%
Total overhead 112,500
Variable cost element 75 £500
37,500
Fixed cost element
75,000
Total overhead for 80% activity
Variable cost element 80 £500 40,000
Fixed cost element
75,000
Total overhead
115,000
MOCK ASSESSMENT C1
442
MOCK ASSESSMENT 1
2006.1
Solution 21
The correct answers are:
repayment of a bank loan
proceeds from the sale of a fixed asset.
Both these items result in a cash flow and would therefore be included in the cash
budget. However, they would not be included in the profit and loss account. The bad
debts write off would be included in the profit and loss account, but not in the cash
budget.
Solution 22
The graph is known as a conventional breakeven chart.
Solution 23
The amount budgeted to be paid to suppliers in September is £289,000.
Workings:
Solution 24
The difference in the values ) between point X and point Y on the profit volume chart
represents profit.
Solution 25
The shaded area on the breakeven chart represents loss.
Solution 26
July August September
£££
Purchases
250,000 300,000 280,000
25% paid in the month of purchase 62,500 75,000 70,000
5% discount allowed (3,125) (3,750) (3,500)
70% paid in the first month 175,000 210,000
5% paid in the second month
12,500
Budgeted payment
289,000
Debit Credit No entry in this account
Material usage variance account 4
Raw material control account 4
Work in progress account 4
FUNDAMENTALS OF MANAGEMENT ACCOUNTING
443
MOCK ASSESSMENT 1
2006.1
Solution 27
The cost of the abnormal loss is £460.
Workings:
Cost per kg ¼£43,700/19,000 ¼£2.30
Cost of abnormal loss ¼£2.30 200 kg ¼£460.
Solution 28
The direct material price variance is £18,000 favourable.
Solution 29
The direct material usage variance is £15,000 adverse.
£
Direct material cost 26,000
Labour cost 12,000
Production overhead cost
5,700
43,700
Kg
Input 20,000
Normal loss
1,000
Expected output 19,000
Actual output
18,800
Abnormal loss
200
Workings:
£
36,000 kg should cost ( £10) 360,000
but did cost
342,000
Variance
18,000 F
Workings:
11,500 units should use ( 3 kg) 34,500 kg
but did use
36,000 kg
Difference 1,500 kg
std price per kg
£10
Variance
£15,000 A
MOCK ASSESSMENT C1
444
MOCK ASSESSMENT 1
2006.1
Solution 30
The direct labour rate variance is £52,000 adverse.
Solution 31
The direct labour efficiency variance is £44,000 favourable.
Workings:
11,500 units should take ( 5 hours) 57,500 hours
but did take
52,000 hours
Difference 5,500 hours
std rate per hour
£8
Variance
£44,000 F
Solution 32
The variable production overhead expenditure variance is £13,000 favourable.
Solution 33
The variable production overhead efficiency variance is £22,000 favourable.
Solution 34
The sales volume contribution variance is £240,000 favourable.
Workings:
Workings:
£
52,000 hours should cost ( £8) 416,000
but did cost
468,000
Variance
52,000 A
Workings:
£
52,000 hours should have cost ( £4) 208,000
but did cost
195,000
Variance
13,000 F
Workings:
Variance in hours from labour efficiency variance ¼5,500 hours
standard variable production overhead per hour £4
Variance
£22,000 F
Actual sales volume 11,500 units
Budget sales volume
10,000 units
Variance in units 1,500 favourable
standard contribution per unit £(250 30 40 20)
£160
Sales volume contribution variance
£240,000 favourable
FUNDAMENTALS OF MANAGEMENT ACCOUNTING
445
MOCK ASSESSMENT 1
2006.1
Solution 35
The sales price variance is £57,500 adverse.
Workings:
£
11,500 units should sell for ( £250) 2,875,000
But did sell for
2,817,500
Sales price variance
57,500 adverse
Solution 36
(a) The total value of the issues to production during April was £1,329.
(b) The value of the closing stock at the end of April was £252.
Workings:
(b) Stock ¼60 units £4.20 ¼£252
Solution 37
The sales revenue that would be needed to generate the original budgeted profit would be
£270,000.
Workings:
Fixed costs are not relevant because they will remain unaltered.
Original budgeted contribution ¼10,000 units £(20 14) ¼£60,000
Revised contribution per unit ¼£(18 14) ¼£4
Required number of units to achieve same contribution ¼£60,000/£4 ¼15,000 units
Required sales revenue ¼15,000 units £18 revised price ¼£270,000
Solution 38
When determining the production plan that will maximise the company’s profit next
period, the company’s products should be ranked according to their contribution per hour
of skilled labour.
£
(a) Issues: 9 April 210 units £4 840
14 April 60 units £4 240
20 units £4.10 82
24 April 10 units £4.10 41
30 units £4.20
126
1,329
MOCK ASSESSMENT C1
446
MOCK ASSESSMENT 1
2006.1
Solution 39
Only reason (b) would contribute to a favourable sales price variance.
Reason (a) would result in an adverse variance. Reason (c) would not necessarily result
in any sales price variance because all the units could have been sold at standard price.
Solution 40
(a) The cost charged to the income statement in respect of this contract was £1,750,000.
(b) The value of the contract debtor is £625,000.
Solution 41
The value of the completed output is £38,500
Workings
Equivalent kg
Input material Conversion costs
Input kg Output kg
Opening WIP 300 Finished output 7,000 7,000 7,000
Input 8,000 Normal loss 800
Abnormal loss 100 100 100
Closing WIP 400 400 70% 280
8,300
8,300 7,500 7,380
Costs
££ £
Opening WIP 1,200 1,000 200
Period costs 41,452 29,475 11,977
Normal loss
(1,600) (1,600)
41,052 28,875 12,177
Cost per
equivalent kg
5.50 3.85 1.65
The value of the completed output is £5.50 7,000 kg ¼£38,500
Workings:
£
Revenue credited 2,500,000
Profit recognised
750,000
Cost charged
1,750,000
Workings:
£
Revenue credited 2,500,000
Less cash received
1,875,000
Debtor balance
625,000
FUNDAMENTALS OF MANAGEMENT ACCOUNTING
447
MOCK ASSESSMENT 1
2006.1
Solution 42
The LIFO stock valuation method results in charges to cost of sales which are close to the
economic cost.
Solution 43
The contribution to sales ratio (P/V ratio) of product T is 34%.
Workings:
Contribution per unit of product T ¼$(53 24 8 3) ¼$18
Contribution to sales ratio ¼18/53 ¼34%
Solution 44
The margin of safety of product T is 61% of budgeted sales volume.
Workings:
Period fixed costs ¼7,200 $7 ¼$50,400
Breakeven point ¼
$50,400
$18
¼ 2,800 units
Margin of safety ¼(7,200 2,800) units ¼4,400 units
Margin of safety as percentage of budgeted sales ¼4,400/7,200 ¼61%
Solution 45
(a) The sales price variance is $(466,500 447,000) ¼$19,500 favourable
(b) The sales volume variance is $(99,000 131,000) ¼$32,000 adverse
Solution 46
(a) The total expenditure variance is $(329,400 348,000) ¼$18,600 favourable
(b) The total budget variance is $(137,100 131,000) ¼$6,100 favourable
Solution 47
Debit Credit No entry in this account
Depreciation of production machinery 4
Work in progress account 4
Production overhead control account 4
MOCK ASSESSMENT C1
448
MOCK ASSESSMENT 1
2006.1
Solution 48
Solution 49
The selling price per unit of product H that will achieve the specified return on investment
is £56.05
Workings:
Required return from capital invested to support product H ¼£290,000 14%
¼£40,600
Required return per unit of product H sold ¼£40,600/4,000 ¼£10.15
Required selling price ¼£45.90 full cost + £10.15 ¼£56.05
Solution 50
Within the relevant range, as the number of cups of coffee sold increases:
(a) the ingredients cost per cup sold will stay the same.
(b) the staff cost per cup sold will decrease.
(c) the rent cost per cup sold will decrease.
Debit Credit No entry in this account
Materials control account 4
Work in progress account 4
Production overhead control account 4
FUNDAMENTALS OF MANAGEMENT ACCOUNTING
449
MOCK ASSESSMENT 1
2006.1
Mock Assessment 2
Mock Assessment 2
Certificate in Business Accounting
Fundamentals of Management Accounting
You are allowed two hours to complete this assessment.
The assessment contains 50 questions.
All questions are compulsory.
Do not turn the page until you are ready to attempt the assessment under timed
conditions.
453 2006.1
Question 1
In an integrated accounting system, the accounting entries to complete the production
overhead control account at the end of the period, when the production overheads
absorbed exceed the actual production overhead incurred are:
Question 2
A company expects to sell h units in the next accounting period, and has prepared the
following breakeven chart.
(a) The margin of safety is shown on the diagram by & (insert correct letter).
(b) The effect of an increase in fixed costs, with all other costs and revenues remaining
the same, will be
Question 3
A company uses the repeated distribution method to reapportion service department
costs. The use of this method suggests
& the company’s overhead rates are based on estimates of cost and activity levels, rather
than actual amounts.
& there are more service departments than production cost centres.
Debit Credit No entry in this account
Production overhead control account && &
Work in progress account && &
Finished goods account && &
Profit and loss account && &
increase decrease stay the same
m will && &
k will && &
f will && &
p will && &
Costs &
Revenues
($)
Output & sales (units)
Total revenue
Total cost
m
n
h
g
0
p
f
k
MOCK ASSESSMENT C1
454
MOCK ASSESSMENT 2
2006.1
& the company wishes to avoid under- or over-absorption of overheads in its
production cost centres.
& the service departments carry out work for each other.
Question 4
The management accountant’s report shows that fixed production overheads were over-
absorbed in the last accounting period. The combination that is certain to lead to this
situation is
Question 5
Which of the following costs would be classified as production overhead cost in a food
processing company (tick all that apply)?
& The cost of renting the factory building.
& The salary of the factory manager.
& The depreciation of equipment located in the materials store.
& The cost of ingredients.
Question 6
The normal loss in process 2 is valued at its scrap value. Extracts from the process account
and the abnormal gain account for the latest period are shown below.
The values to be entered in the abnormal gain account for the period are:
A ¼£
B ¼£
Production activity and Fixed overhead expenditure
& lower than budget & lower than budget
& higher than budget & higher than budget
& as budgeted & as budgeted
Process 2
££
Opening WIP 1,847 Output to finished goods
Conversion costs 14,555 5,100 units 22,695
Input materials 6,490 Normal loss 100 units 120
Abnormal gain 220 units Closing WIP
Abnormal gain
££
Profit and loss account A Process 2 B
FUNDAMENTALS OF MANAGEMENT ACCOUNTING
455
MOCK ASSESSMENT 2
2006.1
The following information is required for questions 7 and 8
The incomplete process account relating to period 4 for a company which manufactures
paper is shown below:
Process account
Losses are recognised at the end of the production process and are sold for $1.75 per unit.
Question 7
Given the outcome of the process, which ONE of the following accounting entries is
needed to complete the double entry in the process account for the abnormal loss or gain?
Question 8
The value of the closing WIP was $ .
Question 9
A machine operator is paid £10.20 per hour and has a normal working week of 35 hours.
Overtime is paid at the basic rate plus 50%. If, in week 7, the machine operator worked
42 hours, the overtime premium paid to the operator would be £
.
Question 10
An engineering firm operates a job costing system. Production overhead is absorbed at the rate
of £8.50 per machine hour. In order to allow for non-production overhead costs and profit, a
mark up of 60% of prime cost is added to the production cost when preparing price estimates.
The estimated requirements of job number 808 are as follows:
The estimated price notified to the customer for job number 808 will be £
.
Materials 100%
Labour 50%
Production overhead 40%
Debit Credit No entry in this account
Process account && &
Abnormal Gain account && &
Abnormal Loss account && &
Direct materials £10,650
Direct labour £3,260
Machine hours 140
Units $ Units $
Material 4,000 16,000 Finished goods 2,750
Labour 8,125 Normal loss 400 700
Production overhead 3,498 Work in progress 700
There was no opening work in process (WIP). Closing WIP, consisting of 700
units, was complete as shown:
MOCK ASSESSMENT C1
456
MOCK ASSESSMENT 2
2006.1
Question 11
The diagram represents the behaviour of a cost item as the level of output changes:
Which ONE of the following situations is depicted by the graph?
& Discounts are received on additional purchases of material when certain quantities
are purchased.
& Employees are paid a guaranteed weekly wage, together with bonuses for higher
levels of production.
& A licence is purchased from the government which allows unlimited production.
& Additional space is rented to cope with the need to increase production.
Question 12
A hospital’s records show that the cost of carrying out health checks in the last five
accounting periods have been as follows:
Using the high–low method and ignoring inflation, the estimated cost of carrying out
health checks on 850 patients in period 6 is £
.
Question 13
The principal budget factor for a footwear retailer is
& the cost item taking the largest share of total expenditure.
& the product line contributing the largest amount to sales revenue.
& the product line contributing the largest amount to business profits.
& the constraint that is expected to limit the retailer’s activities during the budget
period.
Period Number of patients seen Total cost
£
1 650 17,125
2 940 17,800
3 1,260 18,650
4 990 17,980
5 1,150 18,360
Outpu
t
Total
cost ($)
0
FUNDAMENTALS OF MANAGEMENT ACCOUNTING
457
MOCK ASSESSMENT 2
2006.1
The following information is required for questions 14 and 15
Extracts from the budget of H Ltd, a retailer of office furniture, for the six months to
31 December show the following information:
Debtors and creditors are expected to rise by 10 and 5 per cent, respectively, by the end of
the budget period.
Question 14
The estimated cash receipts from customers during the budget period are $ .
Question 15
The profit mark-up, as a percentage of the cost of sales (to the nearest whole number) is &%.
Question 16
Which of the following actions are appropriate if a company anticipates a temporary cash
shortage (tick all that apply)?
(i) & issue additional shares;
(ii) & request additional bank overdraft facilities;
(iii) & sell machinery currently working at half capacity;
(iv) & postpone the purchase of plant and machinery.
Question 17
A company manufactures three products, X, Y and Z. The sales demand and the standard
unit selling prices and costs for the next accounting period, period 1, are estimated as follows:
(a) If supplies in period 1 are restricted to 90,000 kg of raw material and 18,000 hours of
direct labour, the limiting factor would be
& direct labour.
& raw material.
& neither direct labour nor raw material.
$
Sales 55,800
Purchases 38,000
Closing stock finished goods 7,500
Opening stock finished goods 5,500
Opening debtors 8,500
Opening creditors 6,500
XYZ
Maximum demand (000 units) 4.0 5.5 7.0
£ per unit £ per unit £ per unit
Selling price 28 22 30
Variable costs:
Raw material ( £1 per kg) 5 4 6
Direct labour ( £12 per hour) 12 9 18
MOCK ASSESSMENT C1
458
MOCK ASSESSMENT 2
2006.1
(b) In period 2, the company will have a shortage of raw materials, but no other resources
will be restricted. The standard selling prices and costs and the level of demand will
remain unchanged.
In what order should the materials be allocated to the products if the company
wants to maximise profit?
First: product &
Second: product &
Third: product &
Question 18
A performance standard which assumes efficient levels of operation, but which includes
allowances for factors such as waste and machine downtime is known as:
& an allowable standard
& an attainable standard
& an ideal standard
& a current standard
The following information is required for questions 19 and 20
W Ltd makes leather purses. It has drawn up the following budget for its next financial
period:
Question 19
The margin of safety represents &% of budgeted sales.
Question 20
The marketing manager has indicated that an increase in the selling price to $12.25 per unit
would not affect the number of units sold, provided that the sales commission is increased
to 8 per cent of the selling price.
These changes will cause the breakeven point (to the nearest whole number) to be
units.
Selling price per unit $11.60
Variable production cost per unit $3.40
Sales commission 5% of selling price
Fixed production costs $430,500
Fixed selling and administration costs $198,150
Sales 90,000 units
FUNDAMENTALS OF MANAGEMENT ACCOUNTING
459
MOCK ASSESSMENT 2
2006.1
Question 21
Over long time periods of several years, supervisory labour costs will tend to behave as:
& linear variable costs
& step fixed costs
& fixed costs
& semi-variable costs
Question 22
A firm calculates the material price variance when material is purchased. The accounting
entries necessary to record a favourable material price variance in the ledger are:
Question 23
The accounting entries necessary to record an adverse labour efficiency variance in the
ledger accounts are:
Question 24
The following graph shows the wages earned by an employee during a single day:
Which ONE of the remuneration systems listed below does the graph represent?
& Differential piecework.
& A flat rate per hour with a premium for overtime working.
& Straight piecework.
& Piecework with a guaranteed minimum daily wage.
Debit Credit No entry in this account
Material control account && &
Work-in-progress control account && &
Material price variance account && &
Debit Credit No entry in this account
Wages control account && &
Labour variance account && &
Work-in-progress control account && &
Outpu
t
Wages ($)
0
MOCK ASSESSMENT C1
460
MOCK ASSESSMENT 2
2006.1
Question 25
J Ltd absorbs production overheads on the basis of machine hours. The following
budgeted and actual information applied in its last accounting period:
(a) At the end of the period, production overhead will be reported as:
& under-absorbed
& over-absorbed
(b) The amount of the under/over-absorption will be $
.
The following data are to be used to answer questions 26 and 27
E Ltd’s stock purchases during a recent week were as follows:
There was no stock at the beginning of the week. 420 units were issued to production
during the week. The company updates its stock records after every transaction.
Question 26
Using a first in, first out (FIFO) method of costing stock issues, the value of closing stock
would be $
.
Question 27
If E Ltd changes to the weighted average method of stock valuation, the effect on closing
stock value and on profit for the week compared with the FIFO method will be:
Budget Actual
Production overhead $180,000 $178,080
Machine hours 40,000 38,760
Day Price per unit ($) Units purchased
1 1.45 55
2 1.60 80
3 1.75 120
4 1.80 75
5 1.90 130
(a) Closing stock value will be: higher &
lower &
(b) Gross profit for the week will be: higher &
lower &
FUNDAMENTALS OF MANAGEMENT ACCOUNTING
461
MOCK ASSESSMENT 2
2006.1
The following data are to be used to answer questions 28 and 29
The diagram shows the profit-volume chart of Z Ltd for its last accounting period. The
company made a profit of $w during the period.
Question 28
An increase in the fixed costs per period (assuming the selling price per unit and the
variable cost per unit remain unchanged), will cause:
Question 29
The following results were achieved in the last accounting period:
The company expects to make and sell an additional 1,400 units in the next accounting
period. If variable cost per unit, selling price per unit and total fixed costs remain
unchanged, profit will increase by $
.
Question 30
A manufacturer of cell phones is considering the following actions. Which of these is likely
to increase the manufacturer’s C/S (contribution/sales) ratio (tick all that apply)?
(i) & taking advantage of quantity discounts for bulk purchases of material;
(ii) & introducing training programmes designed to improve labour efficiency;
(iii) & following the actions of a competitor who has cut prices substantially;
(iv) & reducing exports to countries where there is intense price competition;
(v) & offering retailers a lower price if they display the product more prominently.
increase decrease remain the same
r to && &
w to && &
t to && &
u to && &
r ¼$50,000 w ¼$16,000 t ¼800 units u ¼2,500 units
Sales unit
s
w
t
u
r
0
Profit ($)
Loss ($)
MOCK ASSESSMENT C1
462
MOCK ASSESSMENT 2
2006.1
Question 31
An advertising agency uses a job costing system to calculate the cost of client contracts.
Contract A42 is one of several contracts undertaken in the last accounting period. Costs
associated with the contract consist of:
Design staff worked 1,020 hours on contract A42, of which 120 hours were overtime. One
third of these overtime hours were worked at the request of the client who wanted the
contract to be completed quickly. Overtime is paid at a premium of 25 per cent of the
basic rate of $24.00 per hour.
The prime cost of contract A42 is $
.
Data for questions 32 and 33
Sales of product G are budgeted as follows.
Company policy is to hold in stock at the end of each month sufficient units of product G
to satisfy budgeted sales demand for the forthcoming two months.
Question 32
The budgeted production of product G in month 2 is units.
Question 33
Each unit of product G uses 2 litres of liquid K. Company policy is to hold in stock at the end
of each month sufficient liquid K for the production requirements of the forthcoming month.
The budgeted purchases of liquid K in month 2 are
litres.
Question 34
The following data have been extracted from the budget working papers of GY Limited.
(a) The total budgeted variable cost per unit is £
(b) The total budgeted fixed cost per period is £
Direct materials $5,500
Direct expenses $14,500
Month 1 Month 2 Month 3 Month 4 Month 5
Budgeted sales units 340 420 290 230 210
Production volume (units) 2,000
£ per unit
3,000
£ per unit
Direct materials 6.00 6.00
Direct labour 7.50 7.50
Production overhead department A 13.50 9.00
Production overhead department B 7.80 5.80
FUNDAMENTALS OF MANAGEMENT ACCOUNTING
463
MOCK ASSESSMENT 2
2006.1
Question 35
A company undertaking long-term building contracts has a financial year end of 30 April.
The following details on the purchase and use of machinery refer to contract A44, which
was started on 1 May year 3 and is due for completion after 27 months.
If the company’s policy is to charge depreciation in equal monthly amounts, the balance
sheet value of machinery on contract A44 at 30 April year 4 will be $
.
Question 36
Data for product W are as follows.
The company requires a 15% return on sales revenue from all products.
The selling price per unit of product W, to the nearest penny, is £
Question 37
G Ltd repairs electronic calculators. The wages budget for the last period was based on a
standard repair time of 24 minutes per calculator and a standard wage rate of $10.60 per hour.
Following the end of the budget period, it was reported that:
Based on the above information, the actual wage rate per hour during the period was
$
.
1 July year 3: Machine 1 was purchased at a cost of $55,000. It is to be
used throughout the contract, and will be sold for $6,400
when the contract finishes.
1 October year 3: Machine 2 was purchased at a cost of $28,600. The machine
will be scrapped at the end of contract A44, and is not
expected to have any saleable value.
Direct material cost per unit £22
Direct labour cost per unit £65
Direct labour hours per unit 5 hours
Production overhead absorption rate £3 per direct labour hour
Mark-up for non-production overhead costs 8% of total production cost
Number of repairs 31,000
Labour rate variance $3,100 (A)
Labour efficiency variance Nil
MOCK ASSESSMENT C1
464
MOCK ASSESSMENT 2
2006.1
Question 38
Which ONE of the following factors could explain a favourable direct material usage variance?
A & More staff were recruited to inspect for quality, resulting in a higher rejection rate.
B & When estimating the standard product cost, usage of material had been set using
ideal standards.
C & The company had reduced training of production workers as part of a cost
reduction exercise.
D & The material price variance was adverse.
Question 39
A company produces a single product B. The company budgets to sell 2,200 units of product
B during period 4 and sales are budgeted to be 10 per cent higher in period 5. It is company
policy to hold stocks of finished goods equal to 20 per cent of the following period’s sales.
The budgeted production of product B for period 4 is
units.
Question 40
The following extract is taken from the delivery cost budget of D Limited:
The flexible budget cost allowance for 6,200 miles travelled is £
.
Data for questions 41 to 49
Standard cost and revenue details for product C are as follows.
Budgeted sales and production for June were 47,200 units. However a machine breakdown
occurred and as a result labour were idle for 150 hours and actual sales and production
were 45,600 units.
Other actual data for June are as follows.
Question 41
The sales price variance for June is £ adverse/favourable
Miles travelled 4,000 5,500
Delivery cost £9,800 £10,475
£ per unit
Selling price 90.50
Direct material 12 kg at £1.70 per kg 20.40
Direct labour 3 hours at £14 per hour 42.00
Variable overhead 12.00
£
Sales revenue 4,058,400
Direct material cost for 539,800 kg purchased and used 944,650
Direct labour cost for 134,100 hours, including 150 idle hours 1,850,580
Variable overhead cost 542,800
FUNDAMENTALS OF MANAGEMENT ACCOUNTING
465
MOCK ASSESSMENT 2
2006.1
Question 42
The sales volume contribution variance for June is £ adverse/favourable
Question 43
The materials price variance for June is £ adverse/favourable
Question 44
The materials usage variance for June is £ adverse/favourable
Question 45
The idle time variance for June is £ adverse/favourable
Question 46
The labour rate variance for June is £ adverse/favourable
Question 47
The labour efficiency variance for June is £ adverse/favourable
Question 48
The variable overhead expenditure variance for June is £ adverse/favourable
Question 49
The variable overhead efficiency variance for June is £ adverse/favourable
Question 50
A company provides a shirt laundering service. The standard cost and revenue for
laundering one batch of shirts is as follows.
Fixed costs incurred each month amount to £15,900.
The number of batches of shirts to be laundered to earn a profit of £4,300 per month is
batches.
£ per batch
Selling price 23
Materials cost (detergent, starch, etc.) 3
Labour cost 14
Variable overhead cost 1
MOCK ASSESSMENT C1
466
MOCK ASSESSMENT 2
2006.1
Second Mock Assessment Solutions
Solution 1
Solution 2
(a) The margin of safety is shown on the diagram by k. This is the difference between the
expected sales level and the breakeven point.
(b) m will decrease (extra fixed cost ¼lower profit)
k will decrease (extra fixed cost ¼higher breakeven point ¼smaller margin of safety)
f will increase (extra fixed cost ¼higher breakeven point)
p will increase (p ¼fixed costs, which have increased)
Solution 3
The use of this method suggests the service departments carry out work for each other.
Solution 4
The combination that is certain to lead to over-absorption is production activity higher
than budget and fixed overhead expenditure lower than budget.
Solution 5
The costs are all production overheads with the exception of the cost of ingredients, which
is a direct cost.
Solution 6
A ¼£715
B ¼£979
Workings:
Cost per complete unit in process 2 ¼£22,695/5,100 ¼£4.45
Cost of abnormal gain units ¼£4.45 220 ¼£979
Debit Credit No entry in
this account
Production overhead control account 4
Work in progress account 4
Finished goods account 4
Profit and loss account 4
MOCK ASSESSMENT 2
FUNDAMENTALS OF MANAGEMENT ACCOUNTING
467
2006.1
Scrap value of normal loss per unit ¼£120/100 ¼£1.20
Forgone scrap value of abnormal gain ¼£1.20 220 units ¼£264
Transfer to profit and loss in respect of abnormal gain ¼£979 £264 ¼£715
Solution 7
Process account ¼credit; abnormal gain account ¼no entry in this account; abnormal loss
account ¼debit.
Abnormal loss ¼(4,000 2,750 400 700) units ¼150 units
Solution 8
The value of the closing WIP was $4,158.
Solution 9
The overtime premium paid to the operator would be £35.70.
Overtime ¼7 hours
Overtime premium per hour ¼£5.10
Overtime premium ¼£35.70
Statement of equivalent units
Total
units
Material
equiv units
Labour
equiv units
Production
overhead
equiv units
Finished goods 2,750 2,750 2,750 2,750
Normal loss 400
Abnormal loss 150 150 150 150
WIP c/fwd 700
700 350 280
3,600 3,250 3,180
$$$
Costs 16,000 8,125 3,498
Scrap value normal loss
(700)
15,300
Cost per equivalent unit $4.25 $2.50 $1.10
Statement of evaluation of WIP
$
WIP c/fwd material (700 $4.25) 2,975
labour (350 $2.50) 875
production overhead (280 $1.10)
308
4,158
MOCK ASSESSMENT C1
468
MOCK ASSESSMENT 2
2006.1
Solution 10
The estimated price notified to the customer for job number 808 will be £23,446.
Solution 11
Discounts are received on additional purchases of material when certain quantities are
purchased. The graph depicts a variable cost where unit costs decease at certain levels of
production.
Solution 12
The estimated cost of carrying out health checks on 850 patients is £17,625.
Solution 13
The principal budget factor for a footwear retailer is the constraint that is expected to limit
the retailer’s activities during the budget period.
£
Direct material 10,650
Direct labour
3,260
Prime cost 13,910
Production overhead (140 £8.50) 1,190
Mark up on prime cost (60%)
8,346
23,446
Patients Total cost
£
High 1,260 18,650
Low
650 17,125
610 1,525
Variable cost per patient ¼
£1,525
610
¼ £2:50
At 650 patients: £
Total cost 17,125
Total variable cost (650 £2.50)
1,625
Total fixed cost
15,500
Total cost of 850 patients: £
Fixed cost 15,500
Variable cost (850 £2.50)
2,125
17,625
FUNDAMENTALS OF MANAGEMENT ACCOUNTING
469
MOCK ASSESSMENT 2
2006.1
Solution 14
The estimated cash receipts from customers during the budget period are $54,950.
Cash received ¼ Sales þopening debtors closing debtors
¼ $ð55,800 þ8,500 9,350Þ
¼ $54,950:
Solution 15
The profit mark-up is 55%.
Cost of sales ¼ Opening stock þ purchases closing stock
¼ $ð5,500 þ38,000 7,500Þ
¼ $36,000
$36,000 þMark up ¼ $55,800
Mark Up ¼ $19,800
Mark Up% ¼
19,800
36,000
100% ¼ 55%:
Solution 16
The appropriate actions are (ii) and (iv). These are short term actions to cover a temporary
cash shortage. Actions (i) and (iii) would be more appropriate for a longer term cash shortage.
Solution 17
(a) The limiting factor would be direct labour.
(b) First: product Y; Second: product X; Third: product Z
X Y Z Total
Material (kg) 20,000 22,000 42,000 84,000
Direct labour (hours) 4,000 4,125 10,500 18,625
XYZ
£££
Selling price 28 22 30
Variable cost
17 13 24
Contribution 11 9 6
Kg 546
Contribution per kg £2.20 £2.25 £1.00
Ranking 2 1 3
MOCK ASSESSMENT C1
470
MOCK ASSESSMENT 2
2006.1
Solution 18
A performance standard which assumes efficient levels of operation, but which includes
allowances for factors such as waste and machine downtime is known as an attainable
standard.
Solution 19
The margin of safety represents 8.3% of budgeted sales.
BEP ¼
$ð430,500 þ 198,150Þ
$11:60 $ð3:40 þ0:58Þ
¼ 82,500 units
Margin of safety ¼
90,000 82,500
90,000
100% ¼ 8:3%
Solution 20
These changes will cause the breakeven point to be 79,879 units.
New BEP ¼
$628,650
$12:25 $ð3:40 þ0:98Þ
¼ 79,879 units:
Solution 21
Over long time periods of several years, supervisory labour costs will tend to behave as
step fixed costs.
Solution 22
Material control account ¼debit; work in progress ¼no entry in this account; material
price variance account ¼credit.
The price variance is calculated at the point of purchase therefore the work in progress
account is not affected. The favourable variance is credited to the variance account and
debited in the material control account.
Solution 23
Wages control account ¼no entry in this account; labour variance account ¼debit; work
in progress control account ¼credit.
The efficiency variance is recorded at the point at which it arises, i.e. in the work in
progress account rather than in the wages control account. The adverse variance is debited
to the variance account.
Solution 24
The graph represents piecework with a guaranteed minimum daily wage.
FUNDAMENTALS OF MANAGEMENT ACCOUNTING
471
MOCK ASSESSMENT 2
2006.1
Solution 25
Production overhead will be reported as $3,660 under absorbed.
Solution 26
Using FIFO, the value of the closing stock would be $76.
Units in stock ¼460 purchased 420 issued ¼40 units.
Issues would have been made at the earliest prices therefore the latest prices paid would
be used to value remaining stock ¼40 units $1.90 ¼$76.
Solution 27
(a) Closing stock value will be lower (prices are rising and FIFO uses latest prices to value
stock held in the stores)
(b) Gross profit for the week will be lower (higher average price charged to cost of sales)
Solution 28
r will increase (r ¼loss at zero activity ¼fixed costs)
w will decrease (w ¼profit ¼lower if fixed costs increase)
t will decrease (t ¼margin of safety ¼lower if fixed costs increase)
u will increase (u ¼breakeven volume ¼higher if fixed costs increase)
Solution 29
Profit will increase by $28,000.
Contribution per unit ¼(w + r)/(t + u) ¼$(16,000 + 50,000)/(800 + 2,500) ¼$20
Increase in profit ¼1,400 additional units $20 ¼$28,000
Solution 30
(i), (ii) and (iv) will increase the contribution/sales ratio.
(i) Lower variable costs per unit, higher contribution per unit ¼higher C/S ratio
(ii) Lower variable costs per unit, higher contribution per unit ¼higher C/S ratio
(iii) Lower selling price per unit, lower contribution per unit ¼lower C/S ratio
(iv) Higher average contribution per unit ¼higher C/S ratio
(v) Lower selling price per unit, lower contribution per unit ¼lower C/S ratio
Machine hour rate ¼$180,000/40,000 ¼$4.50 per machine hour
$
Overheads incurred 178,080
Overheads absorbed (38,760 $4.50)
174,420
Under absorbed
3,660
MOCK ASSESSMENT C1
472
MOCK ASSESSMENT 2
2006.1
Solution 31
The prime cost of contract A42 is $44,720.
Solution 32
The budgeted production of product G in month 2 is 230 units.
Workings:
Solution 33
The budgeted purchases of liquid K in month 2 are 420 litres.
Workings:
Purchases each month will be the quantity required for production the following month.
Production in month 3 ¼210 units (month 5 sales) therefore purchases in month 2 will be
210 2 litres ¼420 litres.
Solution 34
(a) The total budgeted variable cost per unit is £15.30
(b) The total budgeted fixed cost per period is £39,000
Workings:
Department A production overhead ¼fixed cost
¼2,000 units £13.50 or 3,000 units £9.00
¼£27,000
Department B production overhead ¼semi-variable cost
Using the high-low method:
$
Direct materials 5,500
Direct expenses 14,500
Basic staff hours 1,020 hrs $24 24,480
Overtime premium 40 hrs $6
240
44,720
units
Closing stock month 2 (290 + 230) 520
Month 2 sales requirements
420
940
Less opening stock month 2 (420 + 290)
(710)
Budgeted production month 2
230 (i.e. month 4 sales volume)
Units Total cost
£
3,000 17,400
2,000 15,600
1,000 1,800
FUNDAMENTALS OF MANAGEMENT ACCOUNTING
473
MOCK ASSESSMENT 2
2006.1
Variable cost per unit ¼£1,800/1,000 ¼£1.80
Fixed cost ¼£17,400 £(1.80 3,000) ¼£12,000
Total budgeted variable cost ¼£(6.00 + 7.50 + 1.80) ¼£15.30
Total budgeted fixed cost ¼£(27,000 + 12,000) ¼£39,000
Solution 35
The balance sheet value of machinery on contract A44 at 30 April year 4 is $55,060.
Net book value ¼$35,560 + $19,500 ¼$55,060
Solution 36
The selling price per unit of product W, to the nearest penny is £129.60
Workings:
Solution 37
Labour efficiency variance ¼zero, therefore hours worked ¼standard hours for 31,000
repairs.
Hours worked ¼31,000 24/60 ¼12,400 hours
Adverse rate variance per hour ¼$3,100/12,400 ¼$0.25
Therefore, actual wage rate per hour ¼$10.60 + $0.25 ¼$10.85
Solution 38
Option D is the only factor that could explain a favourable direct material usage variance.
Higher priced material may be of a higher quality than standard with the result that scrap
and rejections were lower than standard.
Options A to C are all likely to result in an adverse direct material usage variance.
Machine 1 Machine 2
$$
Cost 55,000 28,600
Depreciation
ð55,000 6,400Þ
25 months
10
19,440
28,600
22 months
7
9,100
35,560 19,500
£ per unit
Direct material cost 22.00
Direct labour cost 65.00
Production overhead absorbed ¼5 hours £3
15.00
Total production cost 102.00
Mark-up for non-production costs ¼8% £102.00
8.16
Full cost 110.16
Profit mark-up ¼15/85 £110.16
19.44
Selling price
129.60
MOCK ASSESSMENT C1
474
MOCK ASSESSMENT 2
2006.1
Solution 39
The budgeted production of product B for period 4 is 2,244 units.
Solution 40
The flexible budget cost allowance for 6,200 miles travelled is £10,790.
Variable cost per mile ¼£675/1,500 ¼£0.45
Fixed cost ¼£10,475 £(0.45 5,500) ¼£8,000
Total cost for 6,200 miles ¼£8,000 + £(0.45 6,200) ¼£10,790
Solution 41
The sales price variance for June is £68,400 adverse.
Workings:
Solution 42
The sales volume contribution variance for June is £25,760 adverse
Workings:
Units
Period 4 sales 2,200
Period 4 closing stock (2,200 1.10 0.20) 484
Period 4 opening stock (2,200 0.20)
(440)
Period 4 budgeted production
2,244
Miles £
High 5,500 10,475
Low
4,000 9,800
1,500 675
£
45,600 units should sell for ( £90.50) 4,126,800
But did sell for
4,058,400
68,400 adverse
Actual sales volume 45,600 units
Budget sales volume
47,200 units
Sales volume variance in units 1,600 units adverse
standard contribution per unit £16.10
£25,760 adverse
FUNDAMENTALS OF MANAGEMENT ACCOUNTING
475
MOCK ASSESSMENT 2
Solution 43
The materials price variance for June is £26,990 adverse
Workings:
Solution 44
The materials usage variance for June is £12,580 favourable
Workings:
Solution 45
The idle time variance for June is £2,100 adverse
Workings:
Idle time variance ¼150 hours idle £14 standard labour cost per hour ¼£2,100 adverse
Solution 46
The labour rate variance for June is £26,820 favourable
Workings:
£
539,800 kg should cost ( £1.70) 917,660
but did cost
944,650
26,990 adverse
45,600 units produced should use ( 12 kg) 547,200 kg
But did use
539,800 kg
Variance in kg 7,400 kg favourable
standard price per kg £1.70
£12,580 favourable
£
134,100 hours should cost (£14) 1,877,400
but did cost
1,850,580
26,820 favourable
MOCK ASSESSMENT C1
476
MOCK ASSESSMENT 2
Solution 47
The labour efficiency variance for June is £39,900 favourable
Workings:
Solution 48
The variable overhead expenditure variance for June is £7,000 adverse
Workings:
Solution 49
The variable overhead efficiency variance for June is £11,400 favourable
Workings:
Solution 50
The number of batches of shirts to be laundered to earn a profit of £4,300 per month is
4,040 batches.
Workings:
Contribution per batch of shirts ¼£(23 3 14 1) ¼£5
Number of batches to achieve required profit ¼£(15,900 + 4,300)/£5 ¼4,040 batches
45,600 units produced should take ( 3 hours) 136,800 hours
But did take (active hours)
133,950 hours
Variance in hours 2,850 hours favourable
standard rate per hour £14
£39,900 favourable
£
133,950 active hours should cost (£4) 535,800
but did cost
542,800
7,000 adverse
Efficiency variance in hours from labour variance 2,850 hours favourable
standard rate per hour £4
£11,400 favourable
FUNDAMENTALS OF MANAGEMENT ACCOUNTING
477
MOCK ASSESSMENT 2
Index
Abnormal costs, 250–2
Abnormal losses/gains, process costing, 250–1
Absorption costing, 54
concepts, 54
definition, 54
fixed production overhead variances, 150
marginal costing, 66–7
predetermination issues, 59–61
profits, 61
reconciliations, 172
standard hours, 123
under/over absorption issues, 59–61
variance analysis, 119–30
see also Overhead costs
Accounting, labour costs, 173
Activity levels, 13–14
Administration costs, 11, 54
Allocation, overhead costs, 53, 54, 123, 128, 212
Architect’s certificates, 220
Assessment:
mock questions/solutions, 424–37, 438–50, 454–66, 467–77
multiple-choice questions, 353
planning, 351–2
preparations, 351–419
questions, 355–86
tips, 352–3
Attribution concepts, 54, 212
Average cost (AVCO), 38, 287
comparisons, 36–7
Bad debts, 312
Balance sheets:
budgetary planning, 304
contract costing, 220–31
Batch costing, 218–19
Behaviour patterns, costs, 12–14, 18–19
Bonus schemes, 153
Bookkeeping, 184, 185–6
Bottom-up budgeting, 306
Breakeven analysis:
calculations, 87
charts, 92
concepts, 87
definition, 87
economist’s breakeven charts, 95
limitations, 94–5
Budget centres, 324
Budgetary controls:
concepts, 323–4
flexible/fixed budgets, 325–33
reports, 324
standard costing, 119–29
variances, 323
Budgetary planning:
actual profit reconciliations, 371
cash budgets, 311
co-ordination, 306
complete exercise, 315–16
concepts, 303, 305
definition, 305
information requirements, 306
interrelationships, 307, 311
manuals, 307–308
master budgets, 308
non-operating functions, 322–3
preparation processes, 305–308
profit reconciliations, 371
purposes, 303–305
rolling budgets, 322
spreadsheets, 308
time periods, 304
types, 305
Building contracts see Contract costing
C/S ratio see Contribution to sales (C/S) ratio
Capacity utilization:
limiting factors, 98
services, 292
Carriage costs, 7
Carriage inwards, 7, 8
Cash budgets:
concepts, 311–15
interpretations, 313–14
preparation processes, 305–308, 309
Cash discounts, 311
Charts:
breakeven analysis, 90–2
contribution breakeven charts, 92
economist’s breakeven charts, 95
profit-volume, 92, 93
Classifications:
concepts, 7–9, 221–2
contract costing, 220–31
costs, 221–2
definition, 7
Co-ordination, planning, 306
Codes:
costs, 282–3
definition, 282
Composite codes, 282–3
Composite cost units, 5
Computer systems:
budgetary planning, 308
integrated systems, 172, 174
job costing, 209–18
spreadsheets, 308
stock recording, 184
479
Consultancy business, service costing, 288
Continuous budgets see Rolling budgets
Continuous operation costing:
concepts, 6–7, 9–10, 33, 145–57
definition, 6
output costing, 12
process costing, 247–61
service costing, 54
Contract costing:
accounts, 220–1
architect’s certificates, 220
balance sheets, 222
classification of costs, 7–9
concepts, 220–31, 260
debtor accounts, 225
depreciation, 221, 312
materials, 221, 224
plant usage, 221
profits/losses, 220, 222, 224, 226
progress payments, 220
retention money, 220
work in progress, 220
worked examples, 222
Contribution breakeven charts, 92
Contribution concepts, 88
Contribution to sales (C/S) ratio, 89
Controllable costs, 151
Controls:
budgets, 331
concepts, 304
feedforward controls, 311
stock control, 188, 189
Cost accounting, 3–20
Cost centres:
absorption costing, 54
concepts, 6, 54
definition, 6
examples, 6
materials, 7
overheads, 6
production cost centres, 54, 56
Cost units:
absorption costing, 54
concepts, 4–6, 56–9, 287
continuous operation costing, 287
cost elements, 9
definition, 9
examples, 3–4
specific order costing, 207, 260
standard costs, 120
Cost-volume-profit (CVP) analysis, 87
see also Breakeven analysis
Costs:
absorption costing, 54
activity levels, 13–16
avoidable costs, 13
behaviour patterns, 12, 13, 15, 16, 18, 19
breakeven analysis, 87, 95
carriage, 7, 122
classification types, 4, 71
codes, 282–4
concepts, 4–9, 12, 14–16, 87–102, 247–51
continuous operation costing, 287
contract costing, 220–2, 227, 230
contribution concepts, 88
decision-making, 87
definition, 87
differential/incremental costs, 12, 66
elements, 9–11
fixed costs, 12–14, 87, 88, 90
job costing, 209, 210, 214
labour, 7, 8, 127
machine hours, 59
marginal costing, 66
materials, 8, 123
nature classification, 7
opportunity costs, 37
order costs, 209–32, 260
output costing, 12, 247
process costing, 247–61
production costs, 54, 56–8
purpose classification, 8
relevant costs, 212
semi-variable costs concepts, 15–19
service costing, 54
specific order costing, 209–32
standard costs, 119–30, 145–57, 184, 185–6
standard hours, 152–3
variable costs, 14, 124–5
variance analysis, 119–30
work certified, 220
see also Overhead costs
Curvilinear variable costs, 15
CVP see Cost-volume-profit (CVP) analysis
Debtor accounts, contract costing, 225
Decision-making:
breakeven analysis, 87
budgetary planning, 304, 305
contribution concepts, 88
costs, 12, 87
limiting factors, 98
marginal costing, 66
minimum price quotations, 59
relevant costs, 212
short-term restrictions, 98
stock, 125
Delivery notes, 210
Depreciation:
cash budgets, 311
contract costing, 220–31
integrated accounts, 171–93
plant, 221, 226
Differential costs, 12, 155
Differential piece rate systems, 155
Direct costs, 8, 210
Direct expenses, 10, 211
Direct labour:
cost variances, 124–5, 127, 130
costs, 8, 10, 11, 19, 59, 128
efficiency variance, 128, 129
overheads, 129
rate variance, 127
specific order costing, 260
total variance, 125, 127, 128
Direct materials:
cost variances, 124–9, 130
price variance, 125, 129
total variance, 125, 128
usage variance, 126
FUNDAMENTALS OF MANAGEMENT ACCOUNTING
INDEX
480
2006.1
Diverted hours, 65
Double-entry bookkeeping, 185
Economies of scale, 95
Economist’s breakeven charts, 95
Efficiency variance:
direct labour, 8, 10, 127, 128, 210
variable overheads, 14, 128, 129
Employers:
employment-related costs, 172
National Insurance Contributions, 7, 172
see also Labour
EOQ see Economic order quantity (EOQ)
Equivalent units, process costing, 252
Examination see Assessment
Expenses, costs, 4, 10–11, 211
Feedback controls, 304
Feedforward controls, 311
FIFO see First in, first out (FIFO)
Financial accounts:
integrated systems, 172, 174
non-integrated systems, 172
First in, first out (FIFO)
comparisons, 36
concepts, 34, 36
Fixed budgets, 325, 327
Fixed costs:
breakeven analysis, 87, 95
concepts, 12
contribution concepts, 88
definition, 12
marginal costing, 66
relevant costs, 212
semi-variable costs, 15, 16
Fixed production overhead variances, 53, 123
expenditure variance, 128
standard hours, 152–3
total variance, 125, 127–8
volume variance, 145
Flexible budgets:
concepts, 325–8
example, 325, 328, 331
planning, 328
preparation processes, 305–308, 328
Formulae, stock control, 192–3
Free stock balance, 192–3, 251
Functional planning, 309
Gains, abnormal, 250
Group incentive schemes, 156, 284
Guaranteed minimum wages, 155
High–low analysis method, 17
Historical data, usage problems, 18
Idle time, 148
Incremental budgeting, 323
Incremental costs, 12, 66
Indirect costs, 8, 10, 53, 66
see also Overhead costs
Inflation, 12, 37, 93, 323
Information requirements:
budgetary planning, 304, 305
standard setting, 120
Information technology see Computer systems
Input/output reconciliations, 253, 254, 257, 259, 276
Intangible cost units, 5, 287
Integrated accounting systems:
computer systems, 171
concepts, 171–93
definition, 171
examples, 174, 177
Interlocking accounting systems see Integrated...
Interpretations:
cash budgets, 311–14
variance analysis, 124
Interrelationships, budgetary planning, 307, 311
Job costing, 209–18
Labour:
accounting treatment, 171
costs, 7, 8, 127, 173
process costing, 247–61
remuneration systems, 154, 460
standard rates, 127
standard times, 120, 153, 173
time recording systems, 210
turnover, 12
see also Direct . . .
Last in, first out (LIFO)
comparisons, 36
concepts, 35
Lead times, 335
Ledger accounts, 184, 214
Level of activities, 87
LIFO see Last in, first out (LIFO)
Limiting factors:
capacity utilisation, 288
decision-making, 98
see also Principal budget factors
Linear variable costs, 14, 94
Losses:
abnormal losses, 250
contract costing, 220–31
normal losses, 249
process costing, 247–61
profit-volume charts, 92, 93
Machine hours, costing, 56, 123, 211, 219
Manuals, budgetary planning, 303
Margin:
concepts, 88, 286
of safety, 88–9
total sales margin variance, 88–9
Marginal costing, 66
absorption costing, 54
concepts, 88, 286
decision-making, 87
profits, 88–9
reconciliations, 145, 172
standard costing, 119–30
variance analysis, 119–30
Mark-up concepts, 12
Master budgets, 308
Materials:
concepts, 10
contract costing, 220–31
cost centres, 6, 54
costs, 7, 123
INDEX
481
FUNDAMENTALS OF MANAGEMENT ACCOUNTING
2006.1
Materials (Continued)
process costing, 247–61
purchases, 193, 308, 311
quotations, 59, 122
requisitions, 210
returned notes, 210
standard prices, 120
standard usage, 121
transfer notes, 37
valuations, 37, 126, 193
see also Direct . . .; Stock . . .
Maximum levels, stock control, 91
Minimum levels, stock control, 173
Minimum price quotations, 59
Mixed costs see Semi-variable . . .
Mock assessment:
questions, 353
solutions, 424–37
see also Assessment
Multiple-choice questions, assessment, 353
National Insurance Contributions, 172
Nature classification, costs, 7
Non-controllable costs, 324
Non-linear variable costs, 94
Non-operating functions, budgets, 322–3
Normal losses concepts, 249
Opening work in progress, costing, 256–8
Operational planning, 305
Opportunity costs, 37, 306
Order costs, 209–32, 260
Output costing, 12, 247
Overhead costs, 53–4, 123, 128, 212
absorption costing, 54
allocation, 54
apportionment, 54–7
concepts, 53, 54, 123, 212
definition, 53
functional analysis, 54
marginal costing, 66
reciprocal servicing, 64, 65
recovery, 56
repeated distribution method, 64
standard costing, 120–1, 122–3, 127
standard hours, 123, 152
under/over absorption issues, 59–61
variance analysis, 119–30
see also Fixed . . .; Variable . . .
Overtime premiums, 172
P/V ratio see Profit-volume (P/V) ratio
Participative budgeting, 306
Pay-as-you-earn (PAYE), 172
Performance levels, standard costing, 121–2
Period costs, 12
Periodic weighted average, 38
Perishable services, 287–8
Perpetual inventory systems, 39
Piecework remuneration systems, 154
Planning:
budgetary planning, 303–305
co-ordination, 306
concepts, 303–34
flexible budgets, 325, 328, 331
types, 305
Plant usage, 221
Preparation processes:
assessment, 351–3
budgets, 145
cash budgets, 311–14
flexible budgets, 325, 328, 331
functional planning, 309
Previous process costs, 256
Prices:
direct materials variance, 125–6
quotations, 122
standards materials, 122, 123
Prime costs, 59
Principal budget factors, 307
see also Limiting factors
Priority-based budgeting see Zero-base budgeting (ZBB)
Process costing, 247, 258, 260
abnormal losses/gains, 250
accounts, 247
concepts, 247, 258, 260
equivalent units, 252
input/output reconciliations, 253, 254
losses, 249
opening work in progress, 256
previous costs, 256
work in progress, 256
Production:
cost centres, 54, 65
costs, 8
Profit-volume (P/V) ratio, 89
see also Contribution to sales (C/S) ratio
Profit-volume charts, 92, 93
Profits:
absorption costing, 54
actual/budgeted profit reconciliations, 145
breakeven analysis, 87, 94, 95
budgetary planning, 303, 304–305
contract costing, 220–31
contribution concepts, 88, 98
margin concepts, 88, 286
margin of safety concepts, 88–9
marginal costing, 12, 66
mark-up concepts, 12, 66
recording variance, 184
stock valuation methods, 126
total sales margin variance, 91
variance analysis, 119–30
Progress payments, 220
Prudence concepts, 222
Purchases:
materials, 126, 311
orders, 56
requisitions, 56
suppliers, 174
Purpose classification, costs, 7, 281–2
Quantification, budgetary planning, 304
Quotations:
materials, 29
minimum prices, 155
Reciprocal servicing, 64, 65
Reconciliations:
absorption/marginal costing, 56–9, 88
actual/budgeted profits, 145–8
process costing, 247–53
INDEX
482
FUNDAMENTALS OF MANAGEMENT ACCOUNTING
2006.1
Recording systems:
stock movements, 210, 221
time, 184
variances, 184
Recovery, overhead costs, 56
Relevant costs, 212
Remuneration systems, 460
see also Labour
Repeated distribution method, 65
Replacement costs, 37
Reports, budgetary controls, 323
Requisitions, materials, 210
Retention money, contract costing, 220
Rolling budgets, 322
Sales:
breakeven analysis, 87, 94, 95
contribution concepts, 88, 98
total sales margin variance, 129, 152
Scattergraph analysis, 17
Selling and distribution costs, 11, 176
Semi-variable costs, 15–16
Service cost centres, 54, 61
Service costing, 54, 64, 65
Short-term decision-making, 98
Special orders, quotations, 66
Specific order costing, 209–32, 260
batch costing, 209, 218–19
concepts, 209–32, 260
see also Contract . . .; Job . . .
Specific price method, 34
Spreadsheets, budgetary planning, 120, 308
Standard costing, 119–30, 184
bookkeeping, 184, 185
budgetary controls, 323
concepts, 119–30, 145–57
definition, 119
ledger accounts, 184–5, 214
marginal costing, 66
modern environment, 124
setting, 122–3
stock, 126
updates, 124
see also Variances
Standard hours, 152
Standard marginal costing, 88
Standards:
concepts, 122–4
definition, 120
Stepped fixed costs, 13, 19
Stock:
absorption/marginal costing contrast, 56
AVCO valuation method, 193
concepts, 126, 192, 193
costs, 192
FIFO valuation method, 34, 36
levels, 316
LIFO valuation method, 36, 45
periodic weighted average, 38
profit effects, 37
valuations, 126, 193
work in progress, 188
see also Materials
Stock control:
computerised systems, 171, 210, 283, 284, 308
control levels system, 54, 303
formulae, 36
free stock balance, 192
holding costs, 307, 311
movement recording systems, 39
order costs, 209
perpetual inventory systems, 39
Stores:
ledger cards, 34–6
materials, 33
Strategic planning, 305
Suppliers:
delivery notes, 210
purchases, 211
Tactical planning, 305
Tangible cost units, 5, 288
Taxation:
PAYE, 172
Time-based remuneration systems, 154
Time issues:
budgetary planning, 304, 305
standard hours, 123, 152
standard labour times, 123, 153, 154
Time recording systems, labour, 210
Timesheets, 210
Total sales contribution variance, 129, 130
Total sales margin variance, 286
Trade discounts, 68
True and fair view, 220
Turnover, labour, 12
Under/over absorption issues, 59–61
Usage variance, direct materials, 126
Valuations, stock, 126, 193
Variable costs:
breakeven analysis, 87, 95
concepts, 14, 124
definition, 88
relevant costs, 212
Variable overheads:
cost variances, 124, 125, 127, 128
efficiency variance, 128, 129
expenditure variance, 128
total variance, 125, 127
Variances:
absorption costing, 54
analysis, 124
budgetary control, 323, 324
causes, 149
concepts, 124, 148–9
fixed production overhead, 54, 123
idle time, 148, 173
interpretations, 149, 371
ledger accounts, 184
marginal costing, 66
production overheads, 148
significance issues, 150, 151
see also Standard costing
Wages see Remuneration systems
Weighted average costs, 35, 38
Work certified costs, 222
Work in progress:
contract costing, 220–2, 227, 230
opening work in progress, 256
process costing, 247, 258, 260
Zero-base budgeting (ZBB), 323
INDEX
483
2006.1
FUNDAMENTALS OF MANAGEMENT ACCOUNTING